Download as pdf or txt
Download as pdf or txt
You are on page 1of 447

SMLE 2019

AUGUST

Corrected by: Glory Team


Glory team August

● Anal fissures with skin tag not respond to drug, want procedure?
A- Internal sphernctomy
B- External sphrenctomy
C- Anal curettage with remove tag
Correct answer is C

N.B: For the skin tag it’s called sentinel pile and it’s come with 20% of patient with anal fissure; it’s one of the
diagnostic factors. (Source: Best practice)

INCLUDEPICTURE
● Pic for cystourethrogram, ask dx: "C:\\var\\folders\\m7\\hk4xywwx6tj_nkdq9bzjtq
sw0000gn\\T\\com.microsoft.Word\\WebArchiv
1- Cystourethra reflex eCopyPasteTempFiles\\page1image29591584"
2- Renal agensis \* MERGEFORMAT
3- Urethta stone
Correct answer is 1

N.B: vesicoureteral reflux

● Q about dx of Mitral stenosis


The best initial test for all valvular heart disease is echocardiogram. Transesophageal echo is
generally both more sensitive and specific than transthoracic echo. The most accurate test is
Catheterization

● Meconium aspirations > nitrous oxides.

● benign and malignant Phyllodes tumor treatment


1- mastectomy
2- wide local excision
Correct answer is 2

rare fibroepithelial breast tumors that can be benign or malignant, 35-55 y/o
Glory team August

small: Wide local excision (wide margined zone)


or lumpectomy ‘with <1cm margined zone’
large: Mastectomy.
malignant you should give chemo or radio with
it.
N.B: No need to take LN with small or larger if
there is no s/s of malignancy, if the question
come without size it’s better to choose excision
not mastectomy ‘Source: breastcancer.org and
Kaplan CK 2 lecture notes’

● Neonatal hypoglycemia> give glucose in central line


Correction of Neonatal hypoglycemia We start with *D10* in *peripheral* line.
If no response Shift to *D12.5* in *central* line.

● Acute pancreatitis with high amylase, ask investigation


A- Amylase urine.
B- CT abdomen.
Correct answer is B

Answer: CT or US will show enlarged pancreas with peripancreatic fluid and fat stranding, abscess,
hemorrhage, necrosis, or pseudocyst. if severe; “sentinel loop” or “colon cutoff sign” on x-ray of the abdomen.

● jaundice with us show stone in GB and in CBD, tx.


a. Laparoscopic cholecystectomy
b. ERCP
Correct answer is B

Answer: since gallstone present in both GB and CBD so case of choledocholithiasis (if with infection present so
Cholangitis ‘Charcot’):-
Initial: NBO, IV fluids, Abx (Cipro + Metronidazole, or Ampicillin + Genta. + Metronidazole).
Urgent: ERCP ‘remove the CBD stone and definitive treatment of cholangitis’. Elective: Lap cholecystectomy
(definitive treatment for gallstones)

● Q about renal impairment with arteriovenous anastomoses or shunt, what’s the type of
arteriovenous:

1- Brachioradialis
2- Radiocephalic
Correct answer is 2

Answer: AV fistula types for dialysis: Radio cephalic (1st choice) at forearm, Brachiocephalic (2nd choice) at
elbow and Brachioradialis.

● Palpable gallbladder, with pain less jaundice, didn’t mention painful gallbladder, ask about Dx:
Glory team August

1. GB cancer
2. Cholthisis
3. Pancreatic cancer.
Correct answer is 3
Primary sclerosing cholangitis with palpable gall bladder + obstructive jaundice is Klatskin tumor
Otherwise any palpable GB with obstructive jaundice is periampullary Ca .

● A lot Q about Abortions.

● placenta abrution dx , mx

● Management of hyperkalemia with/ without ECG changes. ** see glory file.


Glory team August

INCLUDEPICTURE
● About fixed hard lump irregular margins > "C:\\var\\folders\\m7\\hk4xywwx6tj_n
breast cancer. kdq9bzjtqsw0000gn\\T\\com.microsoft.
Word\\WebArchiveCopyPasteTempFil
● Thyroid nodule >>>>>>>> es\\4dbc3792-efe0-4c92-8b33-
fe62852e0972" \* MERGEFORMAT

● Cardiac tamponade best diagnosis by?? ECHO.

● about COPD, total lung capacity, PEFV1, another parameter I can’t remember, ask about which
increase and decrease.
COPD: Increase in Lung capacity or air compliance Decreased in FEV1 & FVC.

● Simple q about adenomyosis> came with Hx of uterus surgery, Adenomyosis usually enlarged uterus
with tenderness. Definitive diagnosis is only made by Hysterectomy, best modality MRI.

● Patient post MI developed palpitation >> ECG >> wide QRS suggests bundle branch block ((asking for
Dx))? LBBB

● COPD exacerbation ((hypoxia, acidosis, hypercapnia 7.99)) >>


management > INCLUDEPICTURE
a. face mask "C:\\var\\folders\\m7\\hk4xywwx6t
b. mechanical ventilation. j_nkdq9bzjtqsw0000gn\\T\\com.mi
Correct answer is B crosoft.Word\\WebArchiveCopyPas
teTempFiles\\page9image29498688
In general non-invasive mechanical ventilation is preferable in COPD " \* MERGEFORMAT
but consider invasive if non-invasive has failed or :-
severe hypoxemia PaO2 <40mmhg
severe resp.Acid ph<7.25. hypercapnia >60mmhg. respiratory arrest.
cardiovascular collapse.
Glory team August

● COPD pt. / developed, c/m only from


left pleuritic chest pain, Pneumothorax
2cm btw chest wall and pleura >>
Management
a. needle aspiration
b. chest tube
c. Observation
Correct answer is B

● pregnant lady known case of asthma, on LABA, ICS, SABA ((uncontrolled/late night attacks)) >> Q.
Management? Montelukast, if not pregnant leukotriene.

● Pt with left Lateral neck mass ((third triangle just below angle of mandible)) ... U/S thyroid normal
and post. Cervical lymph node enlargement, on specimen it’s shows follicular thyroid cells >> dx.
A- thyroid carcinoma/ or lateral aberrant.
B- apparent thyroid gland.
C- ectopic thyroid gland.
D- thyroglossal cyst.
Correct answer is A
● 7 y old female started breast development, and pubic hair start to appear and acne >> dx.

A- ovarian tumor,
B- central
C- gonadotropin
Correct answer is B
● Type of HBV vaccine >>

A- live,
B- attenuated,
C- recombinant
Correct answer is C
● Pt with hx. Of retinal detachment surgery 7 day ago and he is going into surgery.... >>
Q. Wt to use to prevent DVT
A- Enoxaparin
B- Pneumatic INCLUDEPICT
C- mixed pneumatic and enoxaparin URE
D- aspirin "C:\\var\\folders
Correct answer is B \\m7\\hk4xyww
● Immunocompromised pt. his younger brother developed chicken box >> how to manage him
x6tj_nkdq9bzjtq
A- Vaccine sw0000gn\\T\\c
B- IVIG
om.microsoft.W
C- nothing
Correct answer is B
ord\\WebArchiv
N.B: Varicella-zoster immune globulin is recommended for people who cannot receive theeCopyPasteTem
vaccine and 1)
who lack evidence of immunity to varicella, 2) whose exposure is likely to result inpFiles\\8a2547e
infection, and
3) are at high risk for severe varicella. Reference: CDC 1-a7c3-4cd1-
a630-
5e6d2ccca961"
● Hydatid cyst and echinococcus cyst >> Management \*
Medically albendazole MERGEFORM
AT
Glory team August

Surgically if large in size , have multiple daughter cystes , ruptured , and if there is biliary communication

● Pt after head trauma he couldn’t make the spoon reach his mouth to eat >> where is the lesion

A- Temporal
B- partial,
C- occipital,
D- cerebellum
Correct answer is D
● Child after period of illness ((can’t remember is URTI or GI )) developed symmetrical lower limb
ascending weakness >> (( GBS ))

● DM, Lateral malleus ulcer with distal pulse >> Q. Investigation?


venuse US

N.B:Since distal pulse is intact the ulcer is most likely venous,

● DVT w/o distal pulse >> Investigation


Answer: US Doppler

● Damage to the cerebellum can lead to: this is information’s not answers.

1. loss of coordination of motor movement (asynergia),


2. the inability to judge distance and when to stop (dysmetria),
3. the inability to perform rapid alternating movements (adiadochokinesia),
4. movement tremors (intention tremor),
5. staggering, wide based walking (ataxic gait)

● Some of the tests your doctor may rely on to diagnose peripheral artery disease are:

1. Physical exam.
2. Ankle-brachial index (ABI)>
3. Ultrasound.
4. Angiography
5. Blood tests.
Correct answer is 2
ABI for chronic peripheral artery disease.

● Female, unilateral leg swelling, progressive, pitting edema >> Q. Investigation?


Initial: Doppler, best: MRV
Glory team August

● Child with bilateral inguinal hernia >> Management


All children with a bilateral presentation should undergo bilateral inguinal hernia repair under a single
anesthesia, * bilateral lap herniotomy*

● Pt. After going into open hernia repair. developed bulging mass with cough same area >>
Management
Recurrent hernia > If first was lap treat it open, If first was open treat it lap.

● Which of the following indicate searching for organic cause in a child>>?

A- peri umbilical abdominal pain


B- last more than 10 min
C- around morning
D- before sleep time
Correct answer is D

● Meningitis case, LP shows gram- ve diplococci, he is close to his brother. How will you manage the
brother?
oral rifampine

● Pregnant routine screening with UA suggest infection >>


(( asymptomatic UTI )) treated by?
Nitrofurantoin, if after delivery: not treated.

● Pregnant 28w, with UTI. Which antibiotic CANNOT be used?

A- nitrofurantoin,
B- fluoroquinolones,
C- amoxicillin
Correct answer is B

N.B: all fluoroquinolones are contraindicated in children, adolescents, and pregnant or breast-feeding
women. Reference: The fluoroquinolones. in PubMed

● Pregnant ( early pregnancy ), allergic to penicillin developed UTI. Treatment?


Glory team August

A- cephalosporin
B- fluoroquinolones
C- nitrofurantoin
D- amoxicillin
Correct answer is C
N.B: Use of this agent is restricted the last several weeks of pregnancy
Reference: American Academy of Family Physicians and medescape
● ACEI angioedema (ramipril)

Angioedema is a well-described side effect of treatment with both ACE inhibitors and ARBs. The
presentation of this form of angioedema is classical with the predominant changes being found in the
upper respiratory tract. Tongue swelling can be a particularly prominent finding in this disorder. There is
currently no diagnostic test that specifically identifies those at risk for development of angioedema. Once
angioedema has occurred with an ACE inhibitor or an ARB the offending agent should not be
readministered. Angioedema can be treated with antihistamines, epinephrine, and/or steroids, although
the specific efficacy of these agents has not been prospectively studied. Most episodes of angioedema are
managed with antihistamine therapy alone. Common Clinical features: swelling of thelips, tongue, face.
And abdominal pain (intestinal angioedema)

● Mechanism of action of metformin?

A- decrease insulin resistance


B- increase blood glucose consumption
C- increase insulin secretion
Correct answer is A
Metformin reduces liver (hepatic) production of glucose, decreases the intestinal absorption of glucose, and
enhances insulin sensitivity by increasing both peripheral glucose uptake and utilization

● Child with nephrotic syndrome. Management?


Prednisolone mainly steroid
N.B: The most widely used protocol is to initially give oral corticosteroids (60 mg/m 2 per day of
prednisolone), unless there are atypical features. After 4 weeks, the dose is reduced to 40 mg/m 2 on
alternate days for 4 weeks and then weaned or stopped.

● Child develop seizure


A- IV lorazepam
B- intra nasal diazepam
C- rectal diazepam
Correct answer is A
N.B: if Status Epilepticus if vascular access, give IV Lorazepam
If No, give Buccal Midazolam or Rectal Diazepam Reference: Illustrated Textbook of Pediatrics

● SLE Pt developed seizures at home and then come to ER what to give

A- cyclophosphamide, or prednisone if on cyclo.


B- hydroxychloroquine,
Seizures as a rare adverse effect of chloroquine therapy in systemic lupus erythematosus patients: a case
report and literature survey
In neonate phenobarbital
SLE encephalopathy is indicated for Cyclophosphamide but treating acute seizures by phenytoin

● Description of a patient with 3 symptoms of SLE. What the fourth symptom of the 11 according to
(RHEUMATOLOGY, SLE organization) to diagnose this pt with SLE?
Glory team August

1. type of rash can’t remember,


2. RF positive
3. Hemolytic anemia.
4. positive ANA
Correct answer is C

New ameican guidlines no ANA in the criteria plus they mention in exam according to what guidelines you
will diagnose SLE

● Child with mass in his left flank, UA shows hematuria. Dx?


1. wilms tumer
2. neuroblastoma
Correct answer is 1

most common manifestation of Wilms tumor is an asymptomatic abdominal mass; an abdominal mass occurs
in 80% of children at presentation. Abdominal pain or hematuria occurs in 25%. Hypertension, gross
hematuria, and fever are observed in 5-30% of patients
Neuroblastoma is more accurately described as an abdominal mass.

● child with Hx. Of URTI two weeks ago, developed glomerulonephritis. Dx?
PSGN

Answer: RPGN is usually caused by one of the three following mechanisms: anti- GBM antibody disease
without or with pulmonary hemorrhage “Goodpasture`s syndrome”, pauci-immune glomerulonephritis, and
severe immunocomplex glomerulonephritis
Reference: Saudi Journal of Kidney Diseases and Transplantation

● Rapidly progressive glomerulonephritis, Histo SHOWS:


1. crescent shape formation
2. IgA deposition
Correct answer is 1
● 4y old child came with his parents for umbilical hernia. Wt to do?
A- reassurance
B- repair
C- Till age 5Y
D- Repair it at age of 5
Correct answer is A
N.B: Watchful waiting is recommended for children who have no symptoms. In 95% of cases,
umbilical hernias less than 1 cm in diameter usually close on their own within 5 years of age. After
age 5, repair is recommended.
Reference: Amircan College of Surgeons and Medscape

● 4 months Child with mid-shaft hypospadias, came for circumcision. What u will do?
A- Do it
Glory team August

B- not possible since they will use it for the repair


C- should be delayed
Correct answer is B

N.B:The tubularized incised plate (TIP)repair is now commonly used for distal and midshaft hypospadias, and
it need a flap

● Microscopic polyarteritis?
Polyarteritis nodosa is present with Wight loss, Testicular pain, Levidu reticularis, Abdominal pain,
Polyneuropathy and HBV positive

● Granulomatosis with polyangiitis (Wegener's)


Wegener granulomatosis (Granulomatosis with polyangiitis) is present with Glomelronephritis ,
persistent rhinorrhea , mouth and nasal ulcer, saddle nose, bloody from nose-ANCA, Treated by
cyclophosphamide

● Dm diagnosis criteria:
Type 2 Diabetes ADA Diagnosis Criteria
Diagnostic criteria by the American Diabetes Association (ADA) include the following: A fasting plasma
glucose (FPG) level of 126 mg/dL (7.0 mmol/L) or higher, or A 2-hour plasma glucose level of 200
mg/dL (11.1 mmol/L) or higher during a 75-g oral glucose tolerance test (OGTT), or A random plasma
glucose of 200 mg/dL (11.1 mmol/L) or higher in a patient with classic symptoms of hyperglycemia or
hyperglycemic crisis Whether a hemoglobin A1c (HbA1c) level of 6.5% or higher should be a primary
diagnostic criterion or an optional criterion remains a point of controversy.In the presence of
hyperglycemic symptoms (polyurea, polydipsia, polyphagia, weight loss, blurry vision) a confirmatory
test is not required (no need to repeat).
In the absence of hyperglycemic symptoms a repeat confirmatory test is required.

● Cushing case ACTH high cortisone high after suppressing what investigation?
brain CT but MRI is better for sure.

● 2 years old Sickler had a crisis 3 weeks ago with blood transfusion came for 2nd dose hep A asking
when to vaccinate
A- gives 2nd dose now INCLUDEPICTURE
B- check hep A titer "C:\\var\\folders\\m7\\hk4xywwx
C- wait 3 months 6tj_nkdq9bzjtqsw0000gn\\T\\com.
Correct answer is A microsoft.Word\\WebArchiveCopy
● Vaccine contraindicated in pregnancy. PasteTempFiles\\page25image295
A- MMR 23600" \* MERGEFORMAT
B- Varicella
C- Both are contraindicated.

● post hysterectomy type of hormonal replacement


1. estrogen alone (this is the answer if it was with an oophorectomy)
Glory team August

2. combined-

Women who have both the uterus and ovaries removed usually just get estrogen replacement therapy (ERT)
alone. But women who have only the ovaries removed need both estrogen and progestin.

● Pt with DM not hypertensive developed proteinuria>> Start ACE

● contraindications of ECV-12

● Child with wheezing and cough on and off poorly responding to bronchodilators His father is a
smoker and they have family HX of asthma what is the best indicator if the child has C-Fibrosis.
A- poor response to meds, bronchodilators.
B- Poor weight gain.
C- father smoking
D- family hx of asthma
Correct answer is B

also, could be nasal polyp, also sinitis

● Child with Down syndrome what is the most striking feature


A- simian crease
B- hypotonia
C- Small toe
Correct answer is A

● Pt with liver cirrhosis have UGIB what to give before endoscopy


A- vitamin K
B- octreotide
C- PPI
Correct answer is B

Variceal bleeding: Octreotide decreases the inflow of blood to portal system by constricting the splanchnic
arterioles and significantly reduces intravesical pressure.

● 15 months old babbles but he doesn't follow 2-word commands, hearing examination done when he
was 6-week-old and it was normal what to do:
Glory team August

A- refer to ENT
B- schedule audiogram
C- reassure parents as it is normal variation
Correct answer is C

The baby can understand 2 step commands at 2 years of age

● 40 years old female LMP 2 months ago came to ER with on/off vaginal spotting she has been trying
to get pregnant for 3 years what is the first step:

A- confirm the pt state of pregnancy


B- Ask about infertility reason
C- Pelvic US
Correct answer is A
● Small intestine obstruction did appendectomy 5 years back X ray air fluid level, Ct done What
investigations done next:

Small Intestinal obstruction is usually caused by Adhesions, Bulge (hernias) or Cancers. The 1st cause of SBO in
adults is Postoperative adhesions (Hx of appendectomy)
X ray is done -CT also is Done The questions is not complete so it could be The next step or the next
investigation to be done: MRI effective in defining the location and etiology of obstruction.(Medscape)
Tx according to the obstruction type: Complete: Laparotomy and lysis of adhesions Incomplete: Conservative
Tx +NGT decompression (Surgical Recall + Medscape)

● Child with fever conjunctivitis, coryza and cough with intercostal recession and low O2 sat
tachycardic Tachypneic what is the optimal initial step: - i
A- Intubation
B- O2 therapy
Correct answer is B
This is Could be a case of Bronchiolitis so the initial Tx is :O2

Oxygen by nasal prongs must be given for at least 15 hours a day (couldn’t Find Direct information ,but it is
usually canula) Current Medical Diagnosis & Treatment 2019

● Pt with cough and wheeze chest x ray normal spirometer normal What to do next:
A- challenge test
B- repeat spirometer
Correct answer is A

Challenge Test: if spirometry is normal or near-normal and asthma is still suspected; a negative test is helpful
in ruling out asthma.
The Color Atlas and Synopsis of Family Medicine, 3e

● Child born with complete heart block What is the cause


A- Rubella
B- DM
C- lupus
Correct answer is C
Glory team August

In Neonatal Lupus Heart block results from the deposition of antibodies into the cardiac conduction system,
leading to local inflammation and irreversible damage to the atrioventricular node CURRENT Diagnosis &
Treatment: Rheumatology, 3e

● Asherman syndrome what layer of endometrium is affected?


A. Basalis
B. Functionalis
Correct answer is A

Basal layer. the endometrium is divided into a functional layer, which lines the endometrial cavity, and a
basal layer, which regenerates the functional layer
Destruction of the basal endometrium prevents endometrial thickening in response to ovarian steroids.
Williams Gynecology, 3e Chapter 16

● Pt with dorsum of the left-hand numbness over the thump index and middle finger what nerve is
affected
A- Radial
B- median
C- Ulnar
D- Axillary
Correct answer is B

● Case of women pain in her hand or palm and wrist due to overwork in keyboard which is affected?
A- Ulnar
B- Radial
C- Anterior interosuss
D- Posterior interossus

N.B: Median nerve is the answer, C is a branch of it.


N.B: anterior interosseous syndrome will present with motor deficits only. No sensory changes should
be appreciated. And if there’s pain it will be in the forearm and cubital fossa.

● 10-month baby had umbilical mass reducible with cough what management
1. Surgery
2. Observation
Correct answer is 2
90% will close spontaneously, if not surgery intervention after the age 4-5 years

● pregnant 13 weeks came with vaginal bleeding, PV examination: dilated OS, you felt or seen product
of conception near cervical os, Diagnosis ?
A- complete abortion
B- missed
C- incomplete
D- inevitable.

Os dilated
If patient passed tissue > incomplete
If patient passed blood only and by examination you feel tissue > Inevitable

● 15-month boy refused to walk and want his parents to hold him he stepped over toys and twisted his
leg What Dx
Glory team August

A- fracture at the end of the tibia


B- Fracture at the end of Rt fibula
C- Rt Knee injury
D- Proximal tibial fracture
Correct answer is A, (spiral fracture of the tibia is more accurete )

Toddler fractures occur in young ambulatory children (from 9 months to 3 years). A toddler's fracture is a
spiral or oblique undisplaced fracture of the distal shaft of the tibia with an intact fibula. The periosteum
remains intact and the bone is stable. These fractures occur as a result of a twisting injury. Septic arthritis
and osteomyelitis should be excluded.

INCLUDEPICTURE
● Dm pt had nephropathy what the urine finding? "C:\\var\\folders\\m7\\hk4xy
a. Hyaline cast wwx6tj_nkdq9bzjtqsw0000gn
b. RBC cast. \\T\\com.microsoft.Word\\W
c. Hematuria ebArchiveCopyPasteTempFiles
d. Proteinuria \\page72image29795472" \*
MERGEFORMAT

● A method to install good habits (behavioral changes):


Answer: Positive reward (positive Reinforcement)
Example; Teacher should talk about right things students are doing
Reference: https://positivepsychology.com/positive-reinforcement-psychology/ Q) Peritonitis

● Tx. Of SVT in stable patient

● What to do if pt. want female dr. to examine?


respect.

● Case of GDM with high postprandial only what to do?

A- Metformin
B- insuline if diagnosed
C- Repeat test IF not diagnosed
Glory team August

● An athletic guy, who has syncope attacks, what is the diagnosis?


HOCM.

● Case of ped. Came whooping cough?


pertussis.

● Age of cancer screening "colon”

● Child complain from fever and confusion, later developed petechial rash dx?
Meningococcemia

● Bacterial tonsillitis case, most common complication?


A- pharyngitis
B- scarlet fever
C- glomerulonephritis
Correct answer is B

● Pt developed convulsion + other symptoms. Dx?


A- hyperkalemia
B- hypokalemia
C- hyperammonemia
D- hypomagnesemia
Correct answer is D

● Child pulling his ear. otoscope (signs of inflammation) dx?


A- otitis externa
B- otitis media
Correct answer is B
● Couples married and try to conceive for 3y with hx. Of the wife of PID treated... semen analysis was
normal, HSG showed normal uterine cavity, no free spillage of contrast bilaterally. Management?
A- induction of ovulation
B- IVF
Correct answer is B
● Child found to have effusion in hed ear, he eats well, wt to do?
A- Antibiotics
B- refer him to ENT for tympanostomy tube, observation first 3-4 months

● Neuro Brucellosis >> duration of treatment


A. 6weeks
B. 4weeks
C. 4months
D. 6months
Correct answer is D

● H.pylori, treatment?

● Duodenal perforation management?


Graham-omental patch

● Viral infection cause tonsillitis with exudate


A- RSV
B- EBV
Glory team August

C- corona
D- influenza
Correct answer is B

● Pt developed bilateral tingling sensations, with hx. Of gastrostomy, which vitamin deficient
A. B1
B. B2
C. B12
D. FOLIC acid
Correct answer is C

● Child with days of bleeding per vagina and itching >>


A- Abuse, if with stool incontinence and laceration
B- foreign body, only bleeding itching
Correct answer is B

● Q/Eczema sites at 9 month child:

N.B :Infant within 12 months it is face and extensors, Beyond infants it is flexors Distribution pattern of
Eczema: Infancy: face, scalp, extensor surfaces of extremities /Older, long-standing disease: flexural
aspects

● Q/Child with x-ray of colles fracture. Management?


Treatment is with close reduction and long arm cast.
INCLUDEPICTURE
"C:\\var\\folders\\m7
\\hk4xywwx6tj_nkdq9
● Q/ (pic of X-ray) with upper lobe cavitation with air-fluid level. Tx? bzjtqsw0000gn\\T\\co
m.microsoft.Word\\W
ebArchiveCopyPasteT
empFiles\\page61ima
drug therapy with isoniazid (INH), rifampin (Rif), pyrazinamide (PZA), and ethambutol ge29740080" \*
MERGEFORMAT
(ETB). All 4 drugs are continued for the first 2 months. PZA and ETB are then discontinued, and
therapy continues with INH and rifampin for another 4 months.
(USMLE Step 2 CK Lecture Notes 2017 – p238)

● Q/ Colon screening once asked for average risk patient and the second for low risk patient

Single family member: Begin 10 years earlier than the age at which the family member developed
their cancer or age 40, whichever is younger. Repeat the scope every 5 years if the family member is
under age 60.
Three family members, 2 generations, 1 premature (before 50): Hereditary nonpolyposis colon cancer
syndrome (HNPCC) comprises these factors. Start screening at age 25 with colonoscopy every 1 to 2
years.
Familial adenomatous polyposis (FAP): FAP is defined as the presence of thousands of polyps with an
abnormal genetic test known as the adenomatous polyposis coli (APC) test. Start screening with
sigmoidoscopy at age 12 every year.
Previous adenomatous polyp: Patient should have a colonoscopy every 3 to 5 years.
Previous history of colon cancer: Patient should colonoscopy at 1 year
after resection, then at 3 years, then every 5 years.

● Q/ 23y old female married, came for routine checkup, never had c-pap, when to have it?
a. Now
b. after 3y
Glory team August

c. after 5 y with HPV


Correct answer is A

N.B: Based on the age and HPV testing, but in general it is done every 3 years.
N.B: Pap smear if less than 21 years so no need.
N.B: Pap smear if between 21 and 29 years so pap test every 3 years.
N.B: Pap smear if between 30 and 65 years so pap test every 3 years or pap test + HPV test every 5
years. N.B: Pap smear if more than 65 years so no need if previous results are normal.
N.B: Pap smear in married women should be done after 3 years from marriage regardless of the age.

● Q/Vaccines given at 2 months


2- Month vaccinations: IPV -Dtap- Hepatitis B – Hib – PCV13 - Rota INCLUDEPICTURE
"C:\\var\\folders\\m7\\
hk4xywwx6tj_nkdq9bzj
tqsw0000gn\\T\\com.
microsoft.Word\\WebA
● Q/ Pt hasing blood transfusion, start to develop pain at the site + fever,
rchiveCopyPasteTempFi
Diagnosis?
les\\page63image2974
3616" \*
Sexual harassment of a young female, what do u think mostly
MERGEFORMAT
will do this?
A- Supervisor
B- Co workers
C- potential clients
Correct answer is A
● 40s came with dementia and abnormal behavior no FH?
A- Alzheimer
B- Vascular
C- Huntington
D- frontotemporal
Correct answer is D

● Peritonitis case after appendectomy what to do first!


A. AB ampicillin or third generation
B. exposing surgery
Correct answer is A
● Pregnant 24w with fever she does not seek medical attention, why
A- Afraid of doctors
B- Self-medication
C- Neglect
D- Cost treatment
Correct answer is C

● Rapidly growing child complains of tibial pain


A- Osgood
B- Stress fracture
C- Shine splint
Correct answer is A

● Q/ Pregnant with positive GTT once. wt to do?

A- Repeat
Glory team August

B- Do fasting test
C- start treatment
Correct answer is C

N.B : Screening (1-h 50-g OGTT) is performed on all pregnant women 24–28 weeks’ gestation If
screening value ≥140 mg/dL, then proceed to a definitive 3-h 100-g OGTT. If screening
value ≥200 mg/dL, and an FBS is ≥95 mg/dl, GDM is diagnosed and no further OGTT testing is
needed.
● Dm pt. had nephropathy what the urine finding?
A- Hyaline cast
B- RBC cast, with nephritis

Urine tests. Urine samples provide information about kidney function and whether too much protein in the
urine. High levels of a protein called microalbumin may indicate kidneys are being affected by disease.

● A baby try to feed a doll mile stones?


15 months

● V.cholera diarrhea characteristic?


LOW Osmolarity gap

● TTT of pleural effusion caused by malignancy.


o If asymptomatic > observe.
o If symptomatic, 500-1500 ml to relive symptoms
o If recurrent pleural effusion TTT is = chemical pleurodesis.

● Cancer of lung in smoker?


20-37 folds.

● Endocrinol abnormality?
Down: thyroid disease and DM.
Turner: Hypothyroidism and Sex hormone

● Tumor lysis syndrome, hyperkalemia uric acid increased. Wt else?


A- hypercalcemia
B- hypernatremia

● Mass found in the liver, suspecting HCC >> Wt the most risk for it?
a. HBV
b. aflatoxin
Correct answer is HBV
N.B: the most common risk factor for liver cancer is chronic (long-term) infection with hepatitis B virus (HBV)
or hepatitis C virus (HCV).
Reference: American Cancer Society

● Man sustained leg injury through car crash with exposed nerves and tissue, what would u do?
A- Debridement with vacuum
Glory team August

B- Debridement with second response gravting


C- Stabilize and leave wound exposed
Correct answer is B

N.B: Vacuum is not recommended in exposed vessels, tissue. So Best is flap. so flap not C/I in the scenario? No
it is the best. N.B: yes supposed to be flap but if there is arterial damage it requires immediate arterial repair.
Gustilo type IIIB Or C accordingly.

● Female had done cholecystectomy and she has fluid collection behind the gallbladder 12*13, has fever,
what would I do?
A- Laparoscopic drainage
B- Iv antibiotic
C- Per Cutaneous drainage
Correct answer is C
N.B: 12*13 so drainage then antibiotics.

● Post Tubal ligation and she came with spotting, what would u do?
A- Pregnancy testing
B- Vaginal us
C- Urine analysis
D- Laparoscopy
Correct answer is A
● Hemorrhoids class 4 what would u do?
INCLUDEPICTURE
A- Ligation
"C:\\var\\folders\\m7\\hk4x
B- Sclerotherapy
ywwx6tj_nkdq9bzjtqsw0000
C- Hemorrhoidectomy
gn\\T\\com.microsoft.Word\
Correct answer is C
\WebArchiveCopyPasteTemp
Files\\page2image30067760"
● Management of incomplete abortion, vitally stable?
\* MERGEFORMAT
A- D/c
B- Wait
C- IOL
Correct answer is B
N.B: If vitally stable so B conservative except if fever that mean septic so A.
N.B: Incomplete abortion is Vaginal bleeding and uterine cramping leading to cervical dilation, with
some, but not all, conception contents having been passed. Management: Emergency suction D&C if
bleeding is heavy to prevent further blood loss and anemia. Otherwise conservative management
awaiting a spontaneous completed abortion or induce contractions with misoprostol (Cytotec(R))
PGE1.

● Child started to have headache nausea fever??one week ago now came with
edema grade 2 in lower limps and preorbital she was very pale (palm,skin.mucous) brown colored
urine With Htn TACHYcardia FEVER TACHYpnea The most important diagnosis ull be concerned about
A- diagnosed SCA
B- recurrent UTI
C- abdominal trauma one week ago
D- Impetigo something one month ago
Correct answer is A
Glory team August

● Mild Hyperkalemia, No ECG changes what to give?

A- Dextrose plus insulin


B- Bicarb injectable
C- IVF normal saline
Correct answer is A

● Safe life in case of Placenta abruption with severe bleeding How to manage?
A- packed RBC
B- 2 large bolus FFP
C- Blood transfusion
D- Multi system and ICU
Correct answers are B and D

2 large bolus and blood transfusion if present, If 2 large bolus of FFP , choose rapid response N.B: If hgb
dropping or severe anemia so Packed RBCs, But initially bolus of fluids. Because fluids will not reverse
hemoglobin. If failed fluid resuscitation, then transfuse packed rbcs.

● Case of Trauma, Bilateral lung infiltration with Petechiae in Chest wall dx?

A- Hemothorax
B- Pneumothorax
C- Pleural effusion
d-Fat embolism syndrome
Correct answer is D

● Post-menopausal endometrial thickening 13 cm what is dx?


A- Endometriosis
B- Adenomyosis
C- Endometrial cancer
Correct answer is C

● 41 years old female come with heavy menstrual bleeding, thickened endometrium what is the tx?
A- Endometrial ablation
B- D&C
C- OCP
Correct answer is B
N.B: Most common form of AUB is heavy menstrual bleeding. So it is dysfunctional uterine bleeding.
INCLUDEPICTURE
Type of biopsy in AUB: Office endometrial biopsy D&C Hysteroscopy
"C:\\var\\folders\\m7\\hk
4xywwx6tj_nkdq9bzjtqsw0
● ectopic pregnancy what is highest risk factor?
A- Previous ectopic pregnancy (significant) 000gn\\T\\com.microsoft.
Word\\WebArchiveCopyP
B- Intra uterine device
C- PID asteTempFiles\\page7imag
e30064640" \*
Correct answer is A
● appendectomy in histopathology they found carcinoid tumor what to do asMERGEFORMAT
investigation?
A- Colonoscopy
B- Abdomen chest pelvis CT
Correct answer is B
Glory team August

N.B: size is important, Less than 2 cm so nothing, More than 2 cm so CT. But here they asked generally about
investigations so CT most likely.
INCLUDEPICTURE
"C:\\var\\folders\
● baby with fever and mild illness?
\m7\\hk4xywwx6t
A- Give vaccines
j_nkdq9bzjtqsw00
B- Give Abx
00gn\\T\\com.mic
C- Delay vaccine
rosoft.Word\\We
Correct answer is A
bArchiveCopyPast
● Wiskott-Aldrich syndrome triad is?
eTempFiles\\page
N.B: triad of :- Dermatitis eczema. Thrombocytopenia. Immunodeficiency recurrent Infections.
8image29715216"
\*
● Irritable bowel syndrome which is the best advice for diet?
MERGEFORMAT
A- Increase Fruits
B- Decease pepper
C- Decrease lentils
Correct answer is A

● what is the hormone responsible for neuro stress response?


B- Androgen
C- Adrenocorticotrophic hormone

● thalassemia case, beta hemoglobin is LOW what is the dx?


a. Beta thalassemia
b. Alpha thalassemia
Correct answer is A
● 18 months with frontal bossing, electrophoresis HgA2= 50, HgF=20 and Splenomagly, what is the dx?
A- SCD
B- Beta thalassemia
Correct answer is B INCLUDEPICTURE
"C:\\var\\folders\\m7\\hk4xywwx
● what is the hormone responsible for violence and aggressiveness? 6tj_nkdq9bzjtqsw0000gn\\T\\com
A- Increase SSRI .microsoft.Word\\WebArchiveCop
B- Decrease SSRI yPasteTempFiles\\page9image300
Correct answer is A 48880" \* MERGEFORMAT

N.B: Mostly A, but if there is testosterone or androgen it is best answer.


N.B: Ssri toxicity causes neuromuscular excitation, autonomic stimulation. N.B: Taking SSRIs give ppl more
motive to actions like in the first 2 weeks of taking SSRIs sucide risk increases

● Adrenal adenoma was diagnosed accidentally. Most common cause?


B- Subclinical Cushing INCLUDEPICTURE
C- B- Conns "C:\\var\\folders\\m7\\hk4
D- Cancer xywwx6tj_nkdq9bzjtqsw00
E- metastatic 00gn\\T\\com.microsoft.W
Correct answer is A ord\\WebArchiveCopyPast
● the 5 S of smoking and asking about re organized them eTempFiles\\page9image3
N.B: 5 A's": Ask, Advise, Assess, Assist, and Arrange. 0049920" \*
MERGEFORMAT
● Cardiac tamponed definitive Dx ?
A- Echo
B- B- CXR
C- CT
D- ECG
Glory team August

Correct answer is A

N.B: Transesophageal echo is the most accurate, trans thoracic is the initial.

● Cholecystitis, U/S = Fluid collection around GB , Fever 38 management ?


A- Lap chole
B- open chole
C- Antibiotic.
D- ERCP
Correct answer is C then A.

● DNR and Family scenario. The Child with Congenital anomalies decided as a case of DNR, family is
refusing to label/sign their child as DNR. What to do?
A- Ignore
B- Respect
Correct answer is A

N.B: once 3 consultants made the decision of DNR, It is DNR even if family refuse you ignore.

● Mediolateral episiotomy why better than medial?


To avoid 4th degree pineal tear.

● Episiotomy reach the mucosal of rectum what is the degree of tear ?


A- 3rd
B- 2nd
C- 4th
Correct answer is C

N.B: Skin > 1 Muscle> 2 Sphincter > 3 Rectum > 4

● child girl pull her hair during stress in scool ttt?


Olanzapine

N.B: old similar question 2018 :-


young girl who become very stressed during exams and she pull her hair till a patches of alopecia appear
how to ttt:

a) Olanzepin
b) Fluoxetine
C)Psycotherapy ✅

N.B: trichotillomania responds best with psychotherapy/habit reversal therapy but pharmacotherapy
includes olanzapine. SSRIs may not be as effective, according to DynaMed.

● Neonate seizure TTT ?

A- No phenytoin
B- lorazepam
Glory team August

C- Phenobarbital
Correct answer is C

N.B: Initial treatment with phenobarbital should be considered. If seizures persist, phenytoin should be added.
Persistent seizures may require the use of an intravenous benzodiazepine, such diazepam, lorazepam.
N.B: Neonates > anti-epileptic superior to benzo. phenob is 1st tx in neonatal seizure.

● Cases of intusspssion what confirm diagnosis? All the choices were clinical

A- Red currant jelly stool


B- sausage mass on palpation
Correct answer is B

N.B: Suasage mass in the RUQ indicate invagination of the proximal to the distal.
N.B: The hallmark physical findings in intussusception are right hypochondrium sausage-shaped mass and
emptiness in the right lower quadrant* ( *Dance sign* ).

● Cases of intusspssion what management?


A- enema
B- US
C- CT
D- Colonoscopy
Correct answer is A
● Case of subclinical thyroiditis, what are the expected lab findings?

N.B: TSH high, T3 and T4 normal = subclinical hypothyroidism. N.B: TSH low, T3 and T4 normal = subclinical
hyperthyroidism.

● Child with infection i didn’t remember “his brother is immunodeficiency what youʼr action to protect
the brother from infection? There is no isolation or avoid contact in choices
A- Vaccines
B- Immunoglobulin
C- Nothing
Correct answer is B
N.B: if varicella so B. If he means tonsillitis answer is C.
N.B: generally immunodeficiency patient to be protected we give them Vaccines but not live ones, but
here Pt exposed already so vaccine will take time to made antibodies so best is immunoglobulin.
● Which drug cause SLE?
A- Chlorpromazine
N.B: there is a difference between drug induced lupus and SLE flre medications.
N.B: sulfa drug for sle flare.
N.B: in drug induced lupus ; Most common hydralazine (rate roughly 20%), procainamide (rate roughly
20%, 5-8% if taken for 1 y), quinidine, isoniazid, and minocycline, Chlopromazine.
N.B: Most specific lab for SLE is anti DsDNA then antismith. But anti smith is best for disease activity.
N.B: Most specific lab for drug-induced SLE is anti-histone.
Glory team August

● Case of pt on anti TB drug develop increase in liver enzymes? What to do ?


A- Stop pyrazinamide and re-evaluate LFT after 1 wk
B- Stop all anti TB drugs and re-evaluate LFT after 1 wk
C- Stop izonide and rifampicin and re-evaluate LFT after 1 wk
Correct answer is B

N.B: if elevated LFT Triple the upper normal limit or doube with jaundice so stop all anti TB.

● 24 years old girl came to you with a painless mobile breast mass that does not change with menstrual
cycle. It started increasing since 3 months. Now itʼs 3 cm in size. What is the most likely cause?
A- Fibroadenoma
B- Fat tissue
C- Cyst
D- Phyllodes
Correct answer is A

● Old male age pt have weight loss, difficult micturition and pain radiated to back.
Prostate Ca

N.B: If the pt is not old with no weight loss We can consider bladder stones or prostatitis.

● Old male age pt have weight loss, difficult micturition and pain radiated to back. Confirmatory test
A- Transrectal biopsy
B- B- PSA
C- C- CT
Correct answer is A
● diagnosis and treatment?

N.B: Perforated viscous (most common duodenal), CXR shows air under right hemidiaphragm. Treatment by
Resuscitate then OR to do Graham omental patch.
N.B: Pneumoperitoneum most probably due to perforated viscus immediate exploratory laparotomy.

● Management of a patient with gastrointestinal stromal tumor of the stomach?


A- Gastrectomy
B- Wide local excision
C- Radiotherapy
Correct answer is B

N.B: Treatment of Gastrointestinal stromal tumor (GIST) *Small GIST (< 2 cm)*
Stomach: to be observed, endoscopic removal possible, Other localization: resection
*Large GIST (> 2 cm)* : surgical excision ± neoadjuvant and/or adjuvant treatment with imatinib
Palliative treatment using imatinib "TKI", if the tumor is not resectable and has developed metastases.

● Risk of MI in a smoker compared to nonsmoker?

A-2 times
B- 3 mes
C- 4 times
D- 5 me
Correct answer is A
Glory team August

● Female presenting with retraction of the nipple and slits in the nipple, Diagnosis?

A- Duct ectasia
B- Carcinoma of the breast
Correct answer is A
N.B: nipple retraction seen in ectasia as well + in duct ectasia there is slit like nipple retraction.
Both same presentation but Duct ectasia is more common than carcinoma.

● Prophylaxis of DVT in orthopedic case patient?


A- Heparin
B- Warfarin
C- Enxoparine
D- bixaban
Correct answer is C
● Long scenario: Radiologist asked by the treating team to do Ct guided Fluid aspiration, Who should
get the informed consent?

A- Radiologist
B- The resident from the treating team
C- The nurse
Correct answer is B

‫ﺾ ﻳﻮﻗﻊ‬óñ‫ اﻟﻤ‬öõ‫ﺨ‬ó‫ﺾ و‬óñ‫ﺣﻪ ﻟﻠﻤ‬ëíê‫ﺖ و‬ç‫ﺴ‬ã‫ﻮ‬â‫ﻜﺘﺐ اﻟ‬ÉÇ 


Å Ä ‫ ﻗﺼﺪﻫﻢ ﻣ‬-: ‫ﻣﻼﺣﻈﺔ‬.

● screening of the uveitis in a patient with JIA (They didn’t mention the diagnosis but typical scenario)
with ANA negative?
A- Every 3 months
B- Every 6 months
C- Annually
D- every 2 years
Correct answer is B

N.B: +ve ANA > 3 months. N.B: -ve > 6 months.

● Patient with a history of acute pancreatitis, Found in CT to have collection in the lesser sac (He is
feverish = abscess) What is the route of drainage?

A- Endoscopic
B- Open
C- Percutanous
D- Laparoscopic
Correct answer is C
● patient who have undergone laparoscopic cholecystectomy developed subhepatic abscess, How to
drain the abscess?
A- Laparoscopic
B- Ct guided Percutaneous drainage
Correct answer is B
● Married women went to the police complaining about her husband Belief of gender variations
A- Cultural
Glory team August

B- socio economic background


Correct answer is A
● After smoking cessation after how long there will significant decrease risk of coronary heart disease?
A- 1 year
B- 2 years
C- 3 years
D- 4 years
Correct answer is A
● newborn presented to you in ER with Acute meningitis you find that he need Lumpur puncture, and
his parents Rufus? What to do?

● Pt with inguinal hernia and chronic cough you prescribed medication 6 month age and he did take it
and come now he said if you don't do the surgery I will Complaint against you.. What to do?
A- Do surgery
‫ﻠﻢ اﻻدارە‬É‫ﺗ‬

● Brownish vaginal discharge closed cervical os fundal ht is not appropriate to gestational age. Dx?

A- Incomplete
B- Threatened
C- Complete
D- Missed
Correct answer is D
● 12 months healthy boy at well care for vaccination which vaccines to be given?

A-MMR
N.B: PCV, opv, Mmr, Mcv all true.

● Baby with recurrent infections his brother died three years ago due to septic shock, contraindicated
vaccine?

A- MMR
B- Varicella
C- IPV
Correct answer is B

● Boerhaave syndrome triad is?

● Pregnant at 22 weeks gestational age oral glucose challenge test after one hr high after 2 hrs high
after three hrs high.. What is next?

A- Repeat same test


B- HgA1c
C- Fasting blood glucose
D- Random blood glucose
Correct answer is A
● 14 years girl obese came with her mother she has had regular with pain her mother noticed that there
was significant change of her daughter mood after menarche during menstruation she had acne during
examination she was incomfortable?

A- Mood
B- Behavior
Correct answer is B
Glory team August

N.B: Incomfortable, Agitation, Aggressive Inappropriate, All Behavioral changes

● 13 years with regular cycle complaints of pain during menstruation how to manage?
A- Progesterone pills
B- NASID
C- COCP
Correct answer is B
● Female with symptoms and sign of PCO metformin was given what is the role of action metformin on
this lady?
A- Reduce insulin resistance
B- Anti adrenergic
C- Reduce insulin sensitivity
Correct answer is A
● Pt came to ER with pain after long trip (5hours) what next?
A- Angio
B- Hepranization
C- fluid supplementation
D- US doupler
Correct answer is D
● 19 yo girl sexually active present to you with lower abdominal pain, vaginal dischage (perulent) no
fever “typical scenario in PID Ceftriaxone was given, she came after days with no response
Whatʼs do you think the organism
A- Chlymedia
B- Genorrhea
C- Adenovirus
D- Herpes simplex virus
Correct answer is A
● Girl presented with abdominal pain, Pregnancy test -ve , Whatʼs the dx ? (Q asked about the most
common cause of abdominal pain here)
A- Pregnancy
B- ruptured ovarian cyst
C- Ovarian cyst
D- appendicitis
Correct answer is D
● Child fall from hight presented to you in ER crying, Bleeding from the ear, tympanic membrane
bulging and bleed , imaging confirmed basal skull fracture, The nerve which pass through foramen
ovalea injured whatʼs the function which will be affected ?

● Fibroid asymptomatic 7 cm, What is the management?


A- Follow up a er 1 year with US
B- Follow up a er 6 months with US and CBC
C- Myomectomy
Correct answer is C
● Best bathing practices for atopic dermatitis (Eczema)?
Use of non-soap products

● Best screening test for strabismus?


A- Corneal light reflex
B- Cover test
C- Red reflex
Correct answer is B
● Milestone: 4 year old child what can he do ?
Glory team August

A- Know 50-100 words


B- Can tell future sentences
C- Can count up to 4 and Tell story
Correct answer is C

● Couple want a Surrogacy, the doctor knows itʼs not allowed in this hospital that he worked in but he
doesnʼt know about itʼs law in the KSA in general

A- Tell them itʼs maybe not allowed in KSA


B- give them another appointment while he collects information
Correct answer is B
● Elderly pt confused with his son, his son refused the ventilator, what to do?

A- accept his wish


B- ask the patient
C- ventilate the pt
Correct answer is B
● Pt elderly with sever pain, family wants to increase the dose of morphin, you know increasing the dose
will cause a side effects for the patient. you will accept according to ?

A- principle of informed choice


B- principal of totality
c-principle of something
Correct answer is A
● 41 females with dysfunctional uterine bleeding, first step in mx?

A- OCP
B-D/c
C- Hysterectomy
Correct answer is B
N.B: We should obtain endometrial Biopsy in any women age > 40 with abnormal uterine bleeding, why?
uterine bleeding in postmenopausal women is due to malignancy until proven otherwise. Give OCP if over 40
years (with normal endometrial biopsy).
N.B: abnormal uterine bleeding in women less than 40 years so start therapy by COCP or medroxyprogestine.

● Pt post surgery came with fever abdominal pain in examination FB found inside abdomen and infected ,
dr take pt to surgery again , after surgery what would you do ?
™ ¶ ¶
‫ﻖ ﻣﻮ ﻧﻔﺲ اﻟﺠﺮاح‬óñ‫¨ اﻟﻔ‬õ Å ‫اﻟﺨﻄﺎ ﺧﻄﺎ ﺷﺨﺺ اﺧﺮ‬
Glory team August

A- report to consultant
inform patient and apologize

● during cholecystectomy you injured CBD after surgery pt stable and doing well ,
what to done?
A- Inform pt
B- Inform family
C- Donʼt tell
D- write report to consultant committee
Correct answer is A
● Child did accident brought to hospital, he can't speak your same language, the translator of the child
didn't arrive hospital yet. What yo do?

A- Prepare preanasthesia
B- Call nurse to stay with him until translator come
C- put stethoscope on his chest then give him to listen
D- try to calm him till anxiety disappears
Correct answer is D

● 67 years old male, known COPD, come with severe respiratory distress, ABG: pH 7.2, O2 saturation
85, PO2 7.2, PCO2 40, bicarbonate 22. TTT?
A- CPAP
B- BIPAP
C- Mechanical ventilation
D- Observation

INCLUDEPICTURE
● all true except last one A. "C:\\var\\folders\\m7\\hk4xywwx6tj_n
kdq9bzjtqsw0000gn\\T\\com.microsoft.
Word\\WebArchiveCopyPasteTempFile
s\\page25image29645312" \*
MERGEFORMAT

● Bleed +Itchy+ offensive smell most likely?


A- foreign body
B- Sexual abuse
Correct answer is A
● Bleed+ canʼt hold urine or stool most likely?
A- sexual abuse
B- foreign body
Correct answer is A
● Which drug cannot be used with Peptic ulcer?
A- Aspirin

● Picture of single cervical polyp with history of minimal bleeding what is correct?
A- Excision in clinic
B- Follow up after few months
C- Admission and more investigation
Correct answer is A
● Pt pulseless what to do ?
A- Ct angio
Glory team August

B- Us
C- vascular ultrasonograph
Correct answer is B
● Psoriasis in vagina what is correct ?
A- Need biopsy to confirmation
B- Clinical diagnosis is enough
Correct answer is A
● Motion sickness case (patient usually vomit when she is on flight or high-altitude places what is the
treatment?
A- diphenhydramine
B- scopolamine
Correct answer is B
● Obese girl with pcos not comparative with examinations ?
A- Mood
B- Anxiety
C- Depression
Should be beavioral

● old male age pt have difficult micturition radiated to back , Diagnosis ?


A- BPH
B- Prostatic cancer
Correct answer is B
● Gestational HTN BP= 150/110 , what to give ?
a. Alpha methyldopa
b. Nifedipine
c. Verapamil
d. Hydralazine
Correct answer is A
● Case of Peritoneal Hematoma management ?
A- Observe
B- Evacuations
Correct answer is A
● female positive family Hx of breast cancer, Mammogram shows atypical dysphasia of duct What to
do?
A- Wide local excisions
B- Simple mastectomy
C- Chemo therapy
D- Radio therapy
Correct answer is B
● anti HTN that causes angioedema?
Ramipril ACEi

● Cushing case ACTH high cortisone high after suppressing what investigation?
A- CT chest
B- B- Brain CT
Correct answer is A
● post hysterectomy type of hormonal replacment
A. estrogen alone
B. combined Estrogen and progesterone
C. progestin only
Correct answer is A
● Child with fever conginctivitis coryze cough Intercostal resection low O2 sat tachycardic Tachypenic
what is the optimal initial step
A- intubation
Glory team August

B- O2 therapy
Correct answer is B
● Anal fissures with skin tag not respond to drug , want procrdure?
A- Lateral Internal sphernctomy
B- External sphrenctomy
C- Anal cuurtage with remove tag
Correct answer is C
● Palpable gallbladder, with jundice , didnʼt mention painful gallbladder, ask about Q:>
A- GB cancer
B- Panceratic cancer
C- Cholthisis
Correct answer is B
● Neonatal hypoglacemia>
A- give glocuse in central line

● Acute pancreatitis with amyalse is high ask investigatoion>


A- CT abdomen
B- Amalyse urine
Correct answer is A
● jundice with us show stone inGB and in CBD , tx>
A- Laparoscopic cholecystectomy
B- ERCP
Correct answer is B
● Q about renal impariment with arterovenos anastomoses or shunt
A- arterovenous
B- brachicephalic
C- radibrachial
Correct answer is B

● 2 Q about Phyldous tumor treatment, one benign , another malignant ttt?


A- mastectomy
B- wide local excion
Correct answer is B
● Couple has infertility. Upon investigation you noticed bilateral tubal ligation. Who will you tell?
A. Couple
B. The wife
Correct answer is A
● A 15 y/o child presented to ED with S&S of Acute meningitis case and you needed to do LP
What is next:

A. Take consent from parents


B. Take consent from child only
C. No need to take consent as this case is emergency
Correct answer is A

● Menopause wants HRT and you see it is not suitable for her. What to do?
Refuse

● Elderly man with end stage Ca lung, intensivist label do not resuscitate, surgeon plan a debulk
surgery that may have little benefit. Ask about?

A. Go ahead with surgery with another anesthetist


Glory team August

B.Take high risk consent


C. Involve the family in decision
D. Involve hospital ethics committee
Correct answer is B

● Mother with IUFD counselled prior to delivery of liveless baby, postdelivery mother insisted to
stillborn should be admitted in NICU. Asked about the action?

A. Denial
B. Acting out
C. Two other options
D. Neglect
Correct answer is A

● Surrogate?

A- Illegal in ksa
B- ask them to viste you after 2 weeks to read about it
Correct answer is A
● To have an excellent Screening strategy. You need it to be
A. Sensitive
B. Cheap
Correct answer is A
● A child went to a party he developed their lip swelling, difficulty breathing( allergic reaction) his mom
wasnʼt their. Which part of the history that is most important to diagnose the child?

A. Allergy history
B. Family history
C. Nutrition history
Correct answer is A

A man has atrophied buttocks(LONG SCENARIO). Which of the following is the recommended treatment?

A- vasodilator
B- Axiall femoral bypass
C- Ailial femoral bypass
D- Aortoiliac bypass

Correct answer is D

● Female 35 years smoker for 3 years. Have 2nd degree relative with DM and infertile what is the most
significant risk factor for endometrial cancer?
A- Dm in 2nd degree relative
B- SUB Infertile
C- Her age
D- Smoker
Correct answer is B
If no baby u can say subinfertile
Glory team August

● A man has atrophied buttocks (LONG SCENARIO). Which of the following is the recommended
treatment?
A. vasodilator
B. Axiall femoral bypass
C. Ailial femoral bypass
Aorto iliac bypass

● A female developed numbness and tingling in the middle thigh after delivery?
Which nerve effected?

A. obturator
B. Femoral, lateral side.
C. Sciatic
Correct answer is A

● A patient known case of atrial fib. He is on warfarin for 6 month ago. Today presented to your clinic
for regular follow up. What you will do?

A. continue warfarin
B. Stop warfarin and give heparin
Correct answer is A

● A man developed a painless ulcer on his penis after sexual intercourse with extramarital women
What do you suspect he has?
A. Syphilis
B. Herpes simplex
Correct answer is A
● Most common affected organ in Mumps is
A. Testes
B. Parotid
Correct answer is B
● Case of DKA. The child was very thin with below 25 percentiles What is the MOST initial treatment
A. Give fluid
B. Correct electrolytes
C.Give bicarbonate
D. Give insulin
Correct answer is A
● Case of pyloric stenosis What is the initial treatment
A. Fluid
B. Do US
Correct answer is A

● A child regarding his immunization. Mother told his brother died after taking a vaccine What is your
action?
A. give BCG
B. Give IVIG
C. Don’t give
D. Wait for immunization team to evaluate the child
Correct answer is B
● A 10 weeks pregnant woman known case of gallstone present with recurrent resent symptoms with
USS of feature of multiple stones like that, ask about?
Glory team August

A immediate laparoscopic cholecystectomy


B. delay till 2nd trimester
C- delay till 3rd trimester
D- can not remember
Correct answer is B
N.B: Balter lap chole at 2nd trimester.

● A patient had this which is GROUND GLASS appearance It is associated with:


A- Infertility
B- Infection
Correct answer is A

● Barrett’s low-grade dysplasia Tx.

A. PPI, and scope after 6-12 months.


B. Surgical resection
Cimetidine
Correct answer is A
● Early puberty sign in male?
A- Testicular enlargement
B- B- pubic hair
C- C- breast enlargement
Correct answer is A
● years with 2ndry dysmenorrhea and heavy menses, on pv u found symmetrical uterine enlargement.
Dx?

A- endometritis
B- adenomyosis
C- endometrial cancer
Correct answer is B

● Baby has HTN, hematuria, proteinuria, and pale, what is the most important thing to ask in history?

A-Hx of SCD
B- Impetigo history attack
Correct answer is B
N.B: Case of nephritic syndrom they have HTN +HU +proteinure (mild) and it occurs usually after skin infection
like impetigo.
N.B: Nephrotic syndrome usually causes anemia due to loss of transferrin and erythropoietin. But this is not
nephrotic.
N.B: He is pallor you should exclude history of SCD first. Although Pallor goes more with SCD since it is chronic
But HTN and hematuria goes more with PSGN.
N.B: PSGN due to impetigo will not cause pallor, SCD can cause both pallor and renal.
N.B: SCA >>> FSGS so pallor and nephropathy.
N.B: HTN + HU +protein urea = PSGN.
N.B: *Some of the common findings in patients with sickle cell nephropathy include:*

-Self-limiting microhematuria, which can be painless to visible, painful gross hematuria, requiring
transfusions.
Microalbuminuria and proteinuria which increases with age. -Nephrotic syndrome in up to 4% of patients.
Glory team August

-Renal infarction presenting with flank or abdominal pain, nausea, vomiting, and fevers.
-Hyperkalemia and mild hyperchloremic metabolic acidosis (type IV renal tubular acidosis).
-Urinary tract infections and pyelonephritis. -Nephrogenic diabetes insipidus
-Acute kidney injury.
-Progressive chronic kidney disease.
If in the Q gives u lab about SCA choose it

● A patient developed shoulder pain after cholecystectomy What is the next step
A. CT
B. Ultrasound of abdomen
It should be Reassurance, Pneumoperitoneum is common post laparoscopic surgery.
If you have pain it is either from it or from intubation...
Reassurance.. If symptoms persists after adequate analgesia py become septic. Then US or CT

● Bilateral inguinal hernia, man act in military, young age, ttt?


a. Open
b. lap with mesh
c. observation
Correct answer is C

● What is the auscultatory sign you will hear in croup:

A. Stridor
B. Wheezing
Correct answer is A

● Pt healthy then started to have non pitting edema in lower limb. Investigations?
women pt. Middle aged no hx of DVT or hx of arterial insufficiency or any peripheral problems, she is not
smoker Not diabetic and not hypertension no hx of surgery or trauma, three months before, one of
her legs developed increases in size, nonpitting edema, blue color, hard skin, what you will do for
her?

A- venous duple
B- arterial Doppler
C- lymphoscintigraphy
D- CT angio
Correct answer is C
Glory team August

● 70-year-old what medication of osteoporosis approved to hypercalcemia induced by cancer h was


having pain
Labs: Ca: 3.5
A- Alendronate
B- Bisphosphonate
C- teripratide
D- Furosemide
Pamidronate , donusomb , teripratide
Correct answer is C
● Treatment of absence seizure
Ethosuximide
In exam they put something similar to Ethosuximide, I think it is” Ethuscideʼʼ

● Pic of menstrual cycle peak of ovulation?


Peak 24-36 hours after Lh
N.B: Ovulation normally occurs 24 to 36 hours after the luteinizing hormone (LH) surge.

● You called vascular surgery to control perfuse bleeding developed while colectomy operation, what
to do?
A- Packing
B- Clamp of ileocecal
C- clamp infra celiac artery D: Clamp supra celiac artery
Correct answer is A
● Pt presented with perianal bulging, pain and there is anal discharge. Dx?

A- Anorectal abscess
B- Hemorrhoids
C- Anal fissures
D- Anal fistula
Correct answer is A
N.B: A and B are painful, A will be fever and pain and bulging or swelling and tenderness and
discharge, C anal pain during defecation, D discharge.

● pt. pregnant with fundal height more the GA and hemoptysis DDx
A- Complete mole
B- Choriocarcinoma
Correct answer is B
● Drug approved by FDA for tx of Cancer hypercalcemia?
A- Denosumab
Note: Other ttt for hypercalcemia in malignancy: normal saline, calcitonin, bisphosphonate

● FDA approved bisphosphonate used for hypercalcemia related to malignancy :


a. alendronate
b. Etidronate
c. Pamidronate
Correct answer is C

This Question from file called Dr. Doaa file,


Glory team August

● How to confirm if Endotracheal Tube Placement


A. CO2 tidal volume
B. Auscultation
C. Chest movement
Correct answer is A (AKA capnography)
● Best time to examine red reflex?
A- Pre school
B- In age of 3
RIGHT ANSWER SHOULLD BE >> At birth or after 6 weeks.
When to test the red reflex for retinoblastoma It is important to test the red reflex after birth, at the
age of six weeks, during routine consultations, or when parents are concerned about the
child's vision or the appearance of her or his eyes.

● A female did midline episiotomy. She presents now with expanding mass at the site of episiotomy. It
measures 15x3 cm now. How would you manage it?
A. Packing
B. Reassure
C. Aspiration
D. Surgical drain
Correct answer is D

● A female patient had DVT and now presents with dyspnea and tachycardia. Her blood pressure is
normal. She was on heparin for her DVT. What is the most appropriate management?
A. Continue same treatment, he is stable, and close monitring.
B. Thrombolysis, IF HYPOTENSIVE
C. Thrombectomy
Correct answer is A
● 75 year old presents with episodes of syncope upon exertion. During examination, you notices a mid
systolic ejection murmur louder on the right side radiating to carotids. Which of the following is true
about the diagnosis?
A. His murmur is augmented in standing position
B. Having signs of CHF is a poor prognostic factor
C. His symptoms are caused by RBBB
Correct answer is B
● A patient had an inferior MI after few days presents with loud systolic murmur and pulmonary
edema. What is the most likely cause?
A. Papillary muscle rupture
B. Cardiogenic shock
C. Heart failure
Correct answer is A
● A patient had abdominal aortic aneurysm repair before. He presents now with hypotension and
oligurea and he is tachycardiac. You gave him IV fluids with no significant improvement. Which of the
following is the cause of his shock?
A. Anaphylactic
B. Cardiogenic
C. Hemorrhagic
D. Septic
Correct answer is C
● A patient was using heparin and he presents now with low platelets. What is the most appropriate
management?
A. Stop heparin and switch to an antiplatelet (forgot the name only one mentioned with dose)
B. Use fondaparinux
C. Use UFH
D. Switch to warfarin
Glory team August

Drug of choice is argatroban (other bivuridine or lepirudin) !!!


Note = we dont stop heparin in case of HIT unless platlet below 50000
● Which of the following has the worst prognostic indication?
A. Bilirubin
B. Albumin
C. AST
Correct answer is B
● What is the visual defect seen in patient with large nonfunctioning pituitary adenoma?
A. Homonymous hemianopia
B. Bitemporal hemianopia
Correct answer is B
● A smoker is taking carbamazepine for his seizures. Which of the following smoking cessation methods
is contraindicated?
A. Nicotine patch
B. Bupropion
C. Virnicline
D. E cigarettes
Correct answer is B

Bupropion Ci in ashma and epilepsy and HTN.

● A patient has seizures and feels generalized weakness. Which of the following would be seen in his lab
results?
A. Hyperkalemia
B. Hypokalemia
C. Hyperammonemia
D. Hypomagnesemia
Correct answer is D
● Which of the following is the most important risk factor for developing DM?
A. Abdominal obesity
B. BMI
Correct answer is A
● Male had total thyroidectomy, 5 hours later he developed shortness of breath and neck swelling.
What is the management?
- Open the wound
(To evacuate the hematoma)

● Pregnant lady in her second trimester presents with signs of hyperthyroidism. She is taking
carbimazole 10 mg and her symptoms are poorly controlled. In her previous pregnancy her baby was
born with hypothyroidism. Her labs TSH low T high. What is the most appropriate management?
A. Increase carbimazole dose to 20
B. Switch to PTU
C. Do radioiodine ablation
D. Refer her for surgical resection
Correct answer is B
● Which of the following is a character of screening tests?
Glory team August

A. Cheap and easy to do


B. Very specific
C. Used for diseases we don't understand
D. Used for rare diseases
Correct answer is A

● A physician is attending a patient and obtaining history from him. Meanwhile he is having trouble
with his patient in which he talks about why he thinks he got the disease and the physician wants to
know more about his symptoms and diagnosis. What is the best approach?
A. Use open ended question, then
B. Let the patient complete his concern, first
C. Politly interrupt and ask direct questions
D. Let another physician see this patient

● Patent after he took drug for the elevated TG. He developed hot flush Which drug was he taking?
A. Clofibrate
B. Statin
C. niacin

Correct answer is C

● Thoracentesis site

A- 6th intercostal space mid axillary


B- 9TH INTERCOSAL SPACE MID AXILLARy
Correct answer is B
● Which cancer is associated with fatigue and decrease in weight in just 2 weeks
A- Leukemia
B- Lymphoma ,
C- Neuroblastoma
Correct answer is A
if b symptoms choose lymphoma.
● How to know if the treatment worked in ectopic pregnancy?
A- weekly bHCG

● Post total thyroidectomy due to graves disease pt. Developed hypocalcemia not resolved with
multiple tries for replacement, wt to do
A-magnesium level
B-supply him with parathyroid
Correct answer is A
● 18y old female didn’t develop any signs of puberty, pt short with short neck and low hair line... her
parents also short .. >> mother asking wt the sempliest test can b done (( all choices were hormones >>

A-TSH &T4
B-FSH &LH
C-estrogen level

● A female patient had a swelling near hear mandible. Lymph node biopsy showd normal follicular thyroid
cells. How would you manage her?
A. Reassurance
B. Radioiodine ablation
C. Refer for surgical consultation
Correct answer is C
Glory team August

● A patient had vitiligo peripheral numbness after gastric resection. His labs show macrocytic anemia with
low B12. How would you manage him?
A. Folate
B. Iron
C. Oral B12
D. Parenteral B12
Correct answer is D
● A female patient presents with achalasia. What is the most appropriate management?
A. Pneumatic dilatation
B. Botox injection
C. Fundoplication, if GERD.
D. Myotomy
Correct answer is A

● A female patient had episodes of palpitations that last 10-15 minutes and resolve on their own. She
denies syncope and is otherwise normal. ECG report: normal heart rate and sinus rhythm What is your
next step?
A. Exercise ECG, if accompanied with pain.
B. Holter monitor
C. Reassure
Correct answer is C
● Young patient presents with fever and chest pain. He had upper respiratory tract infection a week ago.
Upon examination he is short of breath, feverish, with cough. He has a systolic murmur. CXR shows
interstitial infiltrates.

Lab: Low Hb, High reticulocytes.What is the diagnosis?

A. Sickle disease
B. Steptococcus pneumonia
C. Sequestration crisis
Correct answer is A
● A patient had subcutaneous nodules, fever and arthritis. Which of the following confirms the diagnosis?

A. High CRP
B. Positive culture
C. High ESR
D. Positive antistreptolysin titer
Correct answer is B
● Baby of a diabetic mother has absent Moro reflex. What is the most likely diagnosis?
Erb's palsy

● A female patient complains of urinary dribbling, dyspareunia, dysuria. What is the most likely diagnosis?
A. Overflow incontinence
B. Urethral diverticulum
C. Stress incontinence
D. Urge incontinence
Correct answer is C
● 28 year old pregnant female came for GDM screening. Her BMI is 31. You did glucose tolerance test and
the result was:
Fasting: slightly elevated
1 Hour: normal
2 hours: slightly elevated
3 hours: normal
(all values were given, and they didn't mention if it was 75g)
Glory team August

What is the appropriate next step?


A. Repeat glucose tolerance test
B. Recommend weight reduction > failed diet > insulin.
C. Measure A1c
D. Do fasting glucose
Correct answer is B

● 3 Female with nodule in triangle 3 or 5 of the face, On


FNAC it showed follicular thyroid cell- out side the
LN?
A.ectopic thyroid
B. Cancer
C. metastasis of thyroid cancer
Correct answer is A

● same above question but it was nodule in inside


lymph node of neck, what is the dx ?
A.ectopic thyroid
B. Cancer
C. metastasis of thyroid cancer
Correct answer is C

● A female patient had a swelling near hear mandible. Lymph node biopsy showd normal follicular thyroid
cells. What is the diagnosis?
A. Ectopic thyroid
B. Aberrant thyroid
C. Thyroid cancer
Correct answer is B
● A male patient had dysphagia. His ultrasound showed a central mass. What is the diagnosis?
A Goiter
B Thymoma
C Lymphoma
Correct answer is A
● Which growth chart do you use in down syndrome patients?
A. Normal growth chart
B. Down syndrome growth chart
Correct answer is B
● 35 year old male had an object that fell on his leg. After that he had swelling and pain, his distal pulses
were weak. What is your next step in management?
A. CT angiography
B. Venous dopler
C. MR angiography
D. Measurement of compatment pressure
Correct answer is D
● 35 year old male had an object that fell on his leg. After that he had swelling and pain, his distal pulses
were weak. He had a comminuted fracture. Measurement of posterior compartment pressure was 35
mmHg. What is your management?
A. External fixation
B. Internal fixation
C. External fixation and 4 compartment fasciotomy
Correct answer is C
Glory team August

● A patient was diagnosed with generalized anxiety disorder and has difficulty sleeping. Which of the
following is used for management? (No SSRI in options)
A. Alprazolam
B. Bupropion
Correct answer is A

● Patient came after finishing 2 weeks of H.pylori treatment and he is eager to repeat breath test. What is
the earliest time the test can be repeated?
A 4 weeks
B 2 weeks
C 1 week
D now
Correct answer is A
● Patient came from India last night, he had watery diarrhea without mucous or blood. What is the most
likely organism? Traveler diharhhea
A Rota , if Noro will be more accurate depend on the age group
B E-coli, if fever and abdominal pain mentioned in the Q can come without bloody stool, called
enterotoxigenic.
C Bacillus cereus
Enterotoxigenic E. coli (ETEC) produces a toxin that acts on the intestinal lining, and is the most common
cause of traveler's diarrhea.

● Patient came from Egypt, he had watery diarrhea without mucous or blood. What is the most likely
organism?
A Giardia
B Salamonella
C Shigella
D Amoeba
Correct answer is A

female pt came with post partum hemorrhage due to uterine atony , what is the best to give her :
1- oxytocin, uterine atomy oxytocine
2- Ergometrine
3- Carboprost
4- misiprostol
Correct answer is 1

4-female pt came with post partum hemorrhage , what is the best to give her :
(( same above Q but without atony ))
1- oxytocin
2- Ergometrine
3- Carboprost
4- misiprostol
Correct answer is 1

● Bilateral sever heel pain after trauma GCS 15 what next step?
Check pulse

● Baby who spits after feeds and between feedning and is developing normally, phyiscal exam growth
parameters normal /
a. physiological gerd
b. reassure
c. Hypertrophic lower esophageal sphincter
d. Hypertrophic pyloric sphincter
Glory team August

e. Lower esophageal sphincter pressure


Correct answer is E

Old patient with gluteal atrophy with confirmed aorto-ilial stenosis, What will you do?

A- brachio-femoral bypass

B- Aorto0femoral bypass

C- heparin?

Correct answer is B

E- What would indicate further investigation:


A. Failure to thrive
B. Spitting after feeding
C. Spitting during sleep
D. Forgot but didn't make sense
Correct answer is A

● PCOS twice
Once did not want to get pregnant: OCPs
One wanted to get pregnant: weight loss, metformin and clomiphene citrate

Read about:

● Renal agenesis
● Urethta stone
● pulmonary hypertension
● Mitral stenosis
● Exogenously inhaled NO > decrease pulmonary vascular resistance and improve lung blood flow.
● You need to be able to differentiate between Spherocytosis and Autoimmune hemolytic anemia: You
have to deferentiate by lab or family history in Spherocytosis.
Hereditary spherocytosis and autoimmune hemolytic anemia are characterized by having only spherocytes.
● Hypertrophic cardiomyopathy (HCM).
Hypertrophic cardiomyopathy, although not usually fatal, is the most common cause of heart-related sudden
death in people under 30. It's the most common identifiable Ncause of sudden death in athletes.
● perianal disease pyoderma gangrenous
● Post trauma, hypotensive, increase JVP, distant heart sounds >> Q. Dx / investigation Echo
● Q/ (pic of X-ray) with upper lobe cavitation with air-fluid lever >> Q. Tx.
● Risk for hep c after needle stick? 0.03
● Geritric patient with gram - ve cocco bacilli, prophylaxis for surroundings?
● Meningitis diplococci prophylaxis for surroundings?
● Recurrent miscarriages + PE+ prolonged aPTT dx?
● Pancreatitis Prognostic factors.
● Warfarine and refampcin DDI
● Exudive tonsillitis! Best investigation!
● Septic arthritis organisms?
● Abnormal uterine bleeding and the coses according to the age!
● NIC —> risk factor?
● Primary prevention of bleeding in esophageal varices is >> beta blocker if bleeding occurs then give
octreotide. So I think in this case it’s octreotide but not sure.
Glory team August

● Rheumatoid arthritis pleural effusion characteristics? Low glucose, If in analysis > low glucose

● If in imaging > rheumatoid nodule
● Meconium aspiration syndrome treatment initially by Nitic oxide, if no response so add Surfactant
lavage.

● Phases of paracetamol Toxicity

● What associated with increased risk of C. diffclli

● Kwasaki criteria

● Fetus with both knee and hip flexed? Type of Breech >> so it complete
Glory team August

● All types of shock imp:

● There was a question about pneumonia where he was questioning your CURB>65 memorization
Admit or not to admit??


● duct ectasia> retraction nipple+ slite,

● Circomural nipple retraction > cancer

● Question about pulmonary hypertension. > parasternal heave and clear lung markers.
● Similar immunofluorescent findings in diabetic nephropathy
SLE
Glory team August

Post-op pt. ... treated with unfractionated heparin ... after five days developed petechia, Normal PT/PTT, Dx?
Most likely is type 2 heparin induced thrombocytopenia

PNEUMONIA

CURB>65,
Confusion,
Uremia,
Respiratory distress
BP low,
AGE >65

Interpretation:-
0-1 home
2-3 admit
More > ICU

0-1 is OPD
2 is admission
3 or more is ICU
Glory team August
Glory team August

AUGUST 19th
1- previous admissions for bronchogenic carcinoma with pleural effusion, managed by tab and other things.
presented today with the same complain. What will you give him?
A. High salt
B. human albumin
C. Diuretics.
D. Chemical pleurodesis
Correct answer is D

2- baby with stony dulness.


A. plueral effusion.
B. Lung collapse.
Correct answer is A
3- RTA patient, hypotensive normal air entry with distant heart sounds. x-ray multiple rib fractures and wide
mediastinum. Dx?
A. fat embolisms .
B. Cardiac tamponade.
Correct answer is B

4- after chickenpox, kid is clumsy and weak in his lower limps, on examination, power is normal but has neck
rigidity Dx?
A. GBS
B. Meningioencephilitis.

Correct answer is B
5- Q: pic of baby with rash around his neck asking for the dx?
Answer : candida interigo

6- open fracture of fibula and tibia. Mx?


A. debridement.
B. Internal fixation.
C. Open reduction and internal fixation (ORIF)

N.B: if open fracture in lower limb; nail is the best, debridement is the initial
7- Intussusception after stabilazing the pt
A. NGT.
B. Surgery.
Correct answer is A
8- Q: medication that reduces mortality in CHF.
A. digoxin.
B. ACEI
Correct answer is B

9- RTA patient with blood on urethral meatus. What will you do initially?
A. Cysturtherogram
B. suprapubic cath.
Correct answer is B

10- baby with diaper rash and oral thrush. How will you manage him?
A. oral antifungal.
B. Oral and topical antifungal.
C. Topical antifungal
Glory team August

Correct answer is B
11- post cholecystectomy came with perihepatic collection, what will you do?
A. Abx
B. Lap drainage
Correct answer is B
N.B : Post operative intraabdominal collection with or without fever :
• If less than 5*5 cm -> IV Abx. • If more than 5*5 cm -> Image guided drainage.

14- woman everything normal but with dusky color cervix?


Answer: normal pregnancy ( Dusky cervix = chadowik sign )

12- boy with tenderness localized to the upper pole of the testis. Dx?
A. appendage torsion.
B. Testicular torsion.
Correct answer is A
N.B : Torsion of the appendix testis is more common in children than testicular torsion and may be diagnosed
by the "blue dot sign" (i.e., tender nodule with blue discoloration on the upper pole of the testis

13- scenario of woman post D&C asherman syndrome suspected. Which layer?
A. Functionalis
B. Basalis
Correct answer is B

14- woman with Bicornicate uterus, leopard ex: baby heart beats felt on the mother umbilicus, kicking mainly
on the lower abdomen, soft on biischial spine, What is the fetal presentation?
A. Breech.
B. Shoulder.
C. Vertical.
D. Face or brow
Correct answer is A
15- pt with a fib, treatment?
A. Warfarin 2-3
B. Warfarin 3-4
Correct answer is A
16- old patient with CHF and polymyalgia rheumatica. His labs are High wbc, high lymphocytes otherwise
normal. Dx?
A. lymphoma.
B. viral illness.
C. CLL
Correct answer is C

17- Old pt sever cough then back pain compression mx?


Answer : lung cancer with back bone metastatic , Spinal cord compression , management: Steroid +MRI

18- Old ptient with back pain, high ALP, t score -2.1 x-Ray, multible vertebral fracture. dx?
A. Osteopersosis
B. Osteomalacia
C. Paget
Correct answer is C
N.B : hint is high ALP
Glory team August

19- on athsmatic child with a smoker father a d one answer was smoking cessation consultation for parents
The other question was astha attack at the ER, Management?
A. ignore father and treat child✅
B. call protective services c
C. ontact ethics committee (not sure)
D. condemn the father
Correct answer is A
N.B :If first ER visit; treat child , If multiple and father careless; child protection

20- case was in the clinic asthma is controlled what would you do? Qs not clear
A. Add steriod
B. 2 other answers were to add other medications

N.B : SABA > low dose steroid > LABA > ...

21- Diabetogenec effect in pregnancy


A. Inhabin
B. Lactogen
C. Estradol
D. Progesterone
Correct answer is B
N.B : GDM caused by human placental lactogen

22- Child with radial fracture that penetrates out of the skin 1 cm (how to manage)
Answer : debridement with fixation

23- 6 yo boy femoral fracture 30% angulated how to manage


Answer : open reduction with IMN , Any open fracture first you should give abx & irrigation & debridement

24- Febrile seizure management


A. Lorazepam
B. Phenytoin
C. Phenobarbital
D. Paracetamol

N.B :If in ER so diazepam to control , But best is treat underlying cause by paracetamol

25- Post-term pregnancy , I think bishop is 5-6 ,what to do ?


A. ROM
B. Ripening
C. Oxytocin
D. C/s
Correct answer is B

26- - 2ed stage of labour 3h ,station +2/+3 ,how to manage ?


A. Vacuum
B. Cs
C. Others

27- Case of G6PD “sudden jaundice ..etc fever .. what to avoid ?


A. Aspirin
Glory team August

B. Ibuprofen
C. Paracetamols
D. Amoxicillin
Correct answer is A

28- Abdominal X ray,, not sure if "NO free air underdiaphragm" or "No air fluid level" Next step:
A. Amylase
B. ABG
C. measure Ca
D. Measure Cl
Correct answer is

29- Baby vomiting a lot What to give now ?


A. Normally saline
B. Ringer lactate
C. Kc
Correct answer is A
N.B : bolus of NS

30- 2 days old , fever CSF done -> bacilli + catalse positive, Strept ! “Multiple org.” Tx ?
A. Cefotaxim
B. Amox
C. Genitamicin
D. Carpapenem
Correct answer is A
31- Female patient with talk different language, she is concerning about her child status and so anxious..
what to do?
A. show her u understood her emotions
Correct answer is A

32- Patient on TCA 30mg evening ,complaining of confusion morning.. what to do? S
A. witch to SSRI
Correct answer is A

33- Child 3 y ,what do u suspect?


A. Climbing upstairs
Correct answer is A

34- Obese male complain of sexual dysfunction normal prolactin low LH FSH testarone MRI 2.5cm pituitary
adenoma what is cause of his sexual dysfunction?
A. Obasity
B. Hyperprolactinemia
C. Non functioning pituitary adenoma
Correct answer is A
N.B : Pituitary tumors, or adenomas, are the most common cause of hypopituitarism in adults but since
prolactin is normal it’s not hypopituitarism , So could be obesity , If prolactin was also low definitely
answer will be MRI results or pituitary adenoma . adenoma may cause hypopituitarism, but usually you
will find more than one hormone is low since anterior pituitary produces (TSH,prolactin, gonadotropin)
Glory team August

35- Food poisoning after steak 4h vomiting + watery diarrhea?


A. Staph
B. Salmonella
C. C. jejni
Correct answer is A

36- Female rta already stable then found to be pregnant what indecate Rh immunoglobulin
A. Rh +ve and abdominal trauma
B. Rh +ve and wrist fracture
C. Rh-ve and abdominal trauma ✅
D. Rh-ve and wrist fracture
Correct answer is C

37- Patient post MI developed palpitation >> ECG >> wide QRS
Answer : suggests bundle branch block

38- COPD pt developed , c/m only from left pleuritic chest pain, Pneumothorax 2cm btw chest wall and
pleura >> Q.management
A. needle aspiration
B. chest tube
C. Observation
Correct answer is B
N.B : Spontaneous pneumothorax:
*•primary*: age less than 50 without lung disease or Hx of smoking.
Treatment: -if symptomatic regardless the size start needle aspiration (fail put chest tube)
-if asymptomatic look for size :
Less than 2cm > discharge him and OPD follow-up
More than 2 cm > needle aspiration
*•secondary*: age more than 50 with lung disease or Hx of smoking.
Treatment: - symptomatic by chest tube immediately
-if asymptomatic look for size :
Less than 2cm > observe
More than 2 cm > needle aspiration
Glory team August

39- pregnant lady known case of asthma, on LABA, ICS, SABA (( uncontrolled/late night attacks )) >> Q.
Management
Answer : Add Oral corticosteroids

40- 7y old female started breast development, and pubic hair start to appears and acne >> dx.
A. ovarian tumer
B. central precocious puberty
C. Gonadotropin
Correct answer is B

41- Pt with left Lateral neck mass (( third triangle just below angle of mandible )) ... U/S thyroid normal and
post.cervical lymph node enlargement , on specimen it’s shows follicular thyroid cells >> dx.
A. thyroid carcinoma
B. ectopic thyroid gland
C. apparent thyroid gland
D. thyroglossal cyst
Correct answer is C

42- Pt with hx. Of retinal detachment surgery 7 day ago and he is going into surgery.... >> Q. Wt to used to
prevent DVT
A. Enoxaparine
B. Pneumatic
C. mixed pneumatic and enoxaparine
D. Aspirin
Correct answer is B

43- Pt after head trauma he couldn’t make the spoon reach his mouth to eat >> where is the lesion ?
A. Temporal
B. Parital
C. Occipital
D. Cerebellum.
Correct answer is D

44- Pt. After going into open hernia repair .. developed bulging mass with cough same area >>
Q.management
N.B: answer is lap hernia repair
Glory team August

45- Which of the following indicate searching for organic cause in a child>>
A. peri umbilical abdominal pain
B. last more than 10 min
C. around morning
D. before sleep time
Correct answer is D

46- Description of a patient with 3 symptoms of SLE >> Q. Wt the fourth symptom of the 11 according to
(RHEUMATOLOGY, SLE organization) to diagnose this pt with SLE >>
A. type of rash can’t remember
B. RF positive
C. hemolytic anemia
Correct answer is C

47- Mass found in the liver, suspecting HCC >> Q. Wt the most risk for it
A. HBV
B. Aflatoxin
C. Cirrhosis
Correct answer is C

48- Child with mass in his left flank, UA shows hematuria >> Q. dx
A. wilms tumer
B. Neuroblastoma
Correct answer is A
49- Rapidly progressive glomerulonephritis >> Q. Histo
A. crescent shape formation
B. IgA deposition
Correct answer is A

50- Brown vomiting after mechanical vent for 7 days:


A. Functional dyspepsia
B. Helicobacter gastritis
C. Esophageal gastritis
D. Stress ulcer
Correct answer is D

51- Female stopped menstruation for 8 weeks, came with severe pain followed by severe bleeding for 6
hours, came to ER:
A. Ectopic
B. Threatened abortion
C. Complete abortion
D. Rupture uterus
Correct answer is A

52- epilepsy child on antiepileptic he is having suezure and he has complication and uncontrolled:
Glory team August

A. IPV to OPV
B. Defer DTP
C. Defer all live attenuated vaccines
D. Defer all vaccines
Correct answer is B

53- most common cause of shock in Pedia


A. Septic
B. Hypovolemic
Correct answer is A

54- Anal fissures with skin tag not respond to drug , want procrdure? I
A. nternal sphernctomy
B. External sphrenctomy
C. Anal cuurtage with remove tag
Correct answer is C

55- Meconium aspirations > nitrous oxides

56- Palpable gallbladder, with jundice , didn’t mention painful gallbladder, ask about Q:>
A. GB cancer
B. Panceratic cancer
C. Cholthisis
Correct answer is B

57- Acute pancreatitis with amyalse is high ask investigatoion>


A. CT abdomen
B. Amalyse urine
Correct answer is A

58- 18 months girl with asymmetric breast enlargement with no secondary characteristics? What inv you’ll
do?
A. ACTH
B. MRI
C. CR
D. pelvic US
Correct answer is D

59- Copd patient with exacerbation, his ph is 7.2 and his spo2 is high, with will you do initially?
A. reduce oxygen
B. mechanical ventilation.
Correct answer is B

60- Child throws a ball and draws a straight-line age?


A. 12 months
B. 15 months
C. 18 months.
D. 24 months
Correct answer is D

61- Old patient with gluteal atrophy with confirmed aorto-ilial stenosis, What will you do?
A. brachio-femoral bypass
Glory team August

B. Aortofemoral bypass
C. Heparin
Correct answer is B

62- Baby passed stool after delivery, foul smelling, he’s not eating and irritable, since then hes passing small
amount of stool that is yellow and smelly. presents with bilious vomiting and significant abdominal
distension.
A. Hirschsprungs necrotizing
B. Entercoltitis
Correct answer is B

63- Kawasaki patient managed. How will you follow up?


A. Echo.
Correct answer is B

64- ECG with inferior leads ST elevation, what will you do next?
A. Right ECG.
Correct answer is A

65- 3 years old boy came with bloody diarrhea after eating fast food 5 days prior to his presentation. He is
pale and lethargic mother says his face is swollen and he’s urinating less. What will you do?
A. Splenectomy.
B. Antibiotics.
C. Observe.
Correct answer is C
N.B : Supportive more accurate
66- Patient k/c of HTN DM and stroke came with signs of CHF, calculate his Chad score.
A. 4
B. 5
C. 3
D. 2
Correct answer is B

67- management of Supraventricular tachycardia initially:


A. Adenosine
B. Carotid massage
Correct answer is B

68- -Pt smoker develop pneumothorax 3cm aspirated in fallow up normal cxray what your advice:
A. Not to fly
B. Stop smoking
Correct answer is B

69- -Congenital diaphragmatic hernia after stabilization what to do ?


A. Surgery
B. nasogastric tube
Correct answer is B
Glory team August

70-Young male fall down 4 Metter stable normal examination feeling sever heels pain what to do?
A. pain control
B. x-ray
C. Pulse exam
Correct answer is C

71- patient have a history of CKD and complain of knee pain in wich he take paracetamol for it but the pain is
still and he wants stronger medication:
A. NSAID
N. B : answer is : weak opioid ( Codaine or tramadol ) , Hint : CKD , do not use NSIAD

72- pediatric patient taller than 95centile and weight less than 25% and has heart anomaly what is the dx:
marfan syndrome

73- pregnant woman with yellow milk (clustrum) what to tell her about it:
this milk is rich in protein

74- patien after MVA in ER with 2 ribs fractured and paradoxical chest movement how to manage :
A. chest tube
B. needle decompression
N.B :answer : First Stabilize, analgesic, IVF , If in respiratory failure: intubation
Flail chest supportive unless oxygen saturation less than 60 intubate the pt
75- Sx of GI cancer in small intestine with high 5-HIAA:
A. Carcinoid
B. Lymphoma
C. Gastrointestinal something
Correct answer is A

76- Child can’t handle spoon well But he can walk up the stairs with one hand held What is his age:
A. 18 months
B. 24 months
Correct answer is A

77- Parents were concerned about a Child he can babble but he can’t say 3 words together he is (I forgot his
age <18 months older than 6 months:
A. Reassurance
Correct answer is A

78- Child can keep his hold up while sitting but he needs support to sit he can smile what’s his age:
A. 4 months
B. 5 Months
C. 6 months
Correct answer is A

79- Glomerulonephritis symptoms then I think after one week he developed hemoptysis what’s the
diagnosis:
A. Goodpasture syndrome
B. Rapidly progressive glomerulonephritis
Correct answer is A

80- Gastric cancer 5 cm I think sarcoma ( may be it is adenocarcinoma ) in pylorus? what’s the treatment:
may be not sarcoma
Glory team August

A. Total gastrectomy
B. Wide excision
C. Partial gastrictomy
Correct answer is C
N.B : Treatment of Gastric adenocarcinoma:
•If distal(pylorus): *partial gastrectomy.*
•If Middle(body) and upper(cardia): *total gastrectomy.*
•If some lymph node involved, regardless the site of cancer: *total gastrectomy.*

81- Lady with spider veins need to remove it cosmetically what do you do for investigation
A. CT venogram
B. Duplex Venus
Correct answer is B

82- Gastric pain and supraclavicular lymph node what do you do for investigation
A. Endoscopy
Correct answer is A

83- A child comes with fever hypotension < 80/60... what’s the diagnosis?
A. Septic shock
B. Sepsis
Correct answer is A

84- Patient in the ICU he has hypertension, there is elevated potassium decrease sodium fever i think what’s
the diagnosis:
A. Septic shock syndrome
B. Thyrotoxicosis
C. Adrenal insufficiency
Correct answer is
N.B : HTN IS MISWRITTEN , since adrenal insufficiency presented by hypotension

85- Long case, Flat T wave on ECG what do you find in urine , After recurrent vomiting :
A. High potassium
B. Sodium
C. Aciduria
Correct answer is C
N.B : Na is resorbed, but K is lost via an aldosterone mediated mechanism and this leads to excretion of H
ion resulting in "paradoxical aciduria" in an alkalotic patient , Recurrent vomiting leads to hypokalemic
metabolic alkalosis then acids lost in urine , If Q is : on ECG you find flat T wave. Diagnosis? hypokalemia

86- Lady delivered a baby she has UTI they did culture and it was sensitive for ciprofloxacin nitrofurantoin
and sulfamethoxazole what’s the treatment:
A-Ciprofloxacin
B-Sulfamethoxazole
C-Nitrofurantoin
Correct answer is C

87- Patient with mitral stenosis getting pregnant, what is the physiological change that Happen in pregnancy
affecting or leading to heart failure I can’t remember:
A-Increased RBC
B-Increased stroke volume
Correct answer is B
Glory team August

88- Systolic murmur on the left side with Peaked a wave:


A-PS
B-AS
Correct answer is A
89- X-ray showing radio-opaque mass at McBurney’s point and rebound tenderness... what to do next:
A-Appendectomy
B-CT without contrast
Correct answer is A
90- Patient has a hip problem and there was fluid in his hip what’s next
A-Hip aspiration
Correct answer is A
91- Management of a patient with gastrointestinal stromal tumor of the stomach?
A- Gastrectomy
B- Wide local excision
C- Radiotherapy
Correct answer is B
N.B : Treatment of Gastrointestinal stromal tumor (GIST):
•Small GIST (< 2 cm): *observe and scope.*
•Large GIST (> 2 cm):wide local excision ± neoadjuvant and/or adjuvant treatment with imatinib
•Palliative treatment using imatinib, if the tumor is not resectable and has developed metastases.

92- 5 months old How reassure his mother that he is developmentally Ok:
A-Wave bye bye
B-Reach for objects
Correct answer is B
93- Late complication of meningitis:
A-Seizure
B-Hearing loss
C-Facial nerve palsy
D-Ataxia
Correct answer is B

94- Toxoplasmosis test is +ve what ttt: (treatment different than option given ?)
A. Doxycycline + clindamycin
B. rifampin + one of cephalosporin

N.B : treatment is pyrimethamine+ sulfadiazine

95- Patient with HTN and DIC need thrombus prophylaxis (i don’t remember exactly why may be cause of leg)
(un clear question )
A. unfractionated heparin
B. drug from thrombolytic
Correct answer is A
N.B depends on the case , DIC usually treated by FFP and heparin use is contoversial and rarely used except in
pt presenting predominantly with thrombosis ... so yes we can use it if pt present with thrombotic events

96- Schizophrenia case what is the ttt


A. Bride
B. Lithium
C. Olanzapine or clonzapine
Correct answer is C
97- Pt with hyperkamia ( not mention ECG changes) K is 6.5 What to give ?
A. Calcium gluconate
Glory team August

B. Dextrose with insulin


Correct answer is A
N.B : K 6.5 and above give Ca no matter ECG changes

98- Newborn the family has hx of their son died cus of immunodefiency what to do ?
A. GIVE BCG only
B. Wait for immunization team to evaluate the child
C. Defer all vaccines until dx
D. Give birth vaccines and immunology work up
Correct answer is B
Another choices :
A. Give Hep b only
B. Give birth vaccines
C. Defer Vaccines until full immunity assessment
D. Order the vaccines & inform immunity team ✅
Correct answer is D
N.B : Hep B not harmful , choose immunity check if Hep B not in answer

99- Patient with heart failure came with fluid overload, respiratory distress, o2 sat low, What’s‫ ا‬the
appropriate management?
A.Oxygen.
B.IV furosemide.
Correct answer is A
N.B : Oxygen before Frusemide
Another same Q : If pt present with signs and symptoms of HF with infection, next?
A. ABx
B. Furosemide
Correct answer is A
N.B : Furosemide if symptomatic pulmonary edema or effusion

100- Man presented with constricted pupils, what to give?


A. Naloxone (morphine toxicity)
Correct answer is A
101- Pt came with hypertension CT showed asymmetrical kidneys ?
A. Renal artery stensosis
B. polycystic kidney
Correct answer is A
N.B : PCKD; HTN, one kidney larger, family hx.
RAS; HTN; one kidney smaller, resistant HTN to medications.
Will be clear in exam
go for RAS exept if they are giving a hint for PCKD ,Hematuria + family hx with polycystic kidney
Glory team August

102- - 5 y.o child with bed wetting, when should he learn to go to bathroom :
A. 5
B. 6
C. 7
D. 8
Correct answer is B
103- Achalasia management? Nonsurgical options include
A. Pneumatic dilatio
B. Botox (botulinum toxin type A)
Correct answer is A
104- Patient with bicuspic aortic valve and prosthetic valve is going for hernia repair surgery,, prophylaxis?
A. Amoxicillin
B. No need
Correct answer is B
N.B : prosthetic valve indicated only for dental procedure

105- Patient with bicuspic aortic valve and prosthetic valve is going for hernia repair surgery,, prophylaxis?
A. Amoxicillin
B. No need
Correct answer is B
N.B : prosthetic valve indicated only for dental procedure

106- A case about otitis externa ,, Tx?


A. Topical neomycin
B. topical ciprofloxacin

N.B : Both correct and Neomycin/polymyxin B are a reasonable first-line therapy when the tympanic
membrane is intact , If associated with swimming or DM or perforated TM answer cipro

107- Lady with hyperthyroidism, diffuse thyroid enlargement with two nodule,, Low TSH High T3,T4 Next:
A. Radio iodine scan
B. FNA
C. Surgery
Correct answer is A

108- Pt with hyperpigmentation and tiredness.. (case of adrenal insufficency) best investigation to reach dx :
A. SYNCHAN test
B. Dexamethasone sulression test
Correct answer is A

109- female taking progesterone she has PCOS taken for 5 years then stopped what she might have?
A. Osteoporosis
B. Cervical cancer
C. Endometrial carcinoma
Correct answer is A
N.B : PCOS cause endometrial ca but not in short duration

110- one with ascites after drainage asking what med to use after to not have reacumulation?
A. Spironolactone
Glory team August

B. Loop diuretics
C. Thiazide
Correct answer is A

111- pregnant just delivered has endometritis doctor prescribed cefotaxime and clindamycin she started
taking it for 7 days and during the treatment she had abdominal pain with watery diahrea after culture
showed bacteria with enterotoxin how to ttt?
A. Continue same treatment
B. Give metronidazole
Correct answer is B
N.B : it is C.difficile organism , ttt : metronidazole if not response vanco.
112- patient for adrenalectomy what to give preop
A. Give hydrocortisone
B. Phenyldiazipine
Correct answer is A
N.B : if THEY MENTIONED pheochromocytoma > give alpha blockers

113- cardiac anomaly in down


A. ASD
B. VSD
C. Coarctation
Correct answer is B
N.B : Down syndrome , AVSD then VSD then ASD

114- Risk for Nectrotizing Enterocolitis ?


A. Low birth weight
Correct answer is A

115- male, came with hemoptysis, history of recurrent sinusitis. with a finding of Crescentic necrotizing
glomerulonephritis? (can happen with saddle nose or nose with polyp) ?
Dx: Wegner disease (granulamatous poly angitis GPA)
Next: c-ANCA Tx: Cyclophosohamide

116- Most common cause of disorder sex difference in boys


A. Hypospedias
B. Small Pured
C. Cryptochridism
Correct answer is C

117- Newborn with multiple congenital anomaly’s and cardiac abnormalities + rocker-bottom feet , dx ?
A. Patau syndrome
B. Edward syndrome
Glory team August

118- Pt in PICU after diabetic ketoacidosis , labs show hypokalemia, what is the cause :
A- due to insulin
B- vomiting
Correct answer is A
119- Lady has hyperthyroidism with compressive symptoms ( difficulty swalloing ) best treatment:
A. Methimazole
B. Pylthiouracil
C. Radioactive iodine
D. Steroid
Correct answer is C
120- What is the cause of poor wound healing in controlled DM :
A. Peripheral neuropathy
B. Poor blood supply
Correct answer is B
N.B : answer is limited phagocytosis if not in the choises then B it is the answer.

121- Elderly with the classic triad of DEMENTIA, GAIT APRAXIA, URINARY INCONTINENCE,, imagining shows
enlarged ventricles,, Dx?
A. Normal pressure hydrocephalus
Correct answer is A
122- MI pt what would you do in ER initially ?
A. TPA
B. Stent
C. heparin
Glory team August

D. Cath
Correct answer is C
Another same Q : MI case presented to ER of tertiary care center, what is the most appropriate treatment
after Initial ER management in 3ry center?
A. tpA
B. PCI
C. heparin
D. Cath
Correct answer is C
N.B : Aspirin, BB, pain control , O2 if in hypoxia, nitroglycerin if not inferior MI
Heparin specially for stemi > cath , IF it is inferior STEMI MX is Iv fluid + ECG for right leads + avoid morphine &
nitroglycerin. If Q mentioned most appropriate it is PCI sure

123- Q about spinal stenosis

124- back pain increase when going downhill decrease in going up?
A. Acute disc herniation
B. Cervical spnodulylosis
C. Cervical stenosis
Correct answer is C
N.B: 1-Ankylosing spondylitis: pain in low back and gluteal region, insidious pain with increased pain and
stiffness in the morning, pain decrease with exercise and increase with rest.
2-Disc herniation: more common in age between 30-40 , pain worse with flexion and sneezing and bending
,pain described as shooting or shock like pain.
3- Spinal Stenosis: more common above 60 , pain usually in both leg , aggravate by extension and walking and
standing , relieved by bending forward or sitting
Glory team August

125- intussption case with CT showed complete small intestine obstruction ?


A. surgical resection
B. surgical reduction
C. redution emnma
Correct answer is C
N.B : Intussusception:IV fluid + NGT Then if pediatric: air enema , Adult: surgical resection
In pedia start enema and if failed repeat.
Start with laparotamoy if perforated or any signs of peritonitis

126- COPD with respiratory failure best way to deliver O2 ?


A. Bipap
Correct answer is A
127- COPD with mouth full of secretion :
A. CPAP
B. Nasal canula
C. mechanical ventilation
D. face mask
Correct answer is C

128- RTA before 5 day - Xray- showed white shadow covering half of the lung
A. AB
B. chest tube
C. Needle
D. Observe
Correct answer is B
N.B : it is Hemothorax ttt chest tube
129-LVH ejection fraction 55%:
A. High output heart failure s
B. systolic failure
C. Preserved HF
D. diastole failure
E. hypertrophic cardiomyopathies
Correct answer is C
Glory team August

N.B : If the affect of ejections fruction its in Sytolic , In daistolic ,Heart will be relaxation and EF will preserved

130- ? veteran came with fever back pain for 6 months


A. test tubercle skin test
B. cold agglutination tube
C. sacro something imaging
Correct answer is B
N.B : it is Brucellosis , Brucella agglutination test

131- pt with headache for 2 years 2-3 times in a week mainly in lt and rt frontal sinus what to do :
A. head MRI
B. para nasal sinus CT
C. more history and clinical examination
Correct answer is C
N.B : First next step is history for sure , As investigation yes CT (If most important or appropriate go for CT)

132- ?when to do brain MRI in hyperprolcnemia :


A. visual disturbance
B. prolactin double the normal
Correct answer is B

134- Q garlic order > organophosphate poising


N.B : organophosphate , antidote is atropine

135- 12 or 8 y, healthy until when he visit him father and stibmother complaining of abd. Destenstion … drink
milk, next in management >
A. serum lactos level
B. hydrogen breath
C. lactose after drink
Correct answer is B

136- went to travel to area with malaria, what to do :


A. Vaccination
B. Immunoglobulin
C. Chemo
D. not to go to endeic area
Correct answer is C
N.B: Mefloquine,malarone,doxycycline in area resistance to chloroquine
Chloroquine in rest , *best* regimen for malaria prophylaxis is "Atovaquone / Proguanil" in the choices which
is malarone
Glory team August

137- Most common complication scarlet fever?

138-Patient undergone CT with contrast and then the labs showed high creatinine, what
is the cause?
A. ATN
B. AIN
C. Acute glomerulonephritis
Correct answer is A
N.B : contrast induce nephrotoxicity , also ethanol glycerol cause ATN , Contrast indused nephrology sign:high
creatinin ,decrease in urin output after contrast.

139- 5-year-old girl with adult like body odor and axillary and pubic hair. What
investigation should you order?
A. FSH
B. Testosterone
C. 17-Hydroxyprogesterone
D. Dehydroepiandrosterone
Correct answer is D

140-Patient had MVA with SOB. (No other findings of tension pneumothorax like trachea
shift or absent breath sounds). CXR showed fractures of ribs 1-4 at multiple
locations. How would you manage? (looked like flail chest case)
A. Chest tube
B. Needle decompression
C. Intubation
D. ..
Correct answer is C
N.B: It depends if stable start with analgesics and fluid, If unstable intubate him

141-Patient had end stage cancer. Family didn’t want you to tell the patient.
A. Don’t tell the patient
B. Tell the patient
C. Consult ethical committee
D. Consult legal advisor
Correct answer is B
No option of exploring and assessing whether the patient want to know of not

142-Young male patient presented to the ER with fresh bright rectal bleeding. Rectal
examination showed clotted blood at the anus. NG tube showed greenish secretions
coming out. Colonoscopy done and was negative. What should you do next?
A. capsule endoscopy
B. Technetium-99m scan
C. CT abdomen
D. ..
Correct answer is A
No angiography

143-A child with non-bloody diarrhea, failure to thrive and buttocks atrophy what is the
Glory team August

most likely diagnosis?


A. Crohn’s disease
B. Celiac disease
C. ..
D. ..
Correct answer is B

144-Same case what would you order?


A. Anti-endomysial antibody
B. duodenal biopsy
C. colonoscopy
D. ..
Correct answer is B
N.B: if asked next answer is A , if asked best then biopsy

145-Patient with celiac disease non compliant to gluten free diet, how to assess
compliance?
A. Food diary
B. Anti-endomysial antibody
C. Anti-tissue transglutaminase antibody
D. ..
Correct answer is C

146-Young girl has prolonged watery diarrhea. She is fussy about food and only drinks 3
pints of goat milk per day. Labs showed low Hgb and high MCV. What is cause?
A. Psychogenic deprivation
B. Giardia infection
C. IDA
Correct answer is B
N.B : giardia cause high MCV , because it prevent absorption of B12

147-Patient with SLE with arthritis and skin manifestation, BUN and creatinine were
within normal. What would you give?
A. Hydroxychloroquine and Mycophenolate mofetil
B. Hydroxychloroquine and azathioprine
C. Hydroxychloroquine and Methotrexate
D. Hydroxychloroquine and cyclophosphamide
Correct answer is C

148-Around 40yrs menopause use progesterone to regulate his period , whatis the Risk factor of endometrial
carcinoma :
A. Age
B. Progesterone
Correct answer is A

149-30 yrs Marriad pt sinces 3 yr .. ask about when do pap screening ..


A. Now
B. after 3 yrs
Glory team August

C. don’t do in 30 age
Correct answer is A

150-What most risk factor cause breast ca :


A. Age
B. Nulliparous
C. Early menses
D. Late post menopause
Correct answer is A

151-Old age I think 50yr have headache Increase when he down his head + mostly in back in his neck ..
A. Tenstion headache
B. Subarachnoid hemorrhage
Correct answer is B

152-Women need to do surgery, his husband want to sign but she refuse that , who is responsible about
decision?
A. Patient
B. Husband
Correct answer is A

153-Women use OCP before marriage now his husband want baby and she need to IUCD I think
What to you do?
A. Let them discuss with each other
B. Wife decision
C. Husband decision
Correct answer is A

154-Most common screen :


A. Colorectal
B. Breast
Correct answer is A

155-Old case fatigue high WBC + lymphocytes =


A. CLL
Correct answer is A

156-Old age + endometrial lining =15mm or cm + normal utrus size in examination


What management:
A. Hysteroscopy
B. Endometrial biopsy
Correct answer is B

157-Men came from hajj , with infection :


A. airborn
B. Droplet
Correct answer is B
N.B : meningitis is droplet

158-Mid shaft femur Fx management :


Glory team August

A. Open w/ IMN
Correct answer is A

159-Common complication of meningitis :


A. Hear loss
Correct answer is A
N.B : in children > meningitis , in adult > orchitis .

160-Common complication of mumps in child :


A. Parotid
Correct answer is A

161-Whats the most risk factor for optalmopatgy in graves : :


A. Age
B. No response to medication
C. Level of fT3 and f T4
D. Smoking
Correct answer is D

162-In cs op before suture see bleeding came from up of abdomen . Whats the source.
A. liver hemangioma
B. splenic rapture
C. mesenteric a rupture
Correct answer is A

163-Bleeding After cericumsion : lap prolonged PT +PTT :


A. factor X
B. Factor 13
Correct answer is A
N.B : prolonged PT +PTT , factor X
164-5 years old child with unilateral undescended testes
Tx ‫؟‬
A. orchiectomy
B. Orchiopexy
C. observation .
D. Give him testosterone .
Correct answer is B

165-Case of vesicular rush in mouth tongue and soft palate + positive FHx of eczema :
A. Eczema herpatitis
Correct answer is A

166-Low limb + pulse is normal what to do investigation :


A. Ct angio
B. Venous doppler
Correct answer is B

167-Child his father died in accident he always says that his father in the house and he is talking to him , what
is dx ?
N.B : Less than 1 month > acute psychotic brief, 1 to 6 months > schizophreniphorm , More than 6 months >
schizophrenia
Glory team August

168-Child complaining of abdominal pain in the school his family try to support him with no result , what they
will do for him > N.B : Show for him the positive things about school

169-SlE came with polyarthritis :


What the Ttt :
A. hydroxy+ CS
Correct answer is A

170-Ttt of meningitis in child 7yr : vancomycine + ceftraixone

171-HTN pt take drugs ..... complaine of cough ..


whats the management :
A. Change ACEI to ARB
Correct answer is A

172-Case of epileptic want to stop smoking: what durg is C/I


A. Bupropion
Correct answer is A

173-Hyperkalemia 6.5 + renal impairment : whats the Mx (Many drugs):


A. Ca gluconate
B. Insuline + dextrose
Correct answer is A

174-Pt after delivary asymptomatic has +ve culture of UTI what the mx :
A. give some drugs
B. no need for Ttt
C. Nitro
Correct answer is B

175-Case of swelling of joint arthritis : that complication of which ds .. something like this :
A. cystic fibrosis

176-Pt complains neck pain pain as shoting and numbness of up arm what dx :
A. cervical disk prolapse
Correct answer is A

177-Indication of poor response to IVIG :


A. LOW WBCs
B. High crp
Correct answer is B
N.B : Indication of poor response to IVIG : Hyponatremia ,High CRP ,Neutrophilia
Glory team August

178- 90 yrs old have many ds now has mass advance what mx :
A. Palliative w/ something
B. care or symptomatic ttt something like this
Correct answer is B
N.B: palliative care

179- Case of pit adenoma :3 or 4 cm + high prolactin whats the management?


A. some drugs
B. surgical
Correct answer is A

180- PPH in previous delivery :


A. active 3rd stage with oxytocin
Correct answer is A

181- Case after Cs came w/ vaginal bleeding , mx ?

182- Sx of delivary in 34wk what mx :


A. Give cs
Correct answer is A

183-What we advise to DM pt : not clear Q


A. Low carb
Correct answer is A
N.B : first life style modification within 3 months then metformin in HBA1C mor ethan 7.5 and insulin if HBA1C
more than 8

184-Pt came for screening of Dm , positive FHx pf DM : lap high RBG or FBG + high DA1C .. whats mx ..

185-Decrease risk of MI after Smoking cessation ..


A. 1
B. 2
C. 3
Correct answer is A
N.B : • Patient quits smoking, the risk of ischemia will became like non smoker after *1 year*, The risk of
ischemia in smokers than non smokers is *2 times*

186-RF + carditis + valve ds = prophylaxis for how many :


A. until 40 yrs
Correct answer is A

187- Age of Osteoporosis screening in female :


A. 65-70
Correct answer is A
188-Case pt take NSIAD =
A. cause perforated PU
Correct answer is A

189-Pt take Iron put not effect =


A. think about Thalassemia
Glory team August

Correct answer is A

190- iron toxicity =


A. Iv deferoxamine
Correct answer is A

191- Paracetamol toxicity in stage 2 .. Mx

192- ICU ,Acalcular choecystitis defenitive mx:


A. Cholecystoctomy
B. Cholecystectomy
Correct answer is A

193- Case sudden ROM , fetal bradycardia :


A. Cord collapse
B. Abrupto
C. Vasa previa
Correct answer is A
N.B : previa and abrupto excluded because no abdominal pain and vaginal bleeding
rupture of membrane or (amniotomy) + *Painful vaginal bleeding* + Fetal bradycardia = *Abruptio placenta*
rupture of membrane or (amniotomy) followed by *Painless vaginal bleeding* + Fetal bradycardia = *Vasa
previa*
""
Make sure when you read the case.

194-Mx of thyroid nodule in lap normal TSH + SOLED nodule :


A. Scan
B. FNA
Correct answer is B
N.B : solid non toxic nodule , FNA

195- MG case ask about ttt =


A. Pyridostigmine
Correct answer is A

196- GHTN = Risk of what :


A. IUGF
Correct answer is A

197-IUFD in late preg I think 37 wk ,, what mx?

198-IUFD > mother somker , plaim her self , what to do ? Tell her the cause still not known

199- Sudden infant death .. what is important to ask the perent :


A. Prone sleep
B. Smoker parent
Correct answer is B
Glory team August

200-h. pyloric >> not response to triple mx :


A. change into ppi + bismuth + tetracycline + …
Correct answer is A
201- Pt w/ ascetits + PE + I think ovarian mass =
A. Stromal
Correct answer is A
N.B : it is meige syndrome : Meigs syndrome is defined as the triad of benign ovarian tumor with ascites and
pleural effusion

202-Case of paranpnemonic TB

203= Case of polycythemia = HTN + low erythropotine+ high Hb

204-Old age55yr + loss of wt + dysphagia…


A. urgent endoscopy
B. Ct
C. Colonoscopy
D. Barium
Correct answer is A

205- Wooping cough most complication:


A. Menigitis
B. pOeumonia
C. Pnuemothorax
Correct answer is B

206- Croup mx response to epinephrine after hr reappear same condition ..mx:


A. give epinephrine again
Correct answer is A
207-Exercise induce asthma .. now spirometer is normal .. what next :
A. do challenge test
Correct answer is A

208- High k 6.5 + renal impermeant =


A. Dialysis
B. Ca glco
➡ answer is B
Correct
N.B➡: Emergent dialysis: AEIOU
A ➡ m. Acidosis (pH<7.2)
E electrolytes life threatening/refractory hyperK+ (>6.5mEq/L)/ BUN >100mg/dL
I Intoxications (methanol, aspirin ..etc those cuz renal failure)
Glory team August

O overload (fluid)

U uremic encephalopathy/ pericarditis

• K 6.5 or more
- Initial Ca gluconate
- Then medical ttt like insulin + glucose.
• K 6.5 or more + renal impairment
- Initial Ca gluconate
- Then medical ttt like insulin + glucose
- If refractory, no improvement in renal function then dialysis.

209-Asthma sever=
A. rr more than 25
B. Pef 40
Correct answer is A
N.B : Severe asthma ,HR > 120 ,RR > 30 ,FEV1 <60 ,PEFR <60 ,SaO2 < 90 , paCO2 > 45

210-Wound infection .. most organism ?


A- Cl deficile
B- Cl perfrengens
C- STAPH
Correct answer is C

211- Rectum tumor mx ?


A- low anterior resection
B- abdominoperineal resection
C- chemo
Correct answer is B
N.B : < 6-7 cm from anal verge ; abdominoperineal
> 6-7 cm from anal verge ; low anterior resection

212-Pt w. hyperdyslipdemia ,, take drug cause…. Side effect


What name of this drug?
ANSWER : Niacin

213-SCA pt w/ sudden drop Hb =


A. parovirus 19
Correct answer is A

214- Case of renal failure on Metformin when you stop it?


A- GFR 30-45
B- GFR < 30
C- GFR < 15
D- GFR > 45
Correct answer is B

215- Low abd abd pain + negative HCG , no Hemodynamic instability.


A. Appendicitis
B- ovarian torsion
C- ectopic
Correct answer is A

216-Mother has HBV after delivery, what you will give the baby before 12 hours:
A-HBV vacations just
B-HBV vacations +Immunoglobulin
Glory team August

C-Immunoglobulin
Correct answer is B
217-Vaccination at 2 month:
A-Ipv-hib-HBV-DTap
B-Opt-hib-HBV-DTap

218-Patient has abdominal pain, during endoscopy, you find multiple antrum
ulcers, he not responded for medication, best intervention:
A-total Gastrostomy
B-Partial gastrostomy
C- vagotomy
Correct answer is C

219-Wolff-Parkinson white syndrome , not respond to treatment, next step :


A-Increase the dose
B-Radiofrequency
C-digoxin
Correct answer is B

220-Patent came with unilateral leg pain and edema , non-pitting , next :
A-Massage
B-bypass
Correct answer is B
N.B : • Lower limb non pitting edema
Diagnosis most likely lymphedema
Next to ask about hx of trauma
Next to do lymphoscentigraphy
Next to do lymphatic massage
• Unilateral Lower limb pitting edema
Diagnosis Most likely DVT
Next to ask about history of OCP
Next to do venous doupler
Confirmatory test is Magnetic resonance venogram
Glory team August

TTT; *Heparinization + Warfarin *

221- Patent came with chest pain diagnosis as unstable angina , you give hnim
BB,statin,aspirn, what you will add:
a- Alteplase
b-clopidogrel
Correct answer is B
N.B: treat as in MI except for fibrinolysis

222-How prevent pph :


A-manual placenta extraction
B-put likes sponge inside to prevent bleed
Correct answer is A
223-Child came with abdominal distension and vomiting just, other labs and
clinical exam its unremarkable, what do you think;
a-Hirschberg
b-Lactose intolerance
c-Pyloric stenosis

224-best treatment for pregnancy smoking :


A-bupropion
B-nictone patch
C-CBH
Correct answer is C
N.B : If all the choices drugs,It is Bupropion

225-Adrenal adenoma was diagnosed accidentally, Most common cause?


A- Subclinical Cushing
B- nonfunctional adenoma
Correct answer is B
226-Drug approval by FDA for tx of cancer haypercalcima :
A-denosumab
b-teripratide
Correct answer is A
227-CTG graph with VERY low variability, And the mother showed no symptoms of any
abnormality. Patient on Oxytocin, MgSO4, epidural. What is the cause?
A – Oxytocin
Glory team August

B – MgSO4
C – epidural
Correct answer is B
228-Old patent came to primary health care for his lap, the doctor told him you have
lung cancer , he respond (( you are wrong)) , This represent?
A- Denial
B - Neglect
C-compromising
Correct answer is A
229-Liver lesion 20*15cm, Labs: show amoebic, what the best next step?
a-Percutaneous drainage
b- Open derange
C. Metronedazole

230- A case of uterine fundal cancer and asked about lymph node drainage?
A-Par-aortic
B-Internal iliac
Correct answer is A
N.B : Body and cervix to internal iliac lymph nodes.
fundus to para-aortic lymph nodes.

231- Patent with neck mass , when he do FNA you found medullary cancer , best treatment ;
A-Total thyroidectomy
B-Partial thyroidectomy
C – lobectomy
Correct answer is A

232- most place injured of blunt thoracic :


A. Distal to the subclavian
Correct answer is A
233- ICU + thyroid :
A. High reverse T3
Correct answer is A
N.B : Everything low but high reverse T3 , SICK EUTHYROID SYNDROME
234- 3-pregnant with et side abdomen pain N/V obgyn consultant exclude obstetric causes what is ur dx:
appendicitis
or cholecyctitis
Correct answer is A
235- young healthy couple 6months infertility female normal periods with 4 days how to investigate?
A. Prolactine
B. Thyroid
Glory team August

C. 21 progestrone
D. urine LH/FSH
Correct answer is C

236- HTN patients US showed asymmetrycal :


A. Renal artery stenosis

237- most common HCV genotype in KSA.


A. 1
B. 2
C. 3
D. 4
Correct answer is D
238- wife her husband has HBV ?
A. Resume her sexual relationship
Correct answer is A
N.B : HBV consom ,HCV nothing

239- young female long sexual life without protection came with neuro sx with ct or MRI findings ?
A. HIV serology
B. toxoplasma test
Correct answer is A
240- bronchictasis pt came with basal crepitation most important Tx ?
A. Nubilaiser
B. sputum drainage
Correct answer is B

241- MI pt they preparing him for PCI he collapsed the cause is ?


A. Rt ventricular infarction
B. wall rupture
Correct answer is A
Glory team August

242- pt had prolactenoma visual defect ???

243- 1st antenatal visit goal ?


A. To determine the age
B. risk factor
Correct answer is B
N.B : to do a lab test for HIV , TB , blood group and RH etc...

244-wife has tubal blockage you’ll tell ?


A. Wife
B. Couple

245- HCV pt she won’t be able to breastfeed her baby if ?


A. Crackled nipple
Correct answer is A

246- antiTB with high liver enzymes ?


A. Stopp all meds

247- isolation of meningitis case?


A. 24 h
B. 48 h

248 - lactating came with mastitis 5 cm tx?


A. I&D
B. Mastectomy
C. needle aspiration
D. Abx

N.B : mastitis s&s= fever ,redness ,breast thickening but here there’s a mass so mastitis complicated to
abscess answer is INSICION AND DRAINAGE AND If this breast abcess its very important to look for the site If
peripheral = aspiration , If subarular =incision and drainage, Acute lactational mastitis TTT; Warm
compression, ABx, continue breastfeeding

249- obes BMI 28, tx?


A. Orlistat
B. other choices surgical options
C. Sleeve
D. Bypass

N.B : less than 30 BMI NOT INDICATION FOR SURGERY , start by lifestyle modification if not in choices choose
medical TTT
Glory team August

250- pos-hemorhoidectomy developed swelling & hematoma??


A. Evacuate
B. Sitz path
Correct answer is B
251- post-hernia repair, pidendal block, WOTF won't be affected?
A. Perinium
B. Rectum
Correct answer is B
252- CDH they already stebalaize pt what will you do?
A. Surgical intervention
B. NGT
Correct answer is B
253- most common site of eczema in 8y boy? (inter cubital fossa)
Eczema site: In children >> extensor surface. In general >> flexor surface (inter cubital fossa).

254- Patient had gerd (or UGI BLEEDING ) ithink and did endoscopy what you will give patient :
A. oral ppi
B. oral....
C. IV ppi for 27 h Then oral at home
D. IV ppi for... h then oral at home
Correct answer is C
N. B: Octreotide ; given prior to endoscopy in case of variceal bleeding.
PPI ; given for 72 hours in case of bleeding PUD prior to endoscopy.

255- 37-Breast mass was 3*2 I think "small size" it become large 6 I think.. They said clearly it increases in the
size:
A. firboadenmoa
B. phyllode
C. fibrocyctic change
Correct answer is B
N.B : *Fibrocystic Changes*
*C/P:* Diffuse nodularity that change in relation to menstraution increase in approach to menses and once
menses started it will return to the baseline
*Diagnostic Modality:* PE
*Rx:* Observation
*Fibroadenoma*
*C/P:* well-definded mass freely mobile (composed of glandular and fibrous tissue)
*Diagnostic Modality:* US>>core needle biopsy
*Rx:* Observation unless increasing in size or old age (more than40)
*Fat necrosis*
*C/P:* Hard lump following trauma or Sx.
Glory team August

*Diagnostic Modality:* Mammography


*Rx:* Excision
*Intraductal Papilloma*
*C/P:* unilateral breast bleeding
*Diagnostic Modality:* Core needle Biopsy
*Rx:* Excise the duct
*Phyllodes Tumor:*
Cystosarcoma Phyllodes
Variant of Fibroadenoma tgat has high mitotic rate classified into: benign , intermediate and malignat
*C/P*: large freely mobile mass +/- underlying skin changes
*Diagnosis*: US>> Core needle Bx
*Rx*:
#Smaller: Wide local excision with 1-cm -ve margin
#Larger: simple mastectomy
No Need for sentinle node biopsy
256- What is the best time for abx cs?
A. pre op
B. post op
C. when close the skin
D. when you clump the umbilical
Correct answer is A

257- best Malaria Prophylaxis :


A. Atovaquone
B. proguanil
C. Mefloquine

N.B : Combination A & B if not in Choices > mefloquine

258- Pt do hernia repair "not sure about the type" then he present with mass in the inguinal hernia firm,
regular, transpulsation "there was no Erythema or tender"and no fever :
A. sephenous
B. psudoanyresm
C. Abccese
Correct answer is B
N.B : Non tender no erythma no fever I exclude abscess

259- Pediatric Knee swilling with hx of vomiting and Diarrhea They found e coli :
A. septic arthritis
B. gonococal arthritis
Glory team August

Correct answer is A
N.B : Heamtogenous spread
$ *Reiter’s syndrome* (reactive arthritis): it’s an autoimmune reaction occurring after an infection
% How to remember its classic triad:-
*conjunctivitis*, cannot see.
*urethritis*, cannot pee.
*arthritis*, cannot climb a tree.
& *Management*& :-
*Antibiotics* if there’s active infection such as STD’s (chlymedia).
*Nsaids*, for pain and joints inflammation.

260- BI RAD 4 case , What is the most appropriate next step? The options were:
A. Excisional biopsy
B. FNA
C. Core needle biopsy
D. Follow up only
Correct answer is C

261- erectile dysfunction and had stress in work , whats the initial rx?
A. sandinfil
B. Relaxation
Correct answer is B
262- Ped take acetomenphen mother said she take it 24 h ago.. Pt looks will :
A. give the antidot
B. nothing to be given
C. Lavage
D. Observe more hours
Correct answer is B
N.B : * Acetaminophen (paracetamol) toxicity*
Stages:
*1st*: Asymptomatic or GI symptoms.
*2nd*: hepatotoxicity starts, RUQ pain, LFT’s start to rise.
*3rd*: hepatic failure & encephalopathy, LFT’s peaks + signs & symptoms of hepatic failure.
*4th*: Full recovery or death.

*Management* :
*activated charcoal* :
' be given early, at 1-3 hours post-ingestion, after that no need.
should
*Serum paracetamol level* :
you should measure paracetamol level to plot it in The Rumack-Matthew nomogram then determine either to
( the antidote or no.
give
Serum paracetamol level peaks *4 hours after ingestion* so If patient came immediately after ingestion
(
order paracetamol level after 4 hours.
)
If he came 4 hours or more after ingestion order serum paracetamol level immediately.
""If he came 7 hours after ingestion with symptoms or history suggests toxic dose start antidote immediately
!!!!
✅ because hepatic injury usually starts 8 hours post-ingestion and it’s better to give antidote before 8 hours
otherwise patient may lose his liver while you are waiting for the results.
*Start N-acetylcysteine* :-

Minimum toxic dose is 7-10g, so if you have a definitive history of toxic dose ingestion you can start antidote
based on history.
*liver transplant*:
It is the definitive management if patient ends up with fulminant hepatitis or hepatic failure.
Glory team August

263- Case of patient with fluid collection the in the lesser sac. Ttt?
A. percutaneous drainage
B. other ways of drainage mentioned in the other choices
C. Endoscopic drainge
Correct answer is C
N.B : Endoscopic drainage , If non infective pseudocyst
If infective pseudocyst > Percutaneous drainage
Less than 6 cm , 6 weeks+ Asymptomatic ( conservative)

264- US pic of ovarian mass described as multilobulated mass what to do next? The mass was seems to be
malignant sa I remember and I think they give high ca125 :
A. CA 125
B. Surgery
C. Consult onco with further workup
Correct answer is C

265- Post delivery 2 week uti sensitive to nitrofurantoin and TMX-SMX I think... What you will give:
A. nitrofurantoin
B. MX-SMX
Correct answer is A
N. B : B is not given in breast feeding

266 - Pt with abdominal pain they found e coli when they tab his abdomin,What is the most likely source for
this:
A. hematogenouas spread
B. pefrorated bowel
Correct answer is B
N. B : peritonitis with perforation And gall stones with hematogenous
If there’s signs of peritonitis > doudenal
If with biliary tract > hematogenous
267- Female elderly post menopause 12 y back now start having bleeding for 2 months back with vaginal
irritation and dryness
Glory team August

Whats the cuase of her condition:


A. Atrophy
B. adnomyosis
C. Endometriosis
Correct answer is A

268- bleeding with nodular Uterosacral ligament on ex


A. Endometriosis
B. Adenomyosis
Correct answer is A
N.B : Endometriosis > tender utersacral ligaments
Adenomyosis > enlarged tender uterus ,
+ Symmetric, Soft, Tender

269- Patient did sleeve * I think 6 month back not sure* now present with abdominal pain and distention =
A. Obstruction
Correct answer is A

270- pt post sleeve now came with her family c/o mood change:
A. psychiatry referral
B. reassurance
Correct answer is B

271 - stap wound liver ,, what the initial thing you will do
A. US
B. CT
C. 54
Correct answer is A
Penetrating abdominal trauma if stable: first wound exploration
If suspicious > imagaing US & CT
If unstable or deep injury> Laparotomy exploratory
Immediately to OR in gunshot injury

272- CSF findings of high lymphocyte and low glucose =>


A. Tb meningitis
Correct answer is A

273 - Schistosoma lead to =>


A. Pulmonary HTN
Glory team August

Correct answer is A

274 - Gall Bladder 0.6 mm or cm I forgot on routine investigation what next step to do =>
A. Observation
B. CT
Correct answer is A
N.B : The important is don’t do anything if pt is asymptamtic

275- Anal fistula in a pt with crohn’s disease what Abx to give =>
Metro, cirpo

276- Knee Swelling what Abx to give =>


A. Vancomycin
Correct answer is A

277- Obstructive sleep apnea =>


A. CPAP
Correct answer is A
N.B : OSA in adult > CPAP ,In children > surgery

278- Comminuted fracture tibia, posterior compartment pressure 35, what’s your management =>
A. External fixing with leg elevation or internal fixation with leg elevation or Fasciotomy with leg
elevation
B. B. internal fixation with leg elevation
C. C. Fasciotomy with leg elevation
Correct answer is C

279- pt with UTI and right knee pain and left ankle pain, what treatment to give?
NSAID , D x is reactive arthritis , if active infection start abx

280- midshaft hypospadias with hooded foreskin and chorde asking for name of surgery =>Turpilzied incised
plate

281- facial palsy asking for treatment in pt


came w/n 1 hour =>
A. Corticosteroid
Correct answer is A
282- post ERCP pt RUQ pain and vomiting, what of the following would decrease mortality =>
A. IV fluid
B. Abx
Correct answer is A
283- Diabetic foot ulcer showing psudo-(something) hyperkeratosis, what’s the best next step =>
A. amputation or
B. debridement
C. or control DM
Correct answer is B
284- pt came to ER after MVA hypotensive, having splenic laceration and aortic injury, what’s the
most appropriate next step =>
A. thoracotomy
B. Laparotomy
Correct answer is B
285- 32 years female with bloody discharge from nipple next step =>
A. mammogram
Glory team August

B. US
Correct answer is B
N.B : US initially specially age < 35
approach to ductal papilloma :
US Then Mammo Then Excision

286- pt with controlled hypothyroidism, asking about the dose during pregnancy =>
A. increase the dose
B. continue same dose
Correct answer is A
287- most specific indictor for graves =>
A. exophthalmus
Correct answer is A
288- Newborn with hypoglycemia what to give ?
A-2ml/kg of 10% dextrose
B-2ml/kg of 12%dextrose
C-10ml/kg of 10% dextrose
D-4 ml/kg 12% dextrose
Correct answer is A

289- muscles and bone pain relived with drinking milk , hyper
parathyroidism
Hyper calcium , very Low vit D low phosphate what is the cause ?
A.secondary parathyroid
B. Primary parathyroid
C.milk related alkaline syndrome
Correct answer is B
N.B : milk alkali syndrom is triad of Metabolic alkalosis
Hyper c
Renal impairment
Milk alkali symdrome there’s suppressed PTH level.
You will not see elevated PTH unless in the setting of milk-alkali syndrome with renal failure is caused by
severe secondary hyperparathyroidism So go for primary unless they mentioned renal failure

290- Female Pt done surgery after 12 hrs there was serous leak from the wound ?
A. CT of the abdomen
B. Exploration wound
C. Abx
D. Observation
Correct answer is D

291- Same above q exactly but after 4 Days


What are you going to do ?
A. abx
B. CT of abdomen
C. Exploration wound

N.B : first A then C , if asked what to do or best C

292- Heart failure asymptomatic pt on medication what to add?


A- Diuretic
B- ACEi
C- BB
Glory team August

Correct answer is B
Glory team August

293- Post MI she is on aspirin LMWH valsartan , what to add ?


A. ACEI
B. Spironolactone
C. BB
Correct answer is C
N.B : Drugs that decrease the mortality in pt with MI:
- statins
- BB
Glory team August

- ACEIs
- ARBs
- aspirin which is antiplatelet and NSAID

294- Bleeding drained 2 liters blod what to do?


A.CT angio embolisation
B.Thoracotomy
C. Continue chest tube
Correct answer is C
295- post postmenopausal after 12 years of menopause She has bleeding
since 2 Months what is the cause ?
A. Atrophic vaginitis
B. Endometriosis
C.adenomyosis
Correct answer is A
No endometrial cancer im choices
NB: treatment for atrophic vaginitis is topical estrogen

296- Same above q she want to do hysterectomy ?


A. - refer to oncology
B. - assure her and take more information
C. - admits and do hysterectomy
Correct answer is C
297- End stage of cancer the family said give him strong analgesia but
you said that the medication will have side effect . What is the name of
this principle of saying OK but it has side effects?
A. Informed
B. Advice
C. Double effect
D. Principle of tatality
Correct answer is C

298- Female no menstrual period webbed neck - Turner syndrome -


What is the simplest less expensive test to do!
A. FSH LH level
B. estrogen level
C. TSH and t4
Correct answer is A
No Karyotyping in choices
N.B : Primary amenorrhea A , Secondary C
Glory team August

If the question asks about the SIMPLEST only then yes karyotype
But if the question asks about SIMPLEST + *CHEAPEST* then FSH
+ karyotype simplest *to confirm*

299- Mother concerned about her child growth , When to refer baby for
FTT
Which percentile ?
A. - changing from 5 to 15 percentile
B. - pt stays on 10 percentile
C. - pt Stays on 15 percentile
Correct answer is B
No age no Previous reading provided
Glory team August

Correct answer is A
300- Diabetes insipisous given Vasopressin but no response is called
A. Nephrogenic DI
B. Central DI
C. Psychogenic DI
D. Idiopathic DI
Correct answer is A

301- Basal body temp during ovulation?


A. 36.5
B. 37
C. 37.5
Correct answer is A

302- Most accurate evaluation the responses of asthma exacerbation to


the medication?
A. X ray
B. Peak flow Spirometer
C. ABG
Correct answer is B

303- Baby fall days after came with vomiting and headache.
Investigation?
A. MRI
B. Carotid doupler
C. CT
D. Xray skull
Correct answer is C
Glory team August

304- Pediatric ingested a Bleaching agent ABC were done, whats next? Not clear
A. NGT
B. Xray
C. Endoscopy
Correct answer is B
NB: Initial resuscitate & X-ray
Then endoscopy to assess the degree
If question about most important for severity or treatment > endoscopy
Glory team August

305- Giving a complete breech presentation, then asking about LIE? It


was longitudinal and the choices contained , not clear
A. Complete
B. incomplete
C. frank
Glory team August

D. longitudinal lie

Correct answer is D

306- What to use to monitor growth in trisomy 21?


A. Down symdrome growth chart
Correct answer is A

307- managemnt of recurrent malignancy associated pleural effusion?


A- Pleurodesis
Correct answer is A
N.B : a procedure that uses medicine to adhere your lung to
your chest wall. It seals up the space between the outer lining of your
lung and chest wall (pleural cavity) to prevent fluid or air from
continually building up around your lungs.

308- Prediabetic, BMI 30. besides exercise what is the recommneded


diet to lose wieght?
A. -Low glycemic index diet to lose 5-10% of weigh
B. -low fat diet?
Correct answer is A
Same Q : with these choices:
Exercise
Low fat diet
Decrease calories intake
Correct answer is C

309 - Autoimmune cholangitis invx?


A. AMA✅ anti mitochondrial antibody
Correct answer is A

310 - Post thyroidectomy hematoma with respiratory compromise mx?


A. Open wound at bed ( Open immediately )

311- Uncomplicated Femoral hernia mx?


A. Laproscopic repair
Correct answer is A

312 - Diphtheria pseudomembrane child -ve vaccine Hx, with phryngitis


and respiratory distress, white membrane on tonsils when scraped off
blood dots appear Dx?
Glory team August

A. Diphtheria
Correct answer is A

313- Distal femural fracture with angulation mx?


A. ORIF with IMN
Correct answer is A

314- Cause of low SAAG?


A. TB peritonitis
Correct answer is A

315- Fully dilated cervix, good contractions, fetal head is +something whats next mx?
A. Vacuum
B. CS
C. WAIT FOR VAGINAL
Correct answer is C

316- Daibetic pt presented with high HbA1C next appropriate action?


A. -Next visit after 6 weeks to check HbA1c
B. -FBS after 3-6 months (i chose this)
C. –OGTT
Correct answer is A
NB : HgA1C
If high or changed medications so repeat after 3 m
If controlled so repeat after 6 m

317- Lateral mallulas ulcer Q , The pt


was HTN , Not DM , around the ulcer dark and the center of the ulcer is
viable،†invx:
a) Arterial dopplex
b) Venous dopplex
Correct answer is B
NB : Dark pigmentation around the ulcer is venous

318- Rt upper quadrants pain , dilated itrahepatic ducts and common


bile duct, “ no comment on the gallbladder and the pt wasnʼt febril, dx:
A. Cholecystitis
B. Cholangitis
C. Choledocolithiasis
Correct answer is C
NB: Dilated CBD = C

319- Post Op fever : 5 day


A. Uti
B. Dvt *
Correct answer is B
N.B : *post-op fever*
0-2 days: atelectasis or pneumonia.
3-5 days> UTI
5-7 days > DVT
7 day > wound infection
8-15 days > drug fever or deep abscess
Glory team August

320- Contraindications in External cephalic:


a) Anterior placenta
b) Previous CS
Correct answer is B
NB : Posterior or low lying = previa

321- Most accurate test for Asthma dx:


a) Spirometry
b) methacholine challenge test
Correct answer is B
NB : Most accurate test for COPD spirometry
Most accurate test for asthma methacholine

322- Intestinal obstruction symptoms , wt would you do next :


a) Ct abdomin
B) groin examination
C) Xray plain
Correct answer is B
NB : examine then investigate

323- Child brought with hx of jelly like stool, pt stabel not obstructed,
next managment:
a) NGT and keep NPO
b) warm enema under US
Correct answer is B
NB: Stable or not start with IVF and NGT
Resuscitate by IVF & NGT
Barium reduction if failed repeat if failed three times > operation
signs of peritonitis > operation

324- Child brought with hx of jelly like stool, pt stabel not obstructed,
next managment:
a) NGT and keep NPO
b) warm enema under US
Glory team August

Correct answer is B

325- hx 4yrs groing breast mass, 15*15 cm , Radiology


cysts and .... ” next managment:
a) Simple mastectomy
b) Modified radical mastectomy
c) Neoadjovant chemo
Correct answer is A
NB : dx is phyloid tumor

326- Q How to teach children about poisoning?


give daily lectures about the dangers of poisoning ✅
NB : if there is awareness activities, choose it

327- Musclar pain relived by drinking cold milk:


High : Ca2.83 ( N 2.21-2.6) , PTH
Low: Phosphate , Vit D
All lab with high border line except vit d with very low value :
a) Milk alkali syndrome
b) Primary hyperparathyroidism
c) Secondary hyperparathyroidism
Correct answer is B

327- Q pseudo pancreatic cyst, 3 weeks observation but still


symptomatic and enlarging:
A. percutaneous drainage
B. Endoscopic drainage
C. Excesion of pseodocyst
Correct answer is B

328- SLE pt came with arthiritis, Ttt:


Glory team August

a) hydroxychloroquine + cyclophosphamide
b) hydroxychloroquine + azathioprin
c) hydroxychloroquine+ methotrexate
D) hydroxychloroquine + drug that I forgot
Correct answer is C
NB : SLE ; skin + joint
TTt by Hyrdoxychloroquine + steroid or metho

329- pt with PCOS not cooperative during examination, “The pt was


agitated”
A. Mood
B. Depression
C. Behavioral
Correct answer is C

330 - Smoker, DM, HTN , uncontrolled, one leg palpable femoral puls but
canʼt for poploteal and distal , other leg only canʼt palpate the distal,
managment:
a) Femoral thrombectomy
b) Below knee ambutation
C) something endovascular

331- hrs Post thyroidectomy, pt SOB, resp distress , neck swelling and
pressure symptoms, ttt:
A. Surgical evacuation at bed
Correct answer is A

332- 30ys old , 3mo hx or progressive lower limb swelling “non pitting
edema”, invex :
A. CT angio
B. Convencive venogram
C. Venous doppler
D. .Venography
Correct answer is A
Same Q with Lymphoscintigraphy instead of CT angio
NB :choose lymphoscintigraphy if not in choices then ct angio or mri angio

333- DM pt , controled, with Ingroing nail , pre op:


A. Examin another toes
B. Culture befor
C. ABx
Correct answer is A
334- Mass one cm from anal verge, extended 3 cm cephalic ,
managment :
a) Chemo
b) Low anterior resection
c) Total colectomy with iliostomy
d) Abdominoperineal resection
Correct answer is D

335- Best way to confirm vaginal psoriasis


A. Biopsy
B. Iodine something
Glory team August

C. Clinica
Correct answer is A

336- Post traumatic facial Profusely bleeding through mouth Best way to secure airway:
A. Intubation
B. Trace
C. Mask
Correct answer is A
NB: Hematoma suction first to mentain airway & to insert intubation

337- Ectopic pregnancy


BHCG 5000, live faraway form the hospital and her husband is traveling
US confirmed ectopic pregnancy:
Whatʼs the most important indication to treat her surgically ?
A. Bhcg
B. Us results
C. Her social status
Correct answer is C

338- 71- Best for diagnosing intussusption


A. Us
B. Barium enema
C. Barium meal
Correct answer is B

339- Baby abd distended what is the first inx


A. Xray
B. Ct
C. Us
Correct answer is C
NB: If US useful start
In adult I agree x-ray

340- Post op comlained of seroma and serous discharge from wound


what to do?
A. 1.Change dressings regularly
B. 2. US drainge
C. 3. Explratory lapratomy
D. 4. Abx
Correct answer is A

341- Post op pt had abdominal distention+feculsmaterial was coming out


from the wound and he developed peritonitis
what is the mgt?
A. 1. Exploratory laparotomy
B. 2. IV abx
C. 3.
Correct answer is B
NB : B then A
Glory team August

Most important > laparotomy


Initial or next > abx

342- Mother complaining that her 10 w old child is “spitting up” all of his
formula weight
His wt at birth is 3.5, now 5.5 kg
What is your next step:
A. 1. Referal for gastroenterology
B. 2. Immediate pyloric stenosis workup
C. 3. Educate the mother to make the head elevated during feeding
Correct answer is C

343- Non bloody diarrhea in a child (same case in school)


what to do?
A. Observe
B. Abx
Correct answer is A

344- Female smoker on ocp for 6 years came for surgery ovarian
cystecomy during surgery pt develop tachycardia hypotension
hypoxia ?
A. Hemorrhage
B. Pe
Correct answer is B
NB: Smoker + ocp
Sudden hypoxia > PE

345- Dm pt had nephropathy what the urine finding?


A- Hyaline cast
B- Rbc cast
C- Proteinuria
Correct answer is C

346-
C- diat
D- behavior therapy (mentioned that she is not happy)
Answer is B
347- Inguinal canal contents in female =
A. illioingunila nerve
B. Round lig
C. Broad lig
Correct answer is B
NB : In male > spermatic cord
Female > round ligament
348- Thoracentesis :
Glory team August

A. between 7-8 intercostal spaces


B. 8-10
Correct answer is B

349- Sick euthyroid syndrome:


A. ⬆ rT3 (reverse T3)
Correct answer is A
350- Testicular swelling case of no cremastric reflex - torsion - next step =
A. surgery
Correct answer is A
NB : Conditions if it was clear by history & examination > OR without imaging
Appendicitis
Testicular torsion

351 - another one is with pain and swelling next step is


A. Doppler✅ or culture & urine analysis

352- case of undescened testis and ptn was 5 yrs next step-
A. remove at puberty
B. Now orchiopexy
Correct answer is B

353- L. monocytogenes is a Gram-positive, non-spore-forming, motile, facultatively anaerobic, rod-shaped


bacterium. It is catalase-positive and oxidase-negative, and expresses a beta hemolysin =
A. Ampicillin
Correct answer is A

354- Congenital pda + eye :


A. Rubella
Correct answer is A

355- Eye infection swelling etc. ( PROPTOSIS) is


A. orbital cellulitis
Correct answer is A

356- Vaginal discharge shows Hyphea -


A. Candida
Correct answer is A
NB: *Vaginal discharge microscope analysis :-*
Glory team August

*Pseudohyphae or hyphae* > Candida, treated by fluconazole.


,

, *Flagellated, protozoan, parasite* > Trichomonas, treated by metronidazole & for (husband also)

, *Clue cells*(epithelial cells ) > BV, treated by metronidazole

357- Multiple sclerosis i choose which is common with “ optic neuritis. :


A. Female
Correct answer is A
358- Jia with ANA ve affecting both knees = symmetrical
A. Oligo
B. Poly
Correct answer is B

359- Sclerosing cholebgitis - associated with Ulcerative collitis - dx by


ERCP
MRCP
Correct answer is B
360- Murmur change with position always systolic :
A. Innocent murmur
Correct answer is A
361- Minimum minutes for effective aerobic exercise
A. 10
B. 15
C. 20
D. 5
Correct answer is B
362- Patient with hemolysis picture
Blood amear given what investigation to do : hereditary spherocytosid
A. - osmotic frag test
B. - electrophoresis
C. - dont remember the rest
Correct answer is A
Glory team August

363- Pedia patient with abdominal mass ( around the kidney) , and some lung infiltrates or something..
A. Wilms
B. Nueroblastoma
C. I dunno the rest
Correct answer is A

364- Ortho xryay hip after trauma dx ( i saw femur neck fracture)
A. Femur neck fx
B. Slipped capital femoral
C. DDH
Correct answer is B
NB : Trauma + obese + boy = SCFE
If elderly, neck fracture

365- case of MI , received streptokinase, then complicated with ( i think bleeding or hemorrhagic shock ) ,
what is the most Appropriate antidote?

A. aminocaproic acid
Correct answer is A

366- 151-Around 40yrs menopause use progesterone to regulate his period , whatis the Risk factor of
Osteoporosis:
A. Age
B. Progesterone
NB : Answer here should be menopause or early menopause or low estrogen

If not in choices so Age


they MAY mean age for menopause, because it’s too early

367-Pt with muscle cramps polyuria polydipsia , ,PB 90/60 , labs show hypercalcemia and high parathyroid
hormone , how to manage ?
a-Furosemide
b- normal saline
Correct answer is B

368- 8 and half years female came with breast buds sparse pubic hair , vaginal bleeding , what is the Dx ?
a- normal development ✅
Glory team August

b- precocious puberty
Correct answer is A

NB: 8 to 13 normal devolpment In girls , Boys from 9 to 14

369- 7 years old female with only pubic hair , no breast buds , no enlarged clitoris , every thing is normal ,
what is the Dx ?

a- precocious puberty

b- normal development

c- premature adrenarche

Correct answer is C

370- 4 or 6 years child female came with breast buds ,pubic hair , vaginal bleeding , other symptoms of
puberty , asking what type of ovarian cancer dose she have ?

a- granulosa cell tumor

b- teratoma

c- sertoli leydig cell tumor

Correct answer is A

NB: Sertoli leyding present as young patient with a combination of virilization, elevated testosterone levels
and ovarian / pelvic mass on imaging studies

371. mother bring her child female with vaginal bleeding and red congested edematous painful vulva , during
the last week she passed stool without control but she already toilet trained , what is the cause ?

a- child sexual abuse

b- foreign body

Correct answer is A
372-sexual abuse , the hymen perforated at?

A- 4 o clock

B- 5

C- 6

Correct answer is C

373- gluconeogensis increased in ?


Glory team August

a- diabetes mellitus

b- hypothyroidism

Correct answer is A
NB: hypothyroidism can lead to decreased hepatic gluconeogenesis and glycogenolysis.

374 what is the oral treatment of choice in acne rosace (no doxycycline in choices ) ?

a- clindamycin

b- erthyromycin

Correct answer is B
NB: First line as oral doxy

Second is macrolides

375 old age patient came with back pain , labs show :(low albumin , Ca slightly high , ALP high , creatinine
high , IgG very high , IgA low , IgE low ) , what is the Dx ?

a- multiple myeloma

b- monoclonal gammopathy of unknown origin

Correct answer is A
Glory team August

376- old age with swollen tender worm right hip and point tenderness , range of motion was ok but painful ,
there is No Fever , Dx?

a- septic arthritis

b- trochntric bursitis

Correct answer is

NB: Trochnturic bursitis can cause pain on outer side of hip, thigh and even the buttock. Pain well increase
with activity such as getting up from chir or from car .. the pain will also increase when lying on effected side +
when you press the hip

377- patient came with knee pain and swelling at ER , aspiration was cloudy and high WBCs , what is the
management ?

a- surgical drainage and IV Antibiotcs

b- IV empirical antibiotics for 14 days

Correct answer is B
Glory team August

378- pregnant female came with lower abdominal pain and back pain , other features of pre eclampsia , Labs
show : (uric acid high , platelet 70 K ), asking what is the feature of sever pre eclampsia in this case ?

a- abdominal pain

b- uric acid

c- platelet count

Correct answer is C

379- You are taking history from old patient , you could find answers because he was talking about irrelevant
things , what to do ?

a- ask him open end questions

b- control the interview with direct questions

Correct answer is B

380- post menopausal woman with high ALP and fractures normal Ca , what is the treatment ?

a- bisphosphonate

b- estrogen receptor modulators

c- Ca

Correct answer is A
381- which of the following is high risk to happen in post menopause ?

a- osteoporosis

b- endometrial Ca

c- ovarian Ca

Correct answer is A
382- old patient came in ER with sudden SOB and chest pain require intubation , social and drug Hx unknown
, chest xray shows enlarged heart and pulmonary congestion or edema , ECG : tachycardia and AF no st
changes , what is the investigation ?

a- Echo

b- spiral Ct or Ct angio ( not sure which one was in the choices )

c- V Q perfusion scan

d- coronary angiography

Correct answer is A
NB: Acute decompensated Heart failure so ECHO ,Bcz pulmonary embolism not presented with cardiomegaly

383- 17- elderly have change in bowel habits then he take fiber in diet to overcome this change in his habit ,
then he feel fatigue , he did CBC before one year it was normal , what is the cause of his fatigue ?

a- change in bowel habits


Glory team August

b- change in his diet

c- his age

Correct answer is B
384-what the recommended screening for dyslipidemia ?

a- fasting lipid profile

b -fasting cholesterol

c- LDL

Correct answer is B
385-- old patient with sacral ulcer reaching muscle and fascia , what is the stage ?

a- 1

b -2

c- 3

D-4

Correct answer is D

386 - at which week alpha fetoprotein done to pregnant woman?

a- 13

b- 15

c- 17

d- 19

Correct answer is C
NB: IT BETWEEN 15-20
Glory team August

Corrected 20 august

1. case of ectopic 2*3 cm and Bhcg 2500 she lives 70 km away and her husband in
a mission ?
A. Surgical Admit her
Glory team August

2- Nurse bitten by HBV pt she is already vaccinated what should she take ?

A. hbv booster
B. hbv immunoglobulin
C. hbv vaccines again
D. RABIVEN for 4 weeks.
Correct answer is B
Note: Hepatitis B postexposure prophylaxis: Nonimmunized individuals requires both vaccination and
immunoglobulins. Hepatitis B–immunized individuals and those exposed to hepatitis C do not require any
postexposure prophylaxis.

3- 8 years well toilet trained come with nocturnal enuresis which muscle responsible?
Detrusor

4- at what age does nocturnal enuresis concern you?


a. 5
b. 6
c. 7
d. 8
Correct answer is D

5- RTA pt came to ER hypotensive?


A. IV fluid
B. send him to imaging
C. give morphine
Correct answer is A
6- case of sudden infant death no signs of abuse what you look for?
A. social status
B. drugs mother take during pregnancy
Correct answer is B
7- breast milk rich of which immune globulin?
a. IgM
b. lgA
c. IgG
d. IgE
Correct answer is B
8- patient sensitive to egg which vaccine he shouldn’t take?
a. Yellow Fever

11- 3 cases about atrophy vaginitis .. same presentation but he ask about diagnosis.. treatment.. cause
Estrogen

12.Q about Late decelerations ctg what acid base imbalance for the fetus
Met acidosis

18.16 month previously well and term with family m develop sudden respiratory distress ( foregone body )
what did U show in CXR ?
Glory team August

● Focal hyperlucency and reduced pulmonary markings of the affected lung


● Atelectasis

20. intussusception important step in management?


Resuscitation

22. of hernia , inferior lateral ?


A. femoral

24. (tumor lysis syndrome) pt with sign of malignancy ?


A. hypoCa

25. Q Iron toxicity?


A. IV def
B.Hypcalcemia
Correct answer is A

26.2Q colorectal cancer one is the age of screening ?


A.> 50

27.Pertusis vaccine immunity for how long?


20 years
It needs double checking

28. normal pregnancy at which level? ‫ﺤﺪد أي اﺳﺒ√ع‬Ç ‫ﺪون ﻣﺎ‬º


1 between umblicus and symphysis pubis
2 at level umblicus
3 just above symphysis pupis
4 just above umblicus
Correct answer is A
29. Pregnant with 10 week, history of gallbladders stone, now complaining of recurrent attack for 5 weeks.
When to do laparoscopic cholecystectomy?
a. now immediately
b. after birth
c. 2nd trimester
Correct answer is C

30.Q 2y, with unilateral breast enlargement ?


US pelvis

31.Preterm deliver by c/s developed respiratory distress after 30min ?


RDS

32.went to travel to area endemic for malaria prophylaxis?


● If chloroquine-resistant P. falciparum (most malaria endemic regions): atovaquone-proguanil,
doxycycline, mefloquine
● If chloroquine-sensitive P. falciparum: chloroquine
33.Cerebral perfusion pressure ?
A.MAP-ICP
Glory team August

34. endometrial 13mm thick ?


A. endometrial biopsy

35. IBS not responding to medication what's best managment?

36. 2Q diagnosis of turner syndrome ?


- Short neck short stature htn —- turner

37.Molar pregnancy ?
A.complete molar

38.Q post operative pt worse , I think Xray show something forgetin in abdomen , open the abdomen (
infected guze ) what to do > tell the pt

39.Eclampis ?
A.mgso

41.Beta Blocker antidote ?


A.gluc

42. pertussis -wat to give his brother ? Both are vaccinated


a-no need prophylaxis
b-booster tdap
c-booster only for high risk
d-macrolide prophylaxis
Correct answer is D

43.wilson tx pencillamine

44.streptikinase antidote ->aminocaproic acid

45.pt -signs of infection given empirically cloxacillin -culture showed stph aureus ?
a. change to rifampcin
b. continue same tx
Correct answer is A

49.case of condylmata accuminta?


A.HPV

50. breast feeding witb hbv +ve -?


A.encourge breastfeeding

51. husband hbv +ve ?


A.use condom

52.female lower abdominal pain and rt subchondric -adhesion in pelvic and hepatic area cause?
A. chlamydia ??

53.elderly htn dm ulcer big foot with pearly color(mostly premalignt ?) ?


A.excision
Glory team August

54.pt had cancer family ask dr to give him a high dose of analgesic but dr say it will had adverse effect! What
kind of something he did ihave no clue about this ?
A. totality
B. duble effect
C.informed.
Correct answer is C
Dm with uti what antibiotic contraindications ?
Diabetic, with UTI, creatinine high. What antibiotics is contraindicated?
a. Bactrim
b. Cipro
Correct answer is A
57.Q Father didn't vaccinate his child ?
A. respect
B. explain the benefits of vaccine
Correct answer is B
58.pedia , white patch or coat in tonsillitis > complicat to pharingitis ,
glomeriolonephritis ?
A.scarlet

59.Typical chest pain with trop 10 st depreseen in ant lead dx


NSTEMI

60.prophylaxis of maningitis for his brother ?


A. rifampicin

63.Pt with jundice an abdominal pain with high Liver enzymes what is the worst outcome?
a-Albumin level
b-alanin aminotrans
Correct answer is A

64.Worst prognosis or Risk factors for graves ophthlmopathy ?


A.male
B.no response to thionamide
C.tsh level
NOTE: Smoking or female
65.How to prevent airbag injury for children?
A.Restrained back seat

66.Army solider ,walk long distance has foot pain ?


A.Maybe spring injury

67.19 y with DVT , what to ask ?


A.history of OCP

68.Pregnant came with abdominal pain no effecment no cervix dilation what is dx ?


a. False labor
b. Active labor
c. Apruptio placenta
Correct answer is C?
69. Patient post-peptic ulcer perforation. Duration of PPI given?
Glory team August

a. 24hrs.
b. 48hrs.
c. 72hrs.
Correct answer is C
70.70 yrs old man k/c of IHD, central abdominal pain vitally stable, amylase 600, wbc normal, abdominal
xray: dilated small bowl with thickened wall: what’s the dx?
A. Pancreatitis
b. Intestinal obstruction
c. Perforated ulcer
d. mesenteric vascular occlusion
Correct answer is D
74.Neonate with jaundice, mom Rh-, baby Rh+?
A.maternal abs to fetal RBc’s
Correct answer is A

75.38 yrs old male sx of perforated peptic ulcer disease, what to do next?
A.Erect CXR
Correct answer is A
75.Know the physical findings of cystic fibrosis?
A. gynecomastia (not sure)

76.PCOS wants to get pregnant, initial mx?


A.metformin with weight reduction a
B. clomiphene.
Correct answer is B
78.US findings of PCOS?
A.pearl necklace appearance
Correct answer is A
79.What’s the underlying case of infertility of PCOS ?
A.ovulatory
Correct answer is A
80.Asthamtic on SABA what to add ?
A.ICS
Correct answer is A
81.Another Asthma q she’s on SABA and LABA and ICS, she’s been fine for a year what to do?
A.taper down ICS
Correct answer is A
84.Man with solid thyroid nodule no thyroid function test given?
A.thyroid scan
Correct answer is A
85.Ectopic thyroid mx?
A.reassurance and follow up.
Correct answer is A
Glory team August

86.Mallory-wiess tear endoscopic findings ?

88.Post-gastrectomy patient excessive vomiting with low mood what to do next?


A. Reassurance.
B.Psych evaluation.
Correct answer is A
90.Brucella diagnostic test?
A.tube agglutination test
Correct answer is A
NOTE:
● culture of bacteria from blood or bone marrow
● detection of immunoglobulin G (IgG) and IgM on serology

91.Patient with persistent vomiting for 2 weeks what’s the associated acid-base disturbance?
A.met alkalosis with compensation or without compensation?

92.Complication of strep throat infection?


A.scarlet fever

95.59 yrs old postmenopasual came with vaginal bleeding, endometrial thickness = 15mm?
A. Endometrial biopsy.
96.Same scenario other question what’s the dx?
A.endo Ca

97.Same scenario other question what to do next? No bx in answers


A. Admit the patient.

98.24 yrs MVA conscious and alert ABC’s done, what to do next?
A.Abdominal US

99.Case of abruptio placenta unstable what to do next?


A.transfusion
B.admit for observation
C.Activate rapid response and employ multidisciplinary team
Correct answer is C
Glory team August

101.Status epilepticus: what to give after lorazepam?


A.phenytoin
Correct answer is A
102.Triad of normal pressure hydrocephalus?
gait disturbance, urinary inc, dementia.

104.Abdominal pain, amenorrhea. Vaginal examination normal what to do next?


A.B-HCG
Correct answer is A

107.24 yrs old MVA, chest tube inserted, femoral fx stabilized, patient transferred to a tertiary center, on the
way desatted, tachycardia and tachypnea. What to do next?
A. Check chest tube patency
Correct answer is A

109.Child with kawaski disease.(Bilateral Conjunctivitis) Also rx?


aspirin and IVIG

110.Child with vomiting and diarrhea, mild-moderate dehydration. Mother mentioned


similar case at his school. What to do?
A.Reassure
b.ORS.
Correct answer is B

111.Patient underwent colectomy, operation got bloody, patient is actively bleeding, surgeon tried to pack,
his BP is still dropping. Consulted Vascular sx what will the surgeon do?
A.The best answer was remove packing and repair injury.

112.Treatment of ITP?
A.IVIG
Correct answer is A
113.Treatment of HUS?
A.supportive
Correct answer is A
114.Treatment of GBS?
A.IVIG
Correct answer is A
115.6 yrs with femoral fx with 30% angulation ?
A.ORIF with IM nail.
Correct answer is A
116.Child with supracondylar fx no pulses what to do?
A.surgical exploration.
Correct answer is A

118.Femoral fx what’s important before reduction?


A.check distal pulses.
Correct answer is A
119.Postappendectomy, fever, erythematous wound?
A.surgical exploration.
Correct answer is A
120.When to start PAP smear:
A.20-24yrs
Correct answer is A
Glory team August

121.Colon cancer screening:


A.start at 50 yrs old
Correct answer is A
122.Mammogram every
a. 2 years
Correct answer is A
123.Obstructive sleep apnea mx?
A.C-PAP
Correct answer is A
124.24 yrs old medically free female once to get pregnant after 2 years what screening test to choose?
A. Pap-smear
Correct answer is A
125.Barrets with low grade dysplasia?
A.PPI’s
Correct answer is A
126.Male on max dose of PPI’s, partial improvement what to do next?
A. Add C and A (clindaymycin and amoxicillin)
Correct answer is A
127.Patient came for total thyroidectomy medically free only has MVP with regurgitation, what to give?
a. Cephazolin for wound infection prophylaxis (not sure)
b. Amoxicillin for IE prophylaxis
c. Give both
d. No need for any prophylaxis
Correct answer is d
128.Child post-trauma with bleeding from his ear?
A.basal skull fx.
Correct answer is A
129.Female with unilateral swelling and edema; how to investigate?
A.venous duplex US
Correct answer is A
131.Koplik spots?
A. measles
Correct answer is A

132.Most common sites of infant dermatitis?


A.Extensor
Correct answer is A
133.Pt on warfarin i don’t remember why had now TIA what you will give ?
A. Add aspirin
Correct answer is A
134.Sle with skin and arthritis on hydroxychloroquine?

135.question about viral meningitis :most likely cause ?


a. Epstien barr
b. Cytomegalovirus
c. coronavirus
Correct answer is B

136.how to diagnose endometriosis ?


a. MRI
b. Us
c. Endometrial biopsy
Correct answer is c
137.complication of pertussis
Glory team August

a. pneumonia
b. pneumothorax
Correct answer is A
138. baby 6wks presented with mucus like vaginal discharge. diagnosed as mucoid vaginal discharge. what to
do about the mother concern?
a. take swab
b. urinalysis
c. reassure
Correct answer is C

139.neonate develop jaundice after 12hrs of delivery. after 36hrs it gets worse.. blood film
attached.. hb low what investigation to order ?
a. osmotic fragility test
b. g6pd enzyme activity
● It depends in the blood film

140. treatment of tooplasmosis


a. doxycycline + clindamycine
b. pyrimethamine+sulfasalazine
Correct answer is B

141.criteria indicate bad prognosis in acute pancreatitis : (Ranson criteria)


a. Lipase 300
b. amylase 200
c. hematicrit >44
d. albumi>3.5
Correct answer is C

143.question about knee trauma on the lateral side and swelling on medial side which ligament injury
(macmurry test and lachman test negativa)
a. medial meniscus
b. lateral meniscus
c. medial collateral
d. lateral collateral
Correct answer is c

144.old pt with severe epigastric pain radiate to the back and vomiting hx of cholelithiasis.. aspiration made
reveal E. coliinvestigation high lipase and amylase.. what is the source of the bacteria ?
a. hematologic spread
b. bacterial translocation
c. other options
d. Duodenal perforation
Correct answer is d

148.couple came for in fertility counsel.. wife has bilateral tubal fibrosis or blockage. what
to do?
a. tell wife
b. tell husband
c. tell them both
d. tell family
Glory team August

Correct answer is c

149.babe wa s born 5 days ago at home. he presented with thigh brusis. diagnosis
Hemorrhagic disease of newborn
Or
factor X defici

150.pt has fever and headache for 2 days associated with ecchymosis and red urine investigation show plat
123, fibrin 1.1 ( low) diagnosis ?
a. ITP
b. TTP
c. Heparin induced thrombocytopenia
d. DIC
Correct answer is D

151.pt cant put spoon in his mouth after cerebral incidence. where is the lesion?
cerebellum

152.gastric mass 5cm confirmed gastric ca. what to do ?


total gasterctomy
partial gasterectomy
wide excession
● ACCORDING TO LOCATION Treatment subtotal gastrectomy for lesions in distal third of stomach,
total gastrectomy for lesions in middle or upper stomach or invasive lesions;

154.5 yrs female with pubic hair , no clitoromegaly obese, hight above 90 centile ?
1-Testesteron
2-Dihydro.. sulfate
3- 17 hydroxylase
4- Lh ? Or fsh ?
Correct answer is 2

155.Sudden lower abdominal pain, RLQ tenderness , mass on ex , us 6 cm mass?


A. ovarian torsion (my answer )
Correct answer is A

156.Growing pain what to do ?


Glory team August

A.Reassure and tell the mother to relax


Correct answer is A

157.6 yrs child vaccination ?

158.Pt previous 2 abortions , declared D&C , she is pregnant now .... ( forget )
A. asherman’s
B.cervical incompetence.
Correct answer is A

159.Heart block baby found in?


sle mother

160.Pt high B hcg , stable , no sac bleeding on us


Stable tubal pregnancy
Missed abortion
Cervical pregnancy
Ruptured ectopic
Correct answer is A

161.I well give y dx then we will discuss prognosis and management ) what is called ?
A.Agenda
Correct answer is A

162.Meconium aspirations with pneumothorax ?


A.Nitric oxide
B.chest tube
Correct answer is B

163.Man want to go to Sudan what to give for malaria prevention


a. Chloroquine
b. Malaria vaccine
c. Mefloquine
Correct answer is A
164.Female has fibroid 2*3 become 5*6 , endometrial thickens 5 mm
a. Lyeomyoma
b. Lyomysarcoma
c. Endometrial ca
● Correct answer is C
165.Strongest indication of thyroid surgery
a. Eye sx
b. Sx
c. Failure medications
d. Pediatric age
● Correct answer is C
166.Systolic murmur decreases with valsalva ?
A. hcom
Correct answer is A

167.Systolic murmur radiated to carotid ?


A.AS
Correct answer is A

168.Dawn pt with split s2 pan-systolic ventricular hypertrophy ?


Glory team August

A. avsd
Correct answer is A

169.Pt sudden chest pain-episodes of heaematemesis -supraclavicular subcutaneous emphysema?


A.boerhaave syndrome
Correct answer is A

170.Pt 3rd trimester abortion +ve lupus anticoagulant-?


A.antiphospholipid syndrome
Correct answer is A

171.Old pt-fatigue sign of anaemia hb8 , stools with streak of blood -has hemorrhoids stage 2?
A. Sigmoid ca
B.rectal ca
c.chronic hemorrhoid
Correct answer is B

172.Confirmatory investigation for coarctation?


CT angio
Echo
Aortic angio
Correct answer is B

173.Chicken pox case Developed ataxia Can't stand unless by support There is neck rigidity?
A- neurological cerebellum ataxia
B- meningoencephalitis
Correct answer is B

174. amenorrhea after curettage ?


A. asherman syndrome
Correct answer is A

175. after delivery the pt ws unable to breastfeed her child ?


A.Sheehan's syndrome
Correct answer is A

176.Parkinson's disease :
Mask face .
Correct answer is A

177.Old pt come decreases in hight what is inx?


A.dexa
B.lateral lumber and thoracic scan
Correct answer is A

178.75 yo male he can’t stand up 3 mo history after trauma what do you think is trauma ?
A.Falls
b.gunshots
C.burn
Correct answer is A
Glory team August

179.18 yo female come with N/V , abdominal pain,visual Hallucination ?


A.Cocaine withdrawals
B.schizophrenia
C.intoxicating 2 other drug I can’t remember it
Correct answer is A

180.Contraindications drug for asthma ?


A.Beta blocker
Correct answer is A

181.Long scenario about pt had dialysis 3 day ago central line and now came again for dialysis peripheral line
nurse mentioned he had fever and infection in site of previous dialysis site what to do ?
A.culture and iv abx
B.culture iv abx and remove the center line
Correct answer is A

182.Asthma pt take SABA than ich than LABA what to do next ?


A.Ltra
B.observe how the pt take inhaler
Correct answer is B

183.infertile couple the husband has HIV+ve?


A. informs husband
B.info wife
C.info all
Correct answer is A

184.infertile couple the wife had PID ?


A.tell wife
B.tell husband
C.tell all
Correct answer is A

186.Most comes site for mumps?


parotid
Correct answer is A

187.Pap smear every?


3 years
Correct answer is A

188.Most popular screening test?


My answer was colorectal cancer
Or breast if in the option

189.6 months old who has active seizures on anti epileptic drugs, he has dysmorphic features and hypotonia,
can set with support and can’t roll over. What to do regarding vaccinations:
a. change OPV to IPV
b. defer DTaP
c. defer all live vaccinations
d. defer all vaccinations
● Correct answer is B
Glory team August

190.6 months old , born SVD at 35 of gestations with 2 week of incubator, feeding well and developing well
gaining 50 g each day. What import gross motor milestones?
a. set without support
b. roll over from supine to prone
c. roll over from prone to supine
d. stand with support
● Correct answer is A

191.Women at around 10 week of gestation with previous still birth want to get vaccinated
for everything to prevent pregnancy loss
What is appropriate to give her?
a. rubella
b. measles
c. varicella
d. influenzas
● Correct answer is c

200.Most common heart anomaly with Down syndrome?


a. tetralogy of fallot
b. PDA
c. transportation of great arteries
d. cushioned heart something
● Correct answer is d

201.Old Patient around 60 and smoker with SOB, cough and hemoptysis. on examination decrease tactile
vocal on right side + finding of massive RT plurale effusion but trachea and heart are not displaced What
could explain the finding?
● TB plural effusion
● esophageal rupture
● heart failure
● carcinoma on ipslatral side obstructing main bronchus
DOUBLE check it
202.3 months old with feature suggestive of meningitis after LB with negative diplococci
IV penicillin G was started, she has asymptotic sibling (3 years old and 5-6 years old)
with up to date vaccinations
What to do for siblings?
a. admission+ observation
b. Start IV Penicillin G
c. active immunity
d. I don’t remember
Correct answer is C

203.Child confirmed to have pertussis infection, her 6 year old brother was close to her to take care of her.
What to do for the brother?
a. observation
b. boost with DTaP vaccine
● Correct answer is B

209.Ethics Q >> mother had IUFD confirmed by US what to do?


a. tell the mother
Glory team August

b. tell both parents


c. order more tests
● Correct answer is b

210. pt came her husband lately diagnosed with hib.B & just started to take
medications what to tell her?
a. avoid sexual activity
b. she can return to normal activity
c. protective activity
● Correct answer is C
NOTES
Hep B condom
Hep C no need unless multiple partner or + HIV

211.pt has crohns with perforation delivered , what vaccinations will you give to her baby?
All
a. don’t give life attenuated
b. delay life attenuated for 6 m
c. delay life attenuated for 12 m
Correct answer is C

212.9 yrs boy with legs “growth pain” what will you do ?
a. xray
b. reassure
Correct answer is B

214.Plural effusion in rheumatoid arthritis?


a. Hemorrhagic
b. High pH
c. Low glucose
d. Transudate
Correct answer is c

215.TTT of pleural effusion in malignancy?


a. Ultrafiltration
b. chemical pleurodesis
Correct answer is B
216.5 yrs female with pubic hair , no clitoromegaly obese, hight above 90 centile ?
Testesteron
a. Dihydro.. sulfate
b. 17 hydroxylase
c. Lh ? Or fsh ?
Glory team August

● Correct answer is B

217.Growing pain what to do ?


Reassure and tell the mother to relax
Correct answer is A

218.3 months boy with uti respond to antibiotics what investigation to do


a. Pelvic u/s
b. Vcug
Correct answer is A

220.Female smoker on ocp for 6 years came for surgery ovarian cystecomy during surgery pt develop
tachycardia hypotension hypoxia ?
a. Hemorrhage
b. Pe
● Correct answer is B

221.Mother complaining that her 10 w old child is “spitting up” all of his formula weight
His wt at birth is 3.5, now 5.5 kg What is your next step:
a. Referal for gastroenterology
b. Immediate pyloric stenosis workup
c. Educate the mother to make the head elevated during feeding....
● Correct answer is C

222.Post op pt had abdominal distention+feculsmaterial was coming out from the wound
and he developed peritonitis what is the mgt?
a. Exploratory laparotomy
b. IV ab
● Correct answer is A

223.Post op comlained of seroma and serous discharge from wound what to do?
a. Change dressings regularly
b. US drainge
c. Explratory lapratomy
d. Abx
Correct answer is A

Complications of abdominal surgical incisions seroma


Treatment
1-Small or asymptomatic: manage expectantly
2-Large or symptomatic: exploration and drainage,

225.Baby abd distended what is the first inx


a. Xray
b. Ct
c. Us
● Correct answer is A

227.Non bloody diarrhea in a child (same case in school) what to do?


a. Observe
b. Abx
● Correct answer is A
Glory team August

228.Ectopic pregnancy BHCG 5000, live faraway form the hospital and her husband is traveling
US confirmed ectopic pregnancy: What’s the most important indication to treat her surgically ?
a. Bhcg
b. Us results
c. Her social status
● Correct answer is c
229.39 had vaginal itching squamous cell carcinoma in Pap smear . what is Mgt?
a. Total abdominal hystrectomy
b. Radiation
c. Colposcopy or other investigation
d. Chemo
● Correct answer is C

230.Best way to confirm vaginal psoriasis?


a. Biopsy
b. Iodine something
c. Clinical
● Correct answer is A

231.Post traumatic facial Profusely bleeding through mouth Best way to secure airway?
a. Intubation
b. Trace
c. Mask
● Correct answer is A

232.Elderly with reduced arousal, maybe headache... he admits frequent falls but no injur
I think.. no focal neurological deficit, normal skull x ray:
a. chronic sundural hematoma
b. Post concussion syndrome
● Correct answer is A

233.Elderly recently moved to nursing home. Cries in the morning, lost interest in activities, lost appetite and
weight. In the episodes of sadness he has poor memory?
a. multi infarct dementia
b. Depression
Correct answer is B

235.Pericarditis?

ECG, mild symptoms, initial tx?


a. Ibuprofen
b. prednisone
Correct answer is A

ECG, severe constant symptoms, best tx?


a. Prednisone
b. ibuprofen
Glory team August

Correct answer is A

236.Erythema nodosum (painful rash on shins), pneumonia Sx; what's next/investigation?


A.CXR
Correct answer is A
(mycoplasma most likely)

237.SLE
● what to avoid?
A.smoking
Correct answer is A
● what to avoid (drug)?
A.sulfamethoxazole (know the drugs, they change the choices)
Correct answer is A
● developed seizure, what to add?
A.cychlophosphamide
Correct answer is A

238.RA what to add to MTX for morning stiffness?


a. hydroxychloroquine
b. azathioprine
● Correct answer is A

239. URTI then arthralgia, abdominal pain, non-blanchable rash: hypersensitivity vasculitis
a. drug reaction
b. Factor VII
c. Vit C deficiency
NOTES
Missing info and all options wrong
HSP
● a systemic immune-mediated small vessel vasculitis often following an upper respiratory infection
and characterized by IgA deposition and triad of
● palpable purpura
● arthralgias
● abdominal pain

240.Fall, open wound gauze soaked w blood what’s first?


a. A.IVF
b. B.call surgeon
Correct answer is A

241.Infertility 2 years despite trying, normal semen, cervical exam required (?), monophasic basal body
temperature test result, cause is:
a. ovulation
b. Coital (intercourse)
c. PID
d. Vaginal
● Correct answer is A
242.Baby feeding well but with cough I think, right lung infiltration, didn't miss any vaccine?
A. moraxella catarrhalis
B. mycoplasma pneumonia (probably)
Glory team August

C. Strept pneumonia
D. Haemophilus influenzae b
Correct answer is C
___
243.Lady pregnant (I think 7 weeks) with vaginal spotting and previous Hx of miscarriage in the previous
pregnacny. Os closed:
A. threatened abortion
B. normal pregnancy
Correct answer is A
___
244.Pregnant with reactive CTG, 2-3 contractions/10min, 70% effaced, 1cm os open...what to do to confirm
Dx (don't ask me Dx of what, but maybe labor they meant: ?
A. continuous CTG
B. PV exam 2 hours after
Correct answer is B
Note :
Intrapartum assessment
cervical exam to asses for stage 1 arrest
clinical observation to asses for stage 2 arrest
___

245.Sx of HAV (tender hapatomegaly and abnormal LFT), what will be found:
A. HAV IgM
B. HAV IgG
C. HCV
D. HBV ... (not sure about the last choices)
Correct answer is A
_____
246.Mass in vulvar region and filled with mucin , jelly like discharge?
A. Seqaumus cancer
B. Mucinous cyst ✔
Correct answer is B
___
247.parashtesia and loss of sensation in medial leg and foot after varicose surgery which nerve is injured?
A. Femoral
B. Saphenus
C. Obturator
D. Sciatic
Correct answer is B

___

248.Melanoma 2 question both of them next step?


A. Excision
B. Bunsh biopsy
C. Chemo
D. Radio
Correct answer is A
Glory team August

___
250.Boy 15 month age parents said that he cant put 2 word in sentence best to do
A. reassure as its normal variance
B. Reasses at 24m
Correct answer is B
---
252.Pt with hyperpigmentation and tiredness.. (case of adrenal insufficency) best investigation to reach dx?
A. SYNCHAN test
B. Dexamethasone sulression test
Correct answer is A

256.After smoking cessation after how long there will significant decrease risk of coronary heart disease?
A. 1year
B. 2 years
C. 3 years
D. 4 years
Correct answer is A

___
257.Newborn of illegal pregnancy ( scenario of congenital diaphragmatic hernia) surgeon decided to
dosurgery right now, from who you need to take consent ?

No need for consent


__

258.newborn presented to you in ER with Acute meningitis you find that he need Lumpur puncture, and his
parents Rufus? What to do?
Explain to parent then ethical committee /social service
__
260. Pregnant lady need CS but she is refusing, what to do ?
● There is No "Respect her wishes" or "refer her to another doctor" in the choices .
__
261.When to said this is nephrotic syndrome resistant to steroids?
A. 12 day after daily steroid
B. 4 weeks after daily steroid
C. 6 weeks after daily steroid
D. 12 weeks after daily steroid
Correct answer is B
__
262.What is the best predictor of nephrotic syndrome is resolved?
A. Protein in urine
B. Normal Serum albumin
C. Resolve of oedema
Correct answer is A
Glory team August

263. Pt with brochogenic carcinoma with plural effusion sample was negative?
A. transductive exudates due to malignancy
B. Exuductive exudates malignancy
Correct answer is B
---
265.Pt with tachypnea orthopnea.. (Symptoms and signs) of Rt sided heart failure on lab investigations Liver
enzymes are very high abnormal ?
A. Heart failure
B. Renal failure
C. Liver failure
Correct answer is C
--
266.Baby with recurrent infections his brother died three years ago due to septic shock, contraindicated
vaccine?
A. MMR
B. Varicella
C. IPV
Correct answer is B
--

267.Pregnant at 22 weeks gestational age oral glucose challenge test after one hr high after 2 hrs high after
three hrs high.. What is next?
A. Repeat same test
B. H A1c
Correct answer is B

268.14 years girl obese came with her mother she has had regular with pain her mother noticed that there
was significant change of her daughter mood after menarche during menstruation she had acne during
examination she was uncomfortable..? ?
A. Mood
B. Behavior
Correct answer is B
269.13 years with regular cycle complaints of pain during menstruation how to manage?
A. Progesterone pills
B. NASID
Correct answer is B

270.Female with symptoms and sign of PCO metformin was given what is the role of action metformin on this
lady?
A. Reduce insulin resistance
B. Anti adrenergic
Correct answer is A
271.Case of Peritoneal Hematoma management ?
A. Observe
B. Evacuations
Correct answer is A

272.DNR and Family scenario The Childe with Congenital anomalies what to do ? Ethics
A. ignore
B. Respect
B if not ER if ER ignore

274.Case of Peritonitis , during examination of Fluid he found E.coli , Source ?


A. Homogenous spread
Glory team August

B. Duodenal perforation
C. Enterococcus spread
D. From out Side
Correct answer is B
275.Hernia case , Acute Abdominal pain distended , Skin red , the hernia not reduced , X-ray =
Air fluid level what is it ?
A.Obstractional
B.Incarcerated
C.Strangulated
D.Irreducible
Correct answer is C
276.Case of Truma , Bilateral lung infiltration with Petechiae in Chest wall what is it ?
A. Hemotthorax
B. Pneumothorax
C. Pleural effusion
Correct answer is A
277.hepatocellular carcinoma 5*5 management ?
A. Resections
B. Palliative
C. Chemotherapy
Correct answer is A
278.Case of pt on anti TB drug develop increase in liver enzymes ? What to do ?
A . Stop pyrazinamide and re-evaluate LFT after 1 wk
B . Stop all anti TB drugs and re-evaluate LFT after 1 wk
C . Stop izonide and rifampicin and re-evaluate LFT after 1 wk
Correct answer is B
279.Case of women pain in her hand or palm and rest due to over work in keyboard which is affected ?
A. Ulnar
B. Radial
C. Anterior interosuss
D. Posterior interossus
Correct answer is C
280.6 month old infant brought by his parents with history of repeated vomiting; his pulse was (190). He had
dry mucous membranes and sunken anterior fontanel. What is the appropriate volume of
Fluid that should be given initially?
A. Bolus 10 ml/kg of body weight
B. Slow infusion 10 ml/kg of body weight
C. Bolus 20 ml/kg of body weight
D. Oral rehydration
Correct answer is C
281.Saudi couple with Anemia case the peripheral blood smear show Spherocytic ( and she said Auer rode! )
What to do ?
A. HB electrophoresis
B. Immune challenge test
not formulated very well so Osmotic fragile test as the dx is Hereditary Spherocytosis

283-Cushing case ACTH high cotisone high after suppressing what investigation?
A.brain CT
Correct answer is A
284.2 years old sickler had a crisis 3 weeks ago with blood transfusion came for 2nd dose hep A asking when
to vaccinate?
a.give 2nd dose now
b.check hep A titer
c.wait 3 months
Glory team August

Correct answer is A
285.Post hystrictomy type of hormonal replacement?
A. estrogen alone
B. combined
Correct answer is A
286.Pt with DM not hypertensive develped protienurea?
A. start ACE
Correct answer is A
288.child with wheezing and cough on and off poorly responding to bronchodilators
His father is a smoker and they have family HX of asthma what is the best indicator if the child have CF?
A. .poor response to meds
B. .father smoking
C. .family hx of asthma
Correct answer is A
289.Child with Down syndrome what is the most striking feature ?
A. simian crease
B. hypotonia
C. Small toe
Correct answer is B
290.Pt with liver cirrhosis have UGIB what to give before endoscopy
a. .vitamin K
b. .octerotide
Correct answer is B

291.15 months old babbles doesnt follow 2 word comand hearing examination done when he was 6 week old
was normal what to do
A. -refer to ENT
B. -schedule audiogram
C. -reassure parents as it is normal variation
Correct answer is C
292.40 years old female LMP 2 months ago came to er with on/off vaginal spotting she has been trying to get
pregnant for 3 years what is the first step
A. confirm the pt state of pregnancy
B. Ask about infirtelity reason
C. Pelvic US
Correct answer is A
293.Small intestine obstruction did apendectomy 5 years back Xray air fluid level Ct done What
investigations done next?

● SBO:
initial test = x ray
To confirm = CT
Then go for treatment
1-supportive care intravenous fluids
2-treat underlying condition
3-nasogastric decompression and bowel rest

294.Child with fever conginctivitis coryze cough Intercostal resection low O2 sat tachycardic Tachypenic what
is the optimal initial step?
A. intubation
B. O2 therapy
Correct answer is B
294.Best way to delieved O2 in COPD patient?
Glory team August

A. -mask
B. -canula
Correct answer is B
295. Pt with cough and wheeze chest xray normal spirometer normal What to do next?
A. -challenge test
B. -repeat spirometer
Correct answer is A
296.Child born with completele heart block What is the cause?
A. -rubella
B. -DM
C. -lupus
Correct answer is C
297.asherman syndrome what layer of endometriam affected?
Basal layer
298.pt febrile and palpable lymph node with white caot in tonsil, most common complication?
A. Pharyngitis
B. Pneumonia
C. Scarlet fever
D. GN
Correct answer is C
299.Anal fissures with skin tag not respond to drug , want procrdure?
A. Internal sphernctomy
B. External sphrenctomy
C. Anal cuurtage with remove tag
Correct answer is A

300.Pic for cystourthro gram , ask dx:


a. Cystourethra reflex
b. Renal agensis
c. Urethta stone
Correct answer is A

300.Meconium aspirations ?
A.nitrous oxides
Correct answer is A
301.Thuriod nodule ?
A.do FNA (TSH normal)
Correct answer is A
302.adenmyosis?
A. came with Hx of utres surgery
Correct answer is A
303.Neonatal hypoglacemia?
A.give glocuse in central line
NOTES :
Right as Adenomyosis Associated with other uterine diseases such as leiomyomas and endometriosis
304.Palpable gallbladder, with jundice , didn’t mention painful gallbladder, ask about Q?
A. GB cancer
B. Panceratic cancer
C. Cholthisis
Correct answer is B
305.Acute pancreatitis with amyalse is high ask investigatoion>
A. CT abdomen
B. Amalyse urine
Glory team August

Correct answer is A
306.jundice with us show stone inGB and in CBD , tx?
A.Laparoscopic cholecystectomy
B. ERCP
Correct answer is B
307. about renal impariment with arterovenos anastomoses or shunt what is the type of arterovenous?
A. brachicephalic
B. radibrachial
Correct answer is A
308.Post-op pt ... treated with unfractionated heparin ... after five days developed petechia, Normal PT/PTT
>> Dx?
heparin induced thrombocytopenia
Correct answer is A
310.COPD exacerbation, hypoxia, acidosis, hypercapnia 7.99 )) >> management ?
A. face mask,
B. mechanical ventilation
Correct answer is B
311.COPD pt developed , c/m only from left pleuritic chest pain, Pneumothorax 2cm btw chest wall and
pleura ,managemen?
A. needle aspiration
B. chest tube
C. observation
Correct answer is B
312.Pt with left Lateral neck mass (( third triangle just below angle of mandible )) ... U/S thyroid normal and
post.cervical lymph node enlargement , on specimen it’s shows follicular thyroid cells >> dx?
A. thyroid carcinoma
B. ectopic thyroid gland
C. apparent thyroid gland
D. thyroglossal cyst
Correct answer is C
313.7y old female started breast development, and pubic hair start to appears and acne , dx. ?
A. Ovarian tumer
B. central .....,
C. gonadotropin
Answer is Premature adrenarch

314.Type of HBV vaccine ?


A. live
B. attenuated
C. recombinant
Correct answer is C
315.Pt with hx. Of retinal detachment surgery 7 day ago and he is going into surgery.... >> Q. Wt to used
to prevent DVT?
a. enoxaparine
b. pneumatic
c. mixed pneumatic and enoxaparin
d. aspirin
Correct answer is A
317.Pt after head trauma he couldn’t make the spoon reach his mouth to eat ,where is the lesion ?
A. Temporal
B. Parital
C. Occipital
D. cerebellum
Correct answer is D
Glory team August

318.Child after period of illness (( can’t remember is URTI or GI )) developed symmetrical lower limb
ascending weakness ?
A.GBS
Correct answer is A
319.Female, unilateral leg swelling, progressive, pitting edema ,Q. Investigation ?
Initial : Doppler Us
Best : MRV
320.Child with bilateral inguinal hernia , Q.management?
Lap herniotomy
321.Pt. After going into open hernia repair .. developed bulging mass with cough same area Q.management?
Lap
322.Which of the following indicate searching for organic cause in a child?
A. peri umbilical abdominal pain
B.last more than 10 min
C. around morning
D.before sleep time
Correct answer is C
324.Child develop seizure >> Q.treatment ?
A. IV lorazepam
B. intra nasal diazepam
C. rectal diazepam
Correct answer is A
325.Description of a patient with 3 symptoms of SLE >> Q. Wt the fourth symptom of the 11 according to
(RHEUMATOLOGY, SLE organization) to diagnose this pt with SLE ?
type of rash can’t remember
RF positive
hemolytic anemia

326.Mass found in the liver, suspecting HCC >> Q. Wt the most risk for it ?
A. HBV
B. Aflatoxin
Correct answer is A
Should be HBC
327.Child with mass in his left flank, UA shows hematuria >> Q. dx ?
Glory team August

A. wilms tumer
B. neuroblastoma
Correct answer is A

329.Rapidly progressive glomerulonephritis >> Q. Histo ?


A.crescent shape formation
B.IgA deposition
Correct answer is A

330.4y old child came with his parents for umbilical hernia. Wt to do ?
A.reassurance
B.repair
Correct answer is A

331.4months Child with mid-shaft hypospadias, came for circumcision >> Q. Wt u will do ?
A.do it
B.not possible since they will use it for the repair
Correct answer is B
332.Pregnant routine screening with UA suggest infection ?
A. asymptomatic UTI
Correct answer is A

333.Pregnant 28w, with UTI >> Q.which antibiotic can’t be used ?


A. nitrofurantoin
B.fluoroquinolones,
C.amoxicillin
Correct answer is B
334.Pregnant ( early pregnancy ), allergic to penicillin developed UTI >> Q. Treatment ?
A. cephalosporin,
B. fluoroquinolones
C. nitrofurantoin,
D. amoxicillin
Correct answer is D
334.Protonitis case after appendectomy what to do first?
A. -ABx!
B. -exposing surgery
Correct answer is B
Secondary peritonitis Approach: remove the source of infection and treat the underlying cause

335.What to do if pt. Want female dr. To examin?


A.Respect
Correct answer is A

336.Case of GDM! with high post prandial only what to do?


A. Metformin
B. Incline
C. Repet test
Correct answer is C
(should be OGGT)
337.Pt. On insuin, heparin, frosamind after surgry he developed hepr.K and high pt and ptt What medcation
you have to stop?
a. heparin
Correct answer is A
Glory team August

343.Pt with dorsum of the left hand numbness over the thump+index and middle finger what nerve is
affected
A. Radial
B. Median
C. Ulnar
D. Axillary
Correct answer is B
344.10 month baby had umbilical mass reducible with cough what management?
A. Surgery
B. Observation
Correct answer is B
345.6 week pregnant came with mild vaginal bleeding and passing of fetus with open cervix+ tissue around
cervix ?
A. Threatened abortion
B. Missed abortion
C. Complete
D. Inivetable
incomplete
346.15 month boy refused to walk and want his parents to hold him he stepped over toys and twisted his leg
What Dx?
A. Fracture at the end of Rt tibia
B. Fracture at the end of Rt fibula
C. Rt Knee injury
D. Proximal tibial fracture
Spiral tibial fracture
Toddler’s fracture, also called childhood accidental spiral tibial fracture or CAST fracture, is a fracture
unique to ambulatory infants and young children. It is caused by a twisting injury while tripping, stumbling,
or falling. Children usually present limping or refusing to walk.
347.Dm pt had nephropathy what the urine finding?
A. Hyaline cast
B. Rbc cast
C. Proteinuria
D. Hematuria
Proteinuria
348.Antidote of iron ?
A.Deferoxamine
Correct answer is A
349.Research one Q only : If u r going to screen a population for a disease. However, the disease is
considered of low prevalence in that population. So, most of positive people will be?
a. false +
b. true +
c. false -
d. true –
Most likely A

350. px insists to tale HRT but doctor said u will not benefit from it ,what should dr do? –
a. don’t give it
Correct answer is A
351. px dose not want to know the details about surgery from the dr, dr respected her wishes but
anesthesiologists insists ,what to do?
A. Tell her about anesthesiologists wishes
B. look for another anesthesiologists
C. call the ethic commit
Correct answer is A
Glory team August

352.Treatment of Gastrointestinal stromal tumor (GIST)


*Small GIST (< 2 cm)*
Stomach: to be observed, endoscopic removal possible
Other localization: resection
*Large GIST (> 2 cm)* : surgical excision ± neoadjuvant and/or adjuvant treatment with imatinib Palliative
treatment using imatinib, if the tumor is not resectable and has developed metastases.

353. 18 months girl with asymmetric breast enlargement with no secondary characteristics? What inv youll
do?
A. ACTH
B. MRI
C. CR
D. pelvic US
Correct answer is D
354.Copd patient with exacerbation, his ph is 7.2 and his spo2 is high, with will you do initially?
A. reduce oxygen.
B. mechanical ventilation.
Correct answer is B
355.Patient just came back from the Sub-Saharan Africa with fever most likely diagnosis?
A.malaria
B.dengue fever
C.Zika.
Correct answer is A
356.Cancerous phylloid mx?
a. Simple mastectomy
b. wide local excision.
c. radio -chemo
d. Wide local with chemo
Correct answer is B

357.Old patient with gluteal atrophy with confirmed aorto-ilial stenosis, What will you do?
a. brachio-femoral bypass
b. Aortofemoral bypass
c. heparin
Correct answer is B

358.Baby passed stool after delivery, foul smelling, hes not eating and irritable, since then hes passing small
amount of stool that is yellow and smelly. presents with bilious vomiting and significant abdominal
distension?
A. Hirschsprungs –
Correct answer is A

359.3 years old boy came with bloody diarrhea after eating fast food 5 days prior to his presentation. He is
pal and lethargic mother says his face is swollen and hes urinating less. What will you do?
A. Splenectomy.
B. Antibiotics.
C. Observe.

361.Most common cause of disorder sex difference in boys ?


Glory team August

A. Hypospedias
B. Small
C. Pured
D. Cryptochridism
Correct answer is D

364.Measles features (cough. Coryza. Bilateral conjunctivitis) what is the most likely dx + there was rash?
No Measles in choises
A. Rybella
B. Coxsaki
C. Vaccinia
Correct answer is A

365.Brown vomiting after mechanical vent for 7 days?


A. Functional dyspepsia
B. Helicobacter gastritis (my Answer)
C. Esophageal gastritis
Mostly stress related gastritis✅

366.Female stopped menstruation for 8 weeks, came with severe pain followed by severe bleeding for 6
hours, came to ER?
a. Ectopic
b. Threatened abortion
c. Complete abortion
d. Rupture uterus
Correct answer is B

367. How to teach children about poisoning from food?


A.Answer is give daily lectures about the dangers of poisoning
Correct answer is A

369.1 month old doesn’t move (case of cong hypothyroid) ?


A.give thyroxine for life
Correct answer is A

371.epilepsy child on antiepileptic he is having suezure and he has complication and uncontrolled?
A. IPV to OPV
B. Defer DTP
C. Defer all live attenuated vaccines
D. Defer all vaccines
Correct answer is B
372.most common cause of shock in Pedia?
a. Septic
b. Hypovolemic
Correct answer is A

374.Acute salpingitis with suprapubic pain and discharge and pain at fornices

375.want to get pregnant next winter what vaccine to take?


A.rubella
Correct answer is A

pregnant and has a son asks what vaccine to take?


Glory team August

a.influenzas
Correct answer is A

Q pain at anus 3 days doesn’t go continuous with mass 1*1 CM temp 37.
A. Piles
B. Abcess
Correct answer is B

378.female taking progesterone she has PCOS taken for 5 years then stopped what she might have?
A. Osteoporosis
B. Cervical cancer
C. Endometrial carcinoma
Correct answer is C
379.lichen scelerosis increase risk of what cancer?
A.SCC
Correct answer is A

380.one with ascites after drainage asking what med to use after to not have reacumulation?
A. Spironolactone
B. Loop diuretics
C. Thiazide
Correct answer is A

381.pregnant just delivered has endometritis doctor prescribed cefotaxime and clindamycin she started
taking it for 7 days and during the treatment she had abdominal pain with watery diahrea after culture
showed bacteria with enterotoxin how to ttt?
A. Continue same treatment
B. Give metronidazol
Correct answer is B
382.male going for appendectomy and did not find appendix how to fing?
A. Terminal ilium
B. Ileocecal valve
C. Follow tinea coli
Correct answer is C
383.diabetic nephropathy how to meausure
A. Albumin creatinine ratio
B. Microalbuminuria
Correct answer is B
microalbumin is the screening test of choice for diabetic nephropathy.microalbuminuria is to represent an
early stage of diabetic nephropathy. *initial nephropathy*
A 24-hour urine collection is no longer recommended to assess kidney function in diabetics or quantify
proteinuria

384. patient for adrenalectomy what to give preop ?


A. Give hydrocortisone
B. Phenyldiazipine
Correct answer is A

385.after gastrectomy what is the vitamin deficiency ?


A. Vitamin B12
Correct answer is A

386.female MVA, after stabilization after workup she was found to be pregnant when to give Rhogam A. RH
+ve with abdomen trauma?
Glory team August

A. RH -ve with abdominal trauma


B. RH +ve with wrist fracture
C. RH -ve with wrist fracture
Correct answer is A

387.cardiac anomaly in down


A. ASD
B. VSD
C. Coarctation
Correct answer is B
388.UTI cases all want next step all was ?
a. Urine analysis
Correct answer is A

390.pregnant previous CS pregnant with twins twin A is breeach and B is Cephalic what is the
contraindication ECV:
A.Previous CS
B. Fetal presentation
Correct answer is B

391. Woman with seductive look and ask personal questions to the doctor, the nurse does not understand
her. What will you do?
A. shout at her.
B. send her to another doctor.
C. bring a nurse that speaks her language.
Correct answer is C
392.CKD patient best place for fistula?
A. Brachio something
B. Brachiocephalic
Correct answer is B
394.Typical scenario of trichomonas then asked about how youll treat the partner.
A. nothing to do.
B. metronidazole.
Correct answer is B
396. ECG with inferior leads ST elevation, what will you do next?
A. Right ECG
Correct answer is A

397.Patient came for ECV, what is the contraindications?


A. AFI 22.
B. Low laying placenta.
Correct answer is B
398.Patient came to your clinic regarding consent for hysterectomy. What will you do?
Document.
- Explain.

399.Patient for surgery and youre telling him about the details of the surgery and he interrupted you “I don’t
want to know the details” the anesthetist insisted that you must explain to the patient. What will you do?
A. Tell the pt about the anesthetist opinion.
B. Book for another date.
C. Ethic committee.
Correct answer is A

400.Cardiac tamponade invx?


Glory team August

A.Echo.
Correct answer is A

401. Post-op pt treated with unfractionated heparin and after few days he developed petechial rash. Dx?
A. -heparin induced thrombocytopenia
Correct answer is A

Mx of the above case?

403.Sx of GI cancer in small intestine with high 5-HIAA?


A. Carcinoid
B. Lymphoma
C. Gastrointestinal something
Correct answer is A

404.Glomerulonephritis symptoms then I think after one week he developed hemoptysis what’s the
diagnosis:
A. Goodpasture syndrome
B. Rapidly progressive glomerulonephritis
Correct answer is A

405.Hematuria... renal signs plus lung involvement hemoptysis... Results showed anti glomerular basement
membrane What’s the diagnosis:
A. Goodpasture syndrome
Correct answer is A

406.The doctor tells the patient about his diagnosis, prognosis, treatment...
What is this?
A. Reflection
B. Mirror something
C. Summarization
Correct answer is C

407.Lady with spider veins need to remove it cosmetically what do you do for investigation?
A. CT
B. venogram
C. Duplex Venus
Correct answer is C
408.Lady with spider vein I think she wants to remove it cosmetically what’s the treatment:
a. sclerotherapy
b. ablation
Correct answer is A

410.Gastric pain and supraclavicular lymph node what do you do for investigation ?
A.endoscopy
Correct answer is A

411.Patient in the ICU he has hypertension, there is elevated potassium decrease sodium fever i think what’s
the diagnosis:
A. Septic shock syndrome
B. Thyrotoxicosis
C. Adrenal insufficiency
Correct answer is C
Glory team August

412.Car traveling, had shortness of breath and other symptoms of PE he has this S1Q3T3 pattern on ECG
what is the diagnosis?
A. PE
Correct answer is A

413.DVT prophylaxis but the patient has high creatinine level what is the prophylaxis?
A. Enoxeparin
B. UFH
Correct answer is B
414.Long case, Flat T wave on ECG what do you find in urine?
A. High potassium
B. Sodium
Correct answer is A
415.Lady delivered a baby she has UTI they did culture and it was sensitive for ciprofloxacin nitrofurantoin
and sulfamethoxazole what’s the treatment?
A. Ciprofloxacin
B. Sulfamethoxazole
C. Nitrofurantoin
Correct answer is C
416.Chest pain... there is ST depression what’s the diagnosis?
A. Unstable angina
B. NSTEMI
Correct answer is B
417.Patient with mitral stenosis getting pregnant, what is the physiological change that Happen in pregnancy
affecting or leading to heart failure I can’t remember:?
A. Increased RBC
B. Increased stroke volume
Correct answer is A
418.Systolic murmur on the left side with Peaked a wave:
A. PS
B. AS
Correct answer is B
419.X-ray showing radio-opaque mass at McBurney’s point and rebound tenderness... what to do next:
A. Appendectomy
B. CT without contrast
Correct answer is A
420.Patient has a hip problem and there was fluid in his hip what’s next ?
A.Hip aspiration
Correct answer is A
421.Celiac disease what should she avoid:
Barley
Correct answer is A
422.baby with diaper rash and oral thrush. How will you manage him?
A. oral antifungal.
B. Oral and topical antifungal.
C. Topical antifungal
Correct answer is B
423. after chickenpox, kid is clumsy and weak in his lower limps, on examination, power is normal but has
neck rigidity Dx?
A. GBS
B. meningioencephilitis.
Correct answer is B
424.open fracture of fibula and tibia. Mx?
A. debridement.
Glory team August

B. Internal fixation.
Correct answer is A
425. Intussusception after stabilazing the pt?
A. NGT.
B. Surgery.
Correct answer is A
426. Site of ectopic pregnancy?
A. Tube.
B. Ovary.
Correct answer is A
427.medication that reduces mortality in CHF.
A. digoxin.
B. ACEI
Correct answer is B
428.RTA patient with blood on urethral meatus. What will you do initially? Cysturtherogram done.
A.suprapubic cath.
Correct answer is A
429. surrogacy, doctor not sure if its allowed in KSA or not, but he sure that its not allowed in this hospital.
What to do?
● book another appointment and gather info about it.
● Warn them it “might” not be allowed in KSA.
Correct answer is A
430.post cholecystectomy came with perihepatic collection, what will you do?
A. Lap drainage.
Correct answer is A
431. woman everything normal but with dusky color cervix?
NORMAL
Physical Signs Of Pregnancy (CHUG)
Chadwick`sign:
Vagina = bluish discoloration
Hegar's` sign:
Uterus = softening
Uterine enlargement
Goodell's sign:
Cervix = softening
432. boy with tenderness localized to the upper pole of the testis. Dx?
A. appendage torsion.
B. Testicular torsion.
Correct answer is A

Q/scenario of woman post D&C asherman syndrome suspected. Which layer?


a. Functionalis
b. basalis
Correct answer is B
433.woman with Bicornicate uterus, leopard ex: baby heart beats felt on the mother umbilicus, kicking
mainly on the lower abdomen, soft on bi- ischial spine, What is the fetal presentation?
A. Breech.
B. Shoulder.
C. Vertical.
D. Face or brow
Correct answer is A
434. pt with a fib, treatment?
a. Warfarin 2-3
b. Warfarin 3-4
Glory team August

Correct answer is A
435.old patient with CHF and polymyalgia rheumatica. His labs are High wbc, high lymphocytes otherwise
normal. Dx?
A. lymphoma.
B. viral illness.
C. CLL
Correct answer is C

437. Old ptient with back pain, high ALP, t score -2.1 x-Ray, multible vertebral fracture. dx?
A. Osteopersosis
B. Osteomalacia
C. Paget
Correct answer is A
439.Pt smoker develop pneumothorax 3cm aspirated in fallow up normal cxray what your advice
A. Not to fly
B. Stop smoking
Correct answer is B
441.Congenital diaphragmatic hernia after stabilization what?
A. surgery.
B. nasogastric tube
Correct answer is A
442.Young male fall down 4 Metter stable normal examination feeling sever heels pain what to do?
A. pain control.
B. x-ray
Correct answer is A
443.Youg female complain epigastric pain radiate to bake there is palpable epigastric mass Serum amylase
high, ,,dx ?
A.pancreatic psudo cyst
Correct answer is A
444.Lung cancer with back pain no neurological sx next step?
A. MRI
B. MRI with steroids
C. Radiotherapy
Correct answer is B

445.Old Diabetic patient came with swelling(or mass ) 5x6 cm behind the neck with secretions what’s the
Diagnosis?
A. Abscess
Glory team August

B. Furuncle
C. Carbuncle
Correct answer is C
Young Female with sever vaginal bleeding in less than 21 days every cycle , other Young (15) female with
mensturation every 5-6 weeks ?
● Normal variation of mensturation
Correct answer is A
446.Positive reinforcment example ?
A.More tv time
B.Comment focus on what child do right
C.Give the child complicate assignment
Correct answer is A

448.child after truma loss sensation of medial thigh but the muscle is normal?
A. Femoral
B. Obterator
Correct answer is B
449.eldrly came for elective cholycysetcomy and found in system that he was at ICU for MI?
A. Delay surgery after 6 month
B. Do the surgery
Correct answer is A
450.old man with palpable swilling at lower abd and frequent urination , he mentioned it’s hard to initiate
whats # :type of incontinence?
A. Overflow
B. Urge
C. Stress
Correct answer is A
451.moderate croup given epinephrine what else is indicated ?
A.Steroid
Correct answer is A

452.cummonuted tibia fracture after fall from hight with paresthisa given poserior compartment 35?
A. External fixation with elevation
B. Internal fixation with elevation
C. External fixation and 4 compartment fasiotomy
Correct answer is C
454.patient came with secretion coming out his midline incision ?
A. Inspict the wound
B. Explore the woun
C. Change dressing
Correct answer is A
456.hypotheanr atrophy ?
A. Ulnar
B. Median
C. Radial
Glory team August

Correct answer is B
457.female 38 GA with IUfD she ask about the method of delivering?
A. Induce pregnancy to avoid CS
B. Observing
(The patient can decide the method i guess)

458.5year old swimming ,painful pinna with discharge what to give?


A. Amoxicilin
B. Pnumo smth
C. Acetic acid drops and ear wick
Correct answer is C

560.best method to confirm laryngomalcyia ?


A. CXR
B. CT
C. Bronchoscopy
Correct answer is C
fiber optic/flexible laryngoscopy-- Flexible laryngobronchoscopy (FLB) is the gold standard for diagnosis

561.vaginal bleeding , symtrical smth1?


A. MRI pelvis
B. CT
Correct answer is A
564.uncontrolled DM with 1mg metformin?
A. increase dose
Correct answer is A
565.Pand like pain?
A. tension headache
Correct answer is A
566.Age of start osteoporosis screening?
A. 50-64
Correct answer is A
567.Treatment of cerebral toxoplasmosis?
Pyrimethramine+ sulfadiazine or clindamycin if sulfa allergic

568.travel diarrhea to Indonesia, what organism?


a. E coli
Correct answer is A
570.scenario obstructive jaundice treatment?
A.ERCP
Correct answer is A
571.post op splenoectomy ?
A. subphrenic abscess
Correct answer is A
572. beast investigation suspicion breat cancer ?
A. excisional biopsy
Correct answer is A
573.Failure to pass meconium spontaneously within 48 h of life and Symptoms of bowel obstructionFailure to
thrive?
A.Hirschsprung’s Disease
Correct answer is A
574.Rx meningitis
● Note : age < wk: ampicillin + cefotaxime IV OR ampicillin ± an aminoglycoside IV ■
Glory team August

● wk-3 mo: cefotaxime + vancomycin ■


● age >3 mo: vancomycin ■

577.Lateral mallulas ulcer Q , The pt invx ،was HTN , Not DM , around the ulcer dark and the center of the
ulcer is viable?
A. Arterial dopplex
B. Venous dopplex
Correct answer is A
581.Atrophic vaginitis ttt?
A. estrogen cream
Correct answer is A
583.Most accurate test for Asthma dx?
A.Spirometry
b. methacholine challenge test
Correct answer is B
584. Intestinal obstruction symptoms , wt would you do ?
a) Ct abdomin
B) groin examination
C) X-ray
Correct answer is C
585.Child brought with hx of jelly like stool, pt stabel not obstructed, next managment
a) NGT and keep NPO
b) warm enema under US
Correct answer is A
586.Small intestine Tumor, with calcification
a) Carcinoid
b) Lymphoma
c) Stromal
Correct answer is A

587.“ hx 4yrs groing breast mass, 15*15 cm , Radiology‫–ء‬í


õ ‫ﻢ‬Õ ‫ﻪ‬œ‫ﺎن ﻓ‬Õ ‫ﺲ‬à ‫ﺎﻟﻀ ﻂ‬º ‫ ﻣﺎ أﺗﺬﻛﺮ‬، cysts and ....” next
:management?
a) Simple mastectomy
b) Modified radical mastectomy
c) Neoadjovant chemo
Correct answer is A

589.Musclar pain relived by drinking cold milk


High : Ca2.83 ( N 2.21-2.6) , PTH
Low: Phosphate , Vit D
Å Ä ‫ﻞ ﻣﺎﻋﺪا اﻟﭭﻴﺘﺎﻣ‬œ‫÷ ﺟًﺪا ﻗﻠ‬œ‫ﺎن اﻟﻔﺮق ﻋﻦ اﻟﻄﺒ‬Õ ‫ﻠﻬﻢ‬Õ ‫ﻦ‬â‫ﻟ‬
‫ﺔ ﺷﻮي‬œ‫ﺴ ﺔ ﻧﺰوﻟﻪ ﺗﻌﺘ·‡ ﻋﺎﻟ‬ã ›õ‫ دال ﻫﻮ ا‬ õ
a) Milk alkali syndrome
b) Primary hyperparathyroidism
b) Secondary hyperparathyroidism
Correct answer is B

593.Smoker, DM, HTN , uncontrolled, one leg palpable femoral puls but can’t for poploteal and distal , other
leg :only can’t palpate the distal, management?
A. Femoral thrombectomy
B. Below knee ambutation
C. ‫ﺲ ﻣﻮ ﻣﺘﺬﻛﺮة‬Ã ‫ﻠﺮ‬É‫–ء اﻧﺪوﻓﺎﺳ‬ í
õ
Correct answer is C
ABI >Arterial US> Arteriography if Surgery is needed
Glory team August

594.5hrs Post thyroidectomy, pt SOB, resp distress , neck swelling and pressure symptoms, ttt?
A. Surgical evacuation
Correct answer is A
594.30ys old , 3mo hx or progressive lower limb swelling “non pitting edema”, invex ?
A. CT angio
B. Convencive venogram
C. Venous doppler
D. Venography ....
Correct answer is A
‫ﺪال‬º ‫ﺎرات ﻋﺪا اﻧﻪ‬œ‫ﺔ ﻧﻔﺲ اﻟﺨ‬œ‫وﺟﺎ ﻣﺮة ﺛﺎﻧ‬
CT angio = Lymohoscinitigraphy

595.DM pt , controled, with Ingroing nail , pre op?


A. Examine other toes
B. Culture befor
C. ABx
Correct answer is A
596.Mass one cm from anal verge, extended 3 cm cephalic , managment
a) Chemo
b) Low anterior resection
c) Total colectomy with ileostomy
Abdominal perineal resection

3-patient have a history of CKD and complain of knee pain in wich he take paracetamol for it but the pain is
still and he wants stronger medication?
a. NSAID
b. paracetamol+codien
Correct answer is B

5-long scenario of severe preeclampsia and she was given mgso4 why did we give it to patient:
a. to control hypertension
b. to prevent sizure
Correct answer is B
6-pediatric patient taller than 95centile and weight less than 25% and has heart anomaly what is the dx?
a. marfan syndrome
Correct answer is A
Glory team August

7-pregnant woman with yellow milk (clustrum) what to tell her about it:
a. this milk is rich in protein
Correct answer is A
8- patien after MVA in ER with 2 ribs fractured and paradoxical chest movement how to manage?
A. -chest tube
B. -needle decompression
Correct answer is A
first IVF and analgesia
599.garlic order ?
A.organophosphate poising
Correct answer is A
600.pain with defecation ?
A.lateral internal spherinectomy
Correct answer is A
602.worse headache ever ?
A. subdural hematoma
B cluster headache
C. other type of headache
subarachnoid

603.Rt abd. Pain e tender testis ,elevated and horizontal no erthyma or sign of infection ?
A. torstion
Correct answer is A
604.signs of hyprthyroidesim, no fever or infalmation , diagnosis ?
A.hashmoto
B.thyroditis
C.gravis …
Correct answer is C
605.vascular anomaly on check?
A. I think arterio-venaus anastomosis ..
Correct answer is A
606.brother do appendectomy , now he with crohn's and he is afraid to have crohn's what increase the
affect?
A.Smoking
B.Appendectomy
C.Male
Correct answer is A
607. wilms disese ?
A.eye ring
Correct answer is A
608. 2y I think , with breast enlargement ?
A. premature breast enlargement
B. prencoius puberty
Correct answer is A
609.ebola pt went to go ?
A. sign DAMA
B. call security
C. ethical committee
Correct answer is B
610.MRSA on vancomycine developed rash and other symptoms ?
A. stop vanco
B. slow infution of vanco
C. change or reduce the dose
D. change to ce… or
Glory team August

Correct answer is A
611.parkland formula , Lower limbs burn , 70Kg ?
A.first 6 hours 5L and 5L in 2nd 16 hours
Correct answer is A
613.800 study groups 200 afected , 800 groups 100 afected ?
A. 33%
Correct answer is A
614.baby born C/S , meconium stained amniotic fluid , intubated , saturation down to 80% and before was
90% something , no pneumothoracx ?
A. nitrous oxide
B. O2
C. surfactant
Correct answer is A
616.Pedia, 9month on breast feeding , TIBC high , dx?
A.IDA
Correct answer is A
617.12 or 8 y, healthy until when he visit him father and stib mother complaining of abd Destenstion … drink
milk, next in management ?
A. serum lactos level,
B. breath
C. lactose after drink
Correct answer is B
Hydrogen breath test (or HBT) is used as a diagnostic tool for small intestine bacterial overgrowth and
carbohydrate malabsorption, such as lactose, fructose

620.pedia, fever then rash maculopapular in face , mouth then all body ?
A. rubella
Correct answer is A

621.Polyhydrominus with rupture memberane ?


A. cord prolapsed
Correct answer is A
622.loss wt,..,..,(malignancy) , high JVP I think ?
A.adenocarcenoma
Correct answer is A
623.dysphagia solid , esophageal mass in the middle , (malignancy) most common ?
A.squemuse cell carcinoma
Correct answer is A
625.liver cyst , ttt ?
No albendazol in choices
A.percutanious drainage
Correct answer is A
626.pedia, stylomastoid injury during forcepse ?
A.closure of left eye lid
B.left face sensation loss
C.anterior 2/3 tasts
Correct answer is A
631.trauma , 3rd spleenic injury with fluid surrounding?
A.spleenectomy
Correct answer is A
631.leukemic pt on chemo, with fever ?
A. culture blood and urin…. ,
B. culture blood , urine , sputum
C. oral broad spectrum abX culture blood , urine , sputum!
Glory team August

D. IV broad spectrum abX


Correct answer is D
633.Plural effusion , suspected TB ?
A. culture AFB
B. lymphocyte with exudated.
Correct answer is A
634.female pt warriors about she will be die and sit for 15 min , she afraid to go to the doctor, 3 time in last
two months ?
A. panic attack
B. Malingering .
Correct answer is
634. TTG I think ?
A.gluten free diet
Correct answer is A
635. villus atrophy
A. celiac disease
Correct answer is A
636. pt will do thyroid surgery and she afraid?
A. don't
Correct answer is A

637. Q came from hajj, meslas ?


A. airborne
Correct answer is A
638. Rt plural effusion ?
A. Parasternal
B. , 2nd inter costal midclavicular
C. 6th axillary line
D. 9th axillary
Correct answer is C
640. female with mass RLQ , ER request CT what to do?
A. pregnancy tests
Correct answer is A
641. Q Left ventricle hypertrophy confirm ?
A. CK
B. Troponin T
1st ecg and best confirm with echo

643. Q pelvic bleeding, uncontrolled ?


A. packing of pelvis
B. ligation of celiac artery
packing first then infrarenal clamp ?
644. Q elderly with strong cough , stress fracture ?
A. acetaminophen
B. other drug
C. and the rest about surgeries
Correct answer is A
648. Ecg,with history of female complain of SOB and slight tinge of jaundice in eye?
A. digoxin toxicity
Correct answer is A
649. Female had femur fracture internal fixation was done , the suddenly she developed SOB , and chest
pain ?
A.fat embolism
Correct answer is A
Glory team August

651. Patient had car accident came to ER: with hypotension SOB , elevated
JVP , normal equal bilateral air entry what the diagnosis?
A.cardiac tamponade ,
Correct answer is A
the best diagnostic test ?
echo
Correct answer is A
652. Patient complaint of symptoms of mitral regurgitation what is the best investigation?
A. Transesophageal echo
Correct answer is A
654. What is the sleeping hours associated with obesity?
A. <5H
Correct answer is A
655. Recommended daily average activity duration for children?
A.60m
Correct answer is A
656. Pic of rash in lower limbs, with clear clinical scenario pointing to a child has meningitis and mother
afraid that his brother will be affected what to give him?
a. coral rifampin
Correct answer is A
657. A child started to have URTI and his brother has immunodeficiency also his uncle what is the
diagnosis ?
A. Bruton agammaglobulinemia
Correct answer is A
658. Same case above and ask what to give the child?
a. IVIG
Correct answer is A

659. Patient hypersensitive developed cough and he is taking lisnopril what to give instead?
A. losartan
Correct answer is A
660. A female patient complained of infertility for years us showed endometrial 15 mm thickening what
next?
A. endometrial sampling
Correct answer is A
661. Women complains of galactorrhea and absence of period MRI was done for her and found pituitary
tumour 0.7 cm what is the preferred treatment?
A.Cabergoline
Correct answer is A
662. Female has SLE on medication, she comes with UTI , what antibiotics we don’t use ?
A.sulfamethoxazole
Correct answer is A
663. Female complains of green vaginal discharge and itching, u found strawberry cervix, what is the
treatment?
A. metronidazole
Correct answer is A
664. Male complaining of abdominal pain and diarrhoea u found transmural inflammation what is the
diagnosis?
A.crohn’s disease
Correct answer is A
665. Patient complain of epigastric pain diagnosed as PUD due to H. pylori started on triple therapy what
is the method to confirm eradication?
A. urea breath test
Correct answer is A
Glory team August

666. Symptoms of PUD what is the appropriate treatment?


A. triplet therapy ( PPI , calirhtromycin , amoxicillin)
Correct answer is A
667. Elderly with dementia with aggressive behaviour and has conflicts with anybody he deals with, what
is the most likely diagnosis?
A. Frontotemporal dementia
Correct answer is A
668. Child fall down and recover completely, his family brought him to ER he complains of headache and
projectile vomiting what the best next step ?
A. CT
B. MRI
C. LP
Correct answer is A
669. Man with history of recurrent respiratory infections and infertility due to immotile sperm. Chest x-
ray show situs inversus. What it is most likely diagnosis ?
A.Kartagener syndrome
Correct answer is A
670. Patient with a history of acute pancreatitis, Found in CT to have collection in the lesser sac What is
the route of drainage?
A.Percutaneous
Correct answer is A
671. 50 years old female has a vulvural mas what is most likely diagnosis???
A.Squamous cell carcinoma
Correct answer is A
672. Case of early abortion in 1st trimester what cause ?
A.Chromosomal abnormality
Correct answer is A
673. A pregnant lady GA 20 with previous second trimester pregnancy loss , she has cervical
incompetence what is your management?
A.Cervical cerclage
Correct answer is A
674. A Child with knee pain in his right leg and wrest pain in left hand what is your diagnosis?
A. JRA
Correct answer is A
675. Patient come with anal pain , it was reducible now it is not and it’s red , what is your diagnosis??
A. Thrombosed pile
Correct answer is A
677. Most common affected organ in mumps in children??
A.Parotid
Correct answer is A
678. Management of stress incontinence??
A. Kegel exercises
Correct answer is A
679. Child presented to ER with testicular swelling, on examination you foun it’s not painful, it is red and
edema extend to groin ??
A. idiopathic testicular edema
B. esticular torsion
C. epididymitis
Correct answer is A
680. Lady with symptoms of preeclampsia received MgS4, in active labor received Oxytocin as well, with a
pic of fetal bradycardia then recovered on CTG, what is the cause?
A. epidural anesthesia
Correct answer is A
Glory team August

681. A 75 years old known case of DM HTN , had paraplegia and completely recovered now , what is the
appropriate action??
A. Give him anticoagulation
Correct answer is A
683.management of dysfunctional uterine bleeding?
A. ligating
B. D/C
Correct answer is B
684- Baby have brokern his wrist what is the management ?
A. closed fixation
B. open fixation
Correct answer is A
Only angulation and deformity may require an open reduction and internal fixation or external fixation.

687. Patient came from Sudan with symptoms of high grade fever and splenomegaly that comes and ease off,
you did the blood smear and it turned out negative what will you do in this case?
A- do thin smear test
B- repeat thick smear test
C- do the test once his symptoms return active
D- do it for an interval every eight hours for 2 days
Correct answer is B
689 female post-mastectomy who had developed pain neuropathic in the inner chest area, and asked what is
the nerve responsible for this?
A. Long thoracic
B. Internal pectoral
Correct answer is B
690.Diagnosing Alzheimer's dementia ?
A.Brain-imaging tests ( to exclude vascular dementia)
B.medical history
C.physical examination
D.laboratory tests,
Correct answer is B
Diagnosis of Alzheimer dementia: Neuropsychological testing.

693.Baby with milk spitting after feeds :


A.GERD
B.Physiologic gastro esophagus rephlix
C.Eosinophilic esophagitis
Correct answer is A
694.pregnant , heavy smoker,how to help her to stop smoking ?
A. Nicotine patch
B. -behavioral therapy
Correct answer is B
696.Patient underwent orthopedic surgery. 2 days after the surgery he started to complain of swelling,
erythema and tenderness at the site of surgery. Which organism can cause the wound infection?
A. Clostridium perfringens
Correct answer is A
697.contrast Nephropathy ?
A. ATN
B. AIN AGN
C. Pre renal
Correct answer is A
698.Obese male complain of sexual dysfunction normal prolactin low LH FSH testarone MRI 2.5cm pituitary
adenoma what is cause of his sexual dysfunction?
Glory team August

A. Obasity
B. Hyperprolactinemia
C. Non functioning pituitary adenoma
Correct answer is C
699. Osgood what to do next ?
A. reassure
b. x-ray
Correct answer is A
Tx : conservative (rest , ice , NSAIDs)

700.burned female covered in soot, burned around nose and mouth?


A. intubate and 100% O2
B. hyperbaric O2
this questions need more infor but most likely A
*Airway management: intubation and high flow oxygen therapy is indicated if an inhalation injury is
suspected *
700 Burn on the face , GCS 15 he was alert and speaking. the two striking choosies were
a. Elective intubation
b. ICU addmition for 24 hrs
Correct answer is A

701.Female with progressive pain and redness on left hand with Hx of knife prick (was injured byKnife)?
A. cellulitis
B. gas gangrene
C. NEC FASC - ...
Correct answer is A
702.Patient( Fgt his age) with pulsutile abdominal mass, just above the umbilicus,, everything else is normal .
Abdominal X ray,, NO free air under diaphragm, or No air fluid level,, ( Normal x-ray) Next step?
A. Amylase
B. ABG
C. measure Ca
D. Measure Cl
Correct answer is A
Pancreatic pseudocyst one of the ddx of pulsatile abdominal mass .

703. 20 y/o girl feels embarrassed, dizzy, tremulous whenever she stands to answet in the class, best
treatment for her?
A. -Low dose lorazepam
B. -Behavioural
Glory team August

Correct answer is B
704. Lady has hyperthyroidism with compressive symptoms ( difficulty swalloing ) best treatment?
A. Methimazole
B. Pylthiouracil
C. Radioactive iodine
D. Steroid
Correct answer is C
707. A case with tumor and High level of *5-HIAA* in the urine Dx?
Carcinoid tumor
Correct answer is A
708. X ray shows*Egg on side* appearance Dx?
A.TGA
Correct answer is A
709. How to prevent recurrence of VOC in sickle cell patient?
A.Hydroxyurea*
Correct answer is A
710. A case about otitis externa ,, Tx?
A.Topical neomycin
Correct answer is A
711. PID treated with ceftriaxone,, no improvement what is most likely organism?
A.Chlamydia*
Correct answer is A
713.- Child had brain surgery,, presented with polyurea, polydepsia Hypernatremia
Dx?
A.Central DI
Correct answer is A
714. Gullian-Barr syndrome prognosis?
A.complete resolution*
Correct answer is A
715. Elderly with the classic triad of DEMENTIA, GAIT APRAXIA, URINARY INCONTINENCE,, imagining shows
enlarged ventricles,, Dx?
A.Normal pressure hydrocephalus.
Correct answer is A
716. Risk for Nectrotizing Enterocolitis ?
A.Low birth weight
Correct answer is A
717. Child throws a ball and draws a straight-line age?
A. 24 Months
Correct answer is A
717. Patient came to your clinic regarding consent for hysterectomy. What will you do?
A. Document.
B. Explain
Correct answer is B
718. Patient on TCA 30mg evening ,complaining of confusion morning.. what to do?
A. - switch to SSRI
B. - change it from 30 to 10-10-10 mg/day
Correct answer is A
TCA induced anticholinergic delirium .
You should taper TCA don’t stop it suddenly

720.Post-term pregnancy , I think bishop is 5-6 ,what to do ?


A. ROM
B. Ripening
C. Oxytocin
Glory team August

D. C/s
Correct answer is A

721.2ed stage of labour 3h ,station +2/+3 ,how to manage ?


A. Vacuum
B. Cs
C. Others
Correct answer is A
722.Hemodialysis, how to start him on dialysis ?
A. AV fistula
B. Av ..etc
Correct answer is A
723.Case of G6PD “sudden jaundice ..etc fever .. what to avoid ?
A. Aspirin
B. Ibuprofen
C. Paracetamols
D. Amoxicillin
Correct answer is A
725.Female got abortion,her husband is emotionally distressed .. what to do?
A. explore his emotions and discus mx plan
Correct answer is A
727. child highly suggestive of appendicitis , asked which of the following true regarding this case ?
A. happened to 2% only
B. rare before 3 year old
Correct answer is..
http://knowledge.statpearls.com/chapter/usmle%20step%203/26820/
729. women with hypothyroidism Sx what to give her to treat palpitations ?
A. BB
Correct answer is A
731.MC type of hep C?
A. 4
Correct answer is A
732.After taking PPI for H pylori when do you test for eradication?
A. 3 weeks
B. 6 weeks
Amboss: 4-8weeks
1. Stylomastoid injury affect the nerve, what is possible symptom:
a) 2/3 Taste loss
b) Loss of sensation
Correct answer is A
3. Symptoms of GERD + bitter taste, Dx?
a) Esophagitis
b) Acute Gastritis
c) Boerhaave's syndrome
Correct answer is A
4. Child with symptoms of meningitis + papilledema, parents beside the child are worry, what to tell
him to take care about it?
a) visual loss
b) hearing loss
c) renal failure
d) retinal detachment
Correct answer is B
6. New onset of HTN in Pregnant w/o any other symptoms, Mx?
a) Methyldopa
Glory team August

b) ACEI
c) ARBS
d) BBs
Correct answer is A

8. 12 months which vaccines to give?


- OPV + MMR + PCV + MCV/ varicella

9. Status epilepticus in child 30 minute, what to give?


a) Diazepam (or Lorazepam / Midazolam)
b) Phenytoin
Correct answer is A
10. X-ray of volvulus in 50s y/o pt., ask about the organ?
looks like cecum most common volvulus in old age is Sigmoid
11. Pt. done FBG show slightly increase + OGTT 2h read is slightly increase, pt. have polyuria, what to do
next?
a) HA1c
b) Random blood glucose
c) Repeat test
d) Start "medication" / diagnosis establish (I forget)
Correct answer is D
17. Pt. with barky cough, what is susceptible breath sound?
a) All sound in same "situation"
b) Expiratory prominence + prolonged
c) Inspiratory prominence due to narrowing
d) Inspiratory prominence due to mucus filling the upper
Correct answer is C
18. - Pt with vaginal greenish discharge ttt ?
- Metronidazole
Correct answer is A
19. Regarding Dtap vaccine ?
A) will give u life lone immunity against pertussis
B) immunization and immunity against pertussis will wanes within 3 y
C) within 10y
D) within 25 y
Correct answer is C
21. contraindication to liver transplantation?
A- active liver failure
B- active liver something + active alcohol
C- chronic with ascites
D- chronic with encephalopathy
Correct answer is B
33. Pt sudden chest pain-episodes of hematemesis and supraclavicular subcutaneous emphysema Dx ?’
- boerheave syndrome
Correct answer is A
35. Old pt with fatigue , hb8 , stools with streak of blood has hemorrhoids what
stage?
-chronic hemorrhoid
Correct answer is A
Glory team August

July 21 – 2019 collection


1- What the most common type of hepatitis c in Saudi Arabia?
A- 1
B- 2
C -3
D- 4
Answer is: D
Reference: research study “Hepatitis C virus genotypes in Saudi Arabia
https://www.ncbi.nlm.nih.gov/pmc/articles/PMC5667522/

2- CTG fetal bradycardia by epidural anesthesia


3- CTG late deceleration This type of deceleration indicates there is insufficient blood flow to the
uterus and placenta

4- false increase BNP


N.B: Increases in BNP levels may be caused by intrinsic cardiac dysfunction or may be secondary to other
causes such as pulmonary or renal diseases (e.g., chronic hypoxia)
Reference: American Academy of Family Physician

5- Screen of polycystic kidney disease/ PCOS?


A. US
B. polyscystin antibody
Correct answer is A
N.B: the diagnosis of PCOS is typically made using the Rotterdam criteria, which require two of the following
three to be present: oligo- or anovulation; clinical and/or biological signs of hyperandrogenism; and detection
of polycystic ovaries by ultrasound.
Reference: Medscape
6 - advise to DM
A. Low carb best than low fat in long management
B. low fat decrease 5-10% ?
C. low glycemic index decrease 5-10%

7- penicillin allergy what C/I ?


A. Meropenem
B. ceftrixon
C. pipracellin Tazopactam

answer: pipracellin Tazopactam


N.B: pipracellin is penicillin , Beta lactamase inhibitor

8- on 1st time microalbuminurea in alb/cr ratio what is next?


N.B: Not complete

9 - US renal / relate alb-ratio / stop metformin ?


N.B: Not complete

10 - Platue Bhcg after salpinostomy after 3 week


What to do?
A. Reassure and follow up
B. methotrexate
Answer: methotrexate
Glory team August

N.B: salpingostomy, carries a 5-15% rate of persistent trophoblastic tissue,The average time for β-HCG to
clear the system is 2-3 weeks, but up to 6 weeks can be required , Further medical treatment with
methotrexate or surgery in symptomatic patients may be necessary if β-HCG levels do not decline or persist.
Reference: medscape

11- Honey crusted exudate


A. Impetigo
B. boil
Answer is: Imptigo
N.B: in imptigo Rupture of the vesicles with exudation of fluid leads to the characteristic confluent honey-
coloured crusted lesions.
Boil:These are infections of hair follicles or sweat gland

12- Sign of iron overload in every organ, What investigation ?


A. Iron load from liver biopsy
B. Cupper
C. A 1 anti trypsin
D. Antimitochondrial ab

12- Prolactinoma ttt?


Cabergoline

13- Tonsil exudate and cervical LN , what is most complication?


A- Scarlet
B- PSGN
C- Pharyngitis

Answer: Scarlet

14- adrenal mass lipid size 5 what to do


A- Observe
B- lap adrenalectomy
answer: B

15- Fibroid asymptomatic what to do ?


A- Follow up annually US
B- follow up CBC and us every 2 month

Answer: Follow up annually US

16- Ttt of TOF with cyanotic spell ?


A- Sedation and make him stable(I chose)
B- diuritc
C- Cardiac cathetrization
Answer: sedation and make child calm
Reference: Pediatric EM morsels , to read more:
https://pedemmorsels.com/hypercyanotic-spells/

17- ANA -VE when to screen uvitis,


A- 3 mo
Glory team August

B- 6 mo
C- 12 mo
D- No screening
Answer: every 6 months

18- radial injury most site?


A- Spiral grove of hemurs
B- olecranon
C- axillary
D- wrist
Answer: spiral groove on the posterior aspect of the humerus

19- LT stylomastoid injury during delivery of baby ?


A- Cant close LT eyelid
B- loss of ant 2/3 of taste
Answer: B

20- Meningitis with kernigs sign + and low LP glucose


What organism ??
Incomplete Q

21- DKA management with rapid fluid correction, complication?


A- Cerebral edema

22- What you find in urine in diabetics


A. protein in urine
B- Glucose and lipid
Answer= A: Screening for DM nephropathy by microalbuminuria with a spot urine albumin/creatinine ratio

23- Facial palsy.. what decrease duration of the illnes?


Answer: The main treatment to help reduce inflammation and therefore relieve pressure on the facial nerve is
oral steroids
Reference: Facial palsy UK

24- Hyperextended fingur? Wt musle injuried?

25- What do you see in FAST exam?


A. Intraperitoneum
Correct answer is A

26- Infected mesh?


A. Remove
Correct answer is A

27- Dm with well sharp red demarcted in her leg?


A. Erysipelas
B. Lipidco diabetrica
Answer:Erysipelas
N.B: sharp margins

28- Came for back pain with high Ca


A. give opoid
B. isotonic flude
C. alendronate
Glory team August

D. forusmide
answer: C

29- Drug treat osteoporosis and hypercalcmia of malignancy


A. alendronate
Answer: Aldendronare
N.B: Pamidronate and etidronate can be given intravenously, while risedronate and alendronate may be
effective as oral therapy.
Reference: Medscape

16 years old with fever for 6 days, he is lethargic, fatigued, on examination there was large tender
splenomegaly, what is the culture with highest diagnostic yield ? (That is it, no more details)
A) stool and urin
B) single blood culture
C) multiple blood cultures
D) bone marrow aspirate culture
Correct answer is C

What is the most likely fundal height in normal pregnancy? (Didn't specify GA)
A) around smphisys pubis
B) between smphisys pubis and umbilicus
C) around umbilicus
D) above umbilicus
Correct answer is B

Smoker son had his father diagnosed with lung cancer, the son wants to stop smoking after his father was
diagnosed, the son have a Hx of Epilepsy (not sure if epilipsy or just seizure), which of the following smoke
cassation medication is CI in the son's condition?
A) bupropion
C) nicotin replacement stuff
Answers: brupropion
N.B: increased risk for seizures related to bupropion and varenicline
reference: medscape

Patient with primary hyperparathyroidism did lower left parathyriodectomy, 4 months later presented with
constipation (other symptoms or hypercalcemia), what is the underlying cause to current presentation?
A) missed adenoma
B) parathyroid hyperplesia
Correct answer is A

Q: ..HTn ..... IUGR What else will be associated with it ?


a. GDM
b. Oligohydroamnios
c. polyhydrominos
Correct answer is B

Uterus fundus lypmh node ?


A/ para aortic
B/ internal illaic
C/ external ilaic
D/ superficial ingunal
Correct answer is A

contraindication drug after appendectomy


Glory team August

‫ﻞ ﺷﻮي‬ó√‫ﻮ ﻃ‬ó‫ﺳﻨﺎر‬
‫ﺎرات‬œ‫ ذﻛﺮ ﻣﻦ اﻟﺨ‬nitrous oxide

47 year old female came to surgical clinic with Hx of upper abdominal pain, shortness of breath, and
palpitations after meals, the symtomps been there for several years, the symtomps improve by vomiting,
what is the best diagnostic modality?
A) upper GI endoscopy
B) abdomen CT
C) angio
D) plain abdominal film
Correct answer is A

Young man, had a trauma to his 2nd finger while it was hyperextended, the man can't flex or extend the
distal phalanx, also the distal phalanx was tender to touch, but he can move the proximal phalanx freely, he
also reported feeling tenderness in his palm, where is the the injury ?
A) deep flexor dogitorum tendon
B) superficial flexor digitorum tendon
C) distal interphalngeal joint
D) distal phalanx bone
Correct answer is A

CTG graph with VERY low variability (almost flat line), but FHR was about 150 or 140.
No decelerations or any variability.
And the mother showed no symptoms of any abnormality.
Patient on Oxytocin, MgSO4, epidural.
What is the cause between those three?
a. Oxytocin
b. MgSO4
c. Epidural
Correct answer is B

54 y/o male after lap cholecystectomy 5 days develop abdominal distention


Us showed ascites
What is your next management
A) drain of fluid
Correct answer is A

: Absent moro reflex unilateral side q


Unilateral moror reflex absence Indicate: Ipsilateral brachial injury or ipsilateral fractured clavicular

: In case of death of newborn What should you do immediately for the parents?
a. Ask the father if he would like to file a complaint
b. Let the mother spend sometime with her dead baby
c. arrange a close follow up with the primary team
Correct answer is B

What is the precatnage of *Male* smokers in Saudi Arabia?


a. 5%
b. 10%
c. 37%
d. 51%
Glory team August

Correct answer is C

Pap smear done > inflammatory Then she did colposcopy biopsy shows >> invasive carcinoma / what is next?
1- conization of the lesion
2- clinical staging of the cancer
Correct answer is 2

‫ وﺻﻒ ادوارد ﺳﻨﺪروم‬Ëõ Å ‫ﺟﺎ‬


Prader willi syndrome also
1- Pap smear was ASCUS in 34-year-old, what next?
A. HPV test
B. colposcopy
C. repeat PAP smear
D. a wrong answer
Correct answer is B

N.B: Any high-risk lesion on pap smear (high vaginal swab) should be followed by
Colposcopy.

2- Child his older brother died sudden cardiac death when he goes to work???!
A) PDA
B) VSD
C) ASD
D) Hypertrophic cardiomyopathy
Correct answer is D

N.B: HOCM is (AD inheritance), family history of multiple premature sudden deaths; S&S: asymptomatic
(common, screening is important), SOB on exertion, angina, presyncope/syncope, CHF, arrhythmias, SCD;
palpation.
Evidence: Toronto Notes 2018, p. 106.

3- 7y/o diagnosed with T1DM came to ER with DKA and is comatose. Mother said she stopped his
treatment because she doesn’t think it’s true and wants his pancreas to have time to grow. This
represent?
A. Denial
B. Neglect
C. compromising
Correct answer is A

N.B: N.B: Denial = refusing the diagnosis.


N.B: Neglect = aware about Dx but refused ttt.

4- Vaccination for 1 years old baby??

No choices
Vaccines for 12 months: OPV – MMR – PCV – MCV

5- About 28 years old, came to ER with heavy vaginal bleeding, she is nulliparous, pregnancy test
negative, no pain, regular cycle, but always with menorrhagia, how to stop the bleeding now?
A) give estrogen (just estrogen, no OCP)
B) give progesterone
C) give GnRH
D) inserted levonorgestrel IUD
Glory team August

Correct answer is A

6- Type of anima of 2 Y child not eating meat only on cow milk.


A. Also, in scenario says that his brother has the same issue.
B. Thalassemia a trait
C. Iron deficiency

Correct answer is C
7- year old female on OCP for 12 years, developed gradual RUQ pain (not something acute), CT was
ordered, Hepatic adenoma 5 cm diagnosed, how to manage?
A. stop OCP
B. Excision
C. left hepatectomy
D. liver transplant (not sure, but something wrong)
8- Lots of questions about HUS, TTP, ITP.

9- Questions about post-operative complications (appendectomy, gastric sleeve, wound infection)

10- Patient with perforated appendicitis after surgery had pus from wound, pain localized to surgical site.
No guarding no fever what best initial treatment is:
A. Antibiotics
B. Open drainage
C. Imaging guided drainage
D. Wound exploration
Correct answer is D

11- Patient with rheumatoid arthritis taking Steroids +chloroquine +methotrexate with no remission
what drugs to add:
A. Azathioprine
B. Adalimumab
C. Cyclosporine
D. Mycophenolate moetifil (celsept)
Correct answer is B

12- Patient have "hacking cough" .... Forgot the case

13- More than 5 case about low limb swelling, what is the appropriate investigation??

14- 40yo male came with pain in left leg after travail minimal trauma to his hip.

Leg was swollen and tender. X ray shows spiral tibia fracture. What is the dx?

15- Q about BIRAD score 3The options were:


A. Excisional biopsy
B. FNA
C. Core needle biopsy
Correct answer is B
Glory team August

16- calcified hydatid cyst, 9 in 3 cm, no antibiotics in choices. TTT?


A- per cutaneous aspiration
B- surgical Excision
Correct answer is B

17- Patient came to ER with RLQ pain. CT shows he has appendicitis and 5*13 cm appendiceal abscess
that extends from ... To the abdominal wall. What is the best management?
a. Lap appendectomy
b. Lap appendectomy and drainage of abscess
c. Open appendectomy and drainage of abscess
d. percutaneous abscess drainage

Correct answer is D

18- Neonate developed jaundice 12 hours postpartum, the jaundice persisted until 36 hours, blood film
was taken, depending on the blood film, what is the best diagnostic test for the neonate?
A. osmotic fragility test
B. G6PD
C. electrophoresis (not sure if there)
D. I don't remember

19- case of RA with erosions in the x-ra. Most important management (Not next step in management)?
A. Hydroxychloroquine 400mg daily
B. Corticosteroid daily
C. Indomethacin daily
D. Hydroxychloroquine 400 mg + methotrexate … weekly
Correct answer is D
20- Pt underwent hernial repair for right inguinal hernia, presented now complaining of that ipsilateral
testicular size is decreased after hernial repair. Dx?
A. Testicular artery occlusion
B. tight mesh
C. pampiniform plexus occlusion
Correct answer is C
21- Athletes with constant planter foot pain examination showed central planter tenderness
A. Hallux rigidus
B. Hallux valgus
C. Tibial nerve entrapment
D. Planter facilities
Correct answer is C
22- women presented with her infant he constantly spit after feeding normal weight:
A. It’s normal and he will grow out of it
B. Refer to gastroenterologist
C. Teach her to burb her infant and to hold him upright after feeding
Correct answer is C
23- family of terminally ill adult in pain asked the physician to increase pain meds he agrees after
explaining that it could lead to death what this approach called
A. -Informed choice
Glory team August

B. -Totality

24- patient k/c of chronic hepatitis b presented with jaundice and fatigue

Inv: high Trianinases low HB DNA what to do:


a. liver biopsy
b. Anti-delta antibodies
c. interferon
d. repeat test

25- female with ULQ lump increasing with time, size 15x15 cm, us show solid mass with multiple cystic
spaces with posterior costic enhancement, management?
a. neoadjuvant chemotherapy
b. simple mastectomy
c. radiation
d. modified radical mastectomy
Correct answer is D

26- Post MVA 40 male, thigh exposed, blue color, what is bacterial cause?
a. S. Aures
b. C. Perfigens
c. salmonella
27- Female with problem in chest go for mastectomy we will take from abdominal muscle for graft witch
artery is response for it?
A. superior epigastric
B. Inferior epigastric
Correct answer is b

28- ECG AV block


Glory team August

29- CTG: variable deceleration

30- Picture of Skin Tag


Glory team August

31- Chloasma or melasma?


N.B: Melasma (also known as chloasma faciei, or the mask of pregnancy
when present in pregnant women) is a tan or dark skin discoloration.

32- Anembryonic pregnancy:


N.B: Anembryonic gestation is characterized by a normal-appearing gestational sac, but the absence of an
embryo, management (as IUFD) expectant until 9 weeks age, then induce labour with medical or D&C

33- Picture of Urethral caruncle.


Glory team August

34- Picture of Endometrial Polyp.

35- Picture of Hysteroscope

Fetus Present with orbit nose mouth what is fetal presentation?


a. Face presentation
Correct answer is A

Fetus after delivery by ventuze Nurse noted bleeding in suture lines or something like that diagnosis :
A. Caput seccondum
B. Cephalohematoma
C. Popten ( strange word I forgot)
D. Othet choise ( not subdural hematoma)
Correct answer is A

Yellow fever vaccine egg allergies

41yo Soldier came with a complaint bulge in his groin. Px reveals bilateral inginal hernia. Management?
a. Lap with mesh
b. open with mesh
c. carful watching
d. hernia truss
Glory team August

Correct answer is A

Hernia that Inferior lateral to groin?


a. -Femoral
b. -inguinal
Correct answer is A

Q- child w s&s of anemia and splenomegaly lab shows spherocytosis and schistocytosis ?
a. -Hereditary scherocytosis
b. Autoimmune hemolytic abenia
Correct answer is A

Q - adult w apdomin and muscle complain


And like drink cold milk > lab shows high ca and nl phosphate ?
a. milk alkali syndrome
b. Idiopathic hyper ca
c. Secondary pth ,
d. prinary pth
Correct answer is D
5- About 28 years old, came to ER with heavy vaginal bleeding, she is nulliparous, pregnancy test
negative, no pain, regular cycle, but always with menorrhagia, how to stop the bleeding now?
A) give estrogen (just estrogen, no OCP)
B) give progesterone
C) give GnRH
D) inserted levonorgestrel IUD
Correct answer is B

3- Q about BIRAD score 3The options were:


A. Excisional biopsy
B. FNA
C. Core needle biopsy
D. Follow up
Correct answer is D

21- Athletes with constant planter foot pain examination showed central planter tenderness
A. Hallux rigidus
B. Hallux valgus
C. Tibial nerve entrapment
D. Planter facilities
Correct answer is D

Q pt came w abdominal pain like appendicitis sx then Xray shows stone on RIF, what next ?
a. -Ct without contrast
b. Consult urologist
c. renal us
Correct answer is A

26- Post MVA 40 male, thigh exposed, blue color, what is bacterial cause?
a. S. Aures
b. C. Perfigens
c. salmonella
Glory team August

Correct answer is B

24- patient k/c of chronic hepatitis b presented with jaundice and fatigue
Inv: high Trianinases low HB DNA what to do:
a. liver biopsy
b. Anti-delta antibodies
c. interferon
d. repeat test
Correct answer is B
23- family of terminally ill adult in pain asked the physician to increase pain meds he agrees after
explaining that it could lead to death what this approach called
A. -Informed choice
B. -Totality
Correct answer is B
Q bilateral inguinal hernia in adult management ?
a.lap
Correct answer is A

Q acute pancreatitis then has abdominal mass and observed for 3 w What next?
Internal Drainage
IV antibiotic

Q child w asthma (moderate) , tx ?

Q - us shows snow storm?


a. Complete molar
b. Choriocarcinoma
Correct answer is A

Patient has follicular thyroid cancer and did rt hemithyroidectomy, after surgery they found residual
malignant cells missed but not in vascular way or not invading anything, what you will do?
a. Radioactive ablation
b. RAI scan
c. total hemithyroidectomy
d. observation or follow up
Correct answer is C

DM on metformin 1 or 2 gram, fasting and A1C are still both high, what is the management?
a. add glibtadine
b. increase metformin
c. do nothing
d. forget but another class of DM drugs
Answer: add sulfonylurea

Elderly man has right swollen knee and pain, there table compares normal and abnormal labs ( yellow
muocus knee fluid, WBC normal but 80% pleomorphic nuclear cells, diagnosis?
a. septic arthritis
b. gout
c. pseudogout
d. RA
Correct answer is A

‫ﻒ أول ﻳﻮم وﺟﺎ ب‬œ‫ﺮاﺗ‬Ô‫ ف ﺳﺆال ﻋﻦ ﺑﻮﺳﺖ او‬SOB


Glory team August

‫ وﺳﺆال‬MVA, WARM EXTREMTIS, all invest free

‫ﻔﻞ وﻟﻘﻴﻮ‬œ‫ﺪ اﻳﺮ ﻟ‬ó√‫ ﻓﻠ‬٣ ‫ ﺳﻨﻪ وﻋﻨﺪە‬١٢ öõ‫ ﺟﺎﺗ‬Proximal ileum stricture, severe obstruction

Malignant phylloid management?

Pt after cardiac bypass ,developed hypotension tachycardia and decrease SVR? what is the type of shock

1-hypovolemic
2-cardoiogenic
3-Septic if w/ fever
Correct answer is 2

Young female came to the clinic complaining of episodes of SOB, wheeze and cough for the last 6 months not
related to certain provoker. She says that these episodes comes *suddenly* and resolve suddenly. She used
ventolin but improved slightly. Upon physical exam she has equal air entry with no wheezing. She is not
allergic to anything and no one in her family has the same problem.All test were normal and spirometery is
normal.
What is the dx?
a. foreign body
b. vocal cord dysfunction
c. asthma
Correct answer is B
Women cant breastfeeding her child beacause his nipple is invented, we apply pressure to reduce inversion
and we found slit in the nipple ?
A/ cyst
B/ breast ectasia
C/ carnioma
Correct answer is B
Female with family Hx of breast cancer came with breast mass, core Bx showed atypical ductal hyperplasia,
how to manage ?
A) simple mastectmy
B) wire guided excision
C) MRM
D) ‫ﻤﺎن ﻏﻠﻂ‬Õ ‫ﺣﺎﺟﺔ‬
Correct answer is A

Which type of food rich in potassium?


A / green beans
B/ tomatoes
Correct answer is B
When start screening for osteoporosis on women ??
a. 55-60
b. 65
Correct answer is B

bactarial vagnosis ‫ﻪ ا˙˚‡ ﻧ√ع‬œ‫ اˆﺶ ﻧﻠ˘¯ ﻓ‬cell ??


A/ gaint cell
B/ lactobacillis
C/ granular epethlial cell
‫ﺎرات‬œ‫¨ اﻟﺨ‬õ Å ‫ﺎن ﻣﻮﺟﻮد‬Õ ‫ ﻣﺎ‬clue cell
Correct answer is C
‫‪Glory team August‬‬

‫ﺟﺎن ﺳﺆال ﻏ‪óñ‬ﺐ ﻋﺠ‪œ‬ﺐ ‪:‬‬


‫‪Ventral hernia repair the mesh placed‬‬
‫واﺷ‪œ‬ﺎء ﺛﺎﻧ‪œ‬ﻪ ‪ sub‬واﻟﺨ‪œ‬ﺎرات وﺷﻠﻮن ﺗﺤﻄﻪ ‪º‬ﻪ‬

‫‪ hps‬ﺟﺖ ﻛﻴ¸ﺲ ﺻﻮرە ﺑﺰر ﻋﻨﺪە راش وﺟﺎ‪Ç‬ﻪ اﺳﻬﺎل ‪:‬‬


‫‪ laryngmalacial what the most dx test‬ﺟﺖ ﻛ¸ﺲ‬

‫ واﺣﺪ ﻣﺎﺧﺬ ﺣﺪ‪Ç‬ﺪ زود ﺗﻌﻄ‪œ‬ﻪ اﻻﻧ ˘ ‪ ˇõ‬دوت ﺣﻘﻪ‬


‫ﺟﺖ ﻛ¸ﺴ ‪Å Ä‬‬
‫اﻋﺮﻓﻮا اﻻﻋﺮاض ﺣﻘﺎﺗﻪ ‪pulmonry hypertention‬ﺟﺖ ﻛ¸ﺲ‬
‫– ‪º‬ﺎﻟﻼب ﻣﺮﺗﻔﻊ وﻫﻮ ﻃﺒ‪Polycetmia vera ÷õœ‬‬ ‫‪í‬‬
‫˚‬ ‫‪Õ‬ﻞ ‪õ‬‬
‫– ‪Ç‬ﻌﻄ‪œ‬ﻚ اﻟ‪#‬ﺸﺨ‪œ‬ﺺ ‪ aplastic anemia‬ﻛ¸ﺲ‬ ‫‪í‬‬ ‫‪í‬‬
‫– ﻋﻨﺪە ﻗﺎل وﻃﺒ‪ ÷õœ‬وش ا˙‡ ‪õ‬‬ ‫‪Õbone marrow aspiration‬ﻞ ‪õ‬‬
‫ﻛ¸ﺲ ﻋﻦ ﺟﺪول اﻟﺘﻄﻌ‪œ‬ﻤﺎت ﻋﻨﺪ ﺷﻬ‪óñ‬ﻦ وش ﺗﻌﻄ‪œ‬ﻪ‬

‫– ال ‪:‬‬ ‫‪í‬‬ ‫‪Å‬‬


‫‪finding for PE in ecg what is the dx PE‬ﻛ¸ﺲ ﺟﺎﻳﺒ‪ Ä‬ﻟ‪%‬ﻢ ﻛﺘﺎ‪º‬ﻪ و ‪õ‬‬

‫– ﺛﺎ ‪ Ëõ Å‬وﺻﺎرت ﺣﺎﻟﺘﻪ اﺳﻮا وش ﺻﺎر ﻟﻪ ‪MI case‬‬ ‫‪í Å‬‬


‫؟ وﻋﻄﻮە ﻣﻮرﻓ‪ Ä‬و ‪õ‬‬
‫‪rv infraction‬‬

‫‪ supportive‬؟ ﺑﺰر ﻣﺎ‪º‬ﻪ اﻻ اﻟﻌﺎﻓ‪œ‬ﻪ اﻟﻠﻬﻢ اﺳﻬﺎل وش &ﺴﻮي ﻟﻪ‬

‫؟ ﻛ¸ﺲ ﻏ‪ óñ‬ﻪ ﻣﺎ˙‪œ‬ﻠﻮ ﺑ'‪œ‬ﻠﺮ راش ووش ﺗﺘﻠ˘¯ ﻋﻨﺪە‬


‫‪º‬ﺎﻟﺨ‪œ‬ﺎرات ﻓ‪œ‬ﻪ‬
‫‪Igm measles‬‬
‫‪Igm herps virus 1‬‬
‫‪Igm herps virus 2‬‬
‫‪Igm vzv‬‬
‫اﺗﻮﻗﻊ اول واﺣﺪ ﺣﻄ‪œ‬ﺖ ﻻﻧﻪ ﻣﺎ˙‪œ‬ﻠﻮ ﺑ'ﻴ·‡ راش‬
‫[‬
‫‪Å‬‬ ‫‪Å‬‬
‫ﻛﺜ‪ ‡Ä‬ﺗﻜﺮار ﻣﻦ ﻧﺎﺣ‪œ‬ﺔ ‪Õ‬ﻞ اﻻﻗﺴﺎم اﻻ اﻟﺴ‪‡Ä‬ﺟﺮي ‪Ç‬ﻌ ‪ ˇõ‬ﻓﻮق ﻧﺎدر ‪Ç‬ﺠ‪ Ä‬ﺳﺆال ﻣﺎﻗﺪ ﻗ‪óñ‬ﺘﻪ ﺑﺮا اﻟﺴ‪‡Ä‬ﺟﺮي‬

‫ وﺣﻮﺳﺘﻪ ﻟﻠﻤ · ‪‡Å‬رە ﺗﻜﺮار ﻓﻮق ‪ ١٠‬اﺳﺌﻠﻪ ﻋﻨﻬﻦ ‪Ç‬ﻤﻜﻦ‬


‫اﻏﻠﺐ اﺳﺌﻠﻪ اﻟﻔﺎ˙ﺴ ‪Å Ä‬‬

‫وش ا˙˚‡ ﺳ' ﺐ ﻟﻠ‪ë‬ﻃﺎن اﻟﺜﺪي ﺑ ‪Å Ä‬‬


‫ ذو‪›õ‬‬
‫‪Age‬‬
‫‪Early menarche‬‬
‫‪Late menopause‬‬
‫– ‪Ã‬ﺲ ﺷﺎﻃﺤﻪ ‪Õ‬ﺎﻧﺖ‬ ‫‪í‬‬
‫اﻟﺮا‪º‬ﻌﻪ ‪ã‬ﺴ¸ﺖ و ‪õ‬‬

‫اﻟﺒ‪œ‬ﺪ‪Ç‬ﺎ ﻛ¸ﺲ ﻛﻮاﺳ ‪ ,õ‬ﻳﺒﻮﻧﻚ &ﺸﺨﺼﻪ ﺳﻬﻠﻪ ﻧﻔﺲ اﻟ‪º öõ‬ﺎﻟﺘﺠﻤ‪œ‬ﻌﺎت‬


‫? ‪Gerd is cuz by‬‬
‫‪Les lower sp‬‬

‫ﺟﺖ ﻛ¸ﺲ ﻋﻦ‬


‫? ‪Appendix abesss what is the pathology‬‬
‫– اﻟﺨ‪œ‬ﺎرات ﻟﻼﻣﺎﻧﻪ ‪vasoconstriction‬‬‫ﺣﻄ‪œ‬ﺖ وﻧﺎ ‪õ‬‬

‫? ‪à normal follicular thyroid tissue‬ﺲ ﻗﺎﻟﻮا وش ﺗ‪#‬ﺴﻮي ﻟﻠﻤﻠﻒ ﻧﻮد اﻟ‪ öõ‬ﻓﺤﺼﺘﻪ وﻟﻘ‪œ‬ﺖ ‪º‬ﻪ ‪ apearent thyroid‬ﺟﺖ ﻛ¸ﺲ‬
‫‪º‬ﺎﻟﺨ‪œ‬ﺎرات &ﺸ‪œ‬ﻞ اﻟﻠﻤﻒ ﻧﻮد‬
‫ﺗﺨﻠ‪œ‬ﻪ ع ﺣﺎﻟﻪ‬
‫ﺗﺘﺎ‪º‬ﻌﻪ ‪º‬ﻌﺪ ‪ ٣‬او ‪ ٦‬ﺷﻬﻮر ‪ã‬ﺴ¸ﺖ اﻻﺧ‪‡Ä‬‬
‫‪Glory team August‬‬

‫ﺟﺖ ﻛ¸ﺲ واﺣﺪ ﻋﻨﺪە‬


‫وﺟﺎﻟﺲ ﺗﻌﺎﻟﺠﺠﻪ ‪Hypothroidsm‬‬
‫)وﻓﺤﺼﺘﻪ وﻃﻠﻊ ﻋﻨﺪە ﻣﺸ‪É‬ﻠﻪ )اﻟﺠﺮﻋﻪ ﻣﺎ‪.‬ﺎﻧﺖ ‪Õ‬ﺎﻓ‪œ‬ﻪ‬
‫اﻟﺠﻮاب ﺗﺰود اﻟﺠﺮﻋﻪ ‪Ã‬ﺲ ‪Õ‬ﺎن ﻓ‪œ‬ﻪ ﺧ‪œ‬ﺎر‪ó‬ﻦ ﺗﺰود اﻟﺠﺮﻋﻪ و&ﺸ‪œ‬ﻚ ﻋﻠ‪œ‬ﻪ ‪º‬ﻌﺪ ‪ ٣‬اﺳﺎﺑﻴﻊ او ‪º‬ﻌﺪ ‪ ٦‬اﺳﺎﺑﻴﻊ ﻣﺪري اﻳﻬﻦ ﺻﺢ ‪0‬اﺣﻪ ﺣﻄ‪œ‬ﺖ ‪ ٦‬ﻻﻧﻪ‬
‫اﻧﺪوﻛﺮو‪ó‬ﻦ واﻟﻤﻮﺿ√ع ‪Ç‬ﻄﻮل ‪º‬ﺎﻻﻧﺪو ا‪º‬ﺤﺜﻮا ﻋﻨﻪ‬

‫؟ ‪º‬ﻜ¸ﺲ وﺣﺪە ‪º‬ﻪ ﺑﻮﺳﺖ ‪º‬ﺎرﺗﻢ ﻫ‪œ‬ﻤ‪ñ‬ج ووش ﺗﺘﻌﻄ‪œ‬ﻪ ‪ post partom hermarrge‬ﻋﻼﺟﺎت‬
‫– ﺗﻌﻄ‪œ‬ﻪ ‪ ،،‬اﻟﻤ‪óñ‬ﻀﻪ ﻣﺎﻋﻨﺪە ﻻ ﺿﻐﻂ وﻻ ر‪Ô‬ﻮ‬‫‪í‬‬ ‫‪Å‬‬
‫ﺣﺎﻃ‪Õ Ä‬ﻞ ﻋﻼﺟﺎت اﻟﺒﻮﺳﺖ ‪º‬ﺎرﺗﻢ ووش از‪ó‬ﻦ ‪õ‬‬

‫‪ 4‬ر‪ó‬ﺎﺿﻪ ‪ ١٥٠‬دﻗ‪œ‬ﻘﻪ ‪º‬ﺎﻻﺳﺒ√ع وش ﺗﻨﺼﺨﻪ ﻣﻦ ﻧﺎﺣ‪œ‬ﺔ اﻟﺪا‪Ç‬ﺖ ‪Õ‬ﺎن ﻓ‪œ‬ﻪ ﺧ‪œ‬ﺎر‪ó‬ﻦ ˆﺸ‪É‬ﻜﻦ ‪º‬ﻚ ‪ pre DM‬ﺟﺖ وﺣﺪە‬
‫وﺗ‪#‬ﺴﻮي ‪õ‬‬
‫‪Low glycemic index diet with 5-10% loss of body weight‬‬
‫ﻧﻔﺲ اﻟ‪º öõ‬ﺎﻟﺘﺠﻤ‪œ‬ﻌﻪ ‪0‬اﺣﻪ ﻣﺪري اﻳﻬﻦ ﺻﺢ ‪Low carb diet‬‬

‫؟وﺣﺪە ﺟﺎ‪Ç‬ﻪ ﺳ˘‡وك و‪ó‬ﻮم ﺟﺎء ‪Ç‬ﻄﻠﻊ ﻣﻦ اﻟﻤﺴ‪#‬ﺸ‪ ¯Å‬ووش ﺗﻀ‪œ‬ﻒ ع ﻋﻼﺟﺎﺗﻪ زود ﻋﺸﺎن ﻣﺎﺗﺠ‪œ‬ﻪ ﺳ˘‡وك ﺛﺎﻧ‪œ‬ﻪ‬
‫اﺳ·‡‪ó‬ﻦ‬

‫‪ 4‬اﻟﺠﻮاب ‪ organophosphate poisoning‬ﺟﺎء ﻛ¸ﺲ ﻋﻦ‬


‫اﻋﺮاﺿﻪ ووش اﻟ‪#‬ﺸﺨ‪œ‬ﺺ؟ ‪õ‬‬
‫؟وﺟﺎﻳﺘﻪ ﻛ‪¸óñ‬ﺲ وﺳ ﻠ ‪Å Ä‬‬
‫ ﻣ‪œ‬ﻘ‪ öõ‬وﻫﺬي ﺛﺎﻟﺚ ﻣﺮە ﺗﺠ‪œ‬ﻪ ﻧﻔﺲ اﻟﺤﺎﻟﻪ وش &ﺴﻮي ﻟﻪ ‪ SCA‬اﻳ‪œ‬ﻪ ﺟﺖ ﻛ¸ﺲ ﻋﻦ‬
‫‪Blood transfusions‬‬
‫‪Spleenptmy‬‬
‫اﺗﻮﻗﻊ ﺳ ﻠﻴ‪ç‬ﺘﻤ ‪ ,õ‬ﻻﻧﻪ ﺛﺎﻟﺚ ﻣﺮە‬

‫ﺟﺎي ‪ copd‬ﻓ‪œ‬ﻪ ﻛ¸ﺲ ﻏ‪ óñ‬ﻪ‬


‫‪his oral cavity full of secretions‬‬
‫و‪Ô‬ﺎﻟﺨ‪œ‬ﺎرات ﻓ‪œ‬ﻪ‬
‫‪Nasal Cpap‬‬
‫‪Mechanical vent‬‬

‫– ‪º‬ﺎب ﻻﻧﻪ ﻧﻒ اﻧﻔ¸ﺲ و‪ 8‬اﻋﻠﻢ‬ ‫‪Å‬‬


‫ﺣﻄ‪œ‬ﺖ ﻧ‪‡Ä‬ل ‪õ‬‬

‫ﺳﺆال ﻋﻦ اﻟ˘‡ا‪Ç‬ﺪ ﺣﻖ اﻟ‪%‬ﻠﻮﻧﺠﻴ‪#‬ﺲ ‪Ç‬ﻘﻮل اﺳﻢ اﻟ˘‡ا‪Ç‬ﺪ و‪ê‬ﺴﺎﻟﻚ ﻫﻮ ﻣﻤ ‪ ‡Å Ä‬ﻻي &ﺸﺨ‪œ‬ﺺ؟ ‪Õ‬ﻠﻮﻧﺠﻴ‪#‬ﺲ‬

‫ﻣﻮﺟﻮدە ‪º‬ﻔﺮﺳﺖ ا‪Ç‬ﻔﺖ ﺟﺎن ﺳﺆال ﻋﻨﻪ ‪Prosthetic valve organsim‬‬

‫‪ superimposed inf grapelike‬ﺟﺎء ﺑﺰر وﻋﻨﺪە ا˙‪ó9‬ﻤﺎ وﺻﺎر‬


‫‪Staph‬‬
‫ﺑﺰر وا˙‪ó9‬ﻤﺎ ور‪ó‬ﻜﻨﺖ اﻧﻔﻜﺸﻦ‬
‫‪Wiskot-ald‬‬

‫؟ وش ﺗﻌﻄ‪œ‬ﻪ ﻋﺸﺎن ‪labour‬و‪º 4‬ﺎل ‪ utrine contrarion‬وﻣﺎﻓ‪œ‬ﻪ ‪ ctg‬ﺟ ˘ ‪Å‬‬


‫ ﺻﻮرە‬ ‫‪õ‬‬
‫اوﻛﺴ‪#‬ﺴﻦ‬
‫؟ وش اﻟﺴ‪:‬ﺐ ‪ ctg reduced variable‬ﻛ¸ﺲ‬
‫‪Mgso4‬‬

‫; ﻣﻊ‬ ‫˘‬
‫‪ prosthetics valve‬وش اﻟ ﻜ‡‪ó‬ﺎ اﻟ‪ öõ‬ﺗ · ‪õ‬‬

‫– وﻻ ‪º‬ﻪ ﺣﺮارە اﻟﺨ‪œ‬ﺎرات ‪ LP‬وﺳﺤﺒﻮا ‪ lethargy and convulsion‬ﺟﺖ ﻛ¸ﺲ ﻏ‪ óñ‬ﻪ ﺑﺰر ‪º‬ﻪ‬ ‫‪í‬‬
‫وﻣﺎ‪º‬ﻪ ‪õ‬‬
‫‪Bactrail mengtis‬‬
‫‪Viral‬‬
‫‪Hpyoxic induce encepathy‬‬
‫–‬‫ﻓ‪œ‬ﻪ ﺧ‪œ‬ﺎر راﺑﻊ ﻧﺎ ‪õ‬‬
‫> اﻟﻤﻨﻄ ˘ ‪Õ ¯õ‬ﺎن ‪º‬ﺎﻟ‪ç‬ﺴ ﻪ ‪›õ‬‬
‫‪õ‬‬ ‫ﻮ‬‫ﻟ‬‫ا‬ ‫ﻮ‬ ‫اﻧﺎ ﺣﺎط اﻟﺜﻼث ﻫ‬
Glory team August

‫ﺐ ع اﻟﻌﺸﺎ‬œ‫ب ﺣﻠ‬ëíˆ ‫ﻪ‬º ‫ زﻗﺖ‬،‫ﺪة‬Ç‫ اﺑﻮ ﺗﺰوج ﻣﺮة ﺟﺪ‬،‫ﺔ‬œ‫ﻪ اﻻ اﻟﻌﺎﻓ‬º‫واﺣﺪ ﺑﺰر ﻣﺎ‬
‫ﺎرات‬œ‫اﻟﺨ‬
Hydrogen breath test after lactose ingestion

‫ﺔ ﺗﺮوﻣﺎ ﻋﻨﺪە ال‬Ç‫ واﺣﺪ ﺟﺎ‬ant cervical tissue O2 86/89


Conscious and alert
What u will give him
O2 MASK.
INTUBATION
CRICOTHYROIDECTOMY
‫– زي ال‬ í
õ ‫ﺸ¸ﺖ اﺳﻤﻬﺎ‬ã ‫ﺔ‬œ‫ وﺣﺪة ﺛﺎﻧ‬CRICOTHYROIDECTOMY
^‫ﻣﻦ اﺣﻤﺪ‬

Alcoholic = drinks alcohil and have hematochezia and have vomiting with blood with hx of peptic ulcer
(esophageal rupture with varices) what ttt
Octreotide

Alcoholic with metalic taste = esophagitis

öõ‫¸ﺲ ﺣﻖ اﻻﺛﻜﺲ اﻟ‬â‫ وﺟﺎ اﻟ‬surrogacy ‫ﻘﻮل‬Ç ‫ﻠﻤﻨﺎ ﻋﻨﻪ‬É‫ ﺗ‬öõ‫اﻟ‬


Consultant does not know
>> warn them that it might be illegal in saudi

UC pt with mutiple duct stenosis and have pruritis


lab shows elevated alkaline phosphatase
Dx, primary sclerosing cholangitis

Pt with pleural effusion with pH 7 and clubbing, dx?


-empyema
-TB
-parapneumatic effusion

45 yo women with dyphagia to liquids which test is with the HIGHEST DIAGNOSTIC yield?
-upper endoscopy
-manometry
-barium

30 something women (case of premature ovarian failure with high FSH and LH asking which of the following
is the complication of this

Which test has the highest diagnostic yeild for GERD


ambulatory ph monitoring

Case of serotonin syndrome, what test used to dx

Case of adrenal incidintaloma, what test should be ordered next? Dexa+urinary metanephrines

Case of post gastric sleeve surgery and have vomiting etc, everything is normal what to do? Reassure

Newborn with cyanosis etc. no murmur. Which med should be given immediately?
-dopamine
-duiretic
Glory team August

-PGE1

Infant of diabetic mother case. Loss of moro reflex on one side. What is the cause?
Erbs paly

Many qs from abortion and their ttt

Anembryonic abortion with DIC. What to do?


-cs
-augmentation of labour

Baby with rocker bottom feet cardiac anomalies and overlapping fingers etc. What is ur dx?
Edward
Patau
Others

Child obese and always with hyperphagia (i forget what term they used)
He has dysmorphic facial features and hypotonia
Dx?
-pradar willi
-noonan
-william

Always hungry*

Pyloric stenosis dx ttt just classic case with olive mass

Ectopic pregnancy many cases not sure in some if it is ectopic or complete abortion

What injury ass. wth blunt chest trauma?


Aortic arch
Proximal to subclavian
Distal to subclavian
At ligamentum arteriusom

Moat common injury associated with blunt chest trauma*

Child with only diarrhea. He drinks pints of gout milk everyday. What is the likely dx?
-plastic anemia
-giardiasis
-forgot the others but they were unrelated

Pt morbidly obese + has GERD. Which procedure is used for him?


-rox en Y..

Pt with thyroid lump only 1.5 cm in the report. What is the ttt?
- total thyroidectomy
-subtotal
-other choices.

Pt with hot thyroid nodule. What is the ttt? They did not mention eye manifistations and i think he is a male
-total thyroidectomy
-radioiodine ablation
Glory team August

Pt with thyroid lump only 1.5 cm in the report.


On biopsy he has Mullary thyroid tissue. What is the ttt?
- total thyroidectomy
-subtotal
-other choices.

‫ ﻫﺬا اﻋﺘﻘﺪ ﺗﻮﺗﺎل ﻻﻧﻪ‬mullary ‫ﺲ‬Ã ‫› اﺗﺄ ˙ﺪ‬õ ˇõ · ‫ﺲ ﻳ‬Ã

Pt with thyroid lump midline below the hyoid bone + moves with swallowing No cervical lymphadenopathy.
Dx? (NO THYROID IN CHOICES)
-thyroiglossal cyst
-cysto-

Flat T wave. Which electrolyte disturbance? Hypokalemia*


Hyperkalemia

Pt TPN what electrolyte disturbance? Hypomagnesemia

Women with yellow vision and ECG with characteristic findings of digitalis toxicity. Which is reponsible?
Digitalis

An elderly patient being treated for CLL with chemotherapy. Which electrolyte disturbance is seen?
-hypocalcemia
-hypercalcemia
-hyponatremia
-hypernatremia

Pt with 2 days ssx of cholecystitis


Ttt?
Lap cholecystectomy

: Pt ( i think elderly) undergoing cholecystectomy but they foud gastric tumour. Ttt?
Resection
Cholecystectomy +resection ‫اﻟﻈﺎﻫﺮ ﻫﺬي‬

Sle:
Has skin and arthritis ssx. Ttt? Hydroxy+steroid i think

Also this
Q: which nonpharmacological ttt is used in SLE?
-weight loss
-smoking cessation*
No (avoidance of sun exposure) in the choices

Sickle cell disease many cases


- boy with bone pain.. etc in lab report elongated RBCs
Which medication to give in this disease?
Penicillin and vaccine i think

Lots of cases about cardiac tamponade

Literally 3 cases the answer is pregnancy test

Pt. alcoholic with long hx of pancreatitis. Came with bulky stools 4-5 times. What is your ttt?
Glory team August

-lipase 30.000
-diet modification with low fat and multiple small meals

Young girl with anemia. Lab test (low HB low MCV high TIBC Low iron) dx? IDA

Case of herpetic gingivostomatitis (painfull vesicles on the gingiva) asked about the ttt? Acyclovir with
something i forgot

Case if a neonate with hypotonia and other ssx of CAH. Asked about the ttt? Hydrocortisone

Many cases about testiclar torsion and epidydimoorchitis

Ttt of toxoplasmosis?
Pyrith+sulfa

Child 5.1 kg , the mother feed him and hevomiting all the kind of formula after feeding , his wight now 3.9 kg
?
A/Reassurance
B/ Us

Esophinlic gastritis what is support your diagnose ??


A/ epigastric pain at night
B/ obesity

Case about miliry wise syndrome ask about diagnosis

Pedia croup pertussis bronchiolitis ‫‡ة ﺟﺖ ﻋﻠﻴﻬﺎ ﻏﺎﻟﺒﻬﺎ‬Ä‫ أﺳﺌﻠﺔ ﻛﺜ‬rx

Best test to screen for HCC?


Tri physic CT
US
AFP

US pic of ovarian mass described as multilobulated mass what to do next?


CA 125
Surgery
Consult onco with further workup

Old, wight loss, painless jaundice?


Pancreatic cancer

-Intestinal obstruction 6 y post sleeve? Adhesions


-beri beri disease, what is defecient?
-Charcot triad, next step?
ERCP
US
MRCP

Pancreatic pseudocyst 2q
Type of drainage both large 15*15
One with leukocytosis
Ct guided
Glory team August

Open drainage
Internal drainage
Percutanous

-Morbidly obese, GERD best bariatric surgery for him?


-balloon
-sleeve
-bypass

Pt with severe GERD ppi not improving


What’s next step in management
A) ranitidine
Other options i don’t recall

21st august
-A patient complains of cough and wheezing. His CXR & spirometer were normal, what to do next?
*repeat spirometer *challenge test *oral steroid *observe
-a pregnant lady went into Labor, she complained of numbness and tingling in the middle thigh after
delivery?
Which nerve injured?
* obturator *Femoral *Sciatic
-Long scenario, what is the average basal body temperature during ovulation ?
36, 36.5, 37, 37.5

-Question about rheumatoid arthritis and it’s effect on pregnancy


*abortion *somthing antibodies *retardation

-What sign indicates pancreatic iron toxicity?


*DM *jaundice

-A case of uterine fundal cancer, and asked about lymph node drainage?
- A case of abdominal pain, suggesting ectopic pregnancy ruptured
- Femoral hernia in 70 year old female asked for surgery
*Laparoscopic *open
-A known case of SLE on hydroxychloroquine and cutaneous menifsations what to add
- A pap smear was done showed abnormality, then biosy was done and showed invasive carcima whats next?
*staging *colonization of lesion *repeat
-Percentage of male or female I dont recall smokers in Saudi
- Most common type of hepatitis in Saudi Arabia
-Many questions on abortion
- Many questions on shock
Picture:
- 2 ECGs
Anterolateral MI & pericarditis
- CTG; Late deceleration
- CXR; not sure about the diagnosis, but the options included; Abscess, carcinoma, wegner

My smle 21 August 2019

#What is the most significant risk for breast cancer??


- age
- late menopause
- early menarche
Glory team August

- delay childbirth until age above 30

# 30 female smoker, subfertilty,


what is the risk factor??
- smoking✅
- subfertilty

# 18 month baby the nurse noted he didn't receive vaccin the parents say "vaccine bad for boy brain"
What to do?
- tell him about the benefit of vaccin.✅

# female with lower abdominal pain "bhcg was 2600" give her sc methotrexate after 1 week "bhcg is 6500"
wht ??
- salpingiostomy
- salpingioectomy✅
- continuou methotrexate
- ocp

# 18 month left knee swelling redness refuse to stand on her leg what is the next appropriate investigation ??
- X-ray to left knee
- CBC
- Reassure
- Aspiration✅

#
- Endometrial polyps **
- Subserosal fibroid

# pt retrosternal chest pain

_ anterior mi
- Inferior mi **
- Septal mi

# HIV female pregnant on medication came and went advice ??


- Tell her the the medication don’t prevent transmitted.
- CS prevent transmitted✅
- She can can do breast feeding

https://www.ncbi.nlm.nih.gov/pmc/articles/PMC3428890/
Note: make sure to see the last choice on the exam

# 76 man dignosis septic arthritis in knee joint, aspiration of joint done , the start antibiotics I forget name of
antibiotics” after 3 day result come from microbiologi that show staphylococcus aureus resistant to cefoxitin,
what the next step??!
- add gentamycin
- Start vancomycin✅ not sure
- Joint wash
Glory team August

1-Pt with malar rash with photosensitivity and artharlagia ,what test is used to confirm the DDx?
A – ANA B-DsDNA✅ C- RF

2- PT developed facial and tongue swelling after Anti HTN medication?


A – Ranilapri✅. B- Bisoprolol

3- Long scenario about pt came with Sx of DKA what type of insulin U will give?
A – according to sliding scale B- Fixed rate Iv infusion ✅ C- insulin BID

4- Pt known case of Rheumatoid Arthritis came with RT knee swelling and pain how will U manage = Abx

5- Pt with arthritis with past Hx of UTI (Reactive arthritis)


How to manage? A -Steroid✅ B-

6- Pt 35 YO female her mother died with breast cancer and her sister died with ovarian cancer ,what is the
best screening modality for Breast cance?
A -BRCA Gene B- Mammography✅ C- breast self exam

7- Long case, Flat T wave on ECG what do you find in urine:


A-High potassium
B-Sodium
Note: hyPOkalemia and digitalis toxicity cause flat T wave

8- Pt known asthmatic with Flat T wave on ECG what medication can cause that ? A- Salbutamol✅ B-
Salmetrol
Note: It causes hyPOkalemia

9- Chlid came to the ED with gastroenteritis , U discharge him home after telling him to come to the hospital
if there is warning Sx? U are doing
A – safety Netting✅ and other choices I cant remember

10- Child drinking Cow milk 3 times a day with pallor ? IDA✅ (No B12 in choices)
11- X-ray showing radio-opaque mass at McBurney’s point and rebound tenderness... what to do next:
A-Appendectomy✅
B-CT without contrast
Note: Reed the senario well if it suggests appendicitis then go for appendectomy

12 - Management of a patient with gastrointestinal stromal tumor of the stomach 5 cm No metastasis No


lymph node enlargement ? In the body of
A- Gastrectomy
B- Wide local excision with Free margins✅ not sure
C- Radiotherapy and chemotherapy

13- Late complication of meningitis:


A-Seizure
B-Hearing loss✅
C-Facial nerve palsy
Glory team August

D-Ataxia

14- Looong case of ITP with plt 32 how to manage?


A - IVIG and steroid B -Plt transfusion C- plasmapheresis
Note: Please read this guideline and read the q carefully in the exam
https://emedicine.medscape.com/article/202158-guidelines#g4

15- pt is taking UFH for PE after surgery -CBC was provided with Low platelet count what to do?
A – Continuo heparine
B- Replace with Enoxaprine
C- stop Heparien and replace it with bivalirudin (another med with same sound)✅
Note: it depends on the time of thrombocytopenia
Type1: in the first 2 days ( observe and continue heparin)
Type2: 4-10 days after treatment ( then choose c)

16 – Pt came URTI the he developed abdominal pain and rash DDx?HSP✅

17 – Side Effect from Furosemide? Hypokalemia✅

18 -Pt 55 Yo smoker and came with breathlessness DDx?COPD

19 – Diabetic came to the ED with well demarcated Erythematous lesion on the shin ? Erysepalis

20- Pt came with retrobulbar pain headache and fever? Dengue Fever

21- Pt came with gait instability and urinary incontince ,he is known case of Osteoarthritis and had cervical
laminectomy after cervical degenerative myopathy what is DDx?
A -Cauda Equina B- Recurrent Cervical Myopathy✅

22- Scenario of Parkinson ds ,where is the lesion? Substantia Nigra

23- Man came with Ptosis , after examination Ice test and other test were +ve how to manage?
Pyridostigmine

24 – a case of macrosomic baby and Complications =Erb's palsy

25- Pic of Chloasma gravidarum

26- Pic of urethral cruncle

27- Pt with PE and hypotensive ,management ? Alteplase

28- Pt with UTI and high creatinine which Abx is contraindicated? Cipro

29 -Most common diagnosis of Pt with Fever from Sub-Saharan region?Malaria

30- Child can’t handle spoon well But he can walk up the stairs with one hand held What is his age: 18
months

31-Pt came with pin prick sensation and loss of vibration sensation with loss of ankle reflex and K was 5.5 and
Creatinine was very high what to do?
A – Vitamin B 12 B – Dialysis c- Bicarbonate
Not sure read the senario well
Glory team August

32- child came for routine checkup U found a murmur that is late systolic changing with position ? I think this
is innocent murmur

33- a child with murmur ,I think he was anemic ?treat the cause

34- Pt came with pleural affusion after 2 weeks from pneumonia infection what is the DDx? Parapneumonic
effusion

35- child came after RTA with blood coming through Ear? Basal skull #

36- Pt fall from 4 meters he is vitally stable and GCS is 15 but he is complaining from Bilateral heel pain what
is the next step?
A - xray for Both Feet✅ B- Pain management

38 – Long case of pancreatitis after ERCPs ,which of the following will increase the survival rate of the pt ?
A – abx B- Ringer lactate✅

39- Low grade dysplasia with barret esophagus ?how to manage


A – Resection ✅B – all other choices were medications

40-where do U find the level of uterus in pregnant lady?


A – medway between symphysis pubis and umbilicus B -above Umbalicus

41 – case of molar pregnancy with US pic

42- pt pregnant with fundal height more the GA and hemoptysis DDx
?Choriocarcinom ?

43- A lot of cases with Ectopic pregnancy

44- 3 cases about Trichomonas management in different scenarios

45- case about vaginal candidiasis

46- Drug approved by FDA for tx of Cancer hypercalcemia? Denosumab✅


Note: Other ttt for hypercalcemia in malignancy: normal saline, calcitonin, bisphosphonate

47- most common cancer in pediatric age? Leukemia

48- Polymyalgia Rheumatic with fatigue ?CLL

49-Hashimoto Thyroiditis associated cancer? Lymphoma

50-mamngement of acute Migraine headache? A- Aspirin B- triptan✅

51- management of Kawasaki ?Aspirin

52- Nurse with +ve HCV antibodies and -ve HCV Ig M what to do?

*Elderly with black discoloration and elvated from epithelia surface what to give ?
Chemo
Radio
Glory team August

Bunch
Execiopn

*A pt has malignancy in his face how are you going to deliver the information to pt

Explain the pathophysiology to the pt


Build the trust

Most of the ethics are repeated


No question from biostatistics

*Kawasaki disease case dx , treatment and indicator of poor prognosis. ( 3qs )

Iv Ig

elevated crp

*WIDE FIXED SPLITTING S2 Atrial septal defect

*Pregnant 34 or 36 forgot c/o painless mild vaginal bleeding did us dx as partial placenta prvia pt stable ctg
was done and is reactive what will you do next

-Immediate CS
-Biophysical profile
-Amniocentesis to check for lung maturity

*39 weeks she is in active labour contractions 3 times every 10 minutes, cervical dilation 6 effacement 80%
fetal presentation when u did pv examination u feel mouth nose and orbit
What will you do?

-CS
-Give oxytocin

*Pt 36 weeks presented with strong contactions , os is closed and no effacement.


True labor
False labor

*Question about euthyroid sick syndrome asking about the dx

*Child had pneumonia during examination u heard a murmur although examined the child a month ago there
was no murmur

Innocent murmur
Valvular heart disease ( don’t remember stenosis or regurgitation.)

*A child every thing is normal, u hear a murmur and fixed splitting. What u will do ? ( not sure about the
splitting)

-Refere to cardio for catheterization


Glory team August

-Reassure

-There was no follow up in the choices


Depends on the age

*Questions about post operative complications SSI ( appendectomy, gastric sleev, wound infection)

*Pt 2 year old with sickle cell , has pneumonia and splenomegaly ,
investigation showing
low hemoglobin
low reticulocytes
Wbc high
What is the best initial to do ?
-Ab
-Splenectomy
-Blood transfusion
-Fluid and analgesia

*Pt diagnosed as sinus headache for 2 years , no improvement with medication of his headache , what to do
?

Paranasal sinus ct
Careful review of history and examination
There was no brain ct

Two Senartios of patients post operatief, with mild symptos such as fever, chills.
Analysis: > wbc
a. Bacteremia
b. Sepsis
c. Sever sepsis
d. Systemic inflammatory response

Patient who is tall, reduced body weight, mitral valve problem, cavum expe…:
a. Marfan
b. Dowm
c. Congintal something

Risk of hcv:
a. 3
b. .03
c. .30
d. 30

Child with suncked fontanel, moderately dehydrated, weight 10 kg asking about who much floid should be given?
a. 1000mg
b. 1500mg
c. 2000mg

Female child with hypotension and decreases natruim in serum, by examination large clitros, dr. gave normal saline.
What to be added?
Glory team August

a. Bicarbonate
b. Hydrocortisone

Child who who say mom, put his hand on his mouth and say mouth. How old?
a. 10m
b. 12m
c. 18m
d. 24m

Patient with symptoms of portal disease and jundance, US shows a mass that is mostly malignant, what is
the next step?
a. Biopsy
b. Trifastic ct
c. Laprtomy.

Women with premenstrual syndrome, change in daily activity, affect her word preformace, recurrence visit to
ER, what is affected,
a. Mood swing
b. Irritable

Mother of child with diffeclt behavior has be advised to deal though positive reinforcement, what can help?
a. Parent leaning programe
b.

RTA patient with normal air entry, normal breath sound, chest tube show minumin blood, suddanly patints is
hypotensive with < JVC and distance heart sound, what is you immediate action>
a. Fluid
b. Precardiothesis

Patient with palpitation and sweeting, and discomfort in the neck. Elevated t4 and reduced TSH. What kind of thyoid
disease?

a. Hishamoto
b. Graves
c. Toxic goiter

Patints with cardic pain, mild elevated temp, ECG: diffuse ST elevation, what is the treatment?
a. NISDS
b. Abx

Patient presented with sudden chest pain, radiate to the left arm, ECG ST elevation in AVF, patints started on asprin
and clopdrogyl, what in the next immadated action?
a. Righ ECG
b. Ct angio

Elderly women with PIP and DIP pain, no significant morning stiffness, PIP swelling associated with
something, RA -, what is the diagnosis?
a. Osteoarthritis
b. RA
Glory team August

c. Senonegative RA

Eldary with high blood pressure on several meting, what to do?


a. Repeat test
b. Starts bb
c. Start ccb

Patint with BPH and now need to start hem on antihypertion medication, what to choose, no alfa bloker in
options
a. Ccb
b. Bb

Eldary Women increase frequency to urinate, high voding, and the feeling on incomelet emptying, on examination,
palbale bladder after urination, what is the diagnosis?
1. Urge incont
2. Stress inco
3. Overflow
4. Reflx

Scenario of PE with ecg shows S1Q3T3


Which vacnation whould admisted at 2 month.
Pregnancy vaccination> influnza
Eeg allergy> yellow fever
Mior and major criteria of RA
Esophgal cancer high grade barret> resection
Anti acid 20mg use> increase the does or switch ?

Remember the heart murmur


Systolic: AS, PS, MR, TR
Dystolis: AR, PR,MS,TS.

Beriberi disease deficiency in which


-b1
-naicin
-vit c
-vit e

Case of malaria hb electrophorisi


Hba2 30
Hbf 50
What dx
Sickle cell anemia
Bthalassemia
Glory team August

A thallaseemia

Vaginal discharge milky white fishy odour >>BV

Other question also about BV positive whiff test

Case vaginal discharge strawberry petechia in cerivix yellow green discharge >>> trichomonas vaginalis

Case of malaria ask what is 1 line of treatment


Chloroquine
Mefloquine
Quinine
Artesunate derivative

Child know his name ,feed him self , no anxiety with separation from parent which milstone
1
2
3
4

Aortic stenosis when considered significant if orifice less then


1cm
2 cm
3cm
4cm

Case of infant with mother k/c of sle found baby to have complete heart block and mild pda what cause
Rubella
Neonatal sle

Child say 2 word phrase what age


24 month

Infant present with hx of cough sneezing and frothy secretion


What most likely dx
Bronchial asthma
Bronchiolitis

Case of toxoplasmosis what is treatment


Sulfadiazine and pyrimethamine
Doxycycline +clindamycin
Rifampcin +cephalosporin
Glory team August

Ctg show late decelaration >placenta insufficiency


Other ctg > fetal anemia

Vaccine contraindicated in egg allergy


Yellow fever
Pattern of eczema disturbution in 15 yrs old boy
Face and neck
Popliteal area

Case of post rt apt had injury to lt knee doctor noticed he can extend leg till thigh
Injury In which
Tibial collteral
Fibial collateral
Ant cruciate
Posterior cruciate

Case of hemorrhage 800 ml during c/s placenta still in uterus , uterine atony considered
1ry pph
2ndry pph

Case asherman syndrome curttege involve which layer


Baslis layer

Pt admitted for cholecystectomy surgeon after opening noticed large mass on stomach wht to do
Cholycystectomy
Stop and do nothing
Resect mass
Do cholycystectomy and resect mass

2-pics of melasma +skin tag+miss abortion in Us+3 ECG +CTG

3-pic of stepping reflex (see google)

4-SLE case on NSAID not improve what to add>methotrexate (my answer)

5-abortion type ?many Q

6-ectopic pregnancy ttt? Many Q

7-ethics repeated

8-case of whooping cough>pretsuss

9-case of barking cough not improve with epinephrine what to do>repeat epinephrine dose
Glory team August

10-5As to stop smoking??i think ask assess assist arrange follow up

11-2 Q about postmenpuase bleeding ? I forgot the Q but easy ‫ﻌﺎت‬œ‫ﻣﻦ اﻟﺘﺠﻤ‬

13-5 Q about milestones (easy)

14- many Q about COPD (‫ﻌﺪ‬º ‫ﻌﺎت‬œ‫)ﻣﻦ اﻟﺘﺠﻤ‬

15-Q was about couliflower pattern with gray color something like that?
-anal ca
-condylomata acuminata(my answer)

16-egg allergy>yellow vaccine

17-baby with vaginal bleeding +itching >foreign body

18-treatment of HF >ACEI +diuretic

19-screen of colon ca>50

20-screen of osteoporosis >65

21-target HBA1c in diabetic pt ?

22-target warfarin in afib>2-3

23-treatment of MI?

24-screen for uvitis I didn’t remember if ANA positive or negative!

25-from where we take consent for LP in child 15 years old ?

26-treatment of pericarditis >ibuprofen (my answer)

27-girl with sweating +palpatation and fear of die>panic attack

28-clear scenario of turner ask for diagnosis!


29-case of PCOS with investigations what you would like to ask more >blood glucose and lipid profile (my
answer)

30-pt with long Hx of GERD and there is high dysplasia with atypia by Endoscopy >refer to surgery to do
resction

) After blood transfusion, a patient developed pain at the site of the needle, chest pain or tightness or SOB,
and fever:
- febrile non-allergic reaction (my choice, thought of TRALI)
- allergy/anaphylaxis...

) Patient on methotrexate therapy, after a while he had abnormal LFT's, how do you diagnose injury from
methotrexate:
- LIVER BIOPSY
Glory team August

*most common fibroids cause heavy period ? submucosa

*Abgar score :HR 120,contra red and limbs blue ,reactive by cough and crying ,flexion limb

*prognostic factor in acute hepatitis ,lab show enzymes high and high bilirubin ? alan high ,PT, albumin

* pt has irregular pulse and HR170 stable tx ? adenosine ,amiodaron ,cardiovein (no beta or CCB in choices)

*girl 7 years old has pubic hair, developed breast ,ask about which kind of puberty? central precocious
puberty ,ovarian tumor ,central lesion , gonadotropin adenoma

*case of cronh disease has fistula cause collection of fluid ?tx percutaneous aspirations, open + remove
fistula

*breast lesion change in size increase and cause compression on skin lead to change in its color due to
compression effect ddx? fibroid , Phylloid tumor

*case testicular torsion ,next step ?Doppler us ,surgical exploration

*Gram+ bacilli in CSF tx? ampicillin

*preterm at 27week +CS ,O2:83,destrss after 30 min ask dx (no CXR )? RDS ,TTA

*CP distress was intubation +now o2 and all lab normal (co2&o2) type of respiratory failure? hypoxic,
hypercapnia (he did not tell us what the result before o2 given )

*calculate GCS ,pic late and early CTG , calcite fluid 10 kg & moderate

*rapid correction of dehydrated child has cns sx ? edma due to rapid correction hyponatremia

*postmenopausal has hyperplasia endometrial +atypical tx? through abdominal hysterectomy (other option
was medication & lagtion)

*pt diagnosis MI what next to give to him ?heparin, catheter

*women she over Wight BMI 30 ,had Family history of cancer ,which prevention way ? maintain good weight,
being active

*physical activity for child ?60 min

*celiac came and suspect to be not compliance to diet ,how to conform that ? antibody, endoscopy
duodenum biopsy (he want follow up )

*child has leg abnormal shape and delayed walk, ca high ,alkaline phosphatase high ,normal Pos ?ricks
,familial hypophosphatemia ricks ,renal dystrophy , hypophosphatemia

*child fever has one lesion on scrash patch that later spread to all body, enlarged lymph node in groin & axilla
+pic rash ,tx ?antibiotic ,(i think this is cat scrach deasesa)

*pt har type two DM and now he has HTN ,best antihypertension tx ?ACI
Glory team August

*pt new diagnosis HTN has protein in urine ,best anti hypertension medication? ACE,duricb,CCB

*sle pt has arthritis tx ? hydroxychloroquine 400 mg daily ,MTX every week + hydroxychloroquine 400 mg
daily

*pt has dyspareunia +pain +US show calcified lesion tx ? laparoscopy

*lady has BIRDs score IV next step ?core biopsy ,FNA, excision

*layday has progressed mass large breast, US show s.. tx ? modified breast ,simple ,radical

*case pt has leg swelling +non pitting edema next step ?Us veins ,ct angon, (no lymph study )

*pt gas snoring ,case of obstructive sleep apna dx ? polysomnography

*pt has tonsillitis with exudate ask most likely to happen (not most common) complication? pharyngitis ,
pneumonia ,scart fever ,PSGN

*ovulation when to occur ?after LH 36h

*endometriosis pt fear of this it can cause cancer ? tell her general not risk cancer

*general screen in population ?CRC , ovarian cancer ,oral cancer , thyroid cancer

*next step after stable pt which will give to pt iv before endoscopy? vasopressin, octoid

*pr trauma ,thy did for him surgery remove spleen ask post oprative wil be mild low ? insulin, glucagon
,vasopressin

*pt has jugular vein injury and fracture skull base ,what also may has injury? Dysphagia (most likely due to
injury bas)

*pt has FTT and has whizzing was diagnosed as has BA next step ? more investigation to exclude asthma
mimker

*pregnant has fetal demise and came with low fibrinogen dx? DIC

*newborn sudden death ,what will ask about? ask about social hx

*how to prevent sudden infant death syndrome? smoking out side

*best to dec risk mi ?low the lipid ,stop smoking

*pic of molor us dx? molar

*pt has +pregnancy test +hemoptysis +mass dx? Choriocarcinoma

*thyroid sick syndrome ?low t4,3 and high revs t3

*pt has distress + saliva dx? epiglottis

*pt has ectopic pregnancy 5000 ask mang ?conservative ,medical, surgery

*pt has inspritiy strider +barking cough which will make u consrin? lips blow
Glory team August

*pt has fatigued + other sx of adrenal insufficiency which most accurate test to confirm dx ? ACTh stimulate
test

*pt has HTN and adrenal mass and catecholamine high best medication for HTN ?alpha blocker

*pt has cyniss +small heart .+mark of pulmonary not clear dx? cardiac cath ,pain and sedation

*TOF which of follow is part of defect ? AS,PS

*small vsd +asymptotic tx? wait and watch

*EI tx ?cetixon +vancomycin, gentamycin

*pregnant lady has hyperthyroidism ? antithyroid medication

*most imp indication surgery grave ? eye sx

* score of thyroid 4 next step ? hemithyroidectomy

*pt has diagnosed has CRC next step ? CT abdomen

*pt has cardiac diseases +epigastric pan sudden onset ,pass loos stool once dx ?mesenteric ischemic

*pt has AS and develop lung cruption which of follow is true ?


has poor prognosis due to CHF sx ,has RBBB, mummer incr with stand

*pt has copd ask best oxygen delivery system to copd ?nasal, simple mask ,venti mask

*pt has cough and alot of sputum sometimes with blood +clupping , what is the investigative? high resolution
CT

*pregnant has right side murmur radiates to carotid ? AS

* high k and change ecg tx?ca gluconate

*first sing in boys puberty? Enlarge testes

*hard sing of vasclar injury ? change color ,low pulse, fracture

*pt had Intussusception what is true ? high recurrence rate after surgery, shock common complication with
surgery,tx with emna if has peritonitis

*pt has nephrotic syndrome ,what he at risk to develop? perotenitis

*pt has HGa2 4% and microcytic anemia dx? beta minor ,major

*pt has h/o upper respiratory infection then he came with jaundice all things are normal except indict
bilirinon high ?gilbert syndrome

* pt has splenectomy +low erythropoietin +headache +high platelet?

*pt has spherocytosis + comb test + dx? hemolytic anemia


Glory team August

*pt has high grade metaplasia in esophagus next step? referral to surgery
*pt after 17 days of gastric sleeve persisting vomiting and has mood sx as his family said and alll examination
are normal ,abdomen are lax and Ct normal all thing normal next step referal to psychiatric

*pt elderly has high wbc 20,000,high lymphocytes only and tired ? CLL

*pt has h/o 12 month ago hemorrhoidectomy + pain unable pas stool ? anal stenosis

*child abdomen pain +diarrhea some times with blood ,loss wt ,best to initial start ? 5-ASa

*pt has vitiligo + anemia mcv high , best to give him ? iv b12

*Parkinson has sever dysphagia cant swallow and wt loss best management? gastrostomy

*pt has chronic HBV come for follow up ,liver enzymes 60 above normal but not very high next step?
Interferon ,observation, other two antviral hepatitis

*pt has sold dysphasia next step ? barium swallowing test

*when to do study to make sure he eradication infection after pt finish course of h.plory ?at this time right
away ,after 4 week, after 2week

*ovarian cancer ? c125

*pt has stress and urge UI best start ?pelvic and kajal exercise

*pt has ITP and his platelets 15 and has bruises next step? Give him iv igg (no in choices obsrvtion)

*pt newly married came to gyna OPD which is can give u high lied information ? inspection valva ,digital exam
pelvic

*pt has dm c/o vage chest pain do ecg was normal, next step ? stress ecg

*pt has lesion on cervix visible next step ? biopsy directly

*two case PMS (pt irritable and had conflict with her worker )one ask tx ?SSRI other ask who to dx? clinical

*pt will do D&c which will be one of the complications of this procedure? uterus perforation

*ROM+bleeding +bradycardia fatus? cord prolaps

*pergnant women at 15 weeks presented with nausea and headache. On exam the BP was high "168/100",
otherwise healthy. Dx? chronic HTN

*pt term has sever Preeclampsia ( pain ,+ protein urine +bb)160 —>stable then mg then call anesthesia

*chils has distended abdomen & thining hair problem what he has ? low diet protein (Kwashiorkor,)

* pt cancer of lung came again complain of pleura effusion, he was tx multiple times aspiration, best tx for
him ?—-> chemo pleurodesis

*pt came from Indonesia has campylobacter jejuni infection tx? azthromycin

*fibroid asymptomatic ? follow up every year by us


Glory team August

*complex resion syndrome ? physiotherapy

*copd + oral secretion+ cns change? mechanical ventilation

*asthma cant speak high co2 decrease breathing sound? Mechanical ventilation

*child age 4 and negative ANA how frequency do eye examination? 3,6,12 m

*pt has dysmenorrhea not relive by NAsid , enlarged+ tender utrus , h/o D&C? ademoeseo

1.Echinoccus liver cyst


CT: 12*15cm with multiple intra-daughter cysts, but no calcification.
Best management?
A.Surgical De-roofing
B.Long term albenazol
C.Intra-sclerotic injection
D.Aspiration
N.B: if question (initial or next step start medical)
Best: multiple intra-daughter cysts is an indication for surgical management.

Young pt 21 YO with dyskinesia with agitation and ring on ophtha exam?


A- Wilson ds
UC pt came with 4 days vomiting and bloody diarrhea and transverse colon is 15 cm what to do ?
A-infiximab b- systemic steroid c- total colectomy d- fructocolocetomy with ileal pouch
Pt 22 Yo with no symptoms what to do for her?
A- pap smear
DKA pt after U manage him he still he has hypokalemia what is the cause ?
A- insulin infusion B- Metabolic acidosis
Pt with Kawasaki took the treatment how will you follow up the pt?
A- Echo B- catheterization
Pediatric pt with cardiac ds with cough and fever how would U approach the pt?
A – cathetrization B- consultation to cardio and pulmo
???
8 weeks pregnant lady came with vaginal discharge the crown rumb length is 7 weeks what is the DDX?
A - Anembrionic B- threatened abortion
???
4 month old down syndrome with harsh systolic and fixed splitting ?
A- AVSD
Pt with lower back pain with morning stiffenes and he feels the pain is getting better with paractamol what
to do?
A- physiotherapy B- biofeedback

Pt with A fib how to control the rhythm ? -


A - diltiazem B – Propranolol

Pericarditis ECG what is the treatment?


A- Ibuprofin-
Pt with IBD ,what increases the risk of osteoporosis?-
Glory team August

A -age B -chronoc steroid use


Pt IBD he is crying what to give ?
A- azathioprine B -steroid-
Pt with PCOS what will you obtain?
A – lipid and glycemic profile -
Long scenario about pt with A fib with lower limb pain how to manage ?
A – femoral A thrombectomy b -thrombolytic Tx
C-above knee amputation D -thrombectomy without imaging

Pt with Follicular cancer and he did hemithyrodictomy ,U found cancerous cells after surgery what to do?
A -radioactive B- total thyroidectomy C- observation and follow up

Pt diabetic on metformin with high glycemic profile?


A – increase the dose of metformin
B- add one of the sulpha drugs
C- Do nothing

Elderly man with Rt knee pain and swelling ,WBCs is normal ?


A -Rheumatoid B- septic arthritis C- gout D -pseudogout

Pt with skin manifestation spread to the trunk and face with lymphadenopathy ?
A -penicillin B- steroid C -antiviral
???

Child with distal femur mass ?


A -osteosarcoma
B- Chondrosarcoma

Pt elderly diagnosed with osteosarcoma ,what is the mets workup?


A -Chest X ray B- pet scan

Pt child with septic arthritis ,culture came with MRSA what to do next?
A – shift to Vancomycin
B -Joint wash out

Pt with distended JVP and systolic murmur ?


A -PS
Mid diastolic rumbling murmur?
A -MS
Pericarditis case ,what do U found on examination? Pericardial rub
Pt 21 Y O with whooping cough ?How many years he will be immunized against pertussis?
A – 10 years B -5 years

Pt is with bronchogenic carcinoma he came with distended JVD and quite heart sounds?
A -Echo
b- ECG
C- Chest x ray
Glory team August

Malaria prophylaxis questions

Pt with Hx of PID with normal semen analysis of the husband and she is trying to conceive ,what to do?
A -Ovulation induction
B -intra uterine insemination
C- IVF
D -weigh reduction

What do U find in biopsy of Pt with celiac ds?


A -villous atrophy

PT with G6PD deficiency what drug should be avoided?


A -Amoxicillin
B- Aspirin
C-paracetamol

Copd?
Fev1/fcv ratio decrease
TLC increase
Tidal capctiy normal or decrease

Postpartum blues she’s already in tx but which of the following would make her symptoms become better
Encourage breastfeeding
Include one family member in the tx

H.polryi infection treated and finished the course of tx when to do breath test
Now
2 weeks from now
4 week from now

*child age 4 and negative ANA how frequency do eye examination? 3,6,12 m

*pt has dysmenorrhea not relive by NAsid , enlarged+ tender utrus , h/o D&C?
admission

1.Echinoccus liver cyst


CT: 12*15cm with multiple intra-daughter cysts, but no calcification.
Best management?
A.Surgical De-roofing
B.Long term albenazol
C.Intra-sclerotic injection
D.Aspiration
N.B: if question (initial or next step start medical)
Best: multiple intra-daughter cysts is an indication for surgical management.
Glory team August

2.Another liver scenario


CT: Liver lesion 10*15cm in right lobe, septations abscess,
Labs: entameba histolytica
best next step in management ?
A.Percutenous drainage

B.Aspiration
C.Metronedazole
N.B: best for entameba is metronedazole + Question said initial, start with metronedazole
even if it’s large abscess.

3.one week post-appendicectomy with minimal abdominal pain, examination: minimal


tenderness on surgical site, otherwise normal,
CT: 2*2cm retrocecal collection, Best management ?
A.Intervention radiologist drainage
B.Open surgical drainage
C.Diagnostic Laparoscopy
D.Conservative management
N.B: usually drainage indicated in large collections 4*4cm and above, but I’m not sure 100%
about retroceacal.

4.Known case of BPH and hyperlipidemia, came complaining of high blood pressure 140/88 in
many occasions, his BP now 150/89, cardiac examination normal, no diabetes no protein in
urine, What is the best next step?
A.Amlodipine
B.Lisinopril
C.Life style modification
D.BB
N.B: (Unfortunately no Alpha Blockers were in choices.)

5.Patient came for 3 months neck pain, thyroid function test all normal, ultrasound: single solid thyroid
mass in right lobe.
Best next management ?
A.Thyroid scan
B.FNA
C.Imaging Follow up
D.Right lobectomy.
N.B: if labs show hyperfunctioning go for scan + It’s solid nodule not cystic so I don’t think FNA.

6.Female with breast compliance, biobsy done (malignant phylloid tumor)


Best next step?
A.Wide local excision
Glory team August

B.mastectomy
C.Radiotherapy
D.Chemotherapy

N.B: if it was large go for mastectomy.

7.Symptomatic chronic heart failure echo: ejection fraction 25%


Best drug?
A.Duretics
B.ACEI
C.Spironolactone
D.Digoxin
N.B: HF with systolic dysfunction best drug decreases mortality rate is ACEI
HF with diastole dysfunction (normal EF) best drug decreases mortality rate is Spironolactone

8.Same case above but asymptomatic, you did echo ( EF 40%)


Best next step?
ACE
BB
Furosemide
CCB

9.Cough & fever, after antibiotics he became better but symptoms never gone, What’s the most likely
diagnosis?
Pulmonary embolism.
parapneumonic effusion.
TB.
Heart failure.

10.Indian patient known case of tb, he has tb bronchiectasis, came with fever, dysapnea and massive
hemoptysis, x-ray: right side infiltrate, What’s the best next appropriate management?
A.Pneumectomy
B.Chest tube
C.Right lateral decubitus position
D.Chest physiotherapy
N.B: bronchiectasis is chronic condition treated by Chest physiotherapy.
If acute exacerbation you may give abx.
Glory team August

11.Patient came complains of infertility + sinusitis, cough and hemoptysis,


Semen analysis: normal number but immobility.
imagining: Situs inversus, What’s the most likely diagnosis?
A.Goodpasture syndrome.
B.Cystic fibrosis.
C.Wegener granulomatosis.
D.Kartagener's syndrome.
N.B: sinusitis + hemoptysis are seen in Wegener.
infertility + urti are seen in CF.
Sperms immobility+sinusitis+urti+Situs inversus are seen in Kartagener's syndrome
eyes cannot see what mind doesn’t know.

12.Diarrhea after abx?


A.C.difficle

13.Case of profuse diarrhea


, Analysis revealed C.difficle toxins, treatment?
A.Metronidazole

14.Female with dysuria, vaginal discharge examination: strawberry friable cervix with yellow-green
mucopurulent discharge,
oil immersion analysis: high PMNS
diagnosis?
A.Gardnerella vaginalis
B.Trichomoniasis
C.Chlymedia
D.Gonorrhea
N.B: Gardnerella vaginalis = BV, so excluded
yellow-green and strawberry cervix suggest Trichomoniasis, however lab results is confusing.

15.Female with fishy-smell discharge, analysis: PH: 5.1


What type of cells will you see?
A.multinuclated giant cells
B.Overproduction of lactobacillus.
C.Single nucleated cells
D.Granulated epithelial cells.
N.B: Most likely BV, so clue cells
Clue cells= squamous epithelial cells coated with the anaerobic gram-variable coccobacilli.

16.Known case of TB came with dyspnea & raised JVP, respiratory & cardiac examination normal, What’s
the most likely diagnosis?
A.Myocarditis
B.Cardiomyopathy
C.Cor-pulmonale
D.Constrictive pericarditis
Glory team August

N.B: tb causes Constrictive pericarditis, Constrictive pericarditis causes raised JVP. But read about other
answers.

17.Female came for breast cancer screening because of her positive family history, biobsy revealed
Atypical hyperplasia, next step?
A.Wide local excision.
B.Simple mastectomy.
C.Radiotherapy.

18.Copd exacerbation, drowsy, with secrerions, ABG: Hypercapnea hypoxia, respiratory acidosis, best next
step?
A.PiPap
B.Mechanical ventilation
C.O2 Mask
D.Nasal Cannula
N.B: severe respiratory acidosis and drowsiness are indications for MV in copd & asthma

19.Copd exacerbation with fever and hypoxia, after giving him high flow rate oxygen, his level of conscious
decrease,
ABG: O2: high, pco2: High, PH:low
Best next step?
A.Mechanical ventilation
B.Pipap
C.Reduce oxygen rate
D.CT brain
N.B: He lost his hypoxia drive because he received high flow o2.

21.Guy approaching and asking strangers despite they told him not to ask these questions again, he still
repeating & asking them, he is unaware that he is doing something wrong,
What’s the disorder?
A.preservation
B.Flight of ideas
C.Loosing of associations
N.B: I hate psychiatry, just remember preservation = repetition.

22.Insulin for dka :-


A.IV Fixed rate
B.IV Fixed rate + long acting
C.Sliding SC
D.mixed bid
Glory team August

23.Old female with rapidly growing thyroid mass & weight loss, history of hashimoto thyroiditis 10 years
ago, What’s the most likely diagnosis?
A.Papillary thyroid carcinoma
B.Thyroid lymphoma
N.B: papillary is the most common thyroid cancer but it’s the best prognosis, in this case seems more
aggressive one + old female with history of Hashimoto this is typical case for thyroid lymphoma.

25.Patient with fatigue & hypertension, labs :-


CBC: all normal only high hemoglobin, RBC not mentioned.
erythropoietin : Low
Most likely diagnosis?
A.myelofibrosis.
B.polycythemia rubra vera.
C.Leukemia.
N.B: myelofibrosis is general bone marrow cancer so you may find pacnytopenia or hepatosplenomegaly.
Polycythemia vera: is specific cancer for rbc’s, our patient had high hgb this is a hint, low erythropoietin,
kidney stop producing it because RBC is very high, hypertensive? Remeber high RBC’s will cause high blood
viscosity which ends up with htn.

26.Sickler came with dyspnea & fever, CBC: hgb is 4


causative organism?
CMV
EBV
Parvovirus b19
N.B: sudden critically low hgb think about aplastic crisis, most commonly triggered by Parvo b19

26.6 years old boy with midshaft femur fracture, angulation 30ْ
Best management?
A.Closed reduction & Hip spica
B.Open reduction & plate.
C.Open reduction & IMN.

N.B: (My answer, not sure)

27.Patient on nsaids complains of hypertension episodes, Labs: hypokelemia :


Most likely diagnosis?
A.Pheochromocytoma.
B.Primary hyperaldosteronism.
C.Nsaids induced HTN.
D.essential HTN.
N.B: it’s the only cause of hypokalemia in choices.
Glory team August

28.Female came complains of headache + sudden painful loss of vision, Examination: loss of vision in right
eye, CT brain : pituitary hemorrhage
MRI brain: pituitary mass,hge + compressing on optic chiasma & cavernous sinus.
Best next step?
A.Close observation
B.Medical therapy
C.Inferior petrosal sinus sampling
D.Urgent neurosurgery referral

29.Female came with headache, CT brain : pituitary mass 13mm (1.3cm), best next step ?
A.Prolactin level
B.Inferior petrosal sinus sampling
C.Transsphenoidal surgery
D.Medical therapy
N.B: indication for surgery in pituitary adenoma: more than 1cm or compressing symptoms e.g(headache,
bitemporal hemianopia)

30.One week full term neonate febrile with diarrhea & distended abdominal, he received abx, septic
workup negative,
then ask you(what makes necrotizing enterocolitis least probability?
A.1 week age.
B.Full term baby.
C.Abx.
D.Febrile.

31.Lactating woman came with unilateral breast tenderness,examination: hotness, redness and 5cm mass,
vital: all normal but elevated body temperature, What’s the most important step in management?
A.aspiration
B.I&D
C.abx
D.Close follow-up

32.16 years old female came for evaluation of chronic diarrhea, floating, abdominal pain, no secondary
sexual characters, What’s the most important next step?
A.double contrast enema
B.abdominal x-ray
C-Anti-transglutaminase antibodies (ATA)
D.Lactose tolerance blood test.

33.Acromegaly diagnosed, What of the following he will need in future?


A.Echo
B.Colonscopy
Glory team August

N.B: acromegaly at risk of cardiomyopathy & colon cancer, but they said (in future) so I think they are
pointing to Colonoscopy.

34.51 years old complain of fever, headache & neck pain, What’s the most likely organism?
A.E.coli
B.listeria
C.strept.pneumonea

35.18 months old boy with history of cyanosis came to ER for poor feeding, dyspnea, examination: central
cyanosis, X-ray: small heart with decreased vascular Markings, What’s the most important next step?
A.Digoxin
B.Furosemide
C.urgent cardiac catheterization
D.Ace.

N.B: this is TOF, not necessarily to read boat shape, small heart with decreased vascular Markings are
enough. (Not 100% sure about answer)

36.RA case on prednisone. Methotrexate, hydrochloroquine, he still has symptoms, What to add?
A.Azathioprine
B.Adalimumab
C.cyclophosamaide.
D.Sulfasalazine.
N.B: I think biologic is indicated.

37.Most important factor for esophageal cancer?


A.Barret esophagus.
B.smoking.
C.chronic achalasia.

38.Colon cancer screening in average risk?


A.40y.
B.50y.
C.55y.
D.60y.
According to ACS: 45y, but it wasn’t in choices.

39.Alcoholic patient came for regular follow-up,


labs:-
AST: 140 (high)
ALT: 40 (high)
Ferritin: 400 (high)
Glory team August

HBs ag is negative
HBs ab is positive.
Most likely Diagnosis ?
A.alcoholic hepatitis
B.autoimmun hepatitis
C.Viral hepatitis.
D. hemochromatosis.
N.B: he is immunized or vaccinated for hep b, AST double ALT this is strongly suggest alcoholic hepatitis.
Don’t confuse your self with high ferritin, hemochromatosis is a systemic disease and usually confirmed by
liver biopsy.

40.While you are evaluating pregnant, you find suspicious lesion in cervix, next step?
A.cone biobsy.
B.pap smear.
C.Follow-up after delivery.
D.colposcopy.
N.B: indication for colposcopy: suspicious lesion or abnormal Pap smear (even in pregnant).

41.Female came with lower abdominal pain, N/V PV examination: lesion with poor irregular margins.
Labs: Human Chorionic Gonadotropin: 100.000
Most likely diagnosis?
A.Complete mole.
B.choriocarcinoma.
C.ectopic pregnancy.
D.cervical cancer
N.B: lesion with poor irregular margins with super elevation in HCG this is choriocarcinoma.

42.Same case but she is 10 weeks pregnant, came with hyperemesis gravidarum and lower abdominal
pain, vaginal bleeding,
Labs: Human Chorionic Gonadotropin: 80.000
Most likely diagnosis?
A.partial hydatidiform mole
B.comlpete hydatidiform mole
C.choriocarcinoma.
D.ectopic pregnancy.
N.B: partial mole doesn’t cause hyperemesis gravidarum & very high hcg.
Complete does and more risky for choriocarcinoma

43.Most significant factor for good prognosis post-MI?


A. Smoking cessation.
B. Lowering lipid.
C.exercise.
D.weight loss.
N.B: smoking & hyperlipidemia both are major risk for IHD.

44.Most common cause of death in dialysis patients?


A.IHD
Glory team August

B.infection.
C.PE.
D.hyperkalemia.

45.Mitral stenosis, when to say Critical according to orifice area?


A.<1cm
B.<2cm
C.<3cm
D.<4cm

46.pregnant 13 weeks came with vaginal bleeding,


PV examination: dilated OS, you felt or seen product of conception near cervical os,
Diagnosis ?
A.complete abortion
B.missed
C.incomplete
D. inevitable.

N.B: in incomplete: mother will provide history of passing tissue, if only history of bleeding but os is dilated
this is inevitable.

47.Multipara came for abdominal bulge since 3 years, examination: Diffuse abdominal swelling in midline,
negative cough impulse.
CT: fascia and bowel not involved.
What’s most likely diagnosis?
A. Incisional hernia.
B.epigastric hernia.
C.divarication recti.
D.transverse abdominis weakness .

48.diffuse scrotal swelling and reddness, no pain, no tenderness,


Most likely diagnosis?
A.testicular torsion.
B. orchioepididymitis
C.idiopathic scrotal edema
Glory team August

D.hernia.
N.B: all are tender except C,
hernia? will not cause diffuse swelling + redness

49.testicular pain, examination: absent cremasteric reflex.


Next step?
A.surgical consultation.
B.doppler US.
C.testicular scan.
D.urine analysis.
N.B: absent cremasteric reflex goes with torsion, if u suspected testicular torsion rush to Operation room
or patient will lose his testicle within 6 hours.

50.GI bleeding,
Upper and lower endoscopy done: nothing clear.
Next step?
A.CT.
B.angiogram.
C.capsule endoscope
D.c99m RBC scan.

51.post thyroidectomy, dyspnea, stridor, neck swelling


next step?
A.percuteanous drainage.
B.wound exploration in ward.
C.tracheostomy.
C. cricothyrotomy.

52.pt with vitiligo, with ataxia or difficulty walking . labs findings : low hgb.
what to give ?
A.parentral B12
B.oral B12.
C. Oral iron
D.oral folic acid
N.B: in pernicious anemia, no intrinsic factors in stomach, so don’t give oral.

53.typical Cushing scenario,


Labs: Urinary free cortisol 3 folds high, ACTH: low.Next step?
A.pituitery MRI.
B. Low dose dexamethasone suppression test.
C.CT adrenal.
D.Late-night salivary cortisol test.
N.B: low ACTH rules out pituitary and ectopic causes, Go for adrenal
If ACTH high go for high dose dexamethasone suppression> if suppressed this is Cushing disease go for
pituitary MRI
If not suppressed this is ectopic got for CT chest, abdomen.
Glory team August

Medicine is the easiest & best part, only try to understand.

54.infant with repeated vomiting after feeding, examination: small epigastric mass.
Best next step for diagnosis?
A.barium enema.
B.abdominal US.
C.abdominal x-ray
D.CT abdomen.
N.B: olive-shaped mass > pyloric stenosis

55.Child with flu-like symptoms developed vesicular rash all over the body.
Best lab to confirm diagnosis?
A.HSV-1 IgM
B.HSV-2 IgM
C.Mumps IgM
D.VZV IgM

56.after MVA he has lower limb fracture, orthopedic done surgical correction, after one day sudden
tachypnea, hypoxia, drowsy, examination: patecheal rash over chest and axilla.
Most likely diagnosis??
A.Pulmonary embolism.
B.Fat embolism syndrome.
C.atelectasis.
D.pneumonia.
N.B: patecheal rash over chest and axilla are the cut of between fat embolism and PE.

57.50y old came with epigastric pain and weight loss.


Examination, vitals and labs all normal.
Next step?
A.reassure.
B. Close follow up
C.PPI then evaluate.
D.urgent endoscopy.
N.B: more than 50
weight loss = red flag.

58.pt with cough and fever, CXR: upper lobe cavitation.


Isolation precaution?
A.contact
B.airborne.
C.droplet.
D.fecal-oral.

59.HTN, post MI on ACE, BB, aspirin, what to add?


A.amlodipine.
Glory team August

B.thiazide
B.Atrovastatin.
D.spironolactone.

60.Asthmatic child came to ER frequently and father was smoker and every time the doctor advice him to
quit smoking but he not listening and careless, child right now complain severe attack , what best to do ?

A.Call child protection.


B.Talk to the father about that habit.
C.Tell hospital ethics committee.
D.Ignore father respond and treat the child.

61.patient came to ER with weight loss, tremors and palpitations, ECG: A-fib.
Next step?
A.anticuagulant.
B.echocardiogram
C.Thyroid function test.
D.Chest x-ray.

62.Female came for postcoital bleeding, best next step in management?


A.abdominal US.
B.hormonal assay.
C.examination of pelvic and cervix.
D.pelvic US.
N.B: postcoital bleeding usually cervical cause, could be cervical polyp or other lesion, start by
examination.

63.Pt came with dark lesion in thigh irregular margin and changing in size
Best next step?
A.biobsy
B.Excision
C.close follow up.
D.reassure.
N.B: any suspicion of melanoma go for wide local excision with safety margin, very aggressive cancer and
the worst prognosis in skin cancers.

64.65 years old pt come with weight loss, abdominal distention.


Examination: cauliflower like mass 2cm from anal verge.
Most likely diagnosis?
A.anal cancer
B.rectal cancer.
C.condyloma acuminata.
Glory team August

N.B: cauliflower caused by HPV, HPV is a risk for anal cancer, and weight loss suggesting.

65. child with weakness and numbness in lower limb after URTI, his problem started from down and
ascending upward,
diagnosis?
A.GBS
B.Transverse myelitis
C.Poilomyelitis
N.B: GBS > ascending paralysis followed urti
Poliomyelitis> never causes sensory impairment

66.Couple come by infertility, investigation show bilateral fallopian tube obstruction what will you do ?
A.Tell wife.
B.Tell husband.
C.Tell both.
D.Tell relative.

67.Pregnant came with abdominal pain and uterine contractions, she was given Mgso4 and the
contractions become normal, Then she complained of sudden shortness of breath.
What to do?
A.give her Ca gluconate.
B.discontinue mgso4.
C.measure serum Mg sulfate.
D.IV fluid.
N.B: Mgso4 toxicity, stop it before giving antidote.

68.24y Patient hearing aliens talking to him for months.


what’s the best next step?
A.antidepressants.
B. antipcychotics.
C.benzo.
D.CT brain
N.B: if you see chronic psychosis in elderly do MRI brain, usually tumor or lesion in frontal lobe. Psychotic
disease extremely rare in elderly.

69.child with repeated vomiting,


labs: low k, low na, low cl, high PH, diagnosis?
A. intussusception
B.pyloric stenosis.
C. volvulus
D.GERD.
Glory team August

70.Pregnant 30 weeks lower abdominal pain with proteinuria, BP: 142/88


low platelets and high URIC acid
What of the following indicate severe preeclampsia?
A.her BP
B.Low platelet
C.abdominal pain
D.high uric acid

71.Patient with nervousness irritability and severe symptoms that improves after menses
what is the medical treatment with high evidence to be effective in her case ?
A.intradermal progestogen patch
B.SSRIs
C.Progesterone only pills
D.combined OCP

72.5 years old child with unilateral undescended testes


Tx?
A.orchiectomy
B.Orchiopexy
C.observation.
D.Give him testosterone.

72.patient with chronic diarrhea, abdominal cramping, flushing, respiratory wheezing .


Most important lab to reach diagnosis?
A.anti transglutaminase antibodies.
B.hydrogen breath test.
C.sweat chloride test.
D.5-Hydroxyindoleacetic acid in urine.
N.B: diarrhea, abdominal cramping, flushing, respiratory wheezing all are seen in carcinoid syndrome, (5-
HIAA) urine is very important for diagnosis.

73.14 years eczema:-


A.politeal fossa
B.antecubital fossa
C.face & sclap
D.peri-oral.
Glory team August

74.Female total abdominal hysterectomy and bilateral salpingo-oophorectomy, urine output 35ml/h,
everything normal, doctor decided to discharge.
In the similar case above, what of the following will make the doctor decide not to discharge the patient?
A.absent of oral intake.
B.Fever.
C.analgesia.
D.low urine output.
N.B: fever could be due to atelectasis, pneumonia (concern), or part of systematic inflammatory response
syndrome (no concern)
Low urine output is a could be ureter injury specially in this type of surgery (big concern) or patient is
dehydrated, AKI.
So what would you choose? I don’t know Wallah, follow your heart.

75.Child with seizure for 3 mins, he is febrile, runny nose, examination: congested throat.
best management?
A.phynetoin
B.lorazepam
C.Antibiotics.
D.Paracetamol.
N.B: most likely febrile seizure, runny nose indicate viral so no abx.

76.Case of Tracheomalacia
How confirm diagnosis?
A.CXR.
B.CT.
C.fluroscopy.
D.bronchoscopy
N.B: initially by CXR,CT and fluoroscopy
But Confirmed by bronchoscopy

77.Patient on Lisinopril furosemide, insulin, came for surgery, prophlaxis heparin was given
Labs:High ptt, High K
Which drug will you stop?
A.furosemide.
B.insulin.
C.lisinopril.
D.Heparin.
N.B: heparin also can cause hyperkalemia, but very rare in comparison to ACEI + he need it as prophlaxis.
Glory team August

a mother came with her 17 years dughter whos complaining of amennorrhea, on examination she has
short stature and webbed neck.
Which test are you going to order

1- TSH, FL4
2- FSH, LH
3- estrogen

Patient with gunshot to left chest. Thoracostomy tube drains 2 liters of blood. What is should you do?
- another thotacostomy tube
- thoracotomy
- antibiotic
- embolization

In case of death of newborn What should you do immediately for the parents?
- Ask the father if he would like to file a complaint
- Let the mother spend sometime with her dead baby
- arrange a close follow up with the primary team

What is the precatnage of *Male* smokers in Saudi Arabia?


5%
10%
37%
51%

7 yo diagnosed with T1DM came to ER with DKA and is comatosed. Mother said she stoped his treatment
because she doesn’t think it’s true and wants his pancreas to have time to grow. This represent?
- Denial
- Neglect
- compromising
-?

4 or 7 years old came with his mother she complains he is irritable easily and picky in his food. He drinks 3
glass of *raw* cow milk daily and heats red meat. His mother says that his brother has the same issue.
What is the dx
- iron def
- thalassemia a trait
- something syndrom
-?

40 yo male came with pain in left leg after travail minimal trauma to his hip.
Leg was swollen and tender. X ray shows spiral tibia fracture. What is the dx?
osteoprosis???

Patient came after fall. All is normal except for extreme bilateral heel pain. What should you do first?
- Pain control
Glory team August

- x Ray
- asses distal pulses
-?

BI RAD 3 case ‫اﺣﺔ‬0 ‫ﻮ‬ó‫ﻨﺎر‬A‫ﺴ¸ﺖ اﻟﺴ‬ã


What is the most appropriate next step?
The options were:
Excisional biopsy
FNA
Core needle biopsy
Mammogram after 6 months

Patient came to ER with RLQ pain. CT shows he has appendicitis and 5*13 cm appendiceal abscess that
extends from ... To the abdominal wall. What is the best management?
- Lap appendectomy
- Lap appendectomy and drainage of abscess
- Open appendectomy and drainage of abscess
- percutaneous abscess drainage

In patient with DM at which GFR level should you stop metformin

A- 40-50
B- 30-40
C- less than 30
D- less than 15

Patient with symptoms of portal disease and jundance, US shows a mass that is mostly malignant, what is
the next step?
a. Biopsy
b. Trifastic ct
c. Laprtomy.

Obese male complain of sexual dysfunction normal prolactin low LH


FSH testarone MRI
2.5cm pituitary
- adenoma what is cause of his sexual dysfunction?
- Obesity
- Hyperprolactinemia
- Non functioning pituitary adenoma
Glory team August

Mood of transmition of Congenital adrenal hyperplasia


AR

- vaginal greenish discharge .. trichomonas


Tx ...metro
-tx of community aquired pnemonia
- Ctg pics +managment
-ethics .. mostly repeated
- pic of rash in the hand (maybe HSP) ...what else might have
- baby coes and smile ..age ?
- anal skin tag with fissure tx?
- CBD vs others with jauindince
- meningitis tx (viral vs bacterial)
-cronhs managment
-sarcoma ..investigations if it spread

Patient has COPD what is the best method to deliver oxygen:?


nasal cannula,
venturi mask ,
non rebreather mask

gastric mass 5 cm confirmed gastric ca. what to do :


total gastrectomy

5 yrs female with pubic hair , no clitoromegaly obese, hight above 90 centile ?
DHEAS ) Dihydro.. sulfate)

Short neck short stature htn —- turner

Sudden lower abdominal pain, RLQ tenderness , mass on ex , us 6 cm mass


ovarian torsion my answer

Growing pain what to do ? Reassure and tell the mother to relax

6 yrs child vaccination


Glory team August

* Diphtheria, tetanus, and whooping cough (pertussis) (DTaP) (5th dose)


* Polio (IPV) (4th dose)
* Measles, mumps, and rubella (MMR) (2nd dose)
* Chickenpox (varicella) (2nd dose)
* Influenza (Flu) (every ye

Pt previous 2 abortions , declared D&C , she is pregnant now .... ( forget )


1- asherman’s 2- cervical incompetence.....
Answer: ashermans syndrome

Heart block baby found in ( sle mother )

Pt high B hcg , stable , no sac bleeding on us


- Stable tubal pregnancy

( I well give y dx then we will discuss prognosis and management ) what is called —
- setting Agenda ( my answer

Meconium aspirations with pneumothorax ? - Nitric oxide - chest tube( my answer)

Man want to go to Sudan what to give for malaria prevention


chlorofinquil

Female has fibroid 2*3 become 5*6 , endometrial thickens 5 mm


-Endometrial ca
-leyosarcoma

Strongest indication of thyroid surgery


-Failure medications
-eye symptoms
Glory team August

Systolic murmur decreases with valsalva


- hcom

Systolic murmur radiated to carotid —- AS

Dawn pt with split s2 pan-systolic ventricular hypertrophy — AVSD

3-4qsVaginal discharge 1 candidiasis


2 bv
3 trichomeniasis

Pt sudden chest pain-episodes of heaematemesis -supraclavicular subcutaneous


emphysema->boerhaave syndrome

Pt 3rd trimester abortion +ve lupus anticoagulant-> antiphospholipid syndrome

Old pt-fatigue sign of anaemia hb8 , stools with streak of blood -has hemorrhoids
stage?chronic hemorrhoid
colon cancer?

‫ اﻏﺴﻄﺲ‬٢٠ ‫اﻟﻴﻮم اﺧﺘ ﺎري‬


2Q milestone

)asherman syndrome- < ‫ﻘﻮﻟﻚ‬Ç ‫ واﺿﺤﻪ‬amenorrhea after curettage


Glory team August


‫ﻢ اﺳﺒ√ع ﻣﺎ ﺗﻘﺪر ﺗﺮﺿﻊ‬Õ ‫ﻌﺪ اﻟﻮﻻدە ب‬º ‫ﻘﻮﻟﻚ‬Ç ‫ و‬Sheehan's syndrome

‫ واﺿﺤﻪ ﺟﺪا‬Kawasaki disease

‫ﻌﺪ‬º ‫ﻘﻮﻟﻚ ﻣﺎﺳﻚ ﻓ¸ﺲ واﺿﺤﻪ‬Ç Parkinson's disease-

1. Old pt come decreases in hight what is inx? (- dexa- lateral lumber and thoracic
scan )
x-ray to detect vertebral fracture

75 yo male he can’t stand up 3 mo history after trauma what do you think is trauma:
Falls - gunshots-burn

3. 18 yo female come with N/V , abdominal pain,visual Hallucination ? (Cocaine


withdrawals-schizophrenia - intoxicating 2 other drug I can’t remember it)

Contraindications drug for asthma >Beta blocker

As I said typical case of Parkinson go an read

Long scenario about pt had dialysis 3 day ago central line and now came again for
dialysis peripheral line nurse mentioned he had fever and infection in site of previous
dialysis site what to do ? ( culture and iv abx and stop dialysis - culture iv abx and r.. the center line )
Glory team August

7. Asthma pt take SABA than ich than LABA what to do next ? Ltra- observe how the
pt take inhaler.)

infertile couple the husband has HIV+ve? ( informs husband .- info wife- info all)

infertile couple the wife had PID ? ( tell wife. - tell husband-tell all )

Prophetic endocarditis? Q about drug

Most comes site for mumps > parotid

Pap smear every? > 3 years

Pap smear for start from? 20 to 24 or 25 to 29 or 30 to 34 ‫ﻣﻮ واﺿﺢ اﻟﺴﻮال‬

Colon cancer screening age ? 50

15. Most popular screening test ? My answer was colorectal cancer

6 months old who has active seizures on anti epileptic drugs, he has dysmorphic
features and hypotonia, can’t set with support and can’t roll over.
What to do regarding vaccinations:
1- change OPV to IPV
Glory team August

2- defer DTaP
3- defer all live vaccinations

Picture of pregnancy face mask dx


chloasma in non pregnant
melasma in pregnant lady
???

Elderly with reduced arousal, maybe headache... he admits frequent falls but no injur I think.. no focal
neurological deficit, normal skull x ray:
chronic sundural hematoma
Post concussion syndrome
...

Elderly recently moved to nursing home. Cries in the morning, lost interest in activities, lost appetite and
weight. In the episodes of sadness he has poor memory:
multi infarct dementia
Depression ✅
...

Calculation of GCS

Pericarditis:
ECG, mild symptoms, initial tx? Ibuprofen ✅, prednisone, ...
ECG, sever constant symptoms, best tx? Prednisone, ibuprofen, ...

2-3 PE
I think one ECG was sinus tachycardia
...

Erythema nodosum (painful rash on shins), pneumonia Sx; what's next/investigation > CXR (mycoplasma
most likely)

C. diff two Q's (Abx then watery diarrhea, fever, ... etc), Dx and tx (metronidazole)

SLE multiple Q's:


criteria
what to avoid: smoking
what to avoid (drug): sulfamethoxazole (know the drugs buddy, they change the choices)
developed seizure, what to add: cychlophosphamide
...

RA what to add to MTX for morning stiffness:


hydroxychloroquine
azathioprine
Glory team August

...

Q of URTI then arthralgia, abdominal pain, non-blanchable rash:


hypersensitivity vasculitis
drug reaction
Factor VII deficiency
Vit C deficiency
...!

Burned female covered in soot, burned around nose and mouth:


- intubate and 100% O2 ✅
- hyperbaric O2
- ...

Female with progressive pain and redness on left hand with Hx of knife prick (was injured by a knife):
- cellulitis
- gas gangrene
- NEC FASC
- ...

2 pancreatic cyst Q's, one with fever, one without fever ; management

Peripancreatic fluid: call surgery?


???

Fall, open wound gauze soaked w blood what’s first: IVF, call surgeon (my choice) ✅, ...

Infertility 2 years despite trying, normal semen, cervical exam required (?), monophasic basal body
temperature test result, cause is:
ovulation (probably) ✅
Coital (intercourse)
PID
Vaginal

Know the steps for Dx and Tx of DVT, limb ischemia (pulseless and bad neuro findings), PAD... with all
possible scenarios (multiple Q's)

3 year developmental milestones, 2 Q's, memorize all categories


6 months milestones

Baby feeding well but with cough I think, right lung infiltration, didn't miss any vaccine:
moraxella catarrhalis
mycoplasma pneumonia (probably) ✅
Strept pneumonia
Haemophilus influenzae b
Glory team August

Lady pregnant (I think 7 weeks) with vaginal spotting and previous Hx of miscarriage in the previous
pregnacny. Os closed:
threatened abortion
normal pregnancy
...

Pregnant with reactive CTG, 2-3 contractions/10min, 70% effaced, 1cm os open... what to do to confirm Dx
(don't ask me Dx of what, but maybe labor they meant:
continuous CTG
PV exam 2 hours after
...

Know stages of labor and the management of each abnormality or prolongation.

Sx of HAV (tender hapatomegaly and abnormal LFT), what will be found:


- HAV IgM
- HAV IgG
- HCV
- HBV ... (not sure about the last choices)

‫ﺔ ﻣﻬﻤﺔ او ﺟﺰء ﻣﻦ اﻟﺴﺆال‬Ô√‫ﻌﺾ اﻻﺳﺌﻠﺔ ﻧﺎﻗﺼﻬﺎ اﺟ‬º


ً
‫ ﻣﺜﻼ ﺳﺆال‬surrogacy، ‫ وﻣﻤﻜﻦ‬،‫ﺘﺨﺬ ﻗﺮارە‬ó‫ﻌﻄﻴﻬﻢ ﻣﻮﻋﺪ ﺣ˘ˇ ﻳﺮاﺟﻊ و‬Ç ‫ﺔ أﻧﻪ‬Ô√‫ وﻣﻦ ﺿﻤﻦ اﻷﺟ‬،‫ﺔ‬Ç‫¨ اﻟﺴﻌﻮد‬õ Å ‫ﻤﻪ‬É‫‡ ﻣﺘﺄ ˙ﺪ ﻣﻦ ﺣ‬Ä‫اﻟﺪﻛﺘﻮر ﻏ‬
‫‡ ﻣﺘﺎ˙ﺪ‬Ä‫ﻫﺬا اﻟﺠﻮاب اﻟﺼﺤﻴﺢ ﻣﺎ دام اﻧﻪ ﻏ‬
Glory team August

CTG: early dec (cause), late dec


ECG: PE, pericarditis
X ray: SCFE
CXR of baby with cerebral palsy and resp failure
Skin tag
Molluscum contagiosum

Another Q with erythema nodosum, and lung non-caseating granuloma (sarcoidosis), tx? Steroids

P.falciparum
‫ﻌﺎت‬œ‫¨ اﻟﺘﺠﻤ‬õ Å ‫ﻧﻔﺲ اﻟﺼﻮرة اﻟﻤﻮﺟﻮدة‬

molluscum contagiosum (came today), notice the umbilication (dimple in the center)..

Sle pt with sx of sle on methotrexate wt non pharmacological rx ?


Wt loss
Smoking cessation
——-
Graves , wt inc. orbitopathy — male sex, smoking
———
Sle
Hyroxchlorine and azathioprine
Hydroxycholoro and steroid

——-
Pertusis vaccine — life long immunity , 10 , 20 ,5 yrs
———
Pt trauma driving 130, e seat belt
He is stable wt next?
Us abdn, ct scan , mri
———
60 yrs old male c/o general fatigue and on/off bleeding mixed with stool .
Labs shows low hb . Wt is most imp step in managment?
Colonoscopy
Ct
Us

———
I think smiliar case but on examination there is 2nd degree hemorroide . Wt is the cause of anemia ?
Rectum cancer
Sigmoid cancer
Glory team August

Cecal cancer
Hemorroid
————
Child was in picnic with his family came c/o SOB and wheeze in right lung
Wt you will find in xray
Hyperinflation
Consolidation
???

————
Ride tricycle , use two phrases sentence, difficulty in drawing square
2
3
4
5
————-
Stacks 6 cubes on each other ?
2
3
4
———-
Pt did ERCP and then he developed abdominal and esophageal emphysema . Wt is the cause ,or where is
the injury?
Gastric
Duodenal
Esophagus
—————-
Pancreatic pseudocyst
———-
Dm type 2 developed nephropathy . Wt U will find in UA
Hyaline cast
Rbc cast
Proteinuria
———-
Pt on TPN the he developed weakness vertigo and convulsion . Wt is the cause ?
HypoK
HyperK
HypoMg
Hyperammonia
————
Tumor lysis syndrome — electrolyte abnormality ?
————
Case of wiskott synd.
———-
18 yrs with rheumatic fever has carditis and has valve problem . For how many yrs give prophylactic ?
Age 21
Age 40
Glory team August

??
??
—————
Pt did herniotomy and after 5 days he developed a small mild tender mass at the site of surgery , no
hotness no redness . Dx ?
Hematoma
Seroma
Wound inf.
Recurrence
—————-

Patient with hernia , and he has cough for 6 months and didnt use any treatment for it, came to U and he
was angry and insist to do the surgery and he threatened you if u didnt book him for surgery , wt will u
do?
- do surgery
- consult physician
- inform the administration

2 months , spinal muscular atrophy


With sob, he need intubation
Family refused to do intubation , wt will u do
- ignore and intubate
- consult ethics committee

How to know correct intubation ?


- co2 monitor
- chest movement

Pt taking warfarin and diagnosed with Tb


Take the regimen of tb
Wt will u do ?
- nothing
- stop rifampicin
- increase warfarin

Pcos
Ectopic pregnany ttt

()(
‫ اﺧﺘ·‡ت اﻟﻴﻮم‬٢١/٨
Glory team August

80% ‫ﻢ ﺣ ﺔ ﺻﻌﺐ‬Õ ‫ﺗﻜﺮراااااار ﻣﺴﺘﻮى اﻻﺧﺘ ﺎر ﺳﻬﻞ ﻟﻤﺘﻮﺳﻂ و‬

‫ﻌﺎت‬œ‫‡ ﺗﺠﻤ‬Ä‫¨ ﻛﺜ‬õ ˘ ‫ اﺗﺬﻛﺮ اﻟ ﺎ‬öõ‫ﺣﺎوﻟﺖ ا˙ﺘﺐ ال اﺳﺌﻠﺔ اﻟﺼﻌ ﺔ و اﻟ‬

Q: false increase BNP

- screen poycyctic? US or polyscystin antibody

- advise to DM?
Low carb best than low fat in long mangment / or low fat decrease 5-10% ? / or low glycemic index
decrease 5-10%

- sub gene of HCV in saudi?


1 / 2/3/4

- penciline allergy what C/I ? Meropenem or ceftrixon or tazocin ?

- 1st time microalbuminurea in alb/cr ratio what is next?

Us renal / relate alb-ratio / stop metformin ?


???

ˇõ · ‫¨ ﻏ‬õ ˘ ‫ﻌﺎت و اﻟ ﺎ‬œ‫‡ ﻣﻦ اﻟﺘﺠﻤ‬Ä‫ﻴ‬A‫ﻴ‬A‫ﻛﺜ‬

: Platue Bhcg after salpinostomy after 3 week

What to do? Reassure and follow up or methotrexate

- Honey crusted exudate


Impetigo? Or boil?

- Sign of iron overload in every organ

What investigation ?

Iron load from liver biopsy


Cupper
A 1 anti trypsin
Antimitochondrial ab

]-Prolactinoma ttt? Caprogrlin

-Tonsil exudate and cervical LN , what is most complication? Scarlet / PSGN / pharyngitis (I chose)

- adrenal mass lipid size 5 what to do? Observe or lap adrenalectomy


Glory team August

-Fibroid asymptomatic what to do ? Follow up annually US / fowllo up CBC and us every 2 month?

-Ttt of TOF with cyanotic spell ? Sedation and make him stable(I chose) / diuritc?

- ANA -VE when to screen uvitis ?3 or 6 or 12 month?

- radial injury most site?


Spiral grove of hemurs / olecranon / axillary / wrist

- LT stylomastoid injury during delivery of baby ?


Cant close LT eyelid / loss of ant 2/3 of taste

í
õ ‫ﻪ‬œ‫ﺎدة وﻫﻮ ﻣﺎ ﻓ‬œ‫ˇ و ﺟﺎﻳ'ﺘﻪ ﻟﻠﻌ‬õ · ‫ﺠﻴﻬﺎ ﺑﻴ‬Ç ‫ﻪ ﺳﺆال ﻋﻦ ام اول ﻣﺮە‬œ‫ ﻓ‬percussion ‫ و‬reassurance /
‫– &ﺴﻮي ﻟﻪ‬

Lots of questions about hud, Ttp, itp

Questions about post operative complications ( appendectomy, gastric sleev, wound infection)

Patient with perforated appendicitis after surgery had pus from wound, pain localized to surgical site. No
guarding no fever what is best initial treatment :
Antibiotics
Open drainage
Imaging guided drainage
Wound exploration

Pateint have "hacking cough" .... Forgot the case

Fetus Presnt with orbit nose mouth what is fetal presentation?

Fetus after delivery by ventuze


Nurse noted bleeding in suture lines or something like that diagnosis :
Caput seccondum
Cephalohematoma
Glory team August

Popten ( strange word I forgot)


Othet choise ( not subdural hematoma)

Yellow fever vaccine allergies

1-Pt with malar rash with photosensitivity and artharlagia ,what test is used to confirm the DDx?
A – ANA B-DsDNA C- RF

2- PT developed facial and tongue swelling after Anti HTN medication?


A – Ranilapri B- Bisoprolol

3- Long scenario about pt came with Sx of DKA what type of insulin U will give?
A – according to sliding scale B- Fixed rate Iv infusion C- insulin BID
4- Pt known case of Rheumatoid Arthritis came with RT knee swelling and pain how will U manage = Abx

5- Pt with arthritis with past Hx of UTI (Reactive arthritis)


How to manage? A -Steroid B-

6- Pt 35 YO female her mother died with breast cancer and her sister died with ovarian cancer ,what is the
best screening modality for Breast cance?
A -BRCA Gene B- Mammography C- breast self exam

7- Long case, Flat T wave on ECG what do you find in urine:


A-High potassium
B-Sodium

8- Pt known asthmatic with Flat T wave on ECG what medication can cause that ? A- Salbutamol B-
Salmetrol

9- Chlid came to the ED with gastroenteritis , U discharge him home after telling him to come to the
hospital if there is warning Sx? U are doing
A – safety Netting and other choices I cant remember

10- Child drinking Cow milk 3 times a day with pallor ? IDA (No B12 in choices)
11- X-ray showing radio-opaque mass at McBurney’s point and rebound tenderness... what to do next:
A-Appendectomy
B-CT without contrast

12 - Management of a patient with gastrointestinal stromal tumor of the stomach 5 cm No metastasis No


lymph node enlargement ? In the body of
Glory team August

A- Gastrectomy
B- Wide local excision with Free margins
C- Radiotherapy and chemotherapy

13- Late complication of meningitis:


A-Seizure
B-Hearing loss
C-Facial nerve palsy
D-Ataxia
14- Looong case of ITP with plt 32 how to manage?
A - IVIG and steroid B -Plt transfusion C- plasmapheresis

15- pt is taking UFH for PE after surgery -CBC was provided with Low platelet count what to do?
A – Continuo heparine
B- Replace with Enoxaprine
C- stop Heparien and replace it with bivalirudin (another med with same sound)

16 – Pt came URTI the he developed abdominal pain and rash DDx?HSP


17 – Side Effect from Furosemide? Hypokalemia
18 -Pt 55 Yo smoker and came with breathlessness DDx?COPD
19 – Diabetic came to the ED with well demarcated Erythematous lesion on the shin ? Erysepalis
20- Pt came with retrobulbar pain headache and fever? Dengue Fever
21- Pt came with gait instability and urinary incontince ,he is known case of Osteoarthritis and had cervical
laminectomy after cervical degenerative myopathy what is DDx?
A -Cauda Equina B- Recurrent Cervical Myopathy
22- Scenario of Parkinson ds ,where is the lesion? Substantia Nigra
23- Man came with Ptosis , after examination Ice test and other test were +ve how to manage?
Pyridostigmine
24 – a case of macrosomic baby and Complications =Erb's palsy
25- Pic of Chloasma gravidarum
26- Pic of urethral cruncle
27- Pt with PE and hypotensive ,management ? Alteplase
28- Pt with UTI and high creatinine which Abx is contraindicated? Cipro
29 -Most common diagnosis of Pt with Fever from Sub-Saharan region?Malaria
30- Child can’t handle spoon well But he can walk up the stairs with one hand held What is his age: 18
months

31-Pt came with pin prick sensation and loss of vibration sensation with loss of ankle reflex and K was 5.5
and Creatinine was very high what to do?
A – Vitamin B 12 B – Dialysis c- Bicarbonate
32- child came for routine checkup U found a murmur that is late systolic changing with position ? I think
this is innocent murmur
33- a child with murmur ,I think he was anemic ?treat the cause
34- Pt came with pleural affusion after 2 weeks from pneumonia infection what is the DDx?
Parapneumonic effusion
35- child came after RTA with blood coming through Ear? Basal skull #
Glory team August

36- Pt fall from 4 meters he is vitally stable and GCS is 15 but he is complaining from Bilateral heel pain
what is the next step?
A - xray for Both Feet B- Pain management
37- Soldier with bilateral groin hernia how to manage ?
A -Lap with mesh B- Open with mesh
38 – Long case of pancreatitis after ERCPs ,which of the following will increase the survival rate of the pt ?
A – abx B- Ringer lactate
39- Low grade dysplasia with barret esophagus ?how to manage
A – Resection B – all other choices were medications c-PPI
40-where do U find the level of uterus in pregnant lady?
A – medway between symphysis pubis and umbilicus B -above Umbalicus
41 – case of molar pregnancy with US pic
42- pt pregnant with fundal height more the GA and hemoptysis DDx
?Choriocarcinom ?
43- A lot of cases with Ectopic pregnancy
44- 3 cases about Trichomonas management in different scenarios
45- case about vaginal candidiasis
46- Drug approved by FDA for tx of Cancer hypercalcemia? Denosumab
47- most common cancer in pediatric age? Leukemia
48- Polymyalgia Rheumatic with fatigue ?CLL
49-Hashimoto Thyroiditis associated cancer? Lymphoma
50-mamngement of acute Migraine headache? Aspirin
51- management of Kawasaki ?Aspirin
52- Nurse with +ve HCV antibodies and -ve HCV Ig M what to do?

‫ﻌﺪ‬º Ëõ Å ‫ﻪ ﺳﺆال ﺟﺎ‬œ‫ﻓ‬


Pt after cardiac bypass ,developed hypotension tachycardia and decrease SVR?what is the type of shock

1-hypovolemic
2-cardoiogenic
3-Septic

‫ ﻣﻮﺿ√ع اﻟـ‬ö‫رﻛﺰوا ﻋ‬Nephrotic syndrome

Ëõ Å ‫وﺳﺆال ﺛﺎ‬
Pt after MI he is on clopidogril ,BB ,ACEI
Glory team August

Lipid profile ‫ﻪ ﻧﻮرﻣﺎل‬œ‫– ﻓ‬í


õ ‫ﻞ‬Õ
‫ﺎﻧﺖ‬Õ ‫ﺎرات‬œ‫واﻟﺨ‬
A-Atorvastatin
B-Fibrates
C-Niacin

Ëõ Å ‫اﺧﺮ ﺳﺆال ﺟﺎ‬..

low hdl
3 and the normal as they want more than 3.010

‫ﺲ اﻟﻈﺎﻫﺮ ﻛﺬا‬Ã ‫ ﺗﻔﺮق ﻣﻌﺎﻫﻢ وﻻﻻ‬0.01 ‫ ﻣﺎﻋﺮف‬1

Elderly with black discoloration and elvated from epithelia surface what to give ?
Chemo
Radio
Bunch
Execiopn

*A pt has malignancy in his face how are you going to deliver the information to pt

Explain the pathophysiology to the pt


Build the trust

Most of the ethics are repeated


No question from biostatistics

*Kawasaki disease case dx , treatment and indicator of poor prognosis. ( 3qs )

Iv Ig

elevated crp

*WIDE FIXED SPLITTING S2 à Atrial septal defect

*Pregnant 34 or 36 à forgot c/o painless mild vaginal bleeding did us dx as partial placenta prvia pt stable
ctg was done and is reactive what will you do next

-Immediate CS
Glory team August

-Biophysical profile
-Amniocentesis to check for lung maturity
???

*Asking about presentation u did pv examination you feel nose mouth and orbit

-It is Face presentation

*39 weeks she is in active labour contractions 3 times every 10 minutes, cervical dilation 6 effacement 80%
fetal presentation when u did pv examination u feel mouth nose and orbit
What will you do?

-CS
-Give oxytocin

*Pt 36 weeks presented with strong contactions , os is closed and no effacement.


True labor
False labor

*Question about euthyroid sick syndrome asking about the dx

*Child had pneumonia during examination u heard a murmur although examined the child a month ago
there was no murmur

Innocent murmur
Valvular heart disease ( don’t remember stenosis or regurgitation.)

*Lots of questions about hud, Ttp, itp

*Questions about post operative complications SSI ( appendectomy, gastric sleev, wound infection)

*Patient with perforated appendicitis after surgery had pus from wound, pain localized to surgical site. No
guarding no fever what is best initial treatment :
Antibiotics
Open drainage
Imaging guided drainage
Wound exploration
Glory team August

*Yellow fever vaccine allergies egg allergy à repeated.

*Pt diagnosed as sinus headache for 2 years , no improvement with medication of his headache , what to
do ?

Paranasal sinus ct
Careful review of history and examination
There was no brain ct

21-08-19
Mostly reapted from August and July, however some options did change.

Two Senartios of patients post operatief, with mild symptos such as fever, chills.
Analysis: > wbc
a. Bacteremia
b. Sepsis
c. Sever sepsis
d. Systemic inflammatory response
???

Patient who is tall, reduced body weight, mitral valve problem, cavum expe…:
a. Marfan
b. Dowm
c. Congintal something

Risk of hcv:
a. 3
b. .03
c. .30
d. 30

Child with suncked fontanel, moderately dehydrated, weight 10 kg asking about who much floid should be
given?
a. 1000mg
b. 1500mg
c. 2000mg
???
Glory team August

Female child with hypotension and decreases natruim in serum, by examination large clitros, dr. gave
normal saline.
What to be added?

a. Bicarbonate
b. Hydrocortisone

Child who who say mom, put his hand on his mouth and say mouth. How old?
a. 10m
b. 12m
c. 18m
d. 24m

Patient with symptoms of portal disease and jundance, US shows a mass that is mostly malignant, what is
the next step?
a. Biopsy
b. Trifastic ct
c. Laprtomy.

Women with premenstrual syndrome, change in daily activity, affect her word preformace, recurrence visit
to ER, what is affected,
a. Mood swing
b. Irritable

Mother of child with diffeclt behavior has be advised to deal though positive reinforcement, what can
help?
a. Parent leaning programe
b.
???

RTA patient with normal air entry, normal breath sound, chest tube show minumin blood, suddanly patints
is hypotensive with < JVC and distance heart sound, what is you immediate action>
a. Fluid
b. Precardiothesis

Patient with palpitation and sweeting, and discomfort in the neck. Elevated t4 and reduced TSH. What kind
of thyoid disease?

a. Hishamoto
b. Graves
c. Toxic goiter

Patients with muscle weakness, increase fatigue, dysphagia and other neuomusclar weakness symptoms,
no eye symptoms what is the diagnosis?
a. Motor nuron disease
Glory team August

b. Mystenia graves
c. Mystenia graves syndrome

Patints with cardic pain, mild elevated temp, ECG: diffuse ST elevation, what is the treatment?
a. NISDS
b. Abx

Patient presented with sudden chest pain, radiate to the left arm, ECG ST elevation in AVF, patints started
on asprin and clopdrogyl, what in the next immadated action?
a. Righ ECG
b. Ct angio

Elderly women with PIP and DIP pain, no significant morning stiffness, PIP swelling associated with
something, RA -, what is the diagnosis?
a. Osteoarthritis
b. RA
c. Senonegative RA

Patient with malar rash and other SLE symptoms, patints uses methotaxate, perdnesone and NISDs,
despite compliance comes with sever pain, what to add?
a. Azathioprine
b. Adalimuab

Eldary with high blood pressure on several meting, what to do?


a. Repeat test
b. Starts bb
c. Start ccb

Eldary Women increase frequency to urinate, high voding, and the feeling on incomelet emptying, on
examination, palbale bladder after urination, what is the diagnosis?
1. Urge incont
2. Stress incont
3. Overflow
4. Reflx

Scenario of PE with ecg shows S1Q3T3


Which vacnation whould admisted at 2 month.
Pregnancy vaccination> influnza
Eeg allergy> yellow fever
Mior and major criteria of RA
Esophgal cancer high grade barret> resection
Anti acid 20mg use> increase the does or switch ?
Glory team August

Remember the heart murmur


Systolic: AS, PS, MR, TR
Dystolis: AR, PR,MS,TS.

Case of ihd in patient previously had rhematic fever and have mitral regurgitation

What is best treatment

Digoxin

Diuretic

Surgical repair of mitral regurgitation

Pt has chronic hepatitis B his wife is worried if got infection what lab ask to check for acute hepatis

Hbeag

Hbsag

Anti hb

Mother with hepatitis B the mod of transmission to baby is considered


Primodial
Primary
Secondary
Tertiary

1-Mass i think in forearm (lipoma)?


-incisions
-exicition(my answer)
-core bx

2-pics of melasma +skin tag+miss abortion in Us+3 ECG +CTG


3-pic of stepping reflex (see google)
4-SLE case on NSAID not improve what to add>methotrexate (my answer)
5-abortion type ?many Q
6-ectopic pregnancy ttt? Many Q
7-ethics repeated
8-case of whooping cough>pretsuss
9-case of barking cough not improve with epinephrine what to do>repeat epinephrine dose
10-5As to stop smoking??i think ask assess assist arrange follow up
11-2 Q about postmenpuase bleeding ? I forgot the Q but easy ‫ﻌﺎت‬œ‫ﻣﻦ اﻟﺘﺠﻤ‬
13-5 Q about milestones (easy)
14- many Q about COPD (‫ﻌﺪ‬º ‫ﻌﺎت‬œ‫)ﻣﻦ اﻟﺘﺠﻤ‬

15-Q was about couliflower pattern with gray color something like that?
Glory team August

-anal ca
-condylomata acuminata(my answer)

16-egg allergy>yellow vaccine


17-baby with vaginal bleeding +itching >foreign body
18-treatment of HF >ACEI +diuretic
19-screen of colon ca>50
20-screen of osteoporosis >65
21-target HBA1c in diabetic pt ?
22-target warfarin in afib>2-3
23-treatment of MI?
24-screen for uvitis I didn’t remember if ANA positive or negative!
25-from where we take consent for LP in child 15 years old ?
26-treatment of pericarditis >ibuprofen (my answer)
27-girl with sweating +palpatation and fear of die>panic attack
28-clear scenario of turner ask for diagnosis!
29-case of PCOS with investigations what you would like to ask more >blood glucose and lipid profile (my
answer)
30-pt with long Hx of GERD and there is high dysplasia with atypia by Endoscopy >refer to surgery to do
resction

2 Q's I remembered from yesterday and hadn't seen them anywhere:

) After blood transfusion, a patient developed pain at the site of the needle, chest pain or tightness or SOB,
and fever:
- febrile non-allergic reaction
- allergy/anaphylaxis...
- hemolytic reaction

) Patient on methotrexate therapy, after a while he had abnormal LFT's, how do you diagnose injury from
methotrexate:
- LIVER BIOPSY
???
Glory team August
Glory team August
Glory team August
Glory team August
Glory team August
Glory team August
Glory team August
Glory team August
Glory team August
Glory team August
Glory team August
Glory team August
Glory team August
Glory team August
Glory team August
Glory team August
Glory team August

1- Case about which of the following causes Dyspepsia:


a. Helicobacter Gastritis true
2- Child has Epilepsy on Antiepileptic drugs but not controlled and he’s still having seizures, What
recommended for this patient regarding Vaccines:
a. Change IPV to OPD
b. Defer DTP
c. Defer all live attenuated vaccines
d. Defer all vaccines
3- Most common cause of Shock in pediatric age group:
a. Septic
b. Hypovolemic
c. All other types of shock
4- Patient came with repeated infections:
a. Chronic Granulomatosis Syndrome
5- Case of Arthritis & Splenomegaly
a) Felty syndrome
6- Colon Cancer screening for 40 man pt with no family hx:
a) 45
b) 55
c) 75

(No 50 in options)
7- A lady with suprapubic pain and diffuse vaginal discharge with pain in fornices:
a) Acute salpingitis
8- A woman plans to get pregnant next winter which vaccine should she take?
a) Rubella
b) Influenza
9- Pregnant women has a child in a school that has some contagious infection what should she take?
Varicella IVIG

If they did not mention hints for chicken pox, give influenza vaccine
Glory team August

10- A man with 3 days of consistent anal pain with an anal tender mass 1cm*1cm with normal
temperature:
a) Anal abscess *
b) Prolapsed piles

If thromous pills not in choice anal abcess


Prolapsead hemmoroids usually painless and non-tender it will be painful if thrombosed

11- 49 year old female complain of yellowish discharge ,itching ,and pelvic tenderness what are the
diagnosis?
a) Trichomonas
b) atrophic vaginitis
12- there is case with right lower abdominal pain , tenderness ,and intestinal obstruction ?
a) ectopic pregnancy
b) rapture ovarian cyst
c) PID

If pregnancy test +: abdominal ectopic pregnancy


IF - : PID
13- screening test of colon cancer with medium risk ?
a) 50
b) 45
c) 55
d) 65
Glory team August

14- patient from Mediterranean area he has signs and symptoms of G6PD which of the following drug
we should avoid?
a) aspirin
b) paracetamol
c) amoxicillin
15- 45 year old patient with signs and symptoms of small intestinal obstruction x-ray shows multiple air
fluid what are the diagnosis ?
a) small intestinal valvular
b) mesenteric ischemia

mesenteric ischemia will have finger printing or pneumoatosis intestinalitis


and the most common symptoms are pain after eating and bloody diarrhea
(on examination out of proportion = sever pain with normal examination)
16- 32 week pregnant came for regular follow up, she went to call her husband to be with her, then she
become pale push her abdomen then she fail down like coma, when she wake up she want to know
what happened, her BP is normal and the baby is normal what you will do for her:
a) Admission
b) Termination
c) ask her what happened with her husband ( I choose this one)
17- 6 years complain of itching, vaginal bleeding. On examination, little vaginal blood:
a) Candida
b) F.B
18- Old patient with DM.HTN on medications, have symptoms of TB, started on first line TB drugs, which
medication should be stopped? It was long scenario
a) Rifampicin
b) Other options was anti-HTN medications

Rifampicin will decrease effect of antihypertensive drugs


19- Multi-trauma patient has decrees blood pressure, low O2 saturation and bleeding what you will do
for him
Glory team August

a) intubation
b) OR
c) You will do consultation
d) Code blue ( multi-team)
20- surgeon want to do appendectomy he is not find appendix how is he can get it ? by following?
a) Terminal ilium
b) Iliocecal valve
c) Tenia colitis
21- pt female have RTA after stabilization the doctor know she is pregnant what is indication for Rh
immunoglobulin?
a) RH+ and abdominal trauma
b) RH+ and wrist fracture
c) RH- and abdominal truma
d) RH- and wrist fracture
22- What is fourth in American criteria in lupus?

23- diabetic patient on metformin 1or 2 gram (he didn't remember) fasting blood sugar ha1c are high
what to do next ?
a) increase 1 of gliptin
b) Increase the dose of metformin
c) Do nothing
d) another class of antidiabetic drug (he didn't remember )

“add”
Glory team August

24- adult male compline of Rt knee pain and swelling( Comparison table between normal and abnormal
value) whte the diagnosis? In the table ( yellow, mucos bug , normal wbc , more than 80% of fluid
polymorphonuclear cell ?
a) Septic artharitis
b) Pseudogout
c) Gout
d) Rhomatic artharitis
25- pediatric , come him family complain of skin manifestation : erythematous rash spread in trunk face
limp with enlargement of inguinal and axillary lymph nodes (picture ) how to treat?
a) penicillin
b) steroid
c) antiviral
d) antifungal
26- patient now in case of afib come by lower Limb pain no distal pulse in lower Limb what you will do ?
a) Femoral artery thrombectomy
b) above knee amputation
c) Catheter thrombolytic therapy
d) embolectomy without imaging
27- child patient come complain with distal femoral mass near knee joint , x-ray show curetic lesions
what is the diagnosis ?
a) osteosarcoma
b) chondrosarcoma
28- elderly diagnosed with osteosarcoma Mets work up :
a) bone scan
b) Chest CT

Most common site of mets : LUNG


29- 28 yeah all patient with distance jugular vein increase with respiration , systolyic murmurs ,
shortness of breath :
a) pulmonary stenosis
30- afib INR gole
Glory team August

2-3
31- chest pain , SOB ,CTG PIC of pericarditis : what you will find by clinical examination :
a) Murmur
b) Pericardial rub
c) S4
32- MI develop 2nd heart block, hypotension 46/40 , givin IV atropine, no improvement?
a) Adinosin
b) Temporary pacemaker
33- transient viral SA , lymphoma ?
??
34- Patient treating for bronchogenic carcinoma , came by distended jugular vein quite heart sound ,
what's next step to diagnosis ?
a) Chest x-ray
b) Eco
c) ECG
35- female have SCA child married for second time , screening for SCA :
a) electrophoresis for husband
b) electrophoresis for mother
c) genetic study
36- pt with history of PID recover ,BMI 35 , but no sing of Pcos , did investigation all came normal , hsg
normal and semin analysis of husband normal , Try to conceive 2 year what is the treatment ?
a) Ovulation induction and intercourse
b) Ovulation induction and artificial insemination
c) Reduce BMI
d) IVF
37- When to start oxygen therapy?
a) Less than 7.9 KPa or 7.4-7.8 KPa in presence of cor pulmonale
38- 5 CTG cases
39- Vaccines schedule for 1 year old:
a. MMR, OPV, Pneumococcal & Varicella
Glory team August

40- Pseudopancreatic cyst case mx:


a) Percutaneous drainage
41- Sxs of croup for dx and another Q asks about the organism (organism: influenza, parainflunza)
42- Ectopic pregnancy mx Qs
43- case diagnosed with diabetes since two month ago how we can follow up for nephropathy ?
a) creatinin to albumin ratio
c) microalbuminurea
44- female with ectopic pregnancy and she lives far away why we should go for surgery ?
a) Because she lives far away
45- Case of celiac disease what do you see in biopsy , and there isn't lymphocytes in the choices ?
a) Villus atrophy + crypt hyperplasia
46- 7 year old child with right testis in abdomen what are the ttt?

Look if pre- puberty. Orchydopexy is standerded.


If pt has short vas difference do orchedectomy
(always orchydopexy unless orchydectomy is indicated)
Glory team August

47- 6 years came with mother to ER complaining of vaginal bleeding , the mother said that the bleeding
when she came from school. On examination, there is bruising on the labia majora and vaginal
bleeding.
a) Sexual abuse
48- MVA came to ER with open wound in front of neck , ‫ﻌﺾ‬º ‫ اﻟﺠﻠﺪ ﻣﺘ ﺎﻋﺪ ﻋﻦ‬BP little decrease, O2 74,
what you will do?
a) intubation
b) tracheostomy

cricothyrestomy indication: failure to intubate, failure to oxygenate


49- Trauma patient, on examination: left lung decrease air entry and dullness on percussion, on x ray
black , what you will do:
a) left chest tube
50- Hepatitis B on labs : HB surface ant antigen was positive , I do not remember exactly
a) active hepatitis
b) received vaccine
c) he infected from others

51- pt want to do adrenalectomy what to give him preoperative?


a) Hydrocortisone
b) Benzophnylediazepine

Steroid to avoid adrenal crisis, in case of pheochromocytoma give alpha blocker (phenoxybenzamine)
52- pt do gastrectomy what vitamin will decrease?
a) Vitamin 12
53- What is cardiac anomaly in down syndrome?
Glory team August

a) ASD
b) VSD
c) COArtication of aorta

NO AVSD in choises

54- UTI infection sign and symptoms what is next step


a) u/a and culture

Only UA
55- case pregnant with DIC PT and PTT IS HIGH And fibrinogen is low ?
56- pregnant with hx of previous cs and she is now pregnant by twin one is breech and second is cephalic
what is ci for ecv?
a) Fetal precentation
b) Previous CS
57- Patient have follicular thyroid cancer he did right hemithyroidectomy after surgery they Discover
some malignant cell didn't removed but not in vascular way or invade anything
What you will do?
a) Radio active Scan
b) total thyroidectomy
c) Observation / follow up
58- Child septic arthritis start in antibiotic, culture come for postiveto MRSA What to do next ?
a) shift 2 vanco
b) ultrascope Wash Out

(in septic arthritis you have to drain and give proper AB)
59- Question about acl tear
60- Mid diastolic murmur with history of rheumatic fever :
a) mitral stenosis
61- picture abute cyanotic heart disease congenital heart disease :
a) etralogy of fallot
62- question about nocturnal enuresis
63- Ha1c goal in diabtic
64- pertussis imunity for how long?
a) 5 year
b) 10 year
c) 15 year
d) 25 year
65- 21 year old patient with whooping cough what the diagnosis?
a) Bartodella pertusis
66- what indicate sevirty of asthema ?
a) peak expiratory meter
b) Respiratory rate
Glory team August

c) Saturation
67- Pt respiratory rate more than 20 , peak expiratory meter less than 600

Here the answer is: peak expiratory meter less than 600
68- Pt respiratory rate more than 25 , peak expiratory meter less than 25

Here the answer is: RR more than 25

69- Case about copd exacerbation , patient drowsy excretion in the mouth
a) Nasl CPAP
b) mecanial ventlation
c) O2 face mask

COPD depend on level of conc. If GCS 15/15 bipap , if drawsy intubate


70- malaria prophylaxis to patient who want to go to sudan ?
71- malaria prophylaxis to patient who want to go to Southern Saudi Arabia ?

malaria proph.: Malarone (atovaquone/proguanil), and mafloquine


72- Question about vaginitis , abortion , abruptio placenta, pcos , fibroid .
Glory team August

73- 41 year old with abnormal uterine bleeding , what the mangment ?
a) Oral contraceptive
b) Gnrh agonist
c) Ablation

Second part August 28, 2019 record


1. Female did tubal ligation complete her family came C/O spotting and late period for 6wks what you
will do ?
● Pregnancy *(test to exclude pregnancy)
● Pelvic US
● Laparoscope

2. 30 years female c/o infertility and she is irritable and depressed ‫?? إˆﺶ ﺣﺘﻼﺣﻆ ﻋﻠﻴﻬﺎ‬
● Loss of eye contact

3. 52yrs old pt ē multiple ulcer in lesser curvature hematemesis and dark stool on drugs and no
improvement what is the next step
● Partial gastrictomy*
● Total gastrictomy
● Ablation

4. 13girl ē vesicular rash in trunk and extremity?


● Herps type1
● Herps type 2
● Varicella *
● Meningococcal

5. 38 wks in labor severe preeclampsia magnesium given and oxytocin administered epidural
anesthesia for pain pt stable ctg show early deceleration ?
● Fetus engaged (head compression)*

6. same Q above +Bradycardia ctg >> epidural anesthesia*


● oxytocin
● -fetal head compression
● -mg4

7. 25yrs had one solid nodule in thyroid , what is the next step ?
● CT neck
● FNA
● Radioactive scan
● MRI
8. lady ē anexity can’t deals with co worker
● Give her instruction how to deal with them

9. *Young age c/o painless bleeding during defecation diagnosis?


● anal fissure
● hemorrhoide
● anal cancer
● fistula

10. Diabetic pt uncontrolled with diabetic foot and gangrene , you must do amputation surgery and pt
refuse ?
Glory team August

1-Do surgery
2-let pt sign refuse DAMA
3-refer another doctor
4-inform Ethics committee

11. old pt with cancer colon ,prepared for surgery at time of operation pt BP start become low
hypotension with tachycardia HR 140 what you will do?
● termination of surgery
● Make ECHO
● chest x-ray
● convert of surgery

12. child with flu-like symptoms developed vesicular rash all over the body best lab to confirm diagnosis?
● HSV-1 IGM
● HSV-2 IGM
● Mumps IGM
● VZV IGM

13. infant with repeated vomiting after feeding , examination :small epigastric mass< best next step?
● barium enema
● abdominal sonography
● abdominal Xray
● CTabdomen

14. Old pt bed ridden with basal ganglia and sever loss of weight , absent gag reflex , how feed pt?
● NGT
● OGT
● parentral I.V
● geogenius tube

15. newborn girl with asymmetrical breast fold < next step?
● MRI
● pelvic US
● cortisol level

16. Pregnant came for antenatal checkup, ultrasound showed an empty gestational sac -there was
empty SAC no baby- Cervix closed what is the dx?
A. anembryonic fetus - pregnancy –

17. Man came complains of watery diarrhea (no mucus - no blood ) abdominal pain and discomfort - one
day ago he arrived after a trip to India what is the most likely diagnosis?
● E coli
● salmonella
● Brucella
18. 30 years old female known case of inflammatory bowel disease she plans to be pregnant she is on
medication since 3 years. despite the treatment, she has a permanent Hx of melena she did
sigmoidoscopy or endoscopy (can't remember ) what you will advise her?
● avoid pregnancy
● plan for pregnancy at least after 3 years
● till her, she can be pregnant but there is risk of abortion
● can't remember the 4th choice but it was un logic answer
Glory team August

19- case of IBS (the diagnosis was mentioned ) they ask what is your advice for him?
● decrease fruits in his diet
● decrease vegetables in his diet
● avoid mint oil

19. housewife get oven burn in her hand lead to pustule formation then she noticed green discharge at
the site of burn what is the most likely organism?
● -klebsiella
● -pseudomonas
● -S.pyogen
● -E coli

20. 15 years old boy brought to the Emergency department by his parents.
he has acute meningitis.the doctor wants to do LP immediately. from whom you will get the consent?
● parents only
● parents and child
● child only
● no need for consent it is an emergency case.

21. patient has liver cirrhosis (other clear details ) he has huge abdominal ascites what medication you
will use for him to solve the ascites?
● -aldosterone antagonist
● flutamide-
● -thiazide
● -AECs

22. 2 months old baby (first baby for his mother) feeding well - normal growth but he has vomiting
between meals what will make you think he needs further investigation?
● he is boy-
● failure to thrive
● vomiting after feeding-
● vomiting between meals

23. RTA patient he has leg fracture they did splint for him later on he came complaining of pain and mild
swelling at his leg .. which of the following you will consider it as alarm sign in his case ?
● swelling
● pain-
● pulseless
● paresthesia between his fingers

Recall questions topics


August glory team
1. fetus Hsp pic
2. Svt ecg
3. pic ECG heart bloc k asked about which dgree 1st. 2nd. 3rd. 4th
4. Molluscum contagiosm pic
5. Picture of volvulus Xray , Asked about the part involved?
6. I got 5 pics
1- late deceleration ( causes )
2- normal ctg
Glory team August

3- choalsma
4- endometrial pyolyp
5- growth chart , failure to thrive
7. pic spherocytisis? A.do osmotic fragility
8. pic of sickle cell rbc? A.hb electrophesis
9. Anteroseptal wall MI ECG ?
10. Picture of hysteroscopy
11. DKA management & Nephrotic nephretic dx
12. Child abuse 2 cases
13. Causes of sudden death in infants
14. Cardiology

15. Pic of volvulus ? Which part? Was cecum


16. endometrial cancer ttt investigation

17. ECG
18. Croup mgt

19. Cervical mass pic

20. Xray TB

21. 2 ulcer perforation

22. Crohns pedia


23. SLE managment depending on the presentation

24. Femur angulation management

25. Radial nerve injury

26. orbital cellulitis what to do next

27. Ethics same as collection

28. Hernia cases dx and mx

29. Jaundice cases dx and mx

30. Smears dx and mx

31. Toxic shock syndrome

32. Bowel obstruction dx and mx

33. Osteoporosis, cervical cancer, pap smear screening

34. Complications of GDM and GHTN

35. GDM mx
36. milestone

37. Mile stone


Glory team August

38. COPD

39. Asthma cases

40. Tumor lysis syndrome

41. CLL
42. C.diff (Abx then watery diarrhea, fever, ... etc), Dx and tx (metronidazole)

43. Fetal Brady

44. Placental abruption

45. Kwasaki disease

46. Adrenal mass mx

47. Cardiac temponade

48. Eradication of H.pylori


49. Persistent vomiting mx
50. DIC , HUs Complicated labor , Molar sx & mx
51. thyroid “literally everything”

52. Inferior MI case with ECG

53. Aortic dissection Clinical signs

54. Mitral stenosis


55. PE “5 questions”

56. UTI in Medicine gyne & pedia

57. Aortic regurge case >> Scenario asking about dx by giving you Clinical signs

58. DVT management

59. indications of IV PPI

60. COPD & bronchitis


61. management “COPD”

62. Signs & symptoms

63. Which one has clubbing

64. Dx Asthma
65. PE ( ECG finding ; sinus tackycardia ?)

66. PFT interpretation

67. Management

68. Gall bladder issues


Glory team August

69. Complications

70. Management
71. Multiple CTG pics (3-4) what is the finding?
72. Deferentials of Vaginal spotting in post menopausal?
73. Cervical lesions dx?
74. Dermatological issues of vulva
75. 2 pancreatic cyst Q's, one with fever, one without fever ; management
76. Diff questions. Easy to pick up by scenario.
77. Post-op complications know them by days.
78. Meningococcemia clinical presentation and asked about the dx?
79. question about open fracture management ?
80. vaccination of 2 month
81. maintenance of 10kg babe
82. Rt kidney and Lft kidney ethics question
83. Drop wrist >>radial nerve injury >>site spiral groves
84. Prophetic endocarditis? Q about drug <
85. Infertility causes “why would she can’t get pregnant “ one case she had previously done D&C >> so
Asherman + know the indications of intrauterine insemination
86. Hormonal disturbance in case of PCOS
87. .how to use anticoagulants many questions about it
● Chronic renal failure which anticoagulants?
● Female with fracturer which anticoagulants
88. Presistance breech how to deliver?
89. Endometriosis increases risk of what ?
90. Calculation of GCS
91. Pericarditis?
Glory team August

AUGEST 24 , 25

● predinsolone recovery resistance >


A. 4 weeks
Answer : A

● shock hypovolc shock :


A. increased wdp
Answer : see below table .

● TB side effect gout >>


A. pyrid
Answer : A

● fondaparinux (LMWH) >


A. after surgery
Glory team August

Answer : A
NB: In general anticouaglants are initiated few hours pre op with the exception of fondaparinux can be
started hours post op (which was mentioned in the Q)

● MRSA resistance ttt >


Answer : see below
)Cover-MRSA>
Ceftaroline, Vancomycin, linezolide.
)Skin-MRSA >
TMP/smx, doxycycline, clindamycin

● newborn illegal need ER surgery >>


A. do it
Answer: A

● female post operative PE hepratosation InR normal what u do


A-stop heparin
B-shift to subcutanios
C. stop Heparin, continue Warfarin
Answer: C

● asmtha moderat >


A- saba , icd , laba
Answer: A

● 6 months baby vomiting failure ti thrive > diagnosis ?


A. Pertussis
Answer: A
Glory team August

● flu ike sympy sob :


A- neubliz
B epinephrine
C-steroid
Answer :
NB: Depends:
If mild to moderate >> steroid
If severe > epinephrine
Mild mean stridor with crying or agitated
Severe mean stridor at rest
Glory team August

● steeping reflex for baby disaapper > 2 MONTHS


NB: All reflexes disappeared in 6 months except: rooting in 4 months, stepping in 2 months, babianski in 2
years.

12 - mother dm delivered on examination baby paralysis one side >


A. ERB’S PALSY
Answer : A

● baby MECONIUM ASPIRATION >>


A. nitric oxiad
Answer: A
● IHD on enosaprin , losartin what to add >>
a – bb
Answer: A

● Crescendo decresndo➡
A. AS
Answer : A

● Postperative which we’ll be decrease:


A. Insuline
B. Glucagon
Answer : A
NB: Pt with DM on hypoglycemic agent, we shift him on insulin prior to operation, then we resume oral after
operation
● Sexual abuse:
A. 6 oʼclock
Answer : A

● Croup treatment :
A. nebulizure epinephrine
B. oral streiod
Answer see below :
NB: SAME AS Q 10
19- Coarction of aorta typical feature
( difference between upper lower pulses )

● Lyioma pragnancy changes:


A. red degeneration
Answer : A

● Asked about clinical pic of Kawasaki?


A. ( conjunctival injection without exudate )
Answer : A

● Strawberry cervix itʼs :


A. (trichomatsis vaginalis)
Answer : A

● Ttt of trichomasis ?
A. (metronidazole)
Answer: A

● Trichomoiasis under microscope?


Glory team August

A. (Flagellate )
Answer : A

● Ttt of wolf Parkinsonʼs synd


A. (Digoxin)
Answer : SEE BELOW :
NB: The Heart Rhythm Society Policy Statement on Catheter Ablation states that catheter ablation is
considered first-line therapy (class 1) and the treatment of choice for patients with WPW syndrome.
WPW syndrome ttt
If unstable; cardioversion
If stable; Proconamide
If refractory; Radiofrequency ablation

● thenar musle atrophy .


A. (Median nerve palsy)
Answer: A

● Normally ovulation occur in


A. 1) after 36 hour from LH peak
B. 2) before 36
C. 3) within LH surg peak )
Answer: A
NB: ovulation occurs between 24 and 36 hours after the LH surge.

● rapidly progress glomerulonephritis


A. (creasnet shape)
Answer : A

● Tea color urin Coca Cola urin


A. ( post streptococcus glomerulonephritis )
Answer : A

● exudate tonsils and edematous most comon complication


A. (scarlet fever)
Answer : A

● sleep hour and obesity?


A. ( <5 hour )
Answer : A

● domestic violence->
A. (2nry pregnancy)
Answer: A

● pregnant with UTI TX?


A. (nitrofurantoin)
Answer : A
NB: Nitro not used in 38 weeks of GA or above

● stress urin incontenance 3 cases asking about dx ( cough, laugh , straining) she
doesnʼt control urin

● Pt with TB and he take medication rifampicne , pyrazimnde ,..etc complain now from
Glory team August

neuropathies whatʼs u would want to give


A ( pyridoxine vit B6 )
Answer : A

● Different between anemia of chronic disease | IDA


NB: In IDA: all values low except TIBC is hig
The anemia of chronic disease is opposite + fertin double the normal

● (codeine) is contraindications in eldary

N.B: Appropriate studies performed to date have not demonstrated geriatric-specific problems that would
limit the usefulness of codeine in the elderly. However, elderly patients are more likely to have confusion and
drowsiness, and age-related lung, liver, kidney, or heart problems, which may require caution and an
adjustment in the dose for patients receiving codeine in order to avoid potentially serious side effects.

● Immunoglobulin secret from breastfeeding ( IgA ) ✅

● Psychiatry. 3 q one of them girl increased appetite & weight gain & depressed mood
& agitated? Is a ( BEHAVIOR disorder ) ✅

● Girl fears from weight gain thin ( anorxia nervousa ) ✅

● When psychopaths pt known and has ideas about own disease ( insight ) ✅

● Diabetic female has vaginal itching at night ( candidiasis ) ✅

● Description for genital herpes in case was ( blisters and vesicles ) ✅

● Most common genital abnormalities in boys? ( cryptorchidism ) ✅

● Child with impetigo (honey crusted lesion):


A. - not contagious can go to nursery
B. - not contagious can go to nursery after 5 days
C. - contagious can go to nursery after 3 days
D. - contagious can go to nursery after lesion dry
Answer : D

● Contraindications to circumcision ( hypossodius ) ✅

● Pt her age 55 when do breast screen (mammograms) 2 y ✅


Glory team August

● Perforated peptic ulcer TX ( GRAHAM OMENTAL PATCH ) ✅


N.B: If gastric do gastrectomy

● Clear cases about pancreatitis ( central epigastric pain radiate to back , very high
lipase ) ✅

● Male with gunshot inlet lateral to left nipple exits scapula


JVR , absence of heart
sounds TX ( Pericardiocentesis ) this case of cardiac temponade ✅
NB: DISTENDED jvp
Muffled heart sound
Hypotension
Beck’s triad for tamponade

● Same case ... when u do perscion itʼs was tympanic chest!! TX ( needle )
NB:DX IS Tenstion pnemothorax ✅

● Medial maleulous ulcer DX ( venous Duplex ) ✅

● 22yro-Patient with constipation and painful anal area ( Anal Fissure ) ✅

● .gases Gangrene postoperative organism ( Clostridium perfegnis ) ✅

● 17 years girl run sports athletic suddenly pain ( osgood Schlatte syndromes) ✅

● patient lady noticed lump3x2 cm for 3 months the mass freely mobile no discharge
not related to menstrual, what is the diagnosis? ( fibroadenoma ) ✅

● Patient high TSH + normal T3 T4 what the diagnosis ( subclinical hypothyroidism) ✅

● Long case pregnant on labor has many things abnormal & Bicornate uterus asked
CI for ECV ?? ( Bicornate uterus ) ✅
Glory team August

● Adhesion in pelvis postoperative organism?


A. (chlymdia )
B. E.coli ,
C. Gonreehea
Answer: A
NB: Also perihepatic adhesions = Chlamydia

● 2nd trimester cervix was dilated DX ? ( Cervical incompetence ) ✅

● Female Right abdomen pain Beta HCG +ve but on US no gestational sac seen?
( Extra uterine pregnancy) ✅

● Vaccines contraindicated in pregnancy? (Zoster ) ✅

● Pregnant with mild systolic ejection murmur along the left lower sternal border
radiate to carotid? ( aortic stenosis) ✅

● 21 years old female she did Pap smears before 3 years it was normal what is your
advice for her ?( . pap test with cytology ) ✅

● Hypokalemia in DKA cuz? ( Insulin shift ) ✅

● pt with mass in the Lt side of neck ultrasound show normal thyroid and the mass
was follicular carcinoma dx: ( Metastasis) ✅

● Pt conscious, orientated, but worm extremity, Deminshed air entry in the right side ,
on percussion was tampanic. Dx?
A. Septic,
B. neurogenic,
C. hypovolemic
D. (cardiogenic shock)
ANSWER : D
NB: BAD RECALL , Cardiogenic pale cold extremity , tympanic percussion with tension pneumothorax which
cause cardiogenic shock

● femal post part hem athma CI > PGE 2 ✅


Glory team August

● SLE flare up ( sulfasalazine drug ) ✅

● When do you check hb for a low risk baby?


a. 24m
b. 12m
c. 14m
d. 18m
Answer : B
NB: The American Academy of Pediatrics (AAP) advises screening all infants for anemia with a hemoglobin
test around 12 months of age, along with assessment of risk factors for iron deficiency and iron deficiency
anemia.

● Lady is admitted to psych ward bcs she cut her wrists to get attention from her psychiatrist. At
admission she claimed to have hallucinations but now denies them. She
caused an argument between the nurse and resident caring for her. What is the personality disorder?
a. Schizoid
b. Borderline
c. Obsessive
d. Antisocial
Answer : B
NB: Schizoid usually with emotions problems
● Man is confused and has fever and pneumonia. You want to give abx but the son refuses and
says it would make his condition worse
a. Consult the pt
b. Council the son when the consultant arrives
Answer : A
NB: Confused doesn’t mean incompetent
Always assume the patient is competent unless clear behavioral evidence indicates otherwise.
Competence is a legal, not a medical issue.
2 A diagnosis, by itself, tells you little about a patient’s competence Clear behavioral evidence would be:-
● Patient is grossly psychotic and dysfunctional
● Patient’s physical or mental state prevents simple communication
If you are unsure, assume the patient is competent.
The patient does not have to prove to you that he is competent. You have to have clear evidence to assume
that he is not.

● F with COPD. She has moderate exacerbation, she is on high flow o2 and her sat is 92. there
were some ABGs I don’t remember the values but it wasn’t that bad
a. Reduce o2 flow
b. Inc o2 flow
c. Mechanical ventilation
Answer : C

● M k/c of COPD. Presents with "mild" exacerbation. You put him on high flow O2 but he got
worse
a. Reduce o2 flow ✅
b. Inc o2 flow
c. Mechanical vent
Answer : A
NB: If ABG show Hypercapnea > reduce o2 rate
Glory team August

● At 28 (or 26) weeks gestation. Whet do you check?


a. Hb of the mom ✅
○ OME: 20-28w you have to check for maternal anaemia, alloimmunisation & GDM
Answer : A
NB: Yes CBC is mandatory in every pregnancy
But if there’s OGGT more accurate
RH in 28 and 34 weeks of GA

● Long scenario of a pregnant lady w/ ROM. How do you confirm?


a. Ferning test ✅
Answer : A
NB: if there’s amnisure in first question more accurate

● In normal pregnancy where is the uterus?


a. Above pubic symphysis ✅
b. Between pubic symph & umbilicus
c. At umbilicus
d. Just above the umbilicus
Answer : A
NB : Depends on Gestational age
But all ages it will be above symphisis pubis

● COPD case. What do you expect to see on PFT in terms of FEV1:FVC, TLC & VC?
Glory team August

NB : TLC increase and increase of risidual volum


DLCO decrese
Decrease in FEV/FVC ratio

● Retroperitoneal sarcoma is mainly associated w/?


a. Retroperitoneal invasion ✅
b. Generalised LAD
c. Retroperitoneal bleeding
Answer : A
NB : Lymph node invasion is 5%

● Placental cord found after delivery to have only 1 artery, it is associated w/?
a. African race
b. >80% chance of malformation ✅
c. Inc mortality
Answer :B
NB : mainly associated with Maternal DM

● 71y/o k/c of hypertension controlled on BB. Presents with "mild" obstructive sx and dysuria. On
exam he has "minimal" enlargement of the prostate but otherwise n. P
(n<4). What would you like to do?
a. Cystoscopy
b. TURP
c. Open resection of the prostate
d. Prescribe alpha blocker
Answer : D

● Trauma pt comes with paradoxical chest movement. Chest xray shows break in ribs 2-5 at more
than 1 location. What do you wanna do?
a. Chest tube
b. Thoracotomy
Glory team August

c. Intubate
Answer : C
NB : IT IS FLAIL CHEST , MX : Analgesics + fluid
If oxygen less than 60 intubate the pt

● Pt had a tu 1cm from anal verge with multiple lymph nodes involved and a cranio-caudal
extension of 4cm. Bx shows an adenoCA. What do you want to do?
a. Lower anterior resection
b. Perineal resection
c. Chemoradio
d.abdominalperitoneal resection
Answer : c , Chemo because there’s LN metastasis

● Question with high ca high PTH low phosphate and low vitD.
a. Primary hyperparathyroidism ✅
b. 2ndary hyperparathyroidism
Answer : A
Glory team August

● Child and his parents at your clinic for vaccinations. They are concerned that after his previous
vaccinations he developed fever and erythema at the site of the injection
should you do?
a. Give all
b. Stop all
c. Refer to allergy and immunology
Answer : A

● Picture of a pt with leukocoria asking what do you want to do


a. Brain imaging
b. Reassure
c. Urgent referral to ophtha ✅
Answer : C

● The son of someone w/ TB has 10cm reaction to the skin test. What does that mean?
a. +ve ✅
b. -ve
Answer : A

● Precaution with TB?


a. Standard
b. Airborne ✅
c. Droplet
Answer : B

● Someone w/ echinococcal CYST of the liver. What do you wanna do?


a. MTZ
b. Surgical removal
c. Aspiration
Answer : A
NB ; Ues albendazole , Surgical derrofing if large .
Actually it’s complicated
But remember 3:-
Medical: albendazole/mebendazole
Glory team August

Surgical: PAIR / deroofing

● Post-trauma pt with paramedics had a GCS score of 15. then he had "periodic
unresponsiveness". Xray shows temporal fx. What ur diagnosis
a. Epidural
b. Subdural
c. Basal skull fx
Answer : A

● Trauma pt was taken to a mediocre hospital and they wanna send him to better one. On xray he
had a widened mediastinum. Vitals were okay. What is the most impor
next step
a. Chest tube
b. Needle decompression
c. Call the on-call surgeon of the better hospital and tell him what to expect
Answer : C
NB : Widening of mediastinum > aortic dissection
Shifted mediastinum > tension pneumothorax
Triad of wide mediastenum + epical pleural hematoma + blunting aortic knob = aortic injury

● I don’t remember the exact scenarios but know when to do a TEE vs a TTE
ANSWER : TEE more specific than TTE

● Whats the MCC of PPH?


a. Uterine atony ✅
b. Uterine inversion
c. Retained placenta
Answer :

● Young pt diagnosed w/ WPW. Was put on BB but they didn't help. Whats the best
management?
a. Radiofreq ablation ✅
b. Amiodarone
Answer : A
NB : WPW syndrome ttt
If unstable; cardioversion
If stable; Proconamide
If refractory; Radiofrequency ablation
Glory team August

● Pt w/ positive Babinski reflex and anaemia & unsteady gait


• Hb was low
• MCV high
• Peripheral smear macrocytic RBC
• Whats ur diagnosis?
a. B12 def ✅
b. Folate def
c. Sideroblastic anaemia
Answer : A

● Pt had positive Babinski reflex and anaemia and an usteady gait + vetiligo
• Hb low
• MCV high
• Whats the tx?
a. Oral B12 ✅
b. Folate
c. IV B12
Answer : A

● Whats the pathology in vitiligo


a. I don't remember the choices but there was something about abs attacking melanocytes and I picked that
Answer : Vitiligo = autoimmune

● Vaccines at 1yr : pcv, mcv, opv, mmr

● 2 ur old w/ pneumonia and poor intake and fever. What is the strongest indication that led to
the child getting admitted?
a. Poor intake ✅
b. Fever
c. Age
d. Hypotension (he was not hypotensive)
Answer : A
NB : If he is infant or neonate , FEVER

● Meconium aspiration, initial resuscitation was done and o2 was given. Preductal pressure was 8
and post ductal p was 9. what should you do initially?
a. Surfactant lavage
b. Nitric oxide ✅
Answer : B

● 101- Case of a pt who had a history of an open appendectomy now presents with bowel
obstruction. Which of the following is CI?
a. All of the choices were anaesthesia drugs
ANSWER : NITRIC OXIDE

● Third day post op developed fever. What is the cause?


a. UTI ✅
Glory team August

b. DVT
c. Abscess
d. Pneumonia
ANSWER : A
NB : post-op fever >
0-2days: atelectasis or pneumonia.
3-5 days> UTI
5-7 days > DVT
7 day > wound infection
8-15 days > drug fever or deep abscess

● Milestone question for a 3yo

● 104- 2 sick euthyroid syndrome questions: 1 was a scenario of an ICU pt w/ thyroid fxn tests
asking about the diagnosis. The other was asked what would the thyroid fxn test
ANSWER : High reverse T3 , low T3T4

● Child with joint pain and skin nodules. What else do you need to diagnose rheumatic fever?
a. ASO ✅
b. ESR
c. CRP
d. Blood cultures that are positive
ANSWER : A

blood culture here is wrong


if throat culture will be right
so answer is A

● - Short bowl syndrome developed seizure what electrolyte is responsible?


a. HypoK
b. HyperK
c. HypoMg ✅
d. Hypo PO
ANSWER : C
Glory team August

● - Which electrolyte abn are you going to see in refeeding syndrome

● - Asthma pt on multiple drugs had hypoK+. Which drug is responsible?


a. Salbutamol ✅

● - How do you help someone with hypercalciuria prevent calcium stones from forming
a. Thiazide diuretics
b. Dec calcium intake ✅

● - Lady with swollen knee joint aspiration showed needle shaped crystals -ve birefringent. What
should u give
a. Probenecid
b. NSAId
ANSWER : B
NB : Chronic = alloprinol
If not nsaids in choices > Colchicine
● - What level to perform thoracocentesis? To sample plural effusion?
a. 8-10 mid axillary ✅
b. 5 ics mid axillary
ANSWER : A
NB : 8-10 for thoracocentesis

● - Case of Fall from height or building. GSC score 15 patient has bilateral foot pain. What is the
next step?
a. Xray
b. Analgesia for pain ✅
Glory team August

ANSWER : B
NB : asses distal pulse then analgesics then X-ray
B THEN A

● - Best imaging to evaluate c spine?


a. CT ✅
b. Xray
ANSWER : A

● - Patient has soot nares. Oriented, vitals were normal. How to manage?
a. Elective intubation ✅
b. Admit to ICU 24 hr
ANSWER : A

● What is the MC organism causing infective endocarditis in someone w/ prosthetic valve put 6d
ago?
a. Staph aureus
b. Staph epidermidis ✅
ANSWER : B

● 4 moths old, well breast feed and don’t not use pacific. Has unilateral otorrhea and on
pneumatic otoscope. No fever.
How to manage? (Duration of illness was not mentioned)
a. Abx ✅
b. 24 re-revaluate
c. Refer ENT for tubes
ANSWER : A

● Infants 8 month of age. Jargons. Doesn’t say 2-3 wards parents are anxious.
a. Reassure them, it’s a normal variant ✅
b. Do audiometry
ANSWER : A

● 3 questions about HUS/ITP/TTP asking how to manage

● Patient on TPN his labs were given. Everything was normal except for high INR . (PT was not
given) but aptt was and it was normal. What can explain his labs?
a. DIC
b. Factor 7 deficiency
c. Liver failure
d. Vitamin K deficiency
ANSWER : D
NB : Liver failure = vit k deficiency , Liver is responsible for production of all of coagulation factors except
Factor 8 , All values is normal except high PT is liver failure.
Vitamin K is not a component of the multivitamin preparation added to parenteral nutrition (PN) solutions,
and hospitalized patients receiving parenteral nutrition support are at risk of developing vitamin K deficiency.
TPN is the keyword
Glory team August

● - Pt with effusion asking which of the following if found on paracentesis will indicate the need for
a chest tube?
a. Cells > (I don’t remember)
b. Serum LDH > fluid LDH
c. Proteins in fluid > proteins in serum
ANSWER : C

● - A questions about SAAG. If it was 1 what is ur dx?


a. Cirrhosis
b. CHF
c. TB
ANSWER : C
NB : Saag = serum acitis albumin gradient
If more than 1.1 most likely the cause of acitis is liver lesion(cirrhosis, potral HTN, bud chiarre syndrome) or
congestive heart failure
Less thn 1.1 most likely infections, peritonitis, nephrotic syndrome.

● Placental abruptio at 30 week stable now. What would be the best initial drug?
a. Dexamethasone ✅
Glory team August

b. Oxytocin
c. Terbutaline
ANSWER : A

● - 2rd trimester preg pt had mild brown painless postcoital bleeding. Exam/US showed placenta
covering the os. What do you wanna do?
a. Admit and observe ✅
b. Reassure and discharge
c. CS
ANSWER : A

● - Women with SOB xray showed BHL. What treatment would you start? (Typical sarcoidosis)
a. Steroids ✅
ANSWER : A
NB : DDx:- BHL :
Sarcoidosis
TB
Lymphoma
Metastasis
Indication of using steroid in sarcoidosis :
Hypercalcemia
Neurolgical involved
Progressive of disease

● . Pt on anti-Tb presented with gout falre, labs : high uric acid


Which anti TB is responsible for this presentation?
a. ethimbatol
b. pyrazinmide ✅
c. Isonized
d. Rifampicin
ANSWER : B

● . Child had diarrhoea that was bloody with mucus what is the most likely organism
a. E coli
b. Campylobacter
ANSWER : B
NB : 3 Infectious bloody diarrhea :-
CHESS 4
C ampylobacter
H emorrhagic e.coli
E entameba histolytica
S almonella
S higella

● . Vibreo cholera stool charactarstic?


Glory team August

a. Osmolar gap < 30 ✅


Answer : A

● A lady came back from jedah she has headache and myalgia. With a rash on ger face that
started few days back. What medication would you start her on?
a. Abx
b. Nsaid
c. Steroid
ANSWER : B
NB : DX IS Travel + Mylagia + Back pain = Ankylosing spondylitis

● . Best was to asses fetal growth in early pregnancy? 14-17 weeks i think
a. Femoral length
b. CRL
c. Abdominal circumflex
d. Biparital
ANSWER : B

● . Patient had Femoral hernia asymptomatic what is the appropriate management?


a. laparoscopy with mesh ✅
b. open with mesh
c. laparoscopy only
ASNWER : A

● Newborn 2 weeks with conjunctivitis. And pneumonia. What is the caustive organism ?
a. Gonorrhoea
b. Chlamydia
c. HSV
ANSWER : B
NB: Chemical irritation in eye >> first 24 hours
Gonorrhea in eye >> less than 5 days
Chlamydia in eye >> more than 5 days
Within 24h > silver nitrate
7-14 days > HSV (unilateral)

● . Pediatric (few months old) with absent red eye reflex unilaterally. What is the next step in
investigation?
a. Mri
b. Ct
c. Direct ophlamoscopy under GA
ANSWER : C

● Adult male after MVA had comminuted fracture and Intra compartment pressure of 35.
Numbness on the 1 web space. Pulse is intact.
a. ORIF
b. External fixation and immediate fasciotomy
Glory team August

C.Fasciotomy with leg elevation


ANSWER :C

● . Patient with acute gout, negative birefringent crystals needle shape. Which medication to avoid
in the period?
a. NSAID
b. Colchicine
c. Allopurinol ✅
d. Steroids
ANSWER : C

● Chest pain that increases when laying down. No SOB. ECG was described as diffuse ST elevation
and PR depression.
a. NSAID ✅
b. NSAID and steroids
ANSWER : A

● Contraindication to DTap vaccine?


a. Encephalitis within 7 days of taking the vaccine that is not attributed to another cause ✅
ANSWER : A

● . Child has hx of seizures on AED physical exam shows dysmorphic feature, hypotonia what to do
regarding his vaccines?
a. Give IPV instead of OPV
b. Defer live attenuated
c. Don’t give DTap
d. defer dtp ✅
ANSWER : D

● . Child with maculopapular rash that started at the face and spread to the body. Conjunctivitis
and coughing.
a. Rubeolla ✅
b. Varicella
○ No measles in the choices
ANSWER :A
NB: Rubeolla is ordinary measles
Conjunctivitis , cough , croyza + maculopapular rash started from face to down = measles
Kopliks spots is pathognomonic for measles
Glory team August
Glory team August

● . Newborn with rash of erythematous base over face and body. What to do? (Sounds like
erythema toxicum neonatorum)
a. reassure the parents ✅
ANSWER : A

● . ITP case with low platelets in the labs what would the bone marrow show?
a. Increase megakaryocytes ✅
ANSWER : A

● . Mucocutaneous bleed with the bone marrow result showed Increase megakaryocytes. What is
the diagnosis?
a. ITP ✅
ANSWER : A

● Case of AUB and she’s bleeding a lot. Blood pressure was slightly low 88 systolic. Haemoglobin
was 10 . What to do now?
a. IV line and give FFP
b. IV conjugated oestrogen
c. Hysterectomy
d. A third medical management I forgot.
ANSWER : B
NB : IV fluid > iV estrogen > D/C
should be IV line and fluid
Glory team August

With FFP only if coagulation profile abnormal

● Umbilical hernia in a 2 years old. When to repair?


a. After 5 years of age ✅
ANSWER : A

● . Male with jaundice hepatic ultrasounds showed biliary tree narrowing and dilation. What
investigation would you order to reach the diagnosis?
a. Liver biopsy
b. Antibodies (AMA)
ANSWER : should be P anca or MRCP
Primary biliary cholangitis = AMA
Primary sclerosing cholangitis = P anca, MRCP, ERCP

● . Patient that had urinary urinary incontinence unsteady gate and forgetful. Head CT showed
wide ventricles.
a. Normal pressure hydrocephalus ✅
ANSWER : A

● . Obese child above the 90 percentile BMI 30.


a. gastrectomy
b. lifestyle modification
ANSWER ; B

● . Male obese non-smokers had high BP and day time sleepiness. On exam he was cyanotic. His
wife says he snores a lot. What is the Best Investigation?
a. Polysomnography ✅
b. Spirometry
ANSWER : A

● . Asthma with more than 2 nights symptoms a month and more than 2 day symptoms a week.
He’s on SABA but Uncontrolled what to add?
a. ICS ✅
b. Anti-leukotriene
ANSWER : A

● . COPD exacerbation with PH 7.1. How to deliver oxygen?


a. Mechanical ventilation
b. Oxygen mask
ANSWER : A

● . 4 questions about vaginal infec. 2 were how to tx (trichomatis vaginitis and bacterial vaginosis).
2 were you were supposed to dx it based on colour of discharge and cer
smell.
ANSWER :

● . CTG showing late decelerations. What is the fetus Acid base initially?
a. Respiratory acidosis
Glory team August

b. Metabolic acidosis
ANSWER :B

● . 2 questions about bishop score. I honestly forgot them. Bs one was already calculated and the
other question i had to calculate it ( 50% effacment ,3-4 cm station 1)

● . Placental abruption management?


a. 2 large bore IV Fresh frozen plasma.
b. Call multidisciplinary team. ✅
c. Cryoprecipitate
ANSWER : B
NB: Should be IVF or packed RBC’s , Followed by emergency Cs
If Coagulation profile goes with DIC , with FFP

● . Child with distended abdomen and bilious vomit. Gush of stool on digital rectal exam.
Diagnosis?
a. Hirschsprung disease ✅
ANSWER : A

● . Haemorrhoids stage 4 treatment?


Hemorrhoidectomy

● . 35 years old male with GERD (they said GERD) he was started on PPI esomeprazole 20 mg.
Came back on flollow up saying The medication provided mild symptom relie
still has metalic tase in his mouth when lying flat. No weight loss. No pain. What is the next appropriate step.
• (Disclaimer:The duration of PPI course was not given. The duration of his GERD wa not given.)
a. endoscopy
b. increase esomeprazole to 40 mg
c. change class of PPI
d. h.pylori triple therapy
ANSWER : B

● . Watery diarrhea they did microscopy and it showed protozoa . What is the causative organism?
a. Entamoeba histolytica
b. Campylobacter
c. Giardiasis
ANSWER: C

● . JIA had -ANA how frequent to screening for uveitis?


a. 6 ✅
b. 3 months
ANSWER : A

● . Pt w/ biL knee pain. Which JIA is it?


a. Oligo
b. poly ✅
ANSWER: B

● . Female presented with the complaint of vaginal discharge that had a grey colour.
Glory team August

On examination there was mild fishy discharge with white covered vagina.
• Vaginal PH 5.5
• No other tests were done
• What is the treatment?
a. Tetracycline
b. Azithromycin
c. Metronidazole ✅
d. Fluconazole
ANSWER : C

● . Pregnant diagnosed with GDM on screening tests. Which of the following vaginal infection is
she at risk of having?
a. Candidiasis ✅
b. Bacterial vaginosis
ANSWER : A

● . When to screen for GDM?


a. 24 weeks ✅
ANSWER : A

● . Patient with hepatocellular carcinoma with measurements provided. Which of the following is
strongly related to this condition?
a. Aflatoxin
b. Hepatitis B ✅
c. Wilson’s diseases
d. Cirrhosis
ANSWER : b , The main cause worldwide is hep B

● . If both parents are carriers of a disease and the chance of their next new born child to be
affected is 25%. What is the pattern of inheritance?
a. Autosomal dominant
b. Autosomal recessive ✅
ANSWER : B

● . Female brought by her mom bcs she has multiple café au lait spots (70 I think). The mom
reports that there is a nephew with the same condition. What is the mode of
inheritance?
a. AR
b. XLR
c. XLD
d. AD ✅
ANSWER : D

● . Female with history of D&C. Complaint of amenorrhea. She failed progesterone & oestrogen
withdrawal test. What is her diagnosis?
a. Asherman syndrome ✅
ANSWER:

● . Common cardiac defect in Trisomy 21?


a. TOF
b. Endocardial cushion defect
c. Transposition of great vessels
Glory team August

ANSWER : B

● . ECG picture asking what is the arrhythmia?


a. Third degree block
ANSWER :

● . They did a biopsy of an ulcer and it showed Pseudoepitheliomatous hyperplasia. How to


manage ?
ANSWER : Surgical excision

● . Woman w/ pain in her hand. What component makes the sf palmar arch
a. Radial
b. Ulnar
c. A interossei
d. P interossei
ANSWER : C

● . Pt had numbness on the dorsal aspect of her thumb, index and half of the middle finger. What
nerve is responsible
a. Radial
b. Ulnar
c. Median
ANSWER ; A
NB : Dorsum= radial

● . Patient presented to the ER with hematemesis. He gave a history of repeated vomiting then he
started to vomit blood. How to manage?
a. Endoscopy
b. Conservative mangment
c. Band ligations
ANSWER : B
*Mallory weiss : liner erosion *
1- Repetitive vomiting and also alcholic
>> followed by hematemesis/ upper GI bleeding >> spontinuse recovery >> no leak, only limited to mucosa
((no dysphagia))
Tx. Give epiN if sever and persistence i f sever inject epiN
5Blood stained liquid
*Brerhaave’s syndrom*e (esophageal rupture- tear in full thickness) in is medical E ☄
1- iatrogenic 2- alcohol/ vomiting
>> Hemoptysis and subcutaneous emphysema also after vomiting >> more critical they could be in shock
X ray air-fluid level
** crepitation with palpation around the sternum, Thoracic pain radiates to back, sign of sepsis
Glory team August

Need surgical intervention

● . Vulvar lesion on examination biopsy was done and it showed SCC in situ. How to manage?
a. Wide local resection ✅
b. Radical vulvectomy
c. Close observation
ANSWER : A

● . Rash on uvula was seen. Psoriasis was suspected. How it this lesion diagnosed?
a. Clinically
b. KOH prep
c. Biopsy
ANSWER : C

● . In addition to Tetravalent meningococcal vaccine (ACYW135). What other measures are taken
by MOHE for visitors arriving from sub-sahara Africa to performing hajj in
makkah
a. follow them closely in hajj
b. keep them isolated in a room
c. 500mg ciprofloxin prophylaxis ✅
ANSWER : C

● . It was a patient that admitted for a lung disease (can’t recall COPD or lung infection) she woke
up in the middle of the night confused and disoriented. What is the first action the nurse should
take?
a. elevate the best, give her oxygen and try to orient the patient ✅
b. call doctor to give lorazepam IM
c. call the family
ANSWER : A

● . Retroperitoneal sarcoma is mainly associated with?


a. retroperitoneal invasion
b. generalized lymphadenopathy
c. retroperitoneal bleeding
ANSWER : A
NB: Retroperitoneal sarcoma is usually rounded or oval in shape with satellite lesions so due to this it invades
the proximal tissues and the blood vessels easily

● . Contraindication to DTap vaccine?


A. Encephalitis within 7 days of taking the vaccine that is not attributed to another cause

ANSWER : A

● . a kid who says 1 words and feeds a doll (18 m)


a. 18m ✅
ANSWER :A

● . lady with 2 year history of frontal sinus area headache. The headache usually lasts between 12-
24 hrs. This time the headache is associated with nausea and is continuo
Glory team August

days. She takes over the counter combined decongestant and analgesic but it does not relieve the pain. There
is no papilledema. What are you gonna do?
a. CT scan of the sinus
b. brain MRI
c. more history and physical
ANSWER : CT best for sinusitis but first history & examination

● . A female with long term progressive breast mass. Over the past few months it increased in size
rapidly. Now she has a 15 cm mass that is multicystic with loculations. It
mobile, not attached to the surrounding tissue.
what’s your management?
○ ( there was no biopsy or aspiration in the options. They were all surgical)
a. lumpectomy
b. simple mastectomy ✅
c. radical mastectomy
ANSWER : B

● . Around 37+ or 38 weeks gestation. She presented with regular contractions. At presentation 2
cm dilated and 50% effaced. Now she progressed to 4 cm dilation and 80
effaced. What do you do?
a. CS
b. amniotomy ✅
c. Oxytocin
d. something else for IOL
ANSWER ; B, by exclusion

● . A lady I think 38 weeks gestation. Had a long day at work and by the end she was worried that
she doesn’t feel the fetal movements anymore. She came in and the doct
(something I forgot.)
What are you gonna do?
a. non stress test ✅
b. biophysical profile
ANSWER : A

183. Child with otitis externa. What’s the treatment?


a. topical tazosin
b. oral antibiotics
c. IV
ANSWER :

● . A baby had diaper rash and oral thrush how do you treat?
a. Topical antifungal
b. Oral + topical antifungal ✅
c. Systemic antifungal
ANSWER : B

● . A man with 1.5 cm nodule in his thyroid. He is asymptomatic. Aspiration showed medullary
carcinoma.
a. total thyroidectomy ✅
b. Hemi
c. Subtotal
d. removal of the module
ANSWER : A
Glory team August

● . A man underwent surgery to remove a parathyroid adenoma. 4 months later he presents with
symptoms of hypercalcemia. Whata is the cause
a. Hyperplasia of the remaining parathyroids
b. New adenoma
c. Missed adenoma ✅
ANSWER : C

● . Pap smear done and showed ASCUS. What’s the next step?
a. repeat Pap smear
b. HPV Test
c. colposcopy
d. cone biopsy
ANSWER : A , Repeat after 12 months , if low grade repeat
High great > colposcopy

● . Case of aortic stenosis. Which of the following factors determines the time of the surgery?
a. The patient's symptoms
b. Intensity of murmur
c. Left ventricular hypertrophy
ANSWER : A

● . Case of Parkinson's disease. Which of the following factors present in the patient would have
the highest risk of developing dementia?
a. Word finding when talking
b. Forgetting upcoming appointments
ANSWER : B
NB: Poor prognosis if accompanied with speech problems
Symptoms
Words > bad prognosis Forgetting > could be with dementia
2dayS
Cardiomyopathy > MRI
Glory team August

● . What's the minimum number of days someone should do muscle strengthening activities?
a. 2
b. 3
c. 4
d. 5
ANSWER : A

● . Case of HF. Echo shows mixed dilated and restricted cardiomyopathy. Which of the following
has the highest diagnostic value?
a. Cardiac CT
b. Exercise ECG
c. Coronary Angio
ANSWER : MRI
Glory team August

● . Pregnant woman in labor is 5 cm dilated with adequate contractions. No change in 5 hours.


Which of the following is the appropriate management?
a. IV oxytocin
b. Instrumental delivery
c. Re-examine in 2 hours
ANSWER : C

● . Pregnant woman in labor with past obstetrical history of PPH requiring blood transfusion.
Which of the following is the appropriate management?
a. Active management of 3rd stage of labor ✅
b. Peri-partum IV crystalloids infusion to dec the need for blood post partum
ANSWER :A
Glory team August

● . Case of crohn's disease presenting with symptoms of obstruction. Imaging shows stricture at
the ileum. Which of the following is the appropriate management?
a. Strictuloplasty ✅
b. Resection with ileostomy
ANSWER : A

● . Patients was in an RTA. Xray shows wide mediastinum. (Aortic dissection)


a. Exploratory thoracostomy
b. Aortography with stenting
ANSWER : B IF USTABLE A

● . Female pregnant in the 2nd trimester with hx hyperthyroidism on carbimazole 15mg presents
with symptoms of hyperthyroidism. Thyroid function tests: low TSH, high
Which of the following is the appropriate management?
a. Inc Carbimazole dose to 20
b. Switch Carbimazole to PTU
ANSWER : methimazole
N.B : Pregnancy with hyperthyroidism:
First trimester >> prop
Second and third >> methimazole

● . Patient presents in preterm labor. How do you confirm the diagnosis?


a. US for cervical length
ANSWER : A

● . Pregnant women presents in labor. CTG shows fetal bradycardia. Which of the following is the
cause?
a. Cord prolapse ✅
ANSWER : A

● . A mother brings her 18 month old child because she's concerned about his growth, which of
the following is most concerning?
a. Persistent weight on the 15% percentile
b. Persistent weight on the 10% percentile
c. Inc weight from 5% to 15% percentile
d. Dec weight by 1% percentile but all within normal range
ANSWER : B

● . Laparoscopic cholecystectomy, transection of the bile duct above the cystic duct, what do you
do?
a. Hepatico-duodenostomy
b. Hepatico-jejunostomy
c. Choledocho-duodenostomy
d. Choledoco-jejunostomy
ANSWER : D
If hepatic duct injury > hepatojeujenostomy
If CBD >> choledecojeujenostomy
Glory team August

● Patient diagnosed with advanced ovarian epithelial tumor, treatment?


a. Surgery
b. Chemo
c. Surgery then chemo
d. Surgery then radio
ANSWER : C
Glory team August

● Pregnant female, 28 wks gestation come with a breast mass that has been enlarging for the past
4 months, what do you do?
a. Reassure and reevaluate after delivery
b. Bilateral breast US
c. Bilateral breast mammogram
ANSWER : B

● Female presents after delivery with yellow-white vaginal discharge, asymptomatic. Microscopic
evaluation shows leukoocytes and epithelial cells. Which of the followin
appropriate management?
a. Reassure
b. Urine dipstick
c. Culture of discharge
ANSWER : A

● - Female preg in 1st tri came with hx of bleeding. No passage of conceptus. Cervix is closed.
Ultrasound picture is provided. What would you do?
Glory team August

A.Follow up in a few weeks


B. Reassure and send home
C. Admit for observation
D. Plan for termination of pregnancy
● It’s a blighted ovum, I picked D.
ANSWER : If stable A
Unstable D

● Female pregnancy at 13 weeks gestations presents with history of vaginal bleeding & closed
cervical os. Ultrasound picture provided, Which of the following is the diag

a. Missed abortion
b. Anembryonic pregnancy

ANSWER : B

● - Cause of patient with acute illness (pancreatitis?) admitted into the ICU developed symptoms
of ARDS ?
A-Right heart pressure
B- Pulmonary pressure ✅
C- Precappilar wedge pressure
ANSWER : B
Glory team August

● - : infant deliverd and has an abnormal sound, his face turns blue when he is crying, it gets
better when he is on prone position and worse on supine position what is the diagnosis?
Larngiomalacia ✅
NB: laryngomalacia improves in the prone position ,vascular ring improves with neck extension

● -couples came and the wife wants to conceive, they came for lab results and it showed that the
wife has PCOS, what u should do??
Tell wife
tell husband
tell both ✅
NB: Tell both as women with PCOS are more likely to have trouble conceiving than other women.
● - 2 months yrs boy his parents are concerning that he crys a lot and he has episodes of night
weakening and crying, what to tell the parents? This is normal untill 6 weeks of age
normal untill 2 months
normal untill 3 months
Normal till 6 months. ✅
● Failure to thrive growth chart

Glory team August


NB: Colic typically appears within the first month of life, peaks at about age 6 wk, and reliably and
spontaneously ends by age 3 to 4 mo.
211-pt complains of diarrhoea and he just came from Asia what is the most likely disease?
Zeka
Malaria
dengue fever. ✅
NB: MOST likely dengue because of its Distribution: tropical regions worldwide, particularly Asia (e.g.,
Thailand) but symptoms !! not formulated well

● A patient with COPD presented with SOB. He had elevated JVP and lower limb edema.
(+heptomegaly?). What investigation would you order?
A-spirometry
B-echo
C-CT scan
ANSWER : B

● A patient with COPD arrived confused and in distress. The airway is full of secretions. O2
saturation was in the 70s. How will manage his O2 levels?
A-mask
B-bipap
C-mechanical ventilation
ANSWER : C , C (COPD is GCD 15:15 bipap , if draowsy intubate) , Oxygen saturation is 70 , pt is hypoxic
Glory team August

● A smoker patient presented with ???. O/E he had skin lesions in the back with some of them
being crusted. A CXR was done and showed bilateral infiltrates. How will you manage?
A-antibiotics
B-steroids
C-acyclovir
D-antiseptics
ANSWER : B

● A patient had pleural effusions with an acidic pH. What is the likely cause? (the scenario was
more detailed in the exam)
A-Empyema
B-parapneumonic effusion
C-TB
-
ANSWER : A , B TRUE NEED MORE INFO
Low PH in pleural effusion:
SLE
RA
Malignant
Empyema
Parapneumatic effusion
Esophageal rupture
Glory team August

● A patient presented with pleural effusions what of the following is an indication for tube
thoracostomy?
A-low glucose
B-predominant neutrophils
-
-
ANSWER : A or low ph

● A patient presented with a history of diarrhea. Vital signs: (all within normal) What acid base
state would you expect?
A-metabolic acidosis
B-metabolic alkalosis
C-metabolic acidosis with compensation
D-metabolic alkalosis with compensation
ANSWER : A

● A patient who is known to have Crohn’s is having hypokalemia uncorrectable with oral KCL
supplements. What is your next step?
A-work up for adrenal insufficiency
B-start magnesium sulphate
ANSWER : CORRECT ANSWER MISSING , COULD BE B
NB: Adrenal insufficiency = Addison which cane with hyponatremia and Hyperkalemia

● A patient presented with retro-orbital pain and generalized body aches. What is your diagnosis?
A-Mers-cov
B-Chikungunya
C-Dengue
D-Ebola? (Not sure but it was one of the hemorrhagic fevers)
ANSWER : C , NOT SURE
Glory team August

● A patient presents with headache and neck stiffness.


His CSF analysis revealed: low glucose, high protein and lymphocytosis. What is your diagnosis?
A- Viral meningitis
B- Bacterial meningitis
C- TB meningitis
ANSWER : C
NB : Normal glucose is viral
High protein and lymphocytes = TB
High protein + PMN = bacterial

● A patient with cervical spine disease which was treated surgically previously presents with
Inability to control his bladder. What is the most likely cause?
A-cervical myelitis
B-cauda equina syndrome
C-Recurrence of disease
ANSWER : C
SIMILAR : Pt came with gait instability and urinary incontince ,he is known case of Osteoarthritis and had
cervical laminectomy after cervical degenerative myopathy what is DDx?
A -Cauda Equina
B- Recurrent Cervical Myopathy
ANSWER : B
NB : Cauda equina pt will have sensory loss with lower limb weakness + bladder and bowel inconten...

● A patient presented with weight gain, constipation, coarse skin and heavy menses. Labs:
TSH high (but below 10) T4 normal
What is your next step?
A-start thyroxine
B-reevaluate after ??.
ANSWER : A

● A patient presenting with absence of menses her labs showed:


TSH high T4 low Prolactin high
Glory team August

What is the diagnosis?


A-hypothyroidism
B-hyperprolactinoma
ANSWER : A
NB: High TSH + high prolactin >> hypothyroidism
Normal TSH + high prolactin >> hyperprolactinoma
Low T3 and T4
+ve feedback
High TRH
TRH will stimulate both TSH and Prolactin
So result is high TSH and high prolactin

● A patient presented with amenorrhea. Labs:


TSH high T4 low Prolactin high
What is your next step:
A-start cabergoline
B-start thyroxine
C-pituitary MRI
ANSWER : B

● A patient is giving birth. for 2 hours, the cervix has been 2cm dilated 50% effaced midanterior
position the fetus is in station +3. 2-3 strong uterine contractions every 10 minutes. What is your
next step:
A-artificial rupture of membrane B-induce with oxytocin
C-reevaluate after 2 hours.
D-use cervical ripening measures
ANSWER : C

● A patient is in labor for 5 hours the cervix has been ?cm dilated (forgot how much but it was not
fully dilated as I remember), fully effaced, the fetus is in station -1. 2 uterine contractions every
10 minutes. Reassuring fetal tracings. What is your next step?
A-continue observation
B-oxytocin
ANSWER : MISSED IMPORT INFO , DEPEN ON DILATATION

● A patient in her 37th week of pregnancy is asking for ECV. She had a previous CS for a breech
delivery. What of the following is a contraindication for ECV?
A-Anterior placenta
B-previous CS
C-non-reassuring fetal tracings
D-cervix (2-3cm?) dilated.
ANSWER : C
NB: Previous CS if classical transverse
Otherwise it’s C

● What is used for the acute management of hypertension in pregnancy?


A-hydralazine
Glory team August

B-nitroprusside
-
ANSWER : A
NB : Methyldopa given as maintenance.
In sever case you give hydrazine

A patient had undergone salpingectomy for ectopic pregnancy. How would you follow up? (Bhcg levels are
given and are high):
A-one post up bhcg measurement to ensure it went down
B-weekly measurements until bhcg levels become undetectable.
ANSWER :
Glory team August

● A patient had undergone salpingectomy for ectopic pregnancy. How would you follow up? (Bhcg
levels are given and are high):
A-one post up bhcg measurement to ensure it went down
B-weekly measurements until bhcg levels become undetectable.
Answer : b

● A patient had undergone salpingostomy for ectopic pregnancy. She has been having weekly bhcg
measurements. The last few measurements showed a plateauing of the bhcg level (high). What
is your next step?
A-she is a candidate for methotrexate therapy
B-laparotomy.
C-give contraceptives
D-reassurance
ANSWER : B , DEPEND ON BHCG LEVEL

● A patient was diagnosed with ectopic pregnancy. The sac is 3cm bhcg is (dont remember exactly
but it was below 5000). How will you treat:
A-medical therapy.
B-surgical treatment.
ANSWER : A

A patient had two previous second trimester abortions with dilation and curettage presents in her
early first trimester with bleeding and passage of clots. What is the cause?
A-cervical incompetence
B-chromosomal anomalies
Glory team August

C-Asherman syndrome
ANSWER : B

233- Ovulation occurs:


A-within 72 hours after LH surge
B-within 72 hours before LH surge
C-At the time of LH surge
ANSWER : 24-36H AFTER LH SURGE

What immunoglobulin is present in breast milk?


A-igA
B-igG
D-igE
ANSWER : A

● A child was brought to the hospital with chicken pox. The mother is concerned about his
immunocompromised brother. What is the appropriate prophylactic measure?
A- Vaccinate
B- Isolate the child
C- Give immunoglobulins
D- Give Antivirals?
ANSWER : C

● A couple brought their child with concerns of growth failure. What is the cutoff for referring the
child to a specialist?
A-Growth parameters below the 10th percentile
B- Growth parameters below the 5th percentile
C-A drop of 1 percentile upon follow up remaining within the normal range
ANSWER : B

● A child presented with sunken eyes, flat fontanelles and dry mucous membranes what is the
degree of dehydration?
A-5%
B-10%
C-15%
D-20%
ANSWER : B
Glory team August

● A child who weighs 18 kg presented with dehydration. After resuscitation. How much would be
the fluid maintenance per hour?
A-47
B-56
C-65
D-70
ANSWER : B

● A child was diagnosed with GERD. What would suggest eosinophilic esophagitis?
A-Obesity
B-Hx of snoring
C-excessive food chewing
D-Worse night time symptoms
ANSWER : C OR D

A child was brought by his parents with hx of jaundice, RUQ pain, (hepatomegaly?). Labs revealed
elevated Liver enzymes. What strain of hepatitis virus is the likely cause?
A-A
B-B
C-C
D-D
ANSWER :A

● A few months (<6 months as I remember) old infant was brought by his parents as they noticed
yellowish discoloration in his eyes. He has been growing well and his growth parameters are all
within normal. O/E he is yellowish from the face until the abdomen.
(labs were given showing an elevated bilirubin, but it was not specified if it was direct or not). What
is the cause?
A- Insufficient glucuronosyl transferase enzyme. B- Cows milk allergy
ANSWER : A

● A patient in his late 60s wants surgery for his a reducible inguinal hernia. What is your approach?
A-open surgery
B-laparoscopic surgery
C-non-surgical management
ANSWER : C OR B

● A patient presented post hernial surgery with a bulge in the surgical site. (The mass is …?). It
does not bulge with coughing. The patient is afebrile and vitally stable.
A-seroma
Glory team August

B-hematoma
C-recurrent hernia
ANSWER : A

● A patient presented with a mass in the lower abdomen. CT scan showed an enhancing mass
within the rectus muscle. What is your next step?
A-Surgical management
B-aspiration and advise rest
ANSWER : B
NB: If small & symptomatic > conservative
Could be rectus sheath hematoma

● A patient presented 3 days after blunt abdominal trauma with persistent abdominal pain. CT
scan was done and showed small abdominal collections with a grade 3 splenic injury otherwise
everything was normal. What is your next step?
A-splenectomy
B-spleen conservative surgery
C-evacuation of abdominal collections
ANSWER : B
NB: B if they mean partial resection
Glory team August

● A patient presents to the hospital with an anal mass. On per rectal exam a boggy red tender
lesion with purulent discharge was found. What is the likely diagnosis?
A-Perianal fistula
B-Intersphencteric abscess
ANSWER : B
NB: Boggy abcess

● A patient presents with a 3*2cm thyroid nodule as seen by US. Labs:


TSH normal
T4 slightly high
What is your next step?
A-thyroid scintigraphy scan
B-FNA
C-hemithyroidectomy
ANSWER : A
NB : TSH low > scan
TSH normal > FNA
Glory team August

(For the following three ethics Qs, I’m not sure about the accuracy of the choices)

● What are the 4 principles for effective reporting in child abuse?


A-observe, document, report, self protection
B-observe, document, referrals, follow up
ANSWER :
NB : B NOT SURE

● A fetus was discovered to be dead at one of the antenatal visits. The mother is crying. Her
husband is upset. What is your role?
A-discuss the possibility of future fetal malformations
B-explore, identify their emotions, empathize.
C-explore, recognize their emotions, find solutions.
D-empathy is sufficient
ANSWER : C
NB : CAUZ intrauterine death

● A pregnant lady in week (8?) presented with spotting after intercourse. The couple are panicking
and blaming themselves. What is the correct action?
● ANSWER : I think this question was about miscarriage , And answer was reassure & tell them
nothing clear yet we will do more invs
Glory team August

● Female did tubal ligation complete her family came C/O spotting and late period for 6wks what
you will do ?
● ANSWER : Pregnancy test

A. Pregnancy *(test to exclude pregnancy)


B. Pelvic US
C. Laparoscope

● 30 years female c/o infertility and she is irritable and depressed ‫?? ﻋﻠﯿﮭﺎ ﺣﺘﻼﺣﻆ إﯾﺶ‬

A. Loss of eye contact


ANSWER : A

● 52yrs old pt ē multiple ulcer in lesser curvature hematemesis and dark stool on drugs and no
improvement what is the next step
A. Partial gastrictomy*
B. Total gastrictomy
C. Ablation
ANSWER : A , NOT SURE
NB : According to the given choices it is Persisted PUD
Surgical option preferred to do vagotomy
Management of upper GI bleeding:-
$First: Always stabilize the patient with IV fluids, send blood for Basic labs & coagulation profile.
$then, endoscopy: If bleeding source is peptic ulcer (most common) > injection of epinephrine around
bleeding point + thermal hemostasis.
$If bleeding source is esophageal varices > IV octreotide followed by endoscopic ligation or sclerotherapy.
"Persistent or recurrent varices bleeding?
Glory team August

Consider transjugular itrahepatic portosystemic shunt (TIPS), ballon temponade, Liver transplant.
Long term ttt to decrease risk of recurrent varices bleed?
BB
Nitrates
follow up

● 13girl ē vesicular rash in trunk and extremity?

A. Herps type1
B. Herps type 2
c. Varicella *
D. Meningococcal
ANSWER : C

● 38 wks in labor severe preeclampsia magnesium given and oxytocin administered epidural
anesthesia for pain pt stable ctg show early deceleration ?
A. Fetus engaged (head compression)*
ANSWER : A

● same Q above +Bradycardia ctg >> e


A . pidural anesthesia*
B. oxytocin
C. -fetal head compression
D. -mg4
ANSWER : A
NB : Mgso4 > reduced variablity
Anesthesia > fetal bradycardia
Glory team August

● 25yrs had one solid nodule in thyroid , what is the next step ?
A. CT neck
B. FNA
C. Radioactive scan
D. MRI
ANSWER : B

NB: If serum tsh in choices will be true

● lady ē anexity can’t deals with co worker :


A. Give her instruction how to deal with them
ANSWER : MISSED INFO

● *Young age c/o painless bleeding during defecation diagnosis?


a. anal fissure
B.hemorrhoide
C.anal cancer
D.fistula
ANSWER : B
NB : Pain then bleeding = fissure
Bleeding then Pain = hemorrhoide

● Diabetic pt uncontrolled with diabetic foot and gangrene , you must do amputation surgery and
pt refuse ?
1-Do surgery
2-let pt sign refuse DAMA
3-refer another doctor
Glory team August

4-inform Ethics committee


ANSWER : 2
NB: If he refuses the doctor > refer
If refuse the surgery > dama

● old pt with cancer colon ,prepared for surgery at time of operation pt BP start become low
hypotension with tachycardia HR 140 what you will do?
A. termination of surgery
B. Make ECHO
C. chest x-ray
D. convert of surgery
ANSWER : D
NB : Best to stop ongoing bleeding , SO OPEN SURGERY

○ child with flu-like symptoms developed vesicular rash all over the body best lab to
confirm diagnosis?
A. HSV-1 IGM
B. HSV-2 IGM
C. Mumps IGM
D. VZV IGM
ANSWER : D

○ infant with repeated vomiting after feeding , examination :small epigastric mass< best
next step?
A. barium enema
B. abdominal sonography
C. abdominal Xray
D. CTabdomen
ANSWER : B
NB : Pyloric stenosis

■ Old pt bed ridden with basal ganglia and sever loss of weight , absent gag reflex
, how feed pt?
A. NGT
B. OGT
C. parentral I.V
D. geogenius tube
ANSWER : MAY BE D
NB : PARKINSON , High risk of aspiration try to avoid stomach
Best is jejunestomy
Glory team August

■ newborn girl with asymmetrical breast fold < next step?


A. MRI
B. pelvic US
C. cortisol level
ANSWER : SEE BELOW
NB : It's normal for newborn babiesto have mild or even swollen, enlarged breasts and/or lumps under the
nipple.
They are almost always benign and due to exposure to maternal hormones in the womb.

● Pregnant came for antenatal checkup, ultrasound showed an empty gestational sac -there was
empty SAC no baby- Cervix closed what is the dx?
a. anembryonic fetus –
B. pregnancy –
ANSWER : A

○ Man came complains of watery diarrhea (no mucus - no blood ) abdominal pain and
discomfort - one day ago he arrived after a trip to India what is the most likely
diagnosis?
A. E coli
B. salmonella
C. Brucella
ANSWER : A

● 30 years old female known case of inflammatory bowel disease she plans to be pregnant she is
on medication since 3 years. despite the treatment, she has a permanent Hx of melena she did
sigmoidoscopy or endoscopy (can't remember ) what you will advise her?
A. avoid pregnancy
B. plan for pregnancy at least after 3 years
C. till her, she can be pregnant but there is risk of abortion
D. can't remember the 4th choice but it was un logic answer
ANSWER : There’s Missed information
Glory team August

NB : If uncontrolled A
If controlled She can get pregnant

● case of IBS (the diagnosis was mentioned ) they ask what is your advice for him?
A. decrease fruits in his diet
B. decrease vegetables in his diet
C. avoid mint oil
ANSWER :C
N.B : Because not all IBS patients triggered by same food, we cannot advice all to avoid certain types. Deit
for all IBS patient is fiber-rich foods

○ housewife get oven burn in her hand lead to pustule formation then she noticed green
discharge at the site of burn what is the most likely organism?
A. -klebsiella
B. -pseudomonas
C. -S.pyogen
D. -E coli
ANSWER :B
NB : Should be aureus , If not I think pseudomonas
in burn the most common:
bacterial = pseudomonas
non-bacterial = candida
Glory team August

● 15 years old boy brought to the Emergency department by his parents.he has acute
meningitis.the doctor wants to do LP immediately. from whom you will get the consent?
A. parents only
B. parents and child
C. child only
D. no need for consent it is an emergency case.
ANSWER : Consent parents
Assent child

● patient has liver cirrhosis (other clear details ) he has huge abdominal ascites what medication
you will use for him to solve the ascites?
A. aldosterone antagonist
B. flutamide-
C. -thiazide
D. -AECs
ANSWER : A
NB : Spironolactone , K sparing

● 2 months old baby (first baby for his mother) feeding well - normal growth but he has vomiting
between meals what will make you think he needs further investigation?
A. he is boy-
B. failure to thrive
C. vomiting after feeding-
Glory team August

D. vomiting between meals

ANSWER :B
NB : MAY BE PYLORIC STENOSIS
● RTA patient he has leg fracture they did splint for him later on he came complaining of pain and
mild swelling at his leg .. which of the following you will consider it as alarm sign in his case ?
A. swelling
B. pain-
C. pulseless
D. paresthesia between his fingers
ANSWER : B
NB : Pain + swelling both with compartment
Of the five classic signs of arterial insufficiency (five P’s: pain, pallor, pulselessness, paresthesias,
poikilothermia [cold skin temperature]), only pain is commonly associated with compartment syndrome,
particularly in its early stages.
29 August - Solved by Glory

1- 18 year old lady, came with primary amenorrhea, short stature, short
neck, didn't mention 2ry sexual characteristics, her family height are
below average, asking about dx:
A- Turner ✅ ✔
B- Hypothyroidism
C- Hypopituterism
N.B: Hypothyroidism will cause mainly secondary amenorrhea.

2- Patient k/c of crohn's, i don't remember the hx, but patient came
with flare (intestinal obstruction), i think they did CT and found iliocecal
stricture, Best management?
A- conservative
-> cases like this are left for few days, so inflammation can subsid, if
not, then surgically removed with anastmosis.
B- striculoplasty ✅ ✔
C- give a follow up appointment
N.B: Fistula in Crohn manged conservatively first, but stricture
take pt immediately to OR to avoid perforation and peritonitis.
N.B: Stricure in crohns treated with striculoplasty, if unstable
patient so do immediate laparotomy and surgical resection.

3- Lady came with unresolved rhinitis or something near, i don't


remember, i think it was sinus problem, asking about the best invex:
A- head CT ✅ ✔

4- A boy or girl, complaining from chronic chest infection, nail clubbing,


seems like a cystic fibrosis, asking about the association:
A- nasal polyps ✅ ✔
5- Elderly with TB treatment, deteriorated, admitted in the ICU, his son
was angry and agitated, screaming he will file a complaint: Ethics
A- till him it can happen to anyone especially in his age
B- agree with him and hellp him to get the report of his father's case
C- acknowledge his feeling with sympathy and till him you will
investigate ✅ ✔
D- till him to go and file the complaint

6- Nurse working in flue clinic, came with symptoms, what does she
probably get,
There was a CXR showing Rt upper lobe cavitation:
A- TB ✅ ✔
B- H.influanzae

7- man with TB, what type of pulmonary disease that get him
susceptible to have it:
A- silicosis ✅ ✔
B- asbestos

8- Some patient has SLE, coming with joint or renal symptoms, asking
what to add:
A- hydroxychloroquine
B- cyclophosphamide ✅ ✔
N.B: If joint then A, if renal or CNS then B.

9- Male, i think he was coming with infection, found to have


thrombocytopenia, eczema?
A- Wiskott aldrich's ✅ ✔
B- Bruto agammaglobulinemia
N.B: Wiskott-Aldrich syndrome:
B and T - Cells immunodeficiency
*WATER* mnemonic
- Wiskott
-Aldrich
- Thrombocytopenia purpura
- Eczema
- Recurrent infections

10- Guy with nerve palsy, his Lt eye moving inward when you ask him to
look straight, moving to the right, he can't look to the Lt:
A- 3rd n palsy
B- 6th n palsy
N.B: Failure of abduction in left eye means left 6th CN palsy.

11- Lady came with himanopia, what is the best management or like
radical ?
A- Trans-sphenoidal surgery ✅ ✔
B- couple of drugs that we discussed and mentioned before

12- 3 CTG pics; asking about mx, so your answer it gonna be related to
the ctg result.
N.B:

13- Old man, in his late 60s, had hx of MI and he is diabetic, came with
dyspnea,
Ex stoney dullness, crackles:
A- pleura infiltration ✅ ✔
B- peumonia
N.B: Most likely pleural effusion as a result of heart failure.

14- Pseudocyst enlarging in size, doctor decided to drain it, what is the
best:
(Patient temp 37.9 i think he had leukocytosis.)
A- percutaneous ✅ ✔
B- Endoscopy
C- laparocopy

15- Dead newborn, for couples with infertility for 11 years, his father
was sad:
A- talk to the parents with sympathy and admit the mother in
postpartum ward ✅ ✔
B- Till the father if you can help him with the paper work or something
like that
C- Give them cl

16- Which type of food should be avoided in celiac dx:


A- rice
B- Barley ✅ ✔
C- potatoes
N.B:
17- midshaft femural fracture with angulation 30 degree:
A- cast
B- Interanl fixation with IMN ✅ ✔

18- pregnant lady in labor, l don't remember if the baby was in critical
position, doctor told her that she need to go for cs but she's refusing:
A- do vaginal delivery
B- take the consent from her husband ✅ ✔
C- Refer to another doctor
N.B: If emergency so take consent from husband, if not so respect
her wishes and refer.

19- epicodyler fracture with loss of sonsation or something related to


nerve damage, best?
A- K wire
B- Forgot the rest
C- Open surgery ✅ ✔
N.B: K wire is used for temporary fixation , or fixation of small
bones. But here we have nerve injury we explore better, surgery is
typically needed to allow the nerve to heal correctly.
N.B: K wire is Fixed in an OR so it is the most logic and right one, U
do surgery for nerve then fix with a K.
N.B: If we have neurovascular damage we have to do open
reduction to avoid permanent injuries. I remember it was in Toronto
notes but I donʼt have my lap top with me now.
20- patient with hallucination, not sure about the duration, after
sometime the symptoms subside, what is the dx? ✅ ✔
N.B: Less than 1 month > acute brief psychosis.
N.B: 1 to 6 months > schizophreniphorm.
N.B: More than 6 months > schizophrenia.

21- newly married 37 Diabetic came with her husband planing for
pregnancy, hgA1c: 8?
A- advice not to get pregnant and till to control her blood sugar as the
uncontrolled dm will cause some complications to the baby
B- She can get pregnant ✅ ✔
N.B: 10 or more advice for not get pregnant until DM controlled, <
10 so pregnancy allowed.

22- patient with artificial valve; Aortic and i think mitral, going for hernial
lap, what to do about prophylaxis :
A- 2 Abx in choices
B- Do nothing ✅ ✔
N.B: GI and GU procedures dont require prophylaxis.

23- 87 old gentlemen with metastasis:


A- palliative mx ✅ ✔
B- Analgesia and
C- Palliative chemo

24- in labor (latent phase) with ROM for 20 hrs:


A- give Abx ✅ ✔
N.B: as prophylactic versus chorioamnionitis.

25- lady complaining from a week hx or 9 days, height intolerance and


sweating: low TSH, high T4, what's the dx:
A- hashimotoa thyroiditis
B- Graves disease ✅ ✔
N.B:

26- pregnant complaining from bleeding, GA in third trimester; Ex


showed suspecious part of the cervix laterally:
A- laser
B- Excised
C- Butox
N.B: How to approach pregnant with cervical lesion:
– If low suspicious >> do pap smear.

– If high suspicious or abnormal Pap smear >> do colposcopy.

– If anadequate results in colposcopy >> do cone biopsy in 2nd

trimester.

27- Diabetic pt came complaining from painful spot on the back of his
neck, Ex: wide erythematus with multiple discharge opening:
A- carbuncle ✅ ✔
B- Furunculosis or folliculitis

28- pregnant lady coming with pv bleeding, uterus larger than date,
bhcg around 82,000: us;snow storm
A- Complete mole ✅ ✔
B- Partial mole
29- a patient came with complaint, doctor summerized his
presentation, what the doctor has done?
A- Summarization ✅ ✔

30- pt with appendicitis, after surgery found to have appendix tumor


0,5 cm(located the tumor at the tip), what to do next?
A- nothing and reassure ✅ ✔
B- CT CAP
N.B: If the base involved Rt hemicolectomy regardless the size.
N.B: If non base involved, so according to size if < 2 cm do nothing,
if > 2cm so CT then right hemicolectomy.

31- a lady with hypothyroidism, last f/u her TSH was high so her doctor
increased the dose to 150 levothyroxen, this visit shwoed very high
TSH, what is the reason?
A- ectopic thyroid
B- Inappropriate adjustment of doses
C- Increase the dose to 200 ✅ ✔
N.B: C is more likerly , increase to 200 then re-assess.
N.B: B can happen during staring of tx. You cant tell is it the does
or ectopic unless u start ur plan then if failed u will know why.
32- patient after RTA, with flail chest, Ex shifted trachea and decrease
air entry in one side, next?
A- needle decompression ✅ ✔
B- Intubation and ventilation
C- Chest tube inserrion
N.B: needle first for the Tension pneumothorax, the intubation for
the flail chest.

33- Old age I think 50yr have headache Increase when he down his
head + mostly in back in his neck?
A- Tension headache
B- Subarachnoid hemorrhage ✅ ✔
C- Cluster headache
D- Muscular spasm
N.B: pain Increase when lifting something or when his head down
itʼs SAH, also it is the worst headache ever.

34- Which of the following has the worst prognostic indication?


A. Bilirubin
B. Albumin
C. AST
D. PT ✅ ✔

35- A physician is attending a patient and obtaining history from him.


Meanwhile he is having trouble with his patient in which he talks about
why he thinks he got the disease and the physician wants to know more
about his symptoms and diagnosis. What is the best approach?
A. Use open ended question
B. Let the patient complete his concern ✅ ✔
C. Politely interrupt and ask direct questions
D. Let another physician see this patient

36- Retroperitoneal sarcoma is mainly associated w/?


a. Retroperitoneal invasion ✅ ✔
b. Generalised LAD
c. Retroperitoneal bleeding
N.B: A is true, Lymph node invasion is 5%

37- Placental cord found after delivery to have only 1 artery, it is


associated w/?
a. African race
b. 80% chance of malformation ✅ ✔
c. Inc mortality
N.B: option A and B are wrong, 30% chance of malformation
( trisomay 13 ) not 80%
With white ppl not african. it do inc. risk of mortality, may option D
will be better.

38- Child with repeated “tantrum”, how will you instruct his mom to deal
with her child?
A- Do “time out” strategy ✅ ✔
B- Doing whats he wants you to do
39- Bronchogenic Carcinoma with recurrent pleural effusion that
treated with pleural tap and presented with pleural tap how to manage?
A-Diuretics
B-Chemical pleurodesis ✅ ✔

40- Pt with Liver cirrhosis with two hepatic lesions ,what is the
pathogenesis?
A- hepatitis ✅ ✔
B- medications that I cant remember

41- Young pt 21 YO with dyskinesia with agitation and ring on ophtha


exam?
A- Wilson ds ✅ ✔
B- hemochromatosis
C- Parkinsonism

42- UC pt came with 4 days vomiting and bloody diarrhea and


transverse colon is 15 cm what to do ?
A-infiximab
b- systemic steroid ✅ ✔
c- total colectomy
d- fructocolocetomy with ileal pouch
N.B: B, if not responed then surgery.
N.B: Toxic megacolon; Dx by Xray /ct, ttt by steroid if no response
so surgery.

43- Bacterial vaginosis what do you expect to find on microscope ?


A -Granulomatous epithelial cells ✅ ✔
b- single giant cells
c- multi giant cells

44- Pt 22 Yo with no symptoms what to do for her?


A- pap smear ✅ ✔
B- nothing

45- DKA pt after U manage him he still he has hypokalemia what is the
cause ?
A- insulin infusion ✅ ✔
B- Metabolic acidosis

46- Pt with Kawasaki took the treatment how will you follow up the pt?
A- Echo ✅ ✔
B- catheterization
C- MRI
D- Serial CT

47- Pediatric pt with congenital cardiac ds with cough and fever how
would U approach the pt?
A – cathetrization
B- consultation to cardio and pulmo and Echocardiogram ✅ ✔
N.B: antibiotics then B.
N.B: CHD cause recurrent respiratory infection.

48- 8 weeks pregnant lady came with vaginal discharge the crown
rumb length is 7 weeks what is the DDX?
A - Anembrionic
B- threatened abortion ✅ ✔
N.B:
49- 4 month old down syndrome with harsh systolic and fixed
splitting ?
A- AVSD ✅ ✔
B- TOF

50- Pt with lower back pain with morning stiffenes and he feels the pain
is getting better with paractamol what to do?
A- physiotherapy ✅ ✔
B- biofeedback

51- Pt with A fib how to control the rhythm ?


A - diltiazem
B – Propranolol
C- anti arrhythmic ✅ ✔
N.B: If question said control *heart rate or ventricular rate* >
diltiazam or BB
But if question about heart *rhythm* control > answer is
antiarrhythmic medications such as flecainide, propafenone, or
amiodarone.

52- Pericarditis ECG what is the treatment?


Ibuprofin ✅ ✔
Aspirin
Steroid
N.B: Aspirin if Post MI.
N.B: Pericarditis alone with no complication use NSAID and
Colichicin to prevent from recurence.
N.B: If the pation SLE + Pericarditis = Steroid + immune
suppression.
N.B: Pregnancy + pericarditis = steroid, otherwise NSAID.

53- Pt with IBD ,what increases the risk of osteoporosis?-


A -age
B- chronoc steroid use ✅ ✔

54- Pt IBD he is crying what to give ?


A- azathioprine
B- steroid ✅ ✔

55- Pt with PCOS what will you obtain?


A – lipid and glycemic profile ✅ ✔
B- 17 hydroxy
C- testosterone
N.B: If not diagnosed yet c but if already diagnosed A.

56- Long scenario about pt with A fib now with sudden lower limb pain
how to manage ?
A – femoral A thrombectomy
b -thrombolytic catheter Tx
C-above knee amputation
D- Catheter embolectomy ✅ ✔
N.B: Revascularization should be done to avoid critical limb
ischemia. Catheter embolectomy is recommended.

57- Pt with Follicular cancer and he did hemithyrodictomy ,U found


some papillary cancerous cells after surgery but no invasion what to
do?
A -radioactive
B- total thyroidectomy ✅ ✔
C- observation and follow up
N.B: There is remnants of malignant cells i am with total.
N.B: Radioactive used post op Usually after total for recurrence
and remnants so B.

58- Pt diabetic on metformin with high glycemic profile?


A – increase the dose of metformin
B- add one of the sulpha drugs ✅ ✔
C- Do nothing

59- Elderly man with Rt knee pain and swelling ,WBCs is normal ? Just
like this no more data
A -Rheumatoid
B- septic arthritis
C- gout
D -pseudogout

60- more character of down?


A- single crease
B- hypotonia ✅ ✔
N.B: Common B, Specific A.

61- Pt with skin manifestation spread to the trunk and face with
lymphadenopathy ?
A -penicillin ✅ ✔
B- steroid
C -antiviral
N.B: Dx is secondary syphilis.

62- Child with distal femur mass ?


A -osteosarcoma ✅ ✔
B- Chondrosarcoma

63- Pt elderly diagnosed with osteosarcoma ,what is the mets workup?


A -Chest X ray
B- pet scan
C- CT chest ✅ ✔
N.B: Bone is the origin, Distant mets mainly to lung.

64- Pt child with septic arthritis ,culture came with MRSA what to do
next?
A – shift to Vancomycine ✅ ✔
B - Joint wash out

65- Pt with distended JVP and systolic murmur ?


A -PS ✅ ✔

66- Mid diastolic rumbling murmur?


A -MS ✅ ✔

67- Pericarditis case ,what do U found on examination?


A- Pericardial rub ✅ ✔
B- pleural rub

68- Pt 21 Y O with whooping cough ?How many years he will be


immunized against pertussis?
A – 10 years ✅ ✔
B -5 years
N.B: Pub med > 4-12 years.

68- Dtap vaccine?


- will give u life lone immunity against pertusis
- immunization and immunity against pertussis will wanes within 3 y
- protect within 10y ✅ ✔
- within 25 y
69- Pt is with bronchogenic carcinoma he came with distended JVD
and quite heart sounds?
A -Echo ✅ ✔
b- ECG
C- Chest x ray

70- Pt with Hx of PID with normal semen analysis of the husband and
she is trying to conceive ,what to do?
A -Ovulation induction
B -intra uterine insemination
C- IVF ✅ ✔
D -weigh reduction
N.B: Tubal block so C.

71- What do U find in biopsy of Pt with celiac ds?


A -villous atrophy ✅ ✔
N.B: + lymphatic inflitration + Crypt cell hyperplasia.

72- PT with G6PD deficiency what drug should be avoided?


A -Amoxicillin
B- aspirin ✅ ✔
C- paracetamol

73- Dislocation mandible need?


a. orotracheal tube ✅ ✔
b. naso tracheal tube
c. tracheostomy

74- Patient came after finishing 2 weeks of H.pylori treatment and he is


eager to repeat breath test. What is the earliest time the test can be
repeated?
A 4 weeks
B 2 weeks ✅ ✔
C 1 week
D now
*Urea breath test*
Stop PPI 2 weeks before
Stop abx 4 weeks before
Stop H2 blockers 9h before
N.B: According to uptodate
Pt should stop PPI and Abx 2 and 4 weeks respectively before
doing UBT

75- Hx Infertility sheʼs Depressed. What is her action during interview?


A- Avoid eye contact ✅ ✔
B- Cracks nails

76- Female with right lower abdominal pain and Bhcg 2600 and given
methotrexate after 1 week came with severe abdominal pain and Bhcg
6000 whatʼs the next step?
A- lap Salpingotomy
B- lap Salpingectomy
C- Exploratory laparotomy ✅ ✔
D- Methotrexate
N.B: Usually you should give another dose till four doses before
going to surgery
But 6000 bhcg is contraindicated for metho so either B or C or D. :-
If ruptured and vitally unstable so C and remove tube, ruptured and
vitally stable so lap ectomy, not ruptured and vitally stable so B or
A depends on :-
1- Ectopic in ampulla and wants fertility : tomy, not wants fertility
so ectomy.
2- Isthmus : ectomy.
3- Recurrent at same side : salpingectomy.
4- Recurrent at opposite site ; tomy.
N.B: regarding metho, if you give first dose so follow after 1 week
by BhCG, if decreased < 15% so another dose, if > 15% so continue
follow up no need another dose.

77- Q about truncus arteriosus & bulbus cordis , cause ??!!


A. Vsd
B. Asd
C. Tetralogy of fallot
D. Transposition of great artery ✅ ✔

78- IVIG is superior to Steroid in ITP cuz rapidly acting. ✅ ✔


July Part 4 (Last part) - Glory Group

1- a man with 2nd degree burn what iv to give and the does (donʼt
remember his weight) the want
A. Normal saline
B. Lactate ringer ✅ ✔

2. Child can throw a ball, drow a straight line , put logo toys upon each
other
A. 16 months
B. 14 months
C. 2 years ✅ ✔
 
3. Child 14 or 12 months speak only 2 word
A. Ent referral
B. audiometry
C. Assurance ✅ ✔
 
4. Mode of inheritance for x-linked dominant if the mother and father is
a carrier?
N.B: 25% AR as both are carriers ✅ ✔
 
5. Treatment for bacterial meningitis (7yeas old)
A. Vanco-rifampin
B. Vanco-genta ✅ ✔
N.B: should be Ceftriaxone plus Vanco, Alternatively is Gentamicin
+ Vancomycin.

 
6. 10 yeas old boy abdominal pain and diarrhea sometimes bleeding
With loss of weight
A. Ulcerative colitis
B. Crohn disease ✅ ✔
 
7. About 60 yeas old women small mild abdominal hernia felt mildly with
cough
A. Laparoscopic with mesh
B. Open with mesh
C. Simple repair herniorrhaphy ✅ ✔
D. Observation
 
8. Most commonly cause of leukocoria
A. Myopia
B. Retinoblastoma
C. Congenital catarract ✅ ✔
 
9. 4y/0 left leg swelling with fever next management ✅ ✔
N.B: Could be cellulitis if leg swelling.
N.B: Septic arthritis if knee swelling, ttt by Aspiration + AB.
 
10. Patient came for cut right ear and after general anesthesia u
discovered abnormal in left ear whatʼre u doing ? #
 
11. case about Guillain-Barre syndrome what is the prognosis?
11. case about Guillain-Barre syndrome what is the prognosis?
A. complete recovery ✅ ✔
B. recovery with residual paralysis
C. Respiratory failure
D. Progressive muscle weakness
 
12. patient with pain in joint and has PUD. What medication will you give
for pain?
A. iboven
B. aspirin
C. codin
D. paracetamol ✅ ✔
For people with ulcers, the risky pain relievers are nonsteroidal
anti-inflammatory drugs, or NSAIDs. They include aspirin,
ibuprofen, naproxen sodium, and ketoprofen.
A non-NSAID pain reliever, like *acetaminophen*, may be a safer
choice.

13. patient did gastric sleeve surgery (or gastrectomy) what vitamin will
be decant?
A. B1
B. B2
C. B6
D. B12 ✅ ✔

14. Blood transfusions reaction read it #


 
15. ovary cancer marker?
A. Ca125 ✅ ✔
B. B-hcg
 
16. pt with diarrhea & leg nodule?
A. Ulcerative colitis
B. Crohn ✅ ✔
C. Celiac
N.B: leg nodules = erythrma nodusum.
 
17. drug addicts came with confusion & cyanosis, what to do next?
A. Ask about the drug
B. Take Hx
C. Give O2 ✅ ✔
N.B: Cheery red means CO poisoning just extra information.
 
18. pedia pt came with gum and nose bleeding and he had URTI a while
ago, Dx?
A. ITP ✅ ✔
B. Hemophilia a
C. Henoch-Schonlein purpura
 
19. pedia pt came with dark urine after URTI, Dx?
A. Glomerulonephritis ✅ ✔
 
20. what is the best test to be sure of eradication of H. pylori?
A. Urea breath test ✅ ✔
B. Endoscopic biopsy
 
21. contractions of esophagus on the bariums study "corkscrew" of
esophagus, Dx?
A. Diffuse esophageal spasm ✅ ✔
 
22. mother have anti-Rh positive the infant has anti-Rh negative and
developed jaundice after birth, mechanism?
A. Fetal antibody against mother RBC
B. Maternal Ab against fetal RBC antigen ✅ ✔
 
23. at second stage of labor for 18 hrs, cervix dilated, full effacement,
baby head can be touched, what to do?
A. CS
B. Stop oxytocin
C. Vacuum ✅ ✔
 
24. female most important risk factor for coronary heart disease?
A. High calories diet
B. Inactivity
C. Smoking ✅ ✔
 
25. baby with recurrent infection +his siblings and father have the same
condition, Dx?
A. Bruton agammaglobulinemia ✅ ✔
 
26. Baby Waves hand what is the age?
Answer 9 months ✅ ✔
 
27. Baby Says hi what is the age?
Answer is 12 months ✅ ✔

28. Draws square what is the age?


Answer 5 years ✅ ✔
 
29. child with herpes simplex and going to school?
A. Not contagious
B. After 3 days
C. After 7 days
D. When the pustules become dry ✅ ✔
N.B: like in chicken pox.

30. 4 cases in cystocele, pelvic floor dysfunction management? ✅ ✔


N.B: anterior repair called colporhaphy.
 
31. female had epistomy at 9 oʼclock and came with swelling and pain,
what to do? ✅ ✔
N.B: sitz baths to help decrease pain, swelling, and bruising.
 
32. pregnant diagnoses eclampsia what to give?
A. MgSO4 ✅ ✔
 NOTE:
MgSO4 side-effects: pulmonary edema
MgSO4 overdose: DTR depression (first sign), respiratory
depression, coma, cardiac arrest; Tx calcium gluconate after
stopping the drug.

33. pregnant with high BP only, Dx?


A. Eclampsia
B. Pregnancy hypertension ✅ ✔
C. Preclmpsia
 
34. A 3-year-old girl presents with a 6-week history of right
submandibular region lymphadenopathy. She has been treated with
three courses of antibiotics without resolution but is otherwise healthy.
Her mother states that the child has had no fever or night sweats.
Physical examination reveals a 3-centimeter submandibular lymph node
that is not fluctuant. There is edema and erythema of the overlying skin.
The purified protein derivative test results in 8 mm of induration, and
testing for cat scratch disease is negative. Of the following, the MOST
valuable diagnostic test is:
N.B: sputum culture! But thereʼs no sputum.
N.B: PPD is normal! No I remember its +ve in this age.
N.B: FNA from lymph node and send it for acid fast bacilli or culture.
N.B: If PPD positive go for CXR.
N.B: skin test in less than 5 years is positive if 10 or more.
N.B:
*Unilateral lymphadenopathy*
Congenital neck masses
Submandibular abscess
*Bilateral*
Viral infections: cytomegalovirus, Epstein-Barr virus
Streptococcal pharyngitis
N.B: 6 weeks without resolution + lymph node more than 2 cm in size.
these are *worrisome features* he should be evaluated by biobsy (FNA
or incisional or exisional).

N.B: Blood culture = In febrile or ill-appearing patient


Throat swab and oral swab = In patients with tonsillopharyngitis. +
Serology: if tularemia or cat-scratch disease is suspected
Tuberculin test in chronic cases.
N.B: The diagnosis is : TB lymphadenitis
Best test is : exsional biopsy
PPD is +.

 
35. A 4-month-old boy presents with the complaint of colic. His mother
reports that the child has multiple, daily, painful, brief episodes that are
so severe that he repetitively flexes his upper and lower extremities all
at once. The baby cries after the episodes. He was diagnosed by
another doctor to have infantile colic. What is the most important clue
to get the diagnosis? ✅ ✔
The child is not interested to the surrounding
N.B: infantile colic, is defined as episodes of crying for more than
three hours a day, for more than three days a week, for three weeks
in an otherwise healthy child. Often crying occurs in the evening. It
typically does not result in long-term problems.
N.B: *broadly defined as an otherwise healthy infant < 3 months of
age who is*
1- crying for no apparent reason that lasts
≥ 3 hours/day and ≥ 3 days/week.
 
36. "Most common incidentally detected adrenal mass
A. subclinical Cushing
B. nonfunctional adenoma ✅ ✔
C. conns
D. adrenocortical cancer
 
37. Development delay + oral ulcer. ttt? ✅ ✔
N.B: *DDx*
Crohn Disease Imaging
Pediatric Chickenpox
Pediatric HIV Infection
Pediatric Herpes Simplex Virus Infection
Sprue Imaging
Syphilis
Systemic Lupus Erythematosus (SLE)
T-Cell Disorders
Zoster
 
38. Patient male 65 Y.O. has history of TIA and develop atrial fibrillation
what can be used to prevent the complication of Afib?
A. Aspirin
B. Warfarin INR 2-3 ✅ ✔
C. Warfarin INR 3-4
D. No need
 
39. Patient has DM on medication and BP 143/98 what to add?
A. BB
B. ACEI ✅ ✔
C. CCB
D. Diuretic
N.B: A class of medicine usually used to treat high blood pressure.
Angiotensin-converting enzyme (ACE) inhibitors also appear to
protect people with diabetes from diabetic nephropathy (kidney
disease).
 
40. Which of the following has the highest risk for ischemic heart
disease?
A. Inactive patient
B. BMI > 34
C. FBG > 8.3, 8.4 twice ✅ ✔
D. Abdominal circumference >36

41. Patient has DM, HTN and history of palpitations on exam has mid/
late diastolic murmur, low pitch, rumbling in character, and best heard
at the apex?
A. MS ✅ ✔
B. AS
C. MR
D. AR
 
42. Patient has typical presentation of TB which of the following confirm
the diagnosis of TB?
A. Plural fluid leukocytosis and lymphocytes
B. Pleural fluid AFB ✅ ✔
C. Pleural fluid high pH
D. Plural fluid high protein
 
43. Patient complain of epigastric pain diagnosed as PUD due to H.
pylori started on triple therapy what is the method to confirm
eradication?
A. Blood culture
B. Stool culture
C. Urea breath test ✅ ✔
D. Urea in urine
 
44. Patient 35 Y.O. has long history of heartburn and has fell like a
metallic taste in his mouth what is the most likely diagnosis?
A. Gastritis
B. Esophagitis ✅ ✔
C. Esophageal cancer
D. Bleeding

45. Asthmatic patient on SABA with increasing frequency of day and


night symptoms what is the appropriate next step?
A. Add LABA
B. Add  oral  steroid
C. Add inhaled steroid ✅ ✔
D. Add theophylline
 
46. Patient presented to the ER after MVA with crush injury to the lower
limb, and open fracture after stabilization the patient undergo ORIF
after the operation she develop SOB, petechia, Ferber and headache
what is the most likely diagnosis?
A. Fat embolism ✅ ✔
B. PE
C. Sepsis
D. Wound infection
 
47. Treatment of PU due to H. pylori?
A. Clarithromycin, amoxicillin, PPI ✅ ✔

48. 16-year-old female presented with abdominal pain and chronic


diarrhea undergo colonoscopy which showed skip lesion and redness in
the terminal ilium what is the most likely diagnosis?
A. UC
B. Crohnʼs disease ✅ ✔
C. IBS
D. PUD
 
49. Patient with RUQ pain, obstructive jaundice (ALP, bilirubin total and
direct high), amylase and lipase also high and on US there is stone what
is the most appropriate next step?
A. Lap Cholecystectomy
B. Open Cholecystectomy
C. ERCP ✅ ✔
D. X-ray
 N.B: C then A.

50. Nephrotic syndrome when to see it is steroid resistant?


A. 4 weeks ✅ ✔
B. 4 months
C. 12 weeks
D. 12 months
 
51. Patient heart failure on medication on ECG there is tall, peaked T
Wave what is the cause?
A. High K ✅ ✔
B. High Na
C. Low K
D. Low K
 
52. Patient with vomiting for three days on ECG he has flat T wave or
prominent U wave what is the cause ?
A. High K
B. High Na
C. Low K ✅ ✔
D. Low Ca
 
53. Patient with HTN not responding for three anti hypertensive drugs
on Ultrasound the right kidney is smaller than left what is the most likely
diagnosis?
A. Adrenal hyperplasia
B. Renal artery stenosis ✅ ✔
C. Pheochromocytoma
D. Pyelonephritis
 
54. Female patient what is the screening age for osteoporosis?
A. 60 year
B. 65 year ✅ ✔
C. 59 year
D. 55 year

 
55. Patient has microcytic anemia treated with iron after 3 months there
is NO improvement what is the next step?
A. Bone marrow biopsy
B. Karyotype
C. Hb electrophoresis ✅ ✔
D. CBC
 N.B: it is thalassemia.

56. Patient with fever and headache and bleeding on lab investigation
has anemia and elevated BUN also the PT, PTT, INR are high, and
fibrinogen is low what is the most likely diagnosis?
A. ITP
B. TTP
C. DIC ✅ ✔
D. HUS
 
57. Patient on chemotherapeutic agents develop tumor lysis syndrome
which of the following an occur?
A. High Ca
B. Low K
C. Low Ca ✅ ✔
D. Low uric acid
N.B: Tumor lysis syndrome :-
- Hyperkalemia
- Hyperphosphatemia
- Hyperuricemia
- Hypocalcemia
and high blood urea nitrogen (BUN)
79- case of HF on ACEI , thiazide, CCB come for routine check up,
examination was unremarkable just 4+ bilateral lower limb edema, what
do?
A. Change thiazide to loop diuretic ✅ ✔
B. Add BB
C. Continue same management
N.B: Correct answer is change CCB cuz it causes peripheral
edema, if not in choices so answer A.

80- female with headache for many weeks. MRI show nonfunctional
pituitary adenoma, what do?
A. Reassurance and follow after 6 months
B. Request hormones profile
C. Trans to neurology
D. Neurosurgery consultation ✅ ✔
N.B: For the great majority of patients with symptomatic
endocrine-inactive adenomas, transsphenoidal surgery and
adenoma removal is the preferred and most effective therapy.
N.B: Sheʼs having a headache and an adenoma, headache means
mass effect, compression symptom.
N.B: If size not mentioned, any symptomatic means referral.
N.B: Nausea and vomiting
Loss of appetite
Weight loss
Fatigue, decreased energy
Decreased mental function
Dizziness
Joint pains
Women: infertility, irregular or nonexistent menses
Men: infertility, impotence in men, loss of body and facial hair
Loss of sexual drive.
N.B: *first look at size*
Less than 1 cm reassure
More than 1 cm referral
If there is no size provided in Q , look at the symptoms if
symptomatic so referral.

81- 12 months baby start cow milk before 3 months. Now came with
low Hemoglobin, splenomegaly
What will see below normal?
A. Platelets
B. Reticlocyt
C. MCV ✅ ✔
N.B: C cuz Cow milk is deficient in iron so pt will have IDA and low
MCV.
N.B: Avoid cow's milk before 12 months of age, and limit intake in
patients 1 to 5 years old.
82- picture of vesicles in child and ask about treatment?
A. Acyclovir ✅ ✔
B. Antibiotic
C. Antiseptic
D. Steroid
N.B: Maculopapular rash = Measles or rubella.
N.B: Vesicular rash = Varicella.
N.B: Vesicles or pastules = HSV.

83- false positive increase BNP?


A. Obesity
B. Acei
C. COPD ✅ ✔

84- False -ve BNP?


A- obesity ✅ ✔

85- 51 years old female with typical feature of meningitis and ask about
organism?
A. S. Pneumonia
B. N. Meningitis
C. L. Monocyt ✅ ✔

86- case of sore throat and now came with lung infiltration. What
precaution?
A. Contact
B. Airborn
B. Airborn
C. Droplet ✅ ✔
N.B: Pneumonia + meningitis >> droplets
TB + measles + varicella >> airborne
MRSA > contacts.

87- complication of D/c?


A. Uterine perforation ✅ ✔
N.B: also Asherman syndrome and adhesion results in infertility

88- Lady Young 25 years old, has left adnexal mass 5 cm, pregnancy
test -ve, last cycle came before 2 weeks. Diagnosis?
A- follicular cyst ✅ ✔
B- corpus luteal cyst
C- ectopic pregnancy
D- pelvic kidney
N.B: Follicular Cyst; Most common ovarian mass in women of
reproductive age.
N.B: Corpus Luteal Cyst; Failure of corpus luteum to regress after
ovum release.
N.B: B with menstruation and pregnancy.
N.B: On US *Follicular US thin wall*
*Corpus US thick wall*.
89- risk factor for abrubtio placenta?
A. Preeclampsia ✅ ✔
N.B: Smoking as well.

90- whit veinal discharge with no oder?


A. Candida ✅ ✔
N.B: itching as well.

91- 50 years old man use alcohol come with abdominal pain. LFT high.
What diagnosis?
A. Alcoholic hepatitis ✅ ✔
B. Autoimmune hepatitis

92- antibiotic use in pregnant?


Not detect which week!
A. Nitro ✅ ✔
B. Floxacellin
C. Cipro

93- female came for routine check up. Show low hemoglobin with no
symptoms
A. Thalassemia beta minor ✅ ✔
A. Thalassemia beta minor ✅ ✔
B. Thalassemia beta major
C. SCD
N.B: Thalassemia trait.

94- child with VOC what give him?


A. analgesic and IVF ✅ ✔
B. 2 packet RBC

95- 50 years old male complaining of fatigue and show high LFT?
With no hepatic infection!
What also order?
A. AMA
N.B: question is not complete.
N.B: if female we might consider PBS so AMA.

96- 7 months after bariatric surgery complaining of abdominal pain


What investigation?
A. Abdominal XRAY
B. Abdominal US
C. Abdominal CT
D. endoscope
N.B: maybe CT to look for leakage.

97- obese non smoker with OSA?


A. Venturi mask
B. CPAP ✅ ✔
C. Steroid
N.B: I see this question with option D ( weight loss ) choose it first
then CPAP Nocturnal positive pressure therapy via CPAP is the
therapy of choice in symptomatic OSA after conservative.
N.B: OSA in adult and elderly > CPAP
In children > adenoid tonsillectomy.
N.B: so initially is weight reduction then best is CPAP.

98- 31 years old smoker, after give bronchodilator slightly improve?


A. Chronic bronchitis ✅ ✔
B. Asthma
N.B: his age and hx of smoking + mild improvement on BD goes
with A.
N.B: asthma onset is much earlier.

99- common Complication of Pertussis


A. Pneumonia ✅ ✔
B. Pneumothorax

100- third degree spleen fraction ?


A. Surgery
B. Conservative surgical ✅ ✔
N.B: Grade 1, 2 = conservative medical Treatment
Grade 3 = spleen conservative surgery
Grade 4,5 = splenectomy
If unstable patient immediately laparotomy regardless grade.

101- Female came with RtLQ pain, nausea and vomiting with guarding,
bhcg -ve, what is most appropriate diagnosis?
A- Ectopic pregnancy
B- Appendicitis ✅ ✔
C- Ruptured ovarian cyst
D- Ovarian torsion
N:B: N/V, acute pain not sudden, stable patient, more with
appendicitis.
N.B: Also look for pain
In appendicitis itʼs acute onset.
Torsion or rupture cyst will be sudden pain.
N.B: alvradi score for appendicitis :-

102- intussusception in adult?


A. Resection ✅ ✔
B. Reduction
C. Enema.
N.B: Intussusception :-
IV fluid + NGT
Then if pediatric: US and air enema
Adult: us then surgical resection.
N.B: Just adding to this great notes *Lead point in intussusception*
*Adults* : lead point is malignancy almost
Kids under age 2 (the vast, vast majority): GALT hyperplasia (e.g.,
from GI infection, vaccine responses, etc.)
If you understand lead point you will get the idea behind the
treatment.

103- cystic duct stone?


A- Mirizzi's syndrome ✅ ✔
N.B: MIrizzi syndrome due to obstruction in CHD ethir due to stone
impaction in CD or infandibulam.
104- Kwasaki treatment
A- Aspirin ✅ ✔
No IVIG in choices.
N.B: *Common Indications of IVIG :- *
1- *Kawasaki* (it is superior to Aspirin).
2- *Myasthenia Crisis* (plasma is superior).
3- *GBS* (it is superior to plasma).
4- *ITP* (it is superior to Steroid).

105- The best stage for screening is


Pre Disease stage
Clincal disease stage
Pre clinical disease stage ✅ ✔
Recovary
N.B: Screening, in medicine, is a strategy used in a population to
identify the possible presence of an as-yet-undiagnosed disease in
individuals without signs or symptoms. This can include individuals
with pre-symptomatic or unrecognized symptomatic disease.
N.B: Screening is to detect pathology before clinical manifestations
So C.
N.B: A before the pt affected with disease
C pt has disease but not appear yet.
N.B: Pre disease is to look for risk factors as primary prevention.

106- Face flush?


A- niacin ✅ ✔
B- statin

107- 20 % burn how you know about improving


Central Venous pressure 12
Normalized HR
Normalized BP
Urine output of 0.6 L/Kg ✅ ✔

108- RTA pt come with eye open to painful stimuli incomprehensible


sound and flex his leg when he get pain on it..
4
6
7
8✅ ✔

109- Boy with one umblical artery?


A- African
B- 80 % will have cardiac anomaly ✅ ✔
C- associated with high mortality
N.B: *single umbilical artery is associated with increased likelihood of
congenital abnormalities* for sure for sure for sure.

110- Case of sarcdosis what is the treatment..


A- Gloucocorticoids ✅ ✔
N.B: Indication of steroid in sarcoidosis
Hypercalcemia
Neurolgical involved
Progressive of disease

111- Treatment of gnralized anxiety disorder?


A- Citalopram SSRI ✅ ✔
N.B: *first CBT which is most effective when CBT + SSRI*.
112- 60 y.o with utrin bleeding spoting on the pad.. Source of blood?
Ovary
Utrin ✅ ✔
Lower gentalia
N.B: Utrin more likely to cause bleeding in old female.
N.B: First uterus
2nd crevix
Last vagina.

113- Chromosome that os affected in alzahimer:


21 ✅ ✔
15
13
N.B: 1 - *presenilin-1 (chromosome 14) and presenilin-2
(chromosome 1)
increased risk of early onset AD*
2- *Associated conditions Down syndrome 21*
3- *APOE2 is protective against AD*
N.B: Alzheimer related to four chromosomes:1,14,19,21.

114-,SCA presented with pallor (low Hb and high riticl)


aplastic crisis
hemolytic ✅ ✔

115-,AF case and you eant to do Rate control on patient with heart
failure?
- Digixin ✅ (not sure)
- no BB
N.B: Atrial fibrillation management:
If question said control *heart rate or ventricular rate* > diltiazam
or BB
But if question about heart *rhythm* control > answer is
antiarrhythmic medications such as flecainide, propafenone, or
amiodarone
If unstable pt > cardiversion
N.B: *rate control*
1- β-blocker or nondihydropyridine calcium channel blocker for
*stable*
2- digoxin if the patient is *hypotensive*
N.B: just to summarize
1st choice: beta blockers (esmolol, propanolol, metoprolol) OR
nondihydropyridine calcium channel blockers (diltiazem,
verapamil)
2nd choice: digoxin if came with CHF.

116- Treatment of camblybacter Jejeni?


A- Ciprofloxacin
B- Azithromycin ✅ ✔
117- Watery diarrhea and microscopic examination there is trophozoite?
Geradia ✅ ✔
Ameba

118- Moat common incidental adrenal mass ?


- nonfunctional adenoma ✅ ✔

119- Asthmatic patient on inhaled albetrol/steroid And she got


pregnant.. what to do?
- stop albetrol and continue steroid
- continue albetrol and stop steroid
- continue same mangement ✅ ✔
- use albetrol PRN

120- Patient on thyroxin and she got pregnant.. what to do?


- inc thyroxin ✅
- dec thyroxin
- stop thyroxin

121- PE patient.. how long he should take anticoagulant?


-3W
-6W
-6M✅ ✔
- 12 M
N.B: Provoked > 3 months
Unprovoked > 6 months

122- (Premitive reflex) rubing reflex. When dose it stop?


-2M
-4M
-6M✅ ✔

123- What drug that decrease mortality in esophial varicese


- octriotide
- PPI
-BB ✅ ✔
N.B: beta-blockers reduce mortality, whereas ligation does not.
N.B: in case of preventing second bleeding give pt BP, but if pt DM
give him elective ligation every 3 weeks or give him mono nitrate.

124- patient with UGITB.. what to gibe before endoscopy?


- octriotid ✅ ✔
- PPI
- bb

125- Pregnant lady developed fever. And she didnʼt get medical advice.
Why?
- self-medication
- high cost of medical advice
- neglect ✅ ✔

126- Baby with hemiscrotal swelling. On US he has spermatic cord


swelling
- appendiage tortion ✅ ✔
-testicular torsion
-epididymitis
N.B: Testicular torsion we do immediate detorsion withouy US, so A
mostly, also blue dots in upper pole is A.

127- Post carpal tunnel surgery and developed regional complex pain
syndrome.. she already sought orthopedic surgeon and pain specialist
Next?
- opoid
- phsyotherpy ✅ ✔
- counseling
N.B: *complex regional pain syndrome*
subacute in onset, sever regional burning pain following trauma
(fracture or sprain) or surgery, possibly due to inflammatory
cytokines .
pain with light tough and movement, edema, trophic changes, may
have motor weakness. x ray shows patchy demineralization.
ttt: PT/OP, NSAIDs, TCA or pergablin, topical lidocain or capsaicin.
N.B: *Complex regional pain syndrome*
1-Patient education:
2-Psychological interventions:
3-Physical and occupational therapy: *first-line therapy*
4- medication
N.B: Complex regional pain syndrome: physiotherapy. Carpal
tunnel syndrome: NSAIDs
128- Pneumococcal vaccines in elderly
- not recommended
- PVC13 then PCV23 ✅ ✔
- 23 then 13

129- what to find in plural tab 2Q


one of them in RA patient? (i chose low glucose) ✅ ✔
the other one in TB patient ? ( i choose +ve acid fast) ✅ ✔

130- patient with joint pain and in lab ferretin is 900


A- adult stills disease
B- Hemochromatosis ✅ ✔
N.B: *Most likely Hemochromatosis (Iron overload disease)*
It affects joints, *Typical: symmetrical arthropathy of the MCP
joints II and III*
Laboratory tests
↑ Serum iron
↑ Ferritin in serum > 200 µg/L
↑ Transferrin saturation (> 45%)
↑ Liver enzymes (AST, ALT)
+ TOtal Iron bindings capacity low
This is features of adult still disease :-
131- penciline allergy what C/I ?
Meropenem
ceftrixon
tazocin ✅ ✔

132- sub gene of HCV in Saudi?


1
2
3
4✅ ✔

133- Advise to DM?


- Low carb best than low fat in long management
-low fat decrease 5-10%
- low glycemic index decrease 5-10% ✅ ✔

134- false increase BNP


A- COPD ✅ ✔
B- Obesity

135- screen poycyctic kidney disease?


- US ✅ ✔
- polyscystin antibody

136- 16 yo, with fever for 6 days, he is lethargic, fatigued, on


examination there was large tender splenomegaly, what is the culture
with highest diagnostic yield ? (That is it, no more details)
A) stool and urine
B) single blood culture
C) multiple blood cultures
D) bone marrow aspirate culture (lukemia?)

137- What is the most likely fundal height in normal pregnancy? (Didn't
specify GA)
A) above smphisys pubis ✅ ✔
B) between smphisys pubis and umbilicus
C) around umbilicus
D) above umbilicus

138- Smoker son had his father diagnosed with lung cancer, the son
wants to stop smoking after his father was diagnosed, the son have a
Hx of Epilepsy (not sure if epilipsy or just seizure), which of the
following smoke cassation medication is CI in the son's condition?
A) bupropion ✅ ✔
C) nicotin replacement stuff

139- Patient with primary hyperparathyroidism did lower left


parathyriodectomy, 4 months later presented with constipation (other
symptoms or hypercalcemia), what is the underlying cause to current
presentation?
A) missed adenoma ✅ ✔
B) parathyroid hyperplesia

140- 1st time microalbuminurea in alb/cr ratio what is next?


Us renal
repeate alb-ratio
stop metformin
24h urine albumin

141- Platue Bhcg after salpinostomy after 3 week What to do?


- Reassure and follow up
- Nmethotrexate

142- Honey crusted exudate in eczema patient


- Impetigo
- boil

143- Sign of iron overload in every organ What investigation ?


-Iron load from liver biopsy
- Cupper
- A 1 anti trypsin
- Antimitochondrial ab

144- Prolactinoma next ttt?


Caprogrlin ✅ ✔

145- adrenal mass lipid size 5 what to do?


- Observe
- lap adrenalectomy ✅ ✔

146- Fibroid asymptomatic what to do ?


Follow up annually US
fowllo up CBC and us every 2 months
Follow US every 6 months

147- Bacterial vaginosis what is expected to see?


Multinuculated large cell
Epithelial cells ✅ ✔

148- Ttt of TOF with cyanotic spell ?


Sedation and make him stable ✅ ✔
diuritc

149- ANA -VE when to screen uvitis ?


3 or 6 or 12 month?
Answer; 6 months ✅ ✔

150- radial injury most site?


Spiral grove of hemurs
olecranon
axillary
wrist

151- LT stylomastoid injury during delivery of baby ?


Cant close LT eyelid ✅ ✔
loss of ant 2/3 of taste

152- HTn, IUGR What else will be associated with it ?


GDM
oligohydromenis ✅ ✔
polyhydrominos

153- A diabetic presented with well demarcated painful superficial


erythema, whatʼs the microorganism?
Erysipelas
Necrosis lipidca diabetigenic
154- Cancer have 1 year Came for back pain with high Ca
- give opoid
- isotonic flude
- alendronate
- forusmide

155- Drug treat osteoporosis and hypercalcmia of malignancy


- alendronate
- pamidronate
-zoledronate ✅ ✔

156- PCOS... next?


Glucose and lipid ✅ ✔

157- Facial palsy.. what decrease duration of the illnes?


Steroid ✅ ✔

158- FAST... wt to see?


Intrapertonim

159- Infected mesh?


Remove

160- Pap smear done > inflmatory Then she did colopscy biosy shows
>> invasive carcinoma / what is next?
-conization of the lesion
- clinical staging of the cancer ✅ ✔

161- Anesthesia contraindicated in intestinal obstruction surgery


- Ketamine
- Propofol
- NO ✅ ✔

162- progressive non-petting limb swelling for 3 M and bluish


discoloration. no trauma or travel history. Next stem?
- conventional venography
- MR venography ✅ ✔
- CT angio
- Doppler
No lymphschintographt in choices

163- Bilateral varicose vein in great saphenous


- Sclerotherapy
- Endovascular ablation

164- Patient with gunshot to left chest. Thoracostomy tube drains 2


liters of blood. What is should you do?
-another thoracostomy tube
- thoracotomy ✅ ✔
- antibiotic
- embolization

165- In case of death of newborn What should you do immediately for


the parents?
- Ask the father if he would like to file a complaint
- Let the mother spend sometime with her dead baby ✅ ✔
- arrange a close follow up with the primary team

166- Young man, had a trauma to his 2nd finger while it was
hyperextended, the man can't flex or extend the distal phalanx, also the
distal phalanx was tender to touch, but he can move the proximal
phalanx freely, he also reported feeling tenderness in his palm, where is
the the injury ?
- deep flexor dogitorum tendon
- superficial flexor digitorum tendon
- distal interphalngeal joint
- distal phalanx bone

167- CTG graph with VERY low variability (almost flat line), but FHR was
about 150 or 140. No decelerations or any variability. And the mother
showed no symptoms of any abnormality. Patient on Oxytocin, MgSO4,
epidural. What is the cause between those three.
- Oxytocin
-MgSO4 ✅ ✔
- epidural

168- Child his older brother died sudden cardiac death when he go to
work???!
- pda
- vsd
- asd
- hypertrophic cardiomyopathy ✅ ✔

169- 7yo diagnosed with T1DM came to ER with DKA and is comatosed.
Mother said she stoped his treatment because she doesnʼt think itʼs
true and wants his pancreas to have time to grow. This represent?
- Denial
- Neglect
- compromising
170- About 28 years old, came to ER with heavy vaginal bleeding, she is
nulliparous, pregnancy test negative, no pain, regular cycle, but always
with menorrhagia, how to stop the bleeding now ?
- give estrogen (just estrogen, no OCP)
- give progesteron
- give GnRH
- inserted levenogestrel IUD ✅ ✔

171- Type of anima of 2 Y child not eating meat only on cow milk ?
B12

172- Patient with perforated appendicitis after surgery had pus from
wound, pain localized to surgical site. No guarding no fever what is best
initial treatment :
- Antibiotics
- Open drainage
- Imaging guided drainage
- Wound exploration ✅ ✔

173- Patient with rheumatoid arthritis taking Steroids + chloroquine +


methotrexate with no remission what drugs to add: - Azathioprine
- Adualinomab
- Cyclosporine
- Mycophenolate moetifil (celsept)

174- Pateint have "hacking cough"


TB?

175- Fetus Presnt with orbit nose mouth what is fetal presentation?
Face presentation ✅ ✔
176- Fetus after delivery by ventuze Nurse noted bleeding not crossing
suture lines or something like that diagnosis :
- Caput seccondum
- Cephalohematoma ✅ ✔
N.B: A not blood and cross suture line.

177- Patient came to ER with RLQ pain. CT shows he has appendicitis


and 5*13 cm appendiceal abscess that extends from ... To the
abdominal wall. What is the best management?
- Lap appendectomy
- Lap appendectomy and drainage of abscess
- Open appendectomy and drainage of abscess
- percutaneous abscess drainage ✅ ✔

178- Pt underwent hernial repair for right inguinal hernia, presented


now complaining of that ipsilateral testicular size is decreased after
hernial repair. Dx?
- Testicular artery occlusion
- tight mesh
- pampiniform plexus occlusion ✅ ✔

179- Athletes with constant planter foot pain examination showed


centeral planter tenderness
- Haullax rigidus
- Haullax valgus
- Tibial nerve entrapment
- Planter facilities

180- women presented with her infant he constantly spit after


feeding normal weight
- Itʼs normal and he will grow out of it
- Refer to gastrointrologist
- Teach her to burb her infant and to hold him upright after feedig

181- family of terminally ill adult in pain asked the physician to


increase pain meds he agree after explaining that it could lead to
death what this approach called
- Informed choice
- Totality ✅ ✔

182- patient complains of cough and wheezing. His CXR & spirometer
were normal, what to do next?
- repeat spirometer
- methcholin challenge test ✅ ✔
- oral steroid
- observe

183- pregnant lady went into Labor, she complained of numbness and
tingling in the middle thigh after delivery? Which nerve injured?
- obturator ✅ ✔
- Femoral
- Sciatic

184- What sign indicates pancreatic iron toxicity?


- DM ✅ ✔
- jaundice

185- A case of uterine fundal cancer, and asked about lymph node
drainage?
A- Par-aortic ✅ ✔

186- Femoral hernia in 70 year old female asked for surgery


- Laparoscopic ✅ ✔
- open

187- Serious acute otitis media complication


A- hearing loss ✅ ✔

188- A pap smear was done showed abnormality, then biosy was done
and showed invasive carcima whats next?
-surgical staging ✅ ✔
- colonization cone of lesion
- repeat

189- Pt after cardiac bypass ,developed hypotension tachycardia and


decrease SVR? what is the type of shock
- hypovolemic
- cardoiogenic
- Septic

190- Pt after MI he is on clopidogril ,BB ,ACEI. Wt to give?


- Atorvastatin ✅ ✔
- Fibrates
- Niacin

191- Pt lost a lot of blood post-op. had to be transfused 15 units. Went


to the ICU and started bleeding from venipunctures and NGT
(everywhere). What is the cause
- Transfusion reaction
- Thrombocytopenia
- Hypocalcemia
-DIC ✅ ✔

192- Pt with malar rash with photosensitivity and artharlagia ,what test
is used to confirm the DDx?
A – ANA
B-DsDNA ✅ ✔
C- RF

193- PT developed facial and tongue swelling after Anti HTN


medication?
A – Ranilapri
B- Bisoprolol

194- Long scenario about pt came with Sx of DKA what type of insulin
U will give?
A – according to sliding scale
B- Fixed rate Iv infusion ✅ ✔
C- insulin BID

195- Pt known case of Rheumatoid Arthritis came with RT knee swelling


and pain how will U manage
A- Abx
N.B: Septic arthritis or Popliteal cyst = more info needed. ✅ ✔
196- Pt with arthritis with past Hx of UTI (Reactive arthritis) How to
manage?
A -Steroid
B- NSAID ✅ ✔

197- Pt 35 YO female her mother died with breast cancer and her sister
died with ovarian cancer ,what is the best screening modality for Breast
cance?
A -BRCA Gene ✅ ✔
B- Mammography
C- breast self exam

198- Long case, Flat T wave on ECG what do you find in urine:
A-High potassium
B-Sodium
C-aciduria ✅ ✔

199- Pt known asthmatic with Flat T wave on ECG what medication can
cause that ?
A- Salbutamol ✅ ✔
B- Salmetrol

200- Chlid came to the ED with gastroenteritis , U discharge him home


after telling him to come to the hospital if there is warning Sx? U are
doing
A – safety Netting and other choices I cant remember
201- Child drinking Cow milk 3 times a day with pallor ? IDA (No B12 in
choices)

X-ray showing radio-opaque mass at McBurneyʼs point and rebound


tenderness... what to do next:
A-Appendectomy ✅ ✔
B-CT without contrast

202- Management of a patient with gastrointestinal stromal tumor


of the stomach 5 cm No metastasis No lymph node enlargement ?
In the body of
A- Gastrectomy
B- Wide local excision with Free margins ✅ ✔
C- Radiotherapy and chemotherapy

203- Late complication of meningitis:


A-Seizure
B-Hearing loss ✅ ✔
C-Facial nerve palsy
D-Ataxia

204- Looong case of ITP with plt 32 how to manage?


A - IVIG and steroid ✅ ✔
B -Plt transfusion
C- plasmapheresis

205- pt is taking UFH for PE after surgery -CBC was provided with
Low platelet count what to do?
A – Continuo heparine
B- Replace with Enoxaprine
C- stop Heparien and replace it with bivalirudin (another med with same
sound) ✅ ✔

206- Side Effect from Furosemide? Hypokalemia ✅ ✔

207- Pt 55 Yo smoker and came with breathlessness DDx?


COPD ✅ ✔

208- Pt came with retrobulbar pain headache and fever?


Dengue Fever ✅ ✔

209- Pt came with gait instability and urinary incontince ,he is


known case of Osteoarthritis and had cervical laminectomy after
cervical degenerative myopathy what is DDx?
A -Cauda Equina
B- Recurrent Cervical Myopathy ✅ ✔

210- a case of macrosomic baby and Complications?


Erb's palsy ✅ ✔

211- Pt with PE and hypotensive ,management ?


Alteplase ✅ ✔

212- Pt with UTI and high creatinine which Abx is contraindicated?


Cipro
Septrin ✅ ✔

213- Most common diagnosis of Pt with Fever from Sub-Saharan


region?
Malaria ✅ ✔

214- Child canʼt handle spoon well But he can walk up the stairs
with one hand held What is his age:
18 months

215- Pt came with pin prick sensation and loss of vibration


sensation with loss of ankle reflex and K was 5.5 and Creatinine
was very high what to do?
A – Vitamin B 12
B – Dialysis ✅ ✔
c- Bicarbonate

216- child came for routine checkup U found a murmur that is late
systolic changing with position ?
innocent murmur ✅ ✔

217- a child with murmur ,I think he was anemic?


A- treat the cause ✅ ✔

218- Pt came with pleural affusion after 2 weeks from pneumonia


infection what is the DDx?
Parapneumonic effusion ✅ ✔

219- Long case of pancreatitis after ERCPs , which of the following


will increase the survival rate of the pt ?
A – abx
B- Ringer lactate ✅ ✔
220- Low grade dysplasia with barret esophagus ?how to manage
A – Resection
B – PPI and follow up 6-12 months ✅ ✔

221- pregnant with fundal height more the GA and hemoptysis DDx
Choriocarcinom ✅ ✔

222- most common cancer in pediatric age? Leukemia ✅ ✔

223- Polymyalgia Rheumatic with fatigue ?


CLL ✅ ✔

224- Hashimoto Thyroiditis associated cancer? Lymphoma ✅ ✔

225- mamngement of acute Migraine headache? Aspirin ✅ ✔

226- management of Kawasaki ?


Aspirin (if IVIG its better) ✅ ✔

227- Nurse with +ve HCV antibodies and -ve HCV Ig M what to do?
Immune

228- 54 years male after lap cholecystectomy 5 days develop


abdominal distinction Us showed ascites What is your next
management
- drain of fluid
- other options I forgot

229- pt k/c of chronic hepatitis b presented with jaundice and


fatigue Inv: high Trianinases low HB DNA what to do - liver biopsy
- anti delta antibodies ✅ ✔
- interferon
- repeat test

230- What is the precatnage of Male smokers in Saudi Arabia?


- 5%
- 10%
- 37% ✅ ✔
- 51%

231- Long scenario, what is the average basal body temperature


during ovulation ?
- 36
- 36.5 ✅ ✔
- 37
- 37.5

232- Question about rheumatoid arthritis and itʼs effect on


pregnancy
- abortion
- somthing antibodies
- retardation

You might also like